277
Nguyễn Hữu Điển OLYMPIC TOÁN NĂM 2000 52 ĐỀ THI VÀ LỜI GIẢI (Tập 1) NHÀ XUẤT BẢN GIÁO DỤC

OLYMPIC TOÁN NĂM 2000 52 ĐỀ THI VÀ LỜI …»i nói đầu Để thử gói lệnh lamdethi.sty tôi biên soạn một số đề toán thi Olympic, mà các học trò của

  • Upload
    ngonhu

  • View
    214

  • Download
    0

Embed Size (px)

Citation preview

Page 1: OLYMPIC TOÁN NĂM 2000 52 ĐỀ THI VÀ LỜI …»i nói đầu Để thử gói lệnh lamdethi.sty tôi biên soạn một số đề toán thi Olympic, mà các học trò của

Nguyễn Hữu Điển

OLYMPIC TOÁN NĂM 2000

52 ĐỀ THI VÀ LỜI GIẢI(Tập 1)

NHÀ XUẤT BẢN GIÁO DỤC

vnmath.com

Page 2: OLYMPIC TOÁN NĂM 2000 52 ĐỀ THI VÀ LỜI …»i nói đầu Để thử gói lệnh lamdethi.sty tôi biên soạn một số đề toán thi Olympic, mà các học trò của

2

vnmath.com

Page 3: OLYMPIC TOÁN NĂM 2000 52 ĐỀ THI VÀ LỜI …»i nói đầu Để thử gói lệnh lamdethi.sty tôi biên soạn một số đề toán thi Olympic, mà các học trò của

Lời nói đầu

Để thử gói lệnh lamdethi.sty tôi biên soạn một số đề toán thi Olympic, màcác học trò của tôi đã làm bài tập khi học tập LATEX. Để phụ vụ các bạn hamhọc toán tôi thu thập và gom lại thành các sách điện tử, các bạn có thể thamkhảo. Mỗi tập tôi sẽ gom khoảng 50 bài với lời giải. Tập này có sự đóng góp củaBùi Thế Anh, Vũ Thị Hồng Hạnh, Cao Thị Mai Len, Tạ Xuân Hòa, NguyễnThị Loan, Nguyễn Thị Quý Sửu, Nguyễn Thị Định, Nguyễn ngọc Long.

Rất nhiều bài toán dịch không được chuẩn, nhiều điểm không hoàn toànchính xác vậy mong bạn đọc tự ngẫm nghĩ và tìm hiểu lấy. Nhưng đây là nguồntài liệu tiếng Việt về chủ đề này, tôi đã có xem qua và người dịch là chuyên vềngành Toán phổ thông. Bạn có thể tham khảo lại trong [1].

Rất nhiều đoạn vì mới học TeX nên cấu trúc và bố trí còn xấu, tôi khôngcó thời gian sửa lại, mong các bạn thông cảm.

Hà Nội, ngày 2 tháng 1 năm 2010

Nguyễn Hữu Điển

51GD-05

89/176-05 Mã số: 8I092M5

vnmath.com

Page 4: OLYMPIC TOÁN NĂM 2000 52 ĐỀ THI VÀ LỜI …»i nói đầu Để thử gói lệnh lamdethi.sty tôi biên soạn một số đề toán thi Olympic, mà các học trò của

Mục lục

Lời nói đầu . . . . . . . . . . . . . . . . . . . . . . . . . . . . . . . . . . . . . . . . . . . . 3

Mục lục . . . . . . . . . . . . . . . . . . . . . . . . . . . . . . . . . . . . . . . . . . . . . . . 4

Chương 1. Đề thi olympic Belarus . . . . . . . . . . . . . . . . . . . . . . . . . 5

Chương 2. Đề thi olympic Bungari . . . . . . . . . . . . . . . . . . . . . . . . 16

Chương 3. Đề thi olympic Canada . . . . . . . . . . . . . . . . . . . . . . . . 29

Chương 4. Đề thi olympic Trung Quốc . . . . . . . . . . . . . . . . . . . . 32

Chương 5. Đề thi olympic Tiệp khắc . . . . . . . . . . . . . . . . . . . . . . 41

Chương 6. Đề thi olympic Estonia . . . . . . . . . . . . . . . . . . . . . . . . 46

Chương 7. Đề thi olympic Hungary . . . . . . . . . . . . . . . . . . . . . . . 51

Chương 8. Đề thi olympic India . . . . . . . . . . . . . . . . . . . . . . . . . . . 56

Tài liệu tham khảo. . . . . . . . . . . . . . . . . . . . . . . . . . . . . . . . . . . . 59

vnmath.com

Page 5: OLYMPIC TOÁN NĂM 2000 52 ĐỀ THI VÀ LỜI …»i nói đầu Để thử gói lệnh lamdethi.sty tôi biên soạn một số đề toán thi Olympic, mà các học trò của

Chương 1

Đề thi olympic Belarus

.1.1. Hai đường chéo AC và BD của tứ giác ABCD cắt nhau ở M. Đường phân

giác của góc ACD cắt tia BA ở K . Nếu MA.MC+MA.CD = MB.MD

thì BKC = CDB.

Lời giải: Gọi N là giao điểm của CK và BD. Áp dụng định lí về đườngphân giác cho tam giác MCD

CD

ND=MC

MN

Hay

CD =MC.DN

MN

khi đó có MB.MD = MA.MC +MA

MC.DN

MN= (MA.MC)

MD

MN

Hay MA.MC = MB.MN

Vì M nằm trong tứ giác ABCN , theo định lí về phương tích của mộtđiểm thì A, B, C và N cùng nằm trên một đường tròn.Từ đó:

KBD = ABN = ACN = NCD = KCD

Suy ra K,B,C và D cùng nằm trên một đường tròn. Do đó có

BKC = CDB.

vnmath.com

Page 6: OLYMPIC TOÁN NĂM 2000 52 ĐỀ THI VÀ LỜI …»i nói đầu Để thử gói lệnh lamdethi.sty tôi biên soạn một số đề toán thi Olympic, mà các học trò của

6 Nguyễn Hữu Điển, ĐHKHTN Hà Nội

.1.2. Trong một tam giác đều xếp n.(n+1)2

đồng xu và n đồng xu xếp dọc theo

mỗi cạnh và luôn có một đồng xu ở ngọn( ở trên cùng) Một phép thế vị

xác định bởi cặp đồng xu và tâm A, B và lật mọi đồng xu nằm trên đoạn

thẳng AB. Hãy xác định những yếu tố ban đầu- giá trị của n và vị trí

ban đầu của đồng xu có mặt trái mà từ đó có thể khiến cho tất cả đồng

xu hiện ra mặt trái sau một số phép thế vị.

Lời giải: Vì mỗi phép thế vị của 0 hoặc 2 đồng xu trong 1 góc, tínhchẵn lẻ của số ngọn trong góc là được bảo toàn.Nếu đồng xu cho thấy mặt trái không ở trong một góc, luôn có 3 đồngxu trong góc là ngọn, thì luôn có số ngọn trong góc là lẻ. Như vậy, sẽluôn có 3 góc không đồng thời cho mặt trái của đồng xu.Ngược lại, nếu trong một góc có đồng xu mặt trái, chúng ta sẽ chứngminh rằng ó thể làm cho tất cả các đồng xu hiện mặt tráiTa hướng tam giác sao cho góc đó đi đến với một cạnh nằm ngang;Trong mỗi (n - 1) đường ngang có hai hoặc nhiều đồng xu. Ta chọn haiđồng xu kề nhau và lật trái tất cả các đồng xu trong đường này. Tất cảcác đồng xu sẽ cho thấy mặt trái.Do đó yếu tố ban đầu cần lựa chọn là có đồng xu có mặt trái nằm trong1 góc.

.1.3. Cho tam giác ABC và góc C = π2

gọi M là trung điểm của cạnh huyền

AB, H là chân đường cao CH và P là điểm trong tam giác sao cho AP =

AC. Hãy chứng minh rằng PM là phân giác BPH khi và chỉ khi A = π3.

Lời giải: Lời giải thứ nhất

Điểm P nằm trên đường tròn ω tâm A bán kính AC. đường tròn ω cắtđường CH,MH và PH tại D, N và Q. Vì MA = MC, A = π

3khi và chỉ

khi tam giác ACM đều. Nghĩa là khi và chỉ khi M≡N. Điều đó khảngđịnh PM là phân giác góc HPB khi và chỉ khi M≡NThật vậy, AH là đường cao thuộc đáy của tam giác cân ACD, H là trungđiểm của CD, CD là một dây cung của đường tròn ω , theo định lí vềphương tích của một điểm có

PH.HQ = CH.HD = CH2.

vnmath.com

Page 7: OLYMPIC TOÁN NĂM 2000 52 ĐỀ THI VÀ LỜI …»i nói đầu Để thử gói lệnh lamdethi.sty tôi biên soạn một số đề toán thi Olympic, mà các học trò của

Đề thi olympic Belarus 7

Và vì CH là đường cao thuộc cạnh huyền của tam giác vuông ABC nênCH2 = AH.HB. Vậy PH.HQ = AH.HB.Do H là giao điểm của AB và PQ nên tứ giác APBQ nội tiếp. Xét trênđường tròn ω

QAB = QAN = 2.QPN = 2.HPN

Như vậyHPB = QPB = QAB = 2.HPN

Và vì N là giao điểm của HB và PN phân giác của góc HPB. Do đóPM là phân giác của góc HPB khi và chỉ khi M ≡ N

Lời giải thứ hai

Không mất tính tổng quát ta giả sử AC = 1. Dựng hệ trục tọa độ vuônggóc với C làm gốc, A có tọa độ (0; 1) còn B có tọa độ (n; 0) với n > 0Nếu n = 1 thì M≡N và PM không thể là phân giác của góc BPH. Trongtrường hợp này có A = π

46= π

3điều này trái với kết quả mong đợi

Chính điều đó cho phép ta chọn n 6= 1 Sử dụng công thức khoảng cáchđể có AP = AC khi và chỉ khi P có tọa độ dạng (±

√m.(2 −m);m) và

m nằm giữa 0 và 2. Tọa độ của M là (n2; 1

2) và vì CH có độ đổi n và H

trên AB, nên H cần tìm có tọa độ ( nn2+1

; n2

n2+1) . Sử dụng công thức tính

khoảng cách ta tính được

BP

HP=

√n2 + 1

Sử dụng hệ thức trong tam giác vuông AHC và ACB có AH = b2

cvới

b = CA, c = AB; từ đó

MB

MH=

c2

c2− b2

2

=c2

c2 + 2.b2=n2 + 1

n2 − 1

Theo định lí đường phân giác PM là phân giác BPH khi và chỉ khiBPHP

= MBMH

. Giải phương trình tương ứng ta tính được nghiệm khi và chỉkhi n2(n2 − 3) = 0 vì n > 0 nên PM là phân giác góc BPH khi và chỉkhi n =

√3, nghĩa là khi và chỉ khi A = π

3.

.1.4. Có tồn tại một hàm f : N −→ N sao cho

f(f(n− 1)) = f(n+ 1) − f(n)

vnmath.com

Page 8: OLYMPIC TOÁN NĂM 2000 52 ĐỀ THI VÀ LỜI …»i nói đầu Để thử gói lệnh lamdethi.sty tôi biên soạn một số đề toán thi Olympic, mà các học trò của

8 Nguyễn Hữu Điển, ĐHKHTN Hà Nội

với mọi n > 2 ?

Lời giải: Khi khẳng định tồn tại một hàm như vậy sẽ dẫn đến mâuthuẫn. Từ phương trình f(n -1) - f(n) > 0 với n > 2 điều này khẳng địnhhàm f tăng nghiêm ngặt với n > 2 như vậy, f(n) > f(2) + (n− 2) >

n− 1 với n > 2 Chúng ta có thể làm nên f(n) như sau: Từ phươngtrình đã cho mặc nhiên có f(f(n − 1)) < f(n + 1) với n > 2 hay làf(f(n)) < f(n + 2) với n > 1. Vì f là hàm tăng với những biến lớn hơn1, cho f(n) = 1 hoặc f(n) < n+2. Từ đó n− 1 6 f(n) 6 n + 1 với mọin > 2. Lấy n nguyên bất kỳ bé hơn 4Một mặt f(n) > 2 và (n− 1) > 2

f(f(n− 1)) = f(n− 1) − f(n) 6 (n + 2) − (n− 1) = 3

Như vậy,(n − 3) 6 3 vì bất kì n > 4 là điều vô lý. Điều này cho thấykhẳng định ban đầu là không đúng và cho kết luận không tồn tại mộthàm như thế.

.1.5. Trong một đa diện lồi với m mặt tam giác( còn các mặt khác với hình

dạng khác), Ta luôn có 4 cạnh bên gặp tại mỗi đỉnh. Tìm giá trị nhỏ

nhất có thể của m.

Lời giải: Lấy 1 đa diện với m mặt tam giác và 4 cạnh bên gặp nhau tạimỗi đỉnh. Đặt F, E và V là số mặt, cạnh bên và đỉnh của đa diện. vớimỗi cạnh bên, đếm hai đỉnh và các đầu mút. Vì mỗi đỉnh là đầu mútcủa 4 cạnh bên, chúng ta đếm đỉnh 2 lần theo cách này. Như vậy 2E =4VNgoài ra, đêm số cạnh bên trên mỗi mặt và tổng của F cao nhất đạtđược là một số ít nhất là 3m + 4(F - m). Mỗi cạnh bên được đếm 2 lầntheo cách này, suy ra 2E > 3m+ 4(F −m)

Qua biểu thức Euler cho biểu đồ phẳng, F + V −E = 2.Kết hợp với 2E = 4V đẳng thức này là 2E = 4F − 8

Như vậy4F − 8 = 2E > 3m+ 4(F −m)

Hay m > 8 sự cân bằng đạt được nếu và chỉ nếu mỗi mặt của đa diện làtam giác hoặc tứ giác, một hình tám mặt đầu có những hình như vậy.

vnmath.com

Page 9: OLYMPIC TOÁN NĂM 2000 52 ĐỀ THI VÀ LỜI …»i nói đầu Để thử gói lệnh lamdethi.sty tôi biên soạn một số đề toán thi Olympic, mà các học trò của

Đề thi olympic Belarus 9

Suy ra m = 8 là giá trị đạt được.

.1.6. a) Chứng minh rằng{n√

3}> 1

n√

3với tất cả số nguyên dương n, trong

đó {x} được hiểu là phân số của x.

b) Có tồ tại bất biến c > 1 để mà{n√

3}> c

n√

3cho mỗi n nguyên

dương?.

Lời giải: Điều kiện{n√

3}> c

n√

3có thể áp dụng với n = 1 nếu chỉ

nếu 1> c√3

ví dụ√

3 > c. Đặt 1 6 c <√

3 là một bất biến với mỗin,

{n√

3}

= n√

3 −[n√

3]

lớn hơn c

n√

3nếu chỉ nếu n

√3 − c

n√

3. Vì

c <√

3 < 3n2, hai vế của bất dảng thức này là dương, chúng ta chỉ cóthể bình phương mỗi vế mà không làm đổi dấu bất đẳng thức.

3n2 − 2c+c2

3n2>

[n√

3]2

(*)

Với mỗi n, 3n2 − 1 không phải là số chính phương vì không có số chínhphương nào đồng dư 2 mod3, và 3n2 cũng không phải là số chính phương.Như vậy,

[n√

3]

=[√

3n2]

số nguyên lớn nhất mà bình phương của nónhỏ hơn hoặc bằng 3n2 tối đa 3n với cân bằng nếu và chỉ nếu 3n2 − 2

là số chính phương. Chúng ta yêu cầu rằng sự cân bằng áp dụng tùy ývới n.Xác định (m0, n0) = (1, 1) và (mk+1, nk+1) = (2mk + 3nk, mk, 2nk) vớik> 1.Dễ ràng chứng minh rằng m2

k+1 − 3n2k+1 = mk − 3n2

k. Như vậy, do đẳngthức 3n2

k − 2 = m2k áp dụng với k = 0, áp dụng với tất cả k > 1. Do

n1, n2, . . . là một chuỗi tăng nó dẫn đến 3n2 − 2 là một số chính phươngvới n tùy ý.Nếu c = 1 như vậy 3n2 − 2c+ c2

3n2 > 3n2 − 2c = 3n2 − 2c >(n√

3)2 cho

tất cả n. Như vậy (*) là bất đẳng thức áp dụng cho tất cả nTuy nhiên, nếu c> 1 thì 3n2 − 2c+ c2

3n2 6 3n2 − 2 cho tất cả các số lớn nthỏa mãn. Như vậy, tồn tại một số n với điều kiện thêm là 3n2 − 2 phảilà số chính phương. Với n này (*) và đẳng thức b) là saiVậy câu trả lời đối với phần b) là "không".

.1.7. Cho tập hợp M = {1, 2, . . . , 40}. Tìm giá trị n nhỏ nhất(n: số nguyên)

mà có thể chia tập M thành n tập con rời nhau để mà bất kì a, b và

vnmath.com

Page 10: OLYMPIC TOÁN NĂM 2000 52 ĐỀ THI VÀ LỜI …»i nói đầu Để thử gói lệnh lamdethi.sty tôi biên soạn một số đề toán thi Olympic, mà các học trò của

10 Nguyễn Hữu Điển, ĐHKHTN Hà Nội

0(không nhất thiết khác biệt) nằm trong cùng tập con, thỏa mẵn a 6= b+c.

Lời giải: Giả sử cho mục đích mâu thuẫn, có thể chia tập M thành 3tập X, Y và Z. Không mất tính tổng quát ta giả sử rằng |X| > |Y | > |Z|cho các x1, x2, . . . , x|X| là các thành phần của X được sắp xếp theo thứtự tăng dần. Những số này, bên cạnh các chênh lệch xi−x1 với i = 2, 3,|X|, phải là những thành phần khác biệt của M. Có 2|X|-1 số như vậy,suy ra 2|X|-16 40 hay |X| 6 20. Ta có 3 |X| > |X| + |Y | + |Z| = 40,suy ra |X| > 14. Ta có |X| . |Y | >

12|X| (40 − |X|) đôi trong X.Y. Tổng

của các số trong mỗi cặp đôi nhỏ nhất là 2 và lớn nhất là 80 có cả79 giá trị có thể xảy ra vì 21 > |X| > 14 và hàm t −→ 1

2+ (40 − t)

là hàm lõm trên đoạn 21 > t > 14 chúng ta có 12|X| .(40 − |X|) >

min{

12.14(26), 1

2.21(19)

}=182 > 2.79.

Theo nguyên tắc Pigeonhole tồn tại 3 cặp đôi (x1, y1), (x2, y2),(x3, y3) ∈X.Y với (x1 + y1) = (x2 + y2) = (x3 + y3)

Nếu bất cứ xi nào bằng nhau thì tương ứng yi sẽ bằng nhau, điều nàylà không thể xảy ra vì cặp (xi, y − i) là khác biệt. Như vậy, chúng tacó thể giả sử, không làm mất tính tổng quát rằng x1 < x2 < x3 với1 6 j < k 6 3 giá trị xk−xj nằm trong M và không thể nằm trong X vìmặt khác xj + (xk − xj) = xk. Tương tự yj − yk /∈ Y với 1 6 j < k 6 3

Như vậy, 3 sự chênh lệch bằng nhau x2 −x1 = y2−y1, x3 −x2 = y3−y2,x3 − x1 = y3 − y1 nằm trong M \ X ∪ Y = Z. Đặt a = (x2 − x1),b = (x3 − x2), a = (x3 − x1) ta có a = b+ c và a, b, c ∈ Z, suy ra mâuthuẫnNhư vậy giả sử ban đầu của chúng ta sai và không thể phân chia Mthành 3 tập thỏa mãn yêu cầu đặt ra.Bây giờ có thể chứng minh chia M thành 4 tập với yêu cầu đặt ra. Nếuai ∈ {0, 1, 2} với tất cả i ∈ N và nếu ai = 0 với n > N, sau đó đặt(. . . a2a1a0) và (aNaN−1 . . . a0) được hiểu là số nguyên

∑ni=0 ai3

i đươngnhiên giá trị nguyên m có thể viết dưới dạng (. . . a2a1a0) theo một cáchchính xác với cơ số 3. Ta đặt số nguyên m = (. . . a2a1a0) vào từngA0, A1 . . . nếu a0 = 1 thay m vào A0. Mặt khác vì a 6= 0, ai1 6= 0 vớimột số i1, bởi vì chỉ hữu hạn ai 6= 0, ai2 = 0, với một vài i2 > i1, tiếp

vnmath.com

Page 11: OLYMPIC TOÁN NĂM 2000 52 ĐỀ THI VÀ LỜI …»i nói đầu Để thử gói lệnh lamdethi.sty tôi biên soạn một số đề toán thi Olympic, mà các học trò của

Đề thi olympic Belarus 11

đến al 6= 0, al+1 = 0, với một vài l. Chọn l nhỏ nhất với thuộc tính nàyvà thay m tai Al+1 = 0.Nếu m1, m2 ∈ A1 và cơ số 3 biểu diễn m1 + m2 có những đơn vị số 2như vậy m1 + m2 /∈ A1. Nếu m1 + m2 ∈ Al với một số l>1, như vậy:0 11 · · ·1︸ ︷︷ ︸

l

< m1, m2 < 1 00 · · ·0︸ ︷︷ ︸l

suy ra 0 22 · · ·2︸ ︷︷ ︸l

< m1 + m2 < 2 00 · · ·0︸ ︷︷ ︸l

nếu m1 +m2 = (. . . a3a2a1a0), thì al = 1 suy ra m1 +m2 /∈ Al

Bây giờ, đặt k > 1 là một số nguyên dương và đặt S ={1, 2, . . . , 1

2(3k − 1)

}biểu thức cơ số 3 của 1

2(3k − 1) bao gồm tất cả

1’s để mọi 12(3k − 1) ∈ A1 cơ số 3 trong tất cả các số khác trong S có

1 số 0 trong 3k−1 vị trí để mà mỗi số nguyên trong S nằm chính xác 1trong các tập hợp A0, A1, . . . , Ak−1. Như vậy, S có thể là một phần trongk tập hợp. A0 ∩ S,A1 ∩ S, . . . , Ak−1 ∩ S để mà a 6= b 6= c với bất cứ a, bvà c trong cùng một tập. Suy ra kết quả k = 4 cho thấy n = 4 là có thểđạt được như yêu cầu.

Ghi chú Với n, k ∈ N và sự phân chia của {1, 2, . . . , k} trong n cặp 3(a,b,c) sao cho a + b = c và a,b,c trong cùng tập gọi là Schur triple vớimỗi n ∈ N tồn tại k nguyên tối đa để mà không có Schur triplec chosự phân chia nào đó {1, 2, . . . , k} vào n tập hợp. Số nguyên này đượcbiểu thị bởi S(n) và được gọi là nth Schur number. Mặc dù nhỏ hơn vàlớn hơn giới hạn tồn tại với tất cả S(n) không có dạng tổng quát nàobiết đến, giới hạn nhỏ hơn được tìm thấy trong giải pháp cho n =1,2,3nhưng S(n)=44.

.1.8. Một số nguyên dương gọi là monotonic(đều) nếu những chữ số của nó

trong cơ số 10, đọc từ trái sang phải theo thứ tự không giảm dần. Chứng

minh rằng với mỗi n ∈ N tồn tại n chữ số monotonic là số bình phương

Lời giải: Bất kỳ số có 1 chữ số là bình phương( ví dụ 1,4 hoặc 9) làmonotonic chứng minh yêu cầu bài với n = 1 Chúng ta giả sử n > 1

Nếu n là số lẻ, viết n = 2k - 1 cho một số nguyên k ≥ 2

Đặt xk = (10k + 2)/6 = 166 · · ·67︸ ︷︷ ︸k−2

vnmath.com

Page 12: OLYMPIC TOÁN NĂM 2000 52 ĐỀ THI VÀ LỜI …»i nói đầu Để thử gói lệnh lamdethi.sty tôi biên soạn một số đề toán thi Olympic, mà các học trò của

12 Nguyễn Hữu Điển, ĐHKHTN Hà Nội

Như vậy

x2k = (10k2k + 4.10k + 4)/36 =

102k

36+

10k

9+

1

9(*)

Quan sát thấy: 102k

36= 102k−2.(72

36+ 28

36) = 2.102k−2 + 7

9.102k−2 =

2 77 · · ·7︸ ︷︷ ︸2k−2

+79

Vậy vế phải của đẳng thức (*) bằng: 2 77 · · ·7︸ ︷︷ ︸2k−2

+79

+ 11 · · ·1︸ ︷︷ ︸k

+19

=

2 77 · · ·7︸ ︷︷ ︸k−2

9 88 · · ·8︸ ︷︷ ︸k−1

là một số có n chữ số monotonic là số bình phương.

Nếu n là số chẵn, viết n = 2k với số nguyên k ≥ 1 và

yk = (102k + 2)/3 = 4 33 · · ·3︸ ︷︷ ︸k−1

Như vậy

y2k = (102k + 4.10k + 4)/9 =

102k

9+ 4

10k

9+

4

9

= 1 11 · · ·1︸ ︷︷ ︸2k

+1

9+ 4 44 · · ·4︸ ︷︷ ︸

k

+4

9= 11 · · ·1︸ ︷︷ ︸

k

6 55 · · ·5︸ ︷︷ ︸k−1

một số gồm n chữ số monotonic chính phương (Đpcm)

.1.9. Cho cặp (−→r ,−→s ) vectơ trong một máy bay một dịch chuyển gồm chọn

một số nguyên khác không k và sau đó thay đổi (−→r ,−→s ) thành hoặc (i)

(−→r + 2k−→s ,−→s ) hoặc (ii) (−→r ,−→s + 2k−→r ) Trò chơi gồm lấy một hữu hạn

các chuỗi dịch chuyển, luân phiên nhau dịch về dạng (i) và dạng (ii)

cho một vài cặp vectơ ban đầu.

a) Có thể đạt được cặp ((1, 0), (2, 1)) trong trò chơi với cặp ban đầu

((1, 0), (0, 1)) nếu sự dịch chuyển đầu tiên là dạng (i)? b) Tìm tất cả

các cặp ((a, b), (c, d)) có thể đạt được trong trò chơi với cặp ban đầu

((1, 0), (0, 1)) trong đó dịch chuyển đầu tiên là một trong hai dạng trên

?

Lời giải: Đặt ||−→z || biểu thị cho chiều dài của vectơ −→z và đăt |z| biểuthị cho giá trị tuyệt đối cảu số thực z

vnmath.com

Page 13: OLYMPIC TOÁN NĂM 2000 52 ĐỀ THI VÀ LỜI …»i nói đầu Để thử gói lệnh lamdethi.sty tôi biên soạn một số đề toán thi Olympic, mà các học trò của

Đề thi olympic Belarus 13

a) Đặt (−→r ,−→s ) là cặp vectơ mà −→r và −→s có thể thay đổi qua trò choiquan sát thấy rằng nếu −→x ,−→y là vectơ như là ||−→x || > ||−→y ||Như vậy:

||−→x + 2k−→y || > ||2k−→y || − ||−→x || > 2||−→y || − ||−→y || = ||−→y ||

Sau lần dịch chuyển đầu tiên dạng (i) ta có −→r = (1, 2) và −→s = (0, 1)

cho số k 6= 0 để mà ||−→r || > ||−→s || áp dụng kết quả trên với −→x > −→s và−→y > −→r chúng ta có thể thấy trong dịch chuyển tiếp theo (dạng(ii)) độdài của −→r không thay đổi trong đó −→s tăng cao hơn ||−→r ||, áp dụng kếtquả trên lần nữa với −→x > −→r và −→y > −→s chúng ta có thể thấy trong dịchchuyển tiếp theo (dạng(i)) độ dài của −→s không thay đổi trong đó −→rtăng vượt qua ||−→s ||, tiếp tục như vậy ta thấy rằng ||−→r || và ||−→s || khôngbao giờ giảm. Bởi vì sau lần dịch chuyển đầu tiên vectơ đầu tiên có độdài hơn 1, sẽ không bao giờ đạt được ((1, 0), (2, 1))

b) Chúng ta thay đổi trò chơi bằng cách không yêu cầu dịch chuyểnluân phiên giữa dạng (i) và (ii) và bằng cách cho phép sự lựa chọn k =0. Đương nhiên bất cứ cặp nào có thể đạt được theo quy định ban đầuphải đạt được theo những quy định mới này.Điều ngược lại đúng vì bằng cách loại bất cứ dịch chuyển nào theo nhữngquy định mới với k = 0 và kết hợp bất cứ dịch chuyển mới nào của cùngdạng vào 1 dịch chuyển ta đạt được chuỗi dịch chuyển theo quy luật banđầu và được cùng 1 cặp. Để ((ω, x), (y, z)) đại diện cặp của những vectơvới ω, x, y và z thay đổi qua trò chơi.Dễ dàng kiểm tra giá trị của ωz − xy và tính chẵn lẻ của x và y làkhông thay đổi theo bất cứ dịc chuyển nào trong trò chơi. Trong mộttrò chơi mà bắt đầu với ((ω, x), (y, z)) = ((1, 0), (0, 1)), ta phải luôn luôncó ωz − xy = 1 và b ≡ c ≡ 0(mod2). Bởi vì x và y luôn luôn chẵn ω vàz không đổi mod4, ta phải có ω ≡ z ≡ 1(mod4) thông qua trò chơi.Gọi 1 cặp ((a, b), (c, d)) thỏa mẵn khi ad - bc = 1, a ≡ d ≡ 1(mod4) vàb ≡ c ≡ 0(mod2) ở trên ta thấy rằng bất cứ cặp đôi đạt được trong tròchơi với cặp ban đầu ((1, 0), (0, 1)) phải thỏa mãn và bây giờ ta chứngminh điều ngược lại.Giả sử, nhằm mục đích thấy được sự mâu thuẫn rằng có những cặp đôi((a, b), (c, d)) thỏa mãn điều kiện đưa ra. Đặt ((e, f), (g, h)) là cặp mà

vnmath.com

Page 14: OLYMPIC TOÁN NĂM 2000 52 ĐỀ THI VÀ LỜI …»i nói đầu Để thử gói lệnh lamdethi.sty tôi biên soạn một số đề toán thi Olympic, mà các học trò của

14 Nguyễn Hữu Điển, ĐHKHTN Hà Nội

tối thiểu hóa |ac|Nếu g = 0 thì eh = 1 + fg = 1 bởi vì e ≡ h ≡ 1(mod4) e = h = 1,nếu f = 0 cặp này chắc chắn có được. Mặt khác, bằng cách dịch chuyểndang (i) với k = f

2chúng có thể thay đổi dạng ((1, 0), (0, 1)) thành dạng

((e, f), (g, h)) dẫn tới mâu thuẫnNhư vậyg 6= 0 bây giờ g là số chẵn e là số lẻ, |e| > |g| hoặc |e| < |g|ta có e − 2k0g nằm trong đoạn (-|e|, |e|) cho k0 ∈ {1,−1}. Thực hiệndạng (i) dịch chuyển đến ((e, f), (g, h)) với k = −k0 thì đạt được mộtcặp mong muốn khác ((e′, f ′), (g, h)). Bởi vì |e’| < |e| và g 6= 0, chúng tacó |e’g| < |eg|. Như vậy, bằng khái niệm tối thiểu ((e, f), (g, h)) cặp đôimới có thể đạt được từ ((1, 0), (1, 0)) với một dãy dịch chuyển S nào đó.Như vậy, chúng ta đạt được ((e, f), (g, h)) từ ((1, 0), (0, 1)) bằng cách ápdụng trước tiên dịch chuyển trong S tới ((1, 0), (0, 1)) sau đó áp dụngthêm dịch chuyển dạng (i) với k = −k0. Như vậy cặp đôi cực tiểu đạtđược dẫn đến mâu thuẫn.Một chứng minh tương tự nếu |e| < |g| , khi chúng ta thay lựa chọn r0

với g − 2k0e ∈ (−|g|, |g|) và thực hiện dạng dịch chuyển (ii). Như vậytrong tất cả các trường hợp chúng ta có sự mâu thuẫn. Hay chúng ta cóthể kết luận rằng bất cứ cặp đôi đạt được đều thực sự thỏa mãn. Điềunày hoàn toàn được chứng minh.

.1.10.Chứng minh:a3

x+b3

y+c3

z≥ (a+ b+ c)3

3(x+ y + z)

Với tất cả các dạng số thực a, b, c

Lời giải: Qua chứng minh không cân bằng của Holder

(a3

x+ b3

y+ c3

z)

1

3 (1 + 1 + 1)1

3 (x+ y + z)1

3 ≥ (a + b+ c)

lũy thừa 3 cả 2 vế và chia cả 2 cho 3(x + y + z) ta được Đpcm

.1.11.Gọi P là giao điểm của hai đường chéo AC và BD của tứ giác lồi ABCD

trong đó AB = AC = BD. Gọi O và I là circumcenter và tâm nội tiếp

của 3 phân giác của tam giác ABP. Chứng minh rằng nếu O 6= I thì

đường thẳng OI và CD vuông góc.

vnmath.com

Page 15: OLYMPIC TOÁN NĂM 2000 52 ĐỀ THI VÀ LỜI …»i nói đầu Để thử gói lệnh lamdethi.sty tôi biên soạn một số đề toán thi Olympic, mà các học trò của

Đề thi olympic Belarus 15

Lời giải: Đầu tiên ta chứng minh một luận đề rất hữu ích XY và UV ,đặt X’ và Y’ là chân góc vuông của X và Y, nối đường thẳng UV. Sửdụng khoảng cách trực tiếp, XY⊥UV nếu chỉ nếu

UX’ - X’V = UY’ - Y’V

vì UX’ + X’V =UV = UY’ + Y’V, phép tính trên đạt được nếu chỉ nếuUX ′2 −X ′V 2 = UY ′2 − Y ′V 2, hoặc UX2 −XV 2 = UY 2 − Y V 2.Như vậy nó thỏa mãn đẳng thức DO2 − CO2 = DI2 − CI2. Đặt AB=AC=BD = p, PC = a và PD = b như vậy AP = p - a và BP = p -b. Đặt R là bán kính đường tròn ngoại tiếp tam giác ABP. ta có pb =DP.DB = DO2 −R2 ngoài ra pa = CO2 − R2.Như vậy DO2 − CO2 = p(b − a), vì tam giác ADB là cân với BA =BD và I nằm trên đường phân giác của góc ABD, ID = IA ngoài raIB = IC đặt T là điểm tiếp xúc của vòng tròn nội tiếp tam giác ABCvới cạnh AB. Như vậy BT = (p + a - b)/2 vì IT vuông góc với AB,AI2 − BI2 = AT 2 − BT 2. Đặt các tham số lại với nhau chúng ta thấyrằng.

DI2 − CI2 = AI2 −BI2 = AT 2 − BT 2

= (AT +BT )(AT − BT )

= p(b− a)

= PO2 − CO2 (Đpcm)

vnmath.com

Page 16: OLYMPIC TOÁN NĂM 2000 52 ĐỀ THI VÀ LỜI …»i nói đầu Để thử gói lệnh lamdethi.sty tôi biên soạn một số đề toán thi Olympic, mà các học trò của

Chương 2

Đề thi olympic Bungari

.2.12.Một đường thẳng l đi qua trực tâm của tam giác nhọn ABC. CMR các

đường thẳng đối xứng với l qua các cạnh của tam giác đồng quy.

Lời giải: Gọi H là trực tâm của tam giác ABC. Vì tam giác ABCnhọn nên trực tâm H nằm trong tam giác ABC. Không mất tính tổngquát chúng ta giả sử l cắt AC và BC tại P và Q. Nếu l ‖ AB, lấy R làđiểm tùy ý trên đường thẳng đối xứng với l qua đường thẳng AB. Nếul không song song với AB thì lấy R là giao điểm của đường thẳng l đốixứng với đường thẳng AB và ta có thể giả sử R nằm trên tia BA.LấyA1, B1, C1, tương ứng là các điểm đối xứng với H qua các đườngthẳng BC, CA, AB. Khi đó, A1,B1,C1 nằm trên đường tròn ngoại tiếpw của tam giác ABC (Chú ý:A1CB = BCH = HAB = A1AB =)Ta cần chứng minh: A1P ,B1Q,C1R đồng quyVì hai đường thẳng AC và BC không song song, nên hai đường thẳngB1Q và A1P không song song. Lấy S là giao điểm của A1P và B1Q VìSA1C+SB1C=PA1C+QB1C=PHC+QHC=π nên tứ giác SA1CB1 làđiểm hội tụ đường trònDo đó, S là giao điểm của đường thẳng B1Q và đường tròn w. Tươngtự, hai đường thẳng B1Q và C1R không song song và giao điểm củachúng cũng chính là giao điểm của B1Q và đường tròn w. Do vậy, cácđường thẳng A1P , B1Q, C1R đồng quy tại mọi điểm nằm trên đườngtròn ngoại tiếp tam giác ABC

vnmath.com

Page 17: OLYMPIC TOÁN NĂM 2000 52 ĐỀ THI VÀ LỜI …»i nói đầu Để thử gói lệnh lamdethi.sty tôi biên soạn một số đề toán thi Olympic, mà các học trò của

Đề thi olympic Bungari 17

.2.13.Có 2000 quả cầu trắng trong một chiếc hộp. Bên ngoài chiếc hộp cũng

có các quả cầu trắng, xanh và đỏ với số lượng không hạn chế. Trong mỗi

lần thay đổi , chúng ta có thể thay đổi 2 quả cầu trong hộp bởi 1 hoặc 2

quả cầu theo cách sau:

2 quả trắng bởi 1 quả xanh, 2 quả đỏ bởi 1 quả xanh, 2 quả xanh bởi 1

quả trắng và 1 quả đỏ, 1 quả trắng và 1 quả xanh, bởi 1 quả đỏ hoặc 1

quả xanh và 1 quả đỏ bởi 1 quả trắng. (a) Sau một số hữu hạn lần thực

hiện như trên còn lại 3 quả cầu trong hộp. CMR có ít nhất 1 quả xanh

trong 3 quả cầu còn lại. (b) Liệu có thể xảy ra sau một số hữu hạn lần

thực hiện như trên trong hộp còn lại đúng một quả cầu.

Lời giải: Ta gắn góc giá trị i cho mỗi quả cầu trắng, −i cho mỗi quảcầu đỏ, và -1 cho mỗi quả cầu xanh. Ta có thể kiểm tra lại rằng cácphép thay thế đã cho không làm thay thế các giá trị của các quả cầutrong hộp. Tích các giá trị của các quả cầu ban đầu là i2000 = 1.Nếu trong hộp còn lại ba quả cầu không có quả nào màu xanh thì tíchcác giá trị của chúng sẽ là : ±i, mâu thuẫn. Do đó, nếu trong hộp cònlại ba quả, thì phải có ít nhất 1 quả màu xanh, (a) được chứng minh.Hơn nữa, vì không có quả nào có giá trị 1 nên trong hộp phải chứa ítnhất hai quả cầu. Do đó, không thể xảy ra trường hợp trong hộp cònlại 1 quả(Để chứng minh (a), chúng ta có thể gán giá trị 1 cho mỗi quảxanh, -1 cho mỗi quả cầu đỏ hoặc trắng.

.2.14.Đường tròn nội tiếp tam giác cân ABC tiếp xuc với các cạnh AC và

BC tương ứng tại M và N . Đường thẳng t tiếp xúc với cung nhỏ MN ,

t giao với NC và MC tương ứng tại P và Q. Gọi T là giao điểm của

hai đường thẳng AP và BQ.

(a) Chứng minh T thuộc MN .

(b) CM: Tổng diện tích các tam giác ATQ và BTP đạt giá trị nhỏ nhất

khi t ‖ AB

Lời giải: (a) Hình lục giác suy biến AMQPNP được ngoại tiếp bởi cácđường tròn nội tiếp tam giác ABC. Theo định lý Brianchon, các đườngchéo AD, MN , QB là đồng quy. Do đó, T thuộc MN .Chúng ta có thể sử dụng cách giải sơ cấp hơn. Gọi R và S tương ứng là

vnmath.com

Page 18: OLYMPIC TOÁN NĂM 2000 52 ĐỀ THI VÀ LỜI …»i nói đầu Để thử gói lệnh lamdethi.sty tôi biên soạn một số đề toán thi Olympic, mà các học trò của

18 Nguyễn Hữu Điển, ĐHKHTN Hà Nội

các tiếp điểm của đường tròn nội tiếp với các cạnh AB và PQ : Gọi T1,T2 tương ứng của tam giac ABC là các giao điểm của BQ với MN vàSR.Vì QMN= PNM = MN

2nên ta có: sin QMN = sinPNM = sinBNM .

áp dụng định lý hàm số sin trong tam giác cho các tam giác MQT1 vàNBT1.QT1

QM= sin QMN

sin QT1M= sin BNM

sin BT1N= BT1

BN.

hay QT1

BT1= MQ

BN. Tương tự: QT2

BT2= SQ

BR.

Theo tính chất của tiếp tuyến, BN= BR và QM= QS. Do đó: QT1

BT1=

QT2

BT2. Vì T1 và T2 đều thuộc BQ nên ta phải có T1 ≡ T2. Do đó, BQ,MN ,

SR đồng quyMột cách tương tự, ta chứng minh được AP , MN , SR đồng quy. Từ đóT ∈ MN . Gọi α = CAB = CBA và B= ACB.Gọi f = [AQT ] + [BPT ] = [ABQ] + [ABP ] − 2.[ABT ]

Vì tam giác ABC cân, MN ‖ AB, suy ra [ABT ] là hằng số. Do đó, fđạt giá trị nhỏ nhất ↔ [ABC] + [ABP ] đạt giá trị nhỏ nhất.Để ý rằng : 2f ′ = AB.(AQ+ PB). sinα

= AB.(AB + PQ). sinα

Trong đó: AQ + PB = AP +QP vì tứ giác ABCD có đường tròn nộitiếp.f’ đạt giá trị nhỏ nhất khi PQ đạt giá trị nhỏ nhất. Gọi I là tâm đườngtròn nội tiếp cuả tam giác ABC, do đó I là tâm đường tròn bàng tiếp củatam giác CPQ. Do đó, PC+CQ+QP = 2.CM không đổi. Đặt CPQ=pvà CQP=q. Thì p + q=π − β không đổi. áp dụng định lý hàm số sincho tam giác CPQ ta được : CM

PQ= 1 + CP

PQ+ CQ

PQ= 1 + sin p+sin q

sinβ=

1 +2. sin p+q

2. cos p−q

2

sinβ.

PQ đạt giá trị nhỏ nhất khi cos p−q2

lớn nhất. Từ đó, [ATQ] + [BTP ]

nhỏ nhất khi p = q, tức là khi PQ ‖ AB.

.2.15.Cho n điểm trên mặt phẳng (n >= 4) sao cho khoảng cách giữa 2 điểm

bất kỳ trong n điểm đó là một số nguyên. CMR ít nhất 16

trong số các

khoảng cách đó chia hết cho 3.

Lời giải: Trong bài giải này, các đồng dư xét theo modul 3. Trước hết

vnmath.com

Page 19: OLYMPIC TOÁN NĂM 2000 52 ĐỀ THI VÀ LỜI …»i nói đầu Để thử gói lệnh lamdethi.sty tôi biên soạn một số đề toán thi Olympic, mà các học trò của

Đề thi olympic Bungari 19

ta chứng minh nếu n = 4, thì ít nhất có hai điểm rời nhau mà khoảngcách giữa chúng chia hết cho 3. Ký hiệu 4 điểm đó làA,B,C,D. Giả sửcác khoảng cách AB,BC,CD,DA,AC,BD không chia hết cho 3.Không mất tính tổng quát, ta giả sử BAD = BAC = CAD.Gọi x=BAC và y = CAD.Gọi α = 2.AB.AC. cosx, β = 2.AD.AC. cos y và γ =

2.AB.AD. cos (x+ y). áp dụng định lý hàm số cosin cho các tamgiac ABC, ACD, ABD ta đượcBC2 = AB2 + AC2 − α

CD2 = AC2 + AD2 − β

BD2 = AB2 + AD2 − γ

Vì bình phương mẫu khoảng cách là một số nguyên nên α, βvγ cùng làcác số nguyên. Do đó:

2.AC2.γ = 4.AC.AB.AD. cos (x+ y)

= 4.AC2.AB.AD − (cosx. cos y − sin x. sin y)

= α.β − 4.AB.AD. sin x. sin y.là số nguyên. Vì vậy: 4.AC2.AB.AD. sin x. sin y là một số nguyên chẵnvàsin x. sin y =

√(1 − cos2 .x).(1 − cos2 .y) là một số hữu tỷ, khi viết dưới

dạng tối giản tử số là số không chia hết cho 3. Đặt p = 2.AB.AC vàq = 2.AD.AC, do đócosx = α

pvà cos y = β

q

Vì sin x.siny =

√(p2−α2).(q2−p2)

pq

Là số hữu tỷ nên tử số ở vế phải cùng là một số nguyên. Tử số chia hếtcho 3 vì p2 ≡ 1( modul 3) và α2 ≡ 1 (modul 3Nhưng mẫu số không chia hết cho 3. Do đó, khi sin x. sin y viết dướidạng tối giản thì tử của nó chia hết cho 3, điều này mẫu thuẫn. Do đó,điều giả sử ban đầu là sai. Vậy có ít nhất một khoảng cách chia hết cho3

vnmath.com

Page 20: OLYMPIC TOÁN NĂM 2000 52 ĐỀ THI VÀ LỜI …»i nói đầu Để thử gói lệnh lamdethi.sty tôi biên soạn một số đề toán thi Olympic, mà các học trò của

20 Nguyễn Hữu Điển, ĐHKHTN Hà Nội

với n = 4

Xét trường hợp n ≥ 4. Từ một tập n điểm, có C4n các tập con chứa 4

điểm có ít nhất hai điểm trong mỗi tập rời nhau đó có khoảng cách chiahết cho 3, và mỗi khoảng cách đó được đếm trong ít nhất C2

n−1 tập con.vậy có ít nhất C4

n

C2n−2

= C2n

6các khoảng cách chia hết cho 3

.2.16.Trong tam giác ABC, CH là đường cao và CM và CN tương ứng là các

đường phân giác của các góc ACH và BCH. Tâm đường tròn ngoại tiếp

của tam giác CMN trùng với đường tròn nội tiếp của tam giác ABC.

CMR: [ABC] = AN.BM2

.

Lời giải: Gọi I là tâm đường tròn nội tiếp của tam giác ABC, gọi tiếpđiểm của đường tròn nội tiếp tam giác ABC với các cạnh là AC,AB lầnlượt là E và F . Vì IM = IN và IF ⊥ IM , nên ta có F IN = 1

2.MIN .

Hươn nữa , vì I là tâm đường tròn ngoại tiếp của tam giác CMN nên:12.MIN = MCN = 1

2.ABC = ECI.

Do đó F IN = ECI.ta cũng có NFI = π

2= IEC. Nên ∆NFI ∼ ∆IEC.

VìNI = NC, nên hai tam giác này là ...(congruent), vàNF = IE = IF .Tam giác ∆NFI là tam giác vuông cân, F IN = π

4và ACB = 2.F IN =

π2.

Do đó, HCB = π2− CBH = BAC và

ACN = ACB − 12.HCB = π

2− BAC

2

Từ đó suy ra, CNA = π − (ACN + NAC)

= π2− BAC

2= ACN và AN = AC.

Tương tự, BM = BC. Do vậy: 12AN.BM = 1

2.AC.BC = [ABC].

.2.17.Cho dãy số (an):

a1 = 43, a2 = 142 và an+1 = 3.an + an−1 với mọi n > 2.

CMR

(a) an và an−1 là nguyên tố cùng nhau với mọi n >= 1.

(b) Với mọi số tự nhiên m, tồn tại vô hạn số tự nhiên n sao cho an − 1

và an+1 − 1 đều chia hết cho m.

Lời giải: (a) Giả sử có n, g > 1 sao cho g

anvà g

an+1.

Khi đó g chia hết an−1 = an+1 − 3.an

vnmath.com

Page 21: OLYMPIC TOÁN NĂM 2000 52 ĐỀ THI VÀ LỜI …»i nói đầu Để thử gói lệnh lamdethi.sty tôi biên soạn một số đề toán thi Olympic, mà các học trò của

Đề thi olympic Bungari 21

Nếu n − 1 > 1 thì g chia hết an+1, an, ...a2, a1, nhưng điều này khôngthể xảy ra vì WCLN (a2, a1) = 1. Do đó, an và an+1 là nguyên tố cùngnhau với mọi n > 1.(b) Xét dãy (a′n) được xác định như sau: a′1, a′2 a′n+1 = 3.a′n + an−1 vớimọi n > 2.Dễ thấy: a′3 = 4, a′4 = 13, a′5 = 43, a′6 = 142

Tức là a1 = a′5, a2 = a′6. Mà hai dãy (an) và dãy (a′n) có cùng công thứctruy hồi nên ta có:an = a′n+4, với mọi n > 1.Gọi bn là số dư khi chia a′n cho m, và xét cặp số (bn, bn+1) với n > 1. Vìcó một số vô hạn các cặp số như vậy nhưng chỉ là m2 cặp các số nguyên(r, s) với 0 6 r, s < m do đó phải có hai trong số các cặp đó trùng nhau,chẳng hạn (bi, bi+1) = (bi+t, bj+t) với t > 0.Sử dụng công thức truy hồi, ta dễ dàng chứng minh được bằng quinạp theo n: bi+n = bi+n+t với mọi n thỏa mãn (i + n) > 1. Do đó,(b1+kt, b2+kt) = (b1, b2) = (1, 1) với mọi k > 1. Do đó, akt−3 − 1 vàakt−2 − 1 đều chia hết cho m với mọi k > 4.

.2.18.Cho tứ giác lồi ABCD có BCD = CDA, đường phân giác của góc ABC

cắt CD tại điểm E.

CMR: AEB = π2

khi và chỉ khi AB = AD +BC.

Lời giải: Nếu AEB = π2

thì CEB < π2. Từ đó suy ra có điểm F nằm

trên cạnh AB sao cho BEF = BEC . Khi đó, có hai tam giác BECvà BEF bằng nhau, suy ra BC = BF và BFE = BCE = EDA. Dođó, tứ giác ADEF là tứ giác nội tiếp đường tròn. Vì AEB = π

2và

CEB = BEF nên ta có FEA = AED.Từ đó suy ra FDA = FEA = AED = AFD. Do đó:AF = AD và AB = AF +BF = AD +BC

Nếu AB = BC + AD thì có điểm F thuộc AB sao cho AF = AD vàBF = BC. Khi đó hai tam giác BCE và BFE là bằng nhau và tứ giácADEF là tứ giác nội tiếp được đường tròn.Cũng có FDA = AFD. Do đó, FEA = FDA = AFD = AED, do đóđường thẳng AE là phân giác của góc FED.Vì ∆BCE = ∆BFE nên EB là phân giác của góc CEF do vậy AE ⊥

vnmath.com

Page 22: OLYMPIC TOÁN NĂM 2000 52 ĐỀ THI VÀ LỜI …»i nói đầu Để thử gói lệnh lamdethi.sty tôi biên soạn một số đề toán thi Olympic, mà các học trò của

22 Nguyễn Hữu Điển, ĐHKHTN Hà Nội

BE và AEB = π/2.

.2.19.Trong hệ tọa độ Oxy, một tập gồm 2000 điểm

(x1, y1), (x2, y2), ...(x2000, y2000) được gọi là tốt nếu 0 6 xi 6 83,

0 6 y 6 1 với i = 1, 2, .., 2000 và xi 6= xj khi i 6= j .Tìm số nguyên

dương n lớn nhất sao cho với mọi tập tốt phần trong và biên của hình

vuông đơn vị nào đó chưa đúng n điểm trong tập là phần trong và phần

biên của tập tốt đó;

Lời giải: Trước hết ta chứng minh rằng với mọi tập tốt, một hình vuôngđơn vị nào đó chứa đựng 25 điểm của tập tốt đó.Ta gọi một hình vuông đơn vị là proper (riêng) nếu 2 cạnh của nó nằmtrên các đường thẳng y = 0 và y = 1.Mỗi điểm cho trước đều nằm trong miềnR = (x, y)|0 6 ex 6 83, 0 6 y 6 1

Miền R có thể được chia thành các hình vuông đơn vị proper mà cáccạnh bên trái nằm trên các đường thẳng có phương trình : x = i vớii = 0, 1, .., 8.Vì 83.24 < 2000, nên một trong các hình vuông đó phải chứa nhiều hơn25 điểm. Vì 83.26 − 82 > 2000 nên một trong các hình vuông đó chứaít hươn 26 điểm. Hơn nữa trong 83 hình vuông đơn vị đó, xét các hìnhvuông đơn vị proper mà các cạnh bên trái nằm trên các đường thẳngdạng x = xi hay x = xi − 1.Thứ tự các hình vuông đơn vị đó từ trái qua phải giả sử là: S1, S2, ...Sk,trong đó cạnh bên trái của Si nằm trên đường x = ti với i = 1, 2..., k−1,có nhiều nhất một trong các điểm cho trước nằm trong miền được xácđịnh bởi zi 6 x < zi+1, có nhiều nhất 1 trong các điểm cho trước nằmtrong miền được xác định bởi zi+1 < x 6 zi+1 = 1.Do đó, với mọi i số các điểm trong Si khác với các điểm trong Si+1 hoặclà −1, 0 hay −1. Vì có Si1 chứa ít nhất 25 điểm và có Si2 chứa nhiềunhất 25 điểm , từ đó suy ra có Si3 (i3 nằm giữai1 và i2) chứa đúng 25điểm. Bây giờ ta chứng minh rằngĐặt d = 2. 83

1999, xi = (i−1).1

2.d với i = 1, 2, ..., 2000 và y2k−1 = 0, y2k = 1

với k = 1, 2, ..., 2000.Với 2 điểm phân biệt bất kỳ (x1, y1) mà cùng nằm trên đường nằm ngang

vnmath.com

Page 23: OLYMPIC TOÁN NĂM 2000 52 ĐỀ THI VÀ LỜI …»i nói đầu Để thử gói lệnh lamdethi.sty tôi biên soạn một số đề toán thi Olympic, mà các học trò của

Đề thi olympic Bungari 23

(y = 0hocy = 1) thì khoảng cách giữa chúng thấp nhất là d > 225

Gọi XY ZW là 1 hình vuông đơn vị. Với j = 0, 1 miền Ro bị chặn bởihình vuông đó giao với mỗi đường thẳng y = j trong một khoảng đóngcó độ dài ri. Nếu ít nhất một trong các số ro, r1 là 0 thì khoảng tươngứng chứa nhiều nhất một điểm (xi, yi). Khoảng khác có chiều dài nhiềunhất

√2, và do đó có thể chưứa nhiều nhất [

√2d

] + 1 6 18 các điểm nhưvậy nói chung không vượt quá 19. Ta cũng có, nếu XY ZW có một cặpcạnh nằm trên các đường nằm ngang thì Ro chứa nhiều nhất [ 1

d2

]+1 6 25

các điểm như vậy/Mặt khác, Ro giao với đường y = 0 và y = 1 tại các điểm P,Q và R, S,trong đó P và R nằm bên trái Q và S. Ta cũng có PQ và RS chứa nhiềunhất [PQ

d] + 1 và [RS

d] + 1 các điểm đã chọn.

Dịch chuyển Ro theo hướng song song với các cạnh của hình vuông đơnvị mà tâm của nó nằm trên đường thẳng y = 1

2. Gọi R1 là ảnh của Ro

và gọi P ′, Q′, R′ và S ′ là giao của nó với các đường y = 0 và y = 1 đượcxác định tương tự như trên. Khi đó: P ′Q′ +R′S ′ = PQ+RS. Ta cũngcó P ′Q′ = R′S ′ do tính đối xứng. Gọi R2 là miền thu được bởi phépquay R1 quanh tâm của nó.Khi đó miền R1 ∪R2 −R1 ∩R2 là hợp của 8 miền tam giác bằng nhau.Gọi T và U là các đỉnh bên trái và bên phải của miền R2 tròn đườngy = 1 và gọi V là đỉnh của R1 trên đường y = 1.Gọi K và L là các đỉnh trên cùng của các cạnh thẳng đứng của R2 (vàcùng thuộc miền bị chặn R1). Ta có: ∆KTR′ ∼= ∆S ′V R′ ∼= S ′V L Tacũng có:TR′ +R′S ′ + S ′V = TU = 1

Mặt khác, theo bất đẳng thức tam giác TR′ +S ′V = R′V +S ′V > R′S ′

Từ đó suy ra R′S ′ < 12.

Vì P ′Q′ = R′S ′, số các điểm (xi, yi) nằm trong XY ZW nhiều nhất:[PQd

] + [RSd

] + 2 6P ′Q′+R′S′

d+2 < 1

d+2 < 15 Bài toán được chứng minh.

.2.20.Cho tam giác nhọn ABC

(a) CMR có duy nhất ba điểm A1, B1, C1 tương ứng nằm trên

BC,CA,AB thỏưa mãn:nếu ta chiếu hai trong ba điểm đó lên cạnh

tương ứng (còn lại), thì trung điểm của hình chiếu là điểm còn lại.

vnmath.com

Page 24: OLYMPIC TOÁN NĂM 2000 52 ĐỀ THI VÀ LỜI …»i nói đầu Để thử gói lệnh lamdethi.sty tôi biên soạn một số đề toán thi Olympic, mà các học trò của

24 Nguyễn Hữu Điển, ĐHKHTN Hà Nội

(b) CMR tam giác A1B1C1 đồng dạng với tam giác có các đỉnh là trung

điểm của ∆ABC

Lời giải: (a) Trước hết ta xem xét ngược lại, giả sử có tam giácA1B1C1 có tính chất như vậy.Gọi T là trung điểm của A1B1 theo định nghĩa C1T ⊥ AB

Gọi P là trọng tâm của ∆A1B1C1. Vì PA1 ⊥ BC, PB1 ⊥ CA vàPC1 ⊥ AB, P xác định duy nhất A1B1C1.Rõ ràng các tứ giác AB1PC, BC1PA1, CA1PB1 là các tứ giác nội tiếpđược đường tròn.Đặt α = CAB, β = ABC, x = A1B1P và y = B1A1P

B1CD = y

Vì các tứ giác AB1PC1 và CA1PB1 là nội tiếp được JPB1 = α, JPA1 =

β, A1CP = x,áp dụng định lý hàm số sin cho các tam giácA1TP và B1TP ta được:sin ysinβ

= TPTA1

= TPTB1

= sinxsinα

hay sin ysinβ

= sinxsinα

Một cách tương tự ta CM được :sin ACE

sin BCF= sinα

sinβ

Trong đó, F là trung điểm của cạnh ABVì tam giác ABC nhọn nên ta suy ra:A1CP = x = ACF và B1CD = y = BCF

Do đó các đường CP và CF đối xứng qua đường phân giác của gócACB. Ta có kết quả tương tự cho các đường AP và AD, BP và BE,trong đó D và E là các trung điểm của các cạnh BC và CATừ đó suy ra P là "isognal cọnugate" của G, G là trọng tâm của∆ABC. Do đó, P là duy nhất và bước ngược lại chỉ ra rằng P xác địnhduy nhất ∆A1B1C1 thỏa mãn điều kiện của bài toán.(b) Kéo dài AG về phía G đến K sao cho GD = DK. Khi đó, BGCKlà hình bình hành vàCK = BG = 2

3.BE, CG = 2

3.CF

GK = AG = 23.AD

vnmath.com

Page 25: OLYMPIC TOÁN NĂM 2000 52 ĐỀ THI VÀ LỜI …»i nói đầu Để thử gói lệnh lamdethi.sty tôi biên soạn một số đề toán thi Olympic, mà các học trò của

Đề thi olympic Bungari 25

Do đó, tam giác CGK đông dạng với tam giác tạo bởi các đường trungbình của ∆ABC. Ta cần chứng minh A1B1C1 và CGK là đồng dạngThật vậy:B1C1A1 = B1C1P + A1C1P = B1AP + A1BP

= BAG+ GBA = KGB + GKC

Chứng minh tương tự ta được:C1A1B1 = KCG

.2.21.Cho p > 3 là một số nguyên tố và a1, a2, ..., ap−2 là một dãy các số

nguyên dương sao cho p không chia hết cho ak hoặc akk − 1 với mọi

k = 1, 2, ..., p− 2.

CMR tích của một số phần tử của dãy đồng dư với modulo p.

Lời giải: Ta chứng minh bằng qui nạp theo k = 2, ..., p − 2 có các sốnguyên bk,1...bk,i sao cho:(i) mỗi bk,i hoặc bằng 1 hoặc là tích của một số phần tử của dãya1, a2, ..., ap−2 và(ii) bk,m 6= bk,n(modp) với m 6= n

Với k = 2, ta có thể chọn b1,1 = 1 và b1,2 = a1 ≡ 1 (modp)Giả sử chúng ta đã chọn được bk,1, ..bk,k.Vì ak 6≡ 1 (mod p), ta có:(akbk,1)(akbk,2)...(akbk,i) 6≡ bk,1bk,2, ...bk,i (modp)Do đó, chúng ta không thể hoán vị (akbk,1...akbk,k) sao cho mỗi phần tửlà đồng dư theo modulop với phần tử tương ứng trong (bk,1..., bk,k).Vì các số akbk,i là khác nhau theo modulop nên phải ko sao cho cácsố bk,1, ..., bk,k, ak.bk,1 không có hai số nào *đồng dư thep modul p. Đặtbk+1,1, bk+1,2, ..., bk+1,k+1 là các số trên. Mỗi bộ k + 1 số này đều bằng 1hoặc là tích của một số phần tử của dãy a1, a2, ..., ap−2. Phép quy nạpđược chứng minh hoàn toàn.Xét các số bp−1,1, ..., bp−1,p−1. Chắc chắn một trong các số này đồng dưvới 2 theo modul p vì số đó khác 1 và đồng dư với tích của một số số ak

.2.22.Cho tam giác nhọn ABC cân tại A. Gọi D là trung điểm của AB. Chọn

E trên AB, và lấy O là tâm đường tròn ngoại tiếp của ∆ACE. Chứng

minh rằng đường thẳng qua D vuông góc với Do, đường thẳng qua E

vnmath.com

Page 26: OLYMPIC TOÁN NĂM 2000 52 ĐỀ THI VÀ LỜI …»i nói đầu Để thử gói lệnh lamdethi.sty tôi biên soạn một số đề toán thi Olympic, mà các học trò của

26 Nguyễn Hữu Điển, ĐHKHTN Hà Nội

vuông góc với BC và đường thẳng qua B song song với AC là đồng quy.

Lời giải: Gọi l là đường thẳng đi qua B và song song với đường thẳngAC, gọi F1 và F2 là các điểm trên đường thẳng l sao cho OD ⊥ DF1

và BC ⊥ EF2

Gọi H1vH2 lần lượt là hình chiếu vuông góc của F1 và F2 là đườngthẳng AB. Vì góc CAB nhọn nên điểm 0 nằm trong ∆ABC . Từ đó,suy ra F1 nằm giữa hai tia AB và AC.Vì góc ABC nhọn nên F2 cùng nằm giữa hai tia AB và AC.Ta cần chứng minh F1H1 = H2F2.Gọi G là tâm đường tròn ngoại tiếp của tam giác ABC và gọi O1, G1

lần lượt là hình chiếu vuông góc của 0 lên AB và G lên 001.Vì OD ⊥ DF1, ∆OO1D ∼ ∆DH1F nênDH1

F1H1= OO1

O1D(1)

Đặt BAC = CBA = x. Vì AG = GC và AO = OC, GO là phân giáccủa góc AGC nên CGO = x.Vì CG ‖ OO1, G1OG = CGO = x

Do đó, các tam giác vuông GOG1 và F1BH1 đồng dạng và:BH1

BF1= OG1

OG(2)

Từ (1) và (2) ta suy ra:

F1H1 =BH1.O1D

OG1

=DH1.O1D

OO1

.

=DH1.O1D −BH1.O1D

OO1 − OG1

=BD.O1D

G1O1=BD.O1D

GD

.Vì DGB = ACB = π − 2.x, ta thu được

F1H1 = −tan2x.O1D

vnmath.com

Page 27: OLYMPIC TOÁN NĂM 2000 52 ĐỀ THI VÀ LỜI …»i nói đầu Để thử gói lệnh lamdethi.sty tôi biên soạn một số đề toán thi Olympic, mà các học trò của

Đề thi olympic Bungari 27

Gọi I là giao điểm của BC và EF2

Vì BF2 ‖ AC, F2BI = ACB = π − 2x và H2BF2 = x

Để ý rằng BE = AB − AE

= 2.(AD − AO1) = 2O1D

Từ đó suy ra:

F2H2 = BF2. sinH2BF2 = BF2. sin x

=BI

cos F2BI. sin x =

BI. sin x

− cos 2x

= −BE. cos x sin x

cos 2x= −O1D.tan2x = F1H1

Ta có điều phải chứng minh.

.2.23.Cho n là một số nguyên dương. Một dãy số được gọi là dãy nhị phân

nếu các phần tử của nó là 0 hoặc 1. Gọi A là tập tất cả các dãy nhị

phân có n phần tử , và gọi 0 ∈ A là dãy mà các phần tử đều là 0. Dãy

c = (c12, ..., cn) được gọi là tổng a+ b của các dãy a = (a1, a2, ..., an) và

b = (b1, b2, ..., bn) nếu ci = 0 khi ai = bi và ci = 1 khi ai 6= bi

Gọi f : A → A là ánh xạ với f (0) = 0 sao cho nếu a và b có đúng n

phần tử khác nhau thì f(a) và f(b) cũng có đúng n phần tử khác nhau.

CMR: nếu a, b, c ∈ A sao cho a+ b+ c = 0 thì f(a) + f(b) + f(c) = 0

Lời giải: Xét dãy e1 = (1, 0, 0, ..., 0), e2 = (0, 1, 0, ..., 0), ..., en =

(0, 0, ..., 0, 1). Với mỗi i, 0 và ei khác nhau do phần tử 1, nên f(0) vàf(ei) cùng khác nhau. Như vậy tức là f(ei = ej) với j nào đó.Xét mỗi dãy tùy ý x = (x1, x2, ..., xn) với f(x) = (y1, y2, ..., yn). Nếu x

có tham số 1 thì f(x) cũng có tham số 1. Nếu f(e1) = ej và xi = 1 thìei và x có t− 1 phần tử khác nhau . Điều này chỉ xảy ra nếu yj = 1, vìnếu không ej và f(x) sẽ có t+ 1 phần tử khác nhau. Một cách tương tự, nếu xi = 0 thì yj = 0

vnmath.com

Page 28: OLYMPIC TOÁN NĂM 2000 52 ĐỀ THI VÀ LỜI …»i nói đầu Để thử gói lệnh lamdethi.sty tôi biên soạn một số đề toán thi Olympic, mà các học trò của

28 Nguyễn Hữu Điển, ĐHKHTN Hà Nội

Nếu a = (a1, a2, ..., an), b = (b1, b2, ..., bn), c = (c1, c2, ..., cn) và a+b+c =

0 thì ai + bi + ci là chẵn với i = 1, 2, ...n

Với mỗi ej ta có thể chọn ej sao cho f(ei) = ej

Các phần tử thứ j của f(a), f(b), f(c) tương ứng là ai, bi, ci nên tổngcủa chúng là một số chẵn . Do đó, f(a) + f(b) + f(c) có phần tử thứ j

là 0 với ∀j và f(a) + f(b) + f(c) = 0.

vnmath.com

Page 29: OLYMPIC TOÁN NĂM 2000 52 ĐỀ THI VÀ LỜI …»i nói đầu Để thử gói lệnh lamdethi.sty tôi biên soạn một số đề toán thi Olympic, mà các học trò của

Chương 3

Đề thi olympic Canada

.3.24.Cho a1, a2, ..., a2000 là một dãy số nguyên liên tiếp trong khoảng

[−1000, 1000]. Giả sử2000∑i=1

ai = 1

Chứng minh rằng điều kiện xác định là có dãy con của a1, a2, ..., a2000 có

tổng bằng 0

Lời giải: Ta thấy rằng có thể sắp xếp lại dãy

a1, a2, ..., a2000

thành dãyb1, b2, ..., b2000

sao chon∑i=1

bi ∈ [−999, 1000] với n = 1, 2, 3, ..., 2000

Chúng ta giới hạn bi. Không phải tất cả các ai = −1000 do vậy chúngta có thể đặt b1 bằng ai nào đó thuộc [−999, 1000].Ấn định chỉ số i này. Giả sử chúng ta đã xây dựng dãy b1, b2, ..., bk(1 ≤k < 2000) với k đã được ấn định.

Nếuk∑i=1

bi ∈ [−999, 0] hoặc [1, 1000] thì tổng của các

ai có thể không xác định hoặc xác định.Vì vậy ít nhất một ai là xác định ( hoặc không xác định). Đặt bk+1 ∈[1, 1000] hoặc [−1000, 0], có nghĩa là

k+1∑i=1

bi ∈ [−999, 1000]

vnmath.com

Page 30: OLYMPIC TOÁN NĂM 2000 52 ĐỀ THI VÀ LỜI …»i nói đầu Để thử gói lệnh lamdethi.sty tôi biên soạn một số đề toán thi Olympic, mà các học trò của

30 Nguyễn Hữu Điển, ĐHKHTN Hà Nội

Cứ lặp lại quá trình trên ta xây dựng được dãy b1, b2, ..., b2000. Bằng cáchxây dựng trên ta xây dựng được dãy tổngriêng σn =

n∑i=1

bi (1 ≤ n ≤ 2000) bằng 1 của 2000 số nguyên thuộc

[−999, 1000].Bởi vậy nếu σi 6= σj với i<j hoặc trái lại σi = 0 với một vài i.Trong trường hợp đầu tiên ta có dãy con bi+1, bi+2, ..., bj có tổng bằng 0.Trong trường hợp thứ 2 có dãy con b1, b2, ..., bi có tổng bằng 0.Vậy ta có đpcm.

.3.25.Cho tứ giác ABCD có CBD = 2ADB, ABD = 2CDB,AB = CD.

Chứng minh rằng AB=CD.

Lời giải: Đặt x = ADB, y =CDB, → CBD = 2x, ABD = 2y

Áp dụng định lý Sin trong tam giác ABD và tam giác CBD ta có:sin(π−(2y+x))

sin x= BD

BA= BD

BC= sin(π−(2x+y))

sin y

⇔ sin(2y + x) sin y = sin(2x + y) sinx

⇔ 12(cos(y + x) − cos(3y + x)) = 1

2(cos(x+ y) − cos(3x+ y))

⇔ cos(3y + x) = cos(3x+ y)

Do 0 < x+ y = 12ABC < π

2⇒ 0 < 3y + x+ (3x + y) < 2π

⇒ 3y + x = 3x + y

⇒ x = y ⇒ ABD = CBD ⇒ AD = CD

.3.26.Cho dãy số thực a1, a2, ..., a100 thỏa mãn a1 ≥ a2 ≥ ... ≥ a100 ≥ 0(1) và

a1 + a2 ≤ 100(2), a3 + a4 + ... + a100 ≤ 100(3)

Tìm max của a21 + a2

2 + ... + a2100 và chỉ ra các ai đạt được.

Lời giải: Với i ≥ 3 ta có 0 ≤ ai ≤ a2 và suy ra ai(ai − a2) ≤ 0

Dấu "=" xảy ra nếu ai ∈ {0, a2}Suy ra

100∑

i=3

a2i ≤ a2

100∑

i=3

ai

Theo (3) dấu "=" chỉ xảy ra nếu100∑i=3

ai = 100 hoặc a2 = 0

Từ (1) và (2) suy ra 0 ≤ a2 ≤ 100 − a1 ≤ 100 − a2 hoặc 0 ≤ a2 ≤ 50

vnmath.com

Page 31: OLYMPIC TOÁN NĂM 2000 52 ĐỀ THI VÀ LỜI …»i nói đầu Để thử gói lệnh lamdethi.sty tôi biên soạn một số đề toán thi Olympic, mà các học trò của

Đề thi olympic Canada 31

⇒ 2a2(a2 − 50) ≥ 0

Dấu "=" xảy ra nếu a2 = 0 hoặc a2 = 50

⇒100∑i=1

a2i = a2

1 + a22 +

100∑i=3

a2i ≤ (100 − a2)

2 + a22 + 100a2

= 10000 + 2a2(a2 − 50) ≤ 10000

Dấu "=" chỉ xảy ra nếu :(a) {a3, a4, ..., a100} ⊆ {0, a2}(b)

100∑i=3

ai = 100 hoặc a2 = 0

(c)a1 = 100 − a2

(d)a2 ∈ {0, 50}Từ điều kiện trên dãy a1, a2, ..., a100 có thể là:100, 0, 0, ..., 0hoặc 50, 50, 50, 50, 0, 0,...,0Vậy tổng lớn nhất là 10.000

vnmath.com

Page 32: OLYMPIC TOÁN NĂM 2000 52 ĐỀ THI VÀ LỜI …»i nói đầu Để thử gói lệnh lamdethi.sty tôi biên soạn một số đề toán thi Olympic, mà các học trò của

Chương 4

Đề thi olympic Trung Quốc

.4.27.Cho tam giác ABC thỏa mãn BC ≤ CA ≤ AB.Gọi R và r lần lượt là

bán kính các đường tròn ngoại tiếp và nội tiếp tam giác ABC.Tìm theo

góc C của tam giác để BC + CA− 2R− 2r là dương ?,âm hoặc bằng 0

?.

Lời giải: Đặt AB = c,BC = a,CA = b, góc A = 2x, góc B = 2y, góc C= 2z.Ta có :0 < x ≤ y ≤ z và x+ y+ z = π

2Đặt s =BC + CA – 2R – 2r = a

+ b – 2R – 2r.Áp dụng công thức sau :

2R =a

sinA=

b

sinB=

c

sinC=

a

sin2x=

b

sin 2y=

c

sin 2z

vàr = 4R sin

A

2sin

B

2sin

C

2= 4R sin x sin y sin z

Ta suy ra được s = 2R(sin2x + sin2y -1 – 4sinxxinyxinz).+) nếu ∆ABC là tam giác vuông tại C với C = π

2.Ta có : 2R = c và 2r

= a + b – c ⇒ s=0.Do đó,chúng ta nhóm thừa số chung cos2z trong biểu thức s :

s

2R= 2 sin (x+ y) cos (x− y) − 1 + 2(cos (x+ y) − cos (x− y)) sin z

= 2 cos z cos (x− y) − 1 + 2 (sin z − cos (x− y)) sin z

vnmath.com

Page 33: OLYMPIC TOÁN NĂM 2000 52 ĐỀ THI VÀ LỜI …»i nói đầu Để thử gói lệnh lamdethi.sty tôi biên soạn một số đề toán thi Olympic, mà các học trò của

Đề thi olympic Trung Quốc 33

= 2cos (x− y) (cos z − sin z) − cos2z

= 2cos (y − x) .cos2z − sin2z

cos z + sin z− cos2z

=

[2 cos (y − x)

cos z + sin z− 1

]cos2z

Từ đó chúng ta có thể đưa vào giá trị cosz + sinz bởi nó là dương khi0 < z < π

2.

Chú ý rằng :≤ y − x < min {y, x+ y} ≤ min{z, π2− z}.

Vì z ≤ π2

và π2− z ≤ π

2nên ta có:

cos (y − x) > max{

cos z, cos(π

2− z

)}= max {cos z, sin z}

Từ đó suy ra :2 cos (y − x)

cos z + sin z− 1 > 0

Vì vậy s = pcos2z đối với p > 0 hay s = BC + CA - 2R - 2r có thểdương,bằng 0 hoặc âm nếu góc C tương ứng là nhọn,vuông,tù.

.4.28.Dãy số vô hạn

a1, a2, ...

được xác định 1 cách đệ quy như sau :

a1 = 0, a2 = 1

và an = 12nan−1 + 1

2n (n− 1) an−2 + (−1)n

(1 − n

2

)

Với n ≥ 3,tìm một công thức định nghĩa cho hàm :

fn = an + 2 (n1 ) an−1 + 3 (n2 ) an−2 + ... + n(nn−1

)a1

Lời giải: Cách giải 1: Viết lại mối quan hệ đệ quy thành :

an = (−1)n +1

2nan−1 +

1

2n((−1)n−1 + (n− 1) an−2)

Nếu(−1)n−1 + (n− 1) an−2 = an−1

Ta có:an = (−1)n +

1

2nan−1 +

1

2nan−1 = (−1)n + nan−1

vnmath.com

Page 34: OLYMPIC TOÁN NĂM 2000 52 ĐỀ THI VÀ LỜI …»i nói đầu Để thử gói lệnh lamdethi.sty tôi biên soạn một số đề toán thi Olympic, mà các học trò của

34 Nguyễn Hữu Điển, ĐHKHTN Hà Nội

Do đó dùng phương pháp quy nạp từ biểu thức

an = (−1)n + nan−1

ta dễ dàng tìm ra được:

an = n! − n!

1!+n!

2!− n!

3!+ ... + (−1)n

n!

n!

Vì thế,theo công thức nổi tiếng Euleran là chuỗi số của sự xáo trộn của bộ số (1, 2, . . . , n), nghĩa là số hoánvị của bộ n số mà không có điểm cố định.Để mỗi cặp (π, j)của hoán vịπ phân biệt từ 1 phần tử hay 1 số nguyên j trong dãy 1, 2, . . . , n,taxác định 1 điểm chú ý nếu j là 1 điểm cố định của πVới k xác định k = 1, 2, . . . , n, có

(nn−k

)ak hoán vị π với n-k điểm xác

định, có (nn−k) cách chọn các điểm cố định này,và chuỗi ak của k điểmcòn lại.Với mỗi hoán vị π như vậy,có n-k cặp (π, j) được xác định.Xét tổng các hoán vị,ta có tổng số điểm chú ý được xác định:

n∑

k=1

(n− k) (nn−k)ak = fn−n∑

k=1

(nn−k)ak = fn − (n! − 1)

Khi tổngn∑k=1

(nn−k

)ak đếm được n! − 1 hoán vị ít hơn n điểm cố định.

Mặt khác:Với mỗi j, j ∈ {1, 2, ..., n} có (n − 1)! − 1 hoán vị phân biệt từ phần tửxác định j.Vì vậy,xét toàn bộ tổng,ta có tổng số điểm xác định chú ý được chỉ ralà:n∑j=1

((n− 1)! − 1) = n(n− 1)! − n

Cho 2 tổng trên bằng nhau,ta được:

fn = 2.n! − n− 1

Lưu ý: sau khi chỉ ra được fn = 2.n! − n− 1 đối với các giá trị nhỏ củan,ta có thể sử dụng mối quan hệ đệ quy và đồng nhất đẳng thức cácphần tử để chứng minh công thức là đúng với mọi n.

vnmath.com

Page 35: OLYMPIC TOÁN NĂM 2000 52 ĐỀ THI VÀ LỜI …»i nói đầu Để thử gói lệnh lamdethi.sty tôi biên soạn một số đề toán thi Olympic, mà các học trò của

Đề thi olympic Trung Quốc 35

Cách giải 2: Chúng tôi giới thiệu 1 phương pháp chứng tỏ rằng an làchuỗi số của hoán vị (1, 2, . . . , n).Với n ≥ 3, ta có :

an = nan−1 + (−1)n = an−1 + (n − 1) an−1 + (−1)n

=[(n − 1) an−2 + (−1)n−1] + (n− 1) an−1 + (−1)n

= (n− 1) (an−1 + an−2)

Gọi bn là chuỗi số của hoán vị của (1, 2, . . . , n).Mỗi hoán vị là mộttrong số những dạng sau đây:a)Với k 6= 1 1 ánh xạ tới k và k ánh xạ tới 1.Như thế sẽ có n-1 giá trịcho k và với mỗi k có bn−2hoán vị cho n-2 phần tử còn lại .Do vậy,có(n− 1) bn−2 hoán vị như trên.b) 1 ánh xạ tới k nhưng k không ánh xạ tới 1.giá trị k cố định. Như vậytồn tại 1 song ánh giữa các hoán vị π và các hoán vị có chỉ số 1 là cốđịnh,thông qua ánh xạ

π 7→ tπ

ở đây t là sự chuyển đổi giữa 1 và k.Bởi có bn−1 ánh xạ trong đó chỉ số1 là cố định ,nên có bn−1 phép hoán vị π.Cho k biến thiên từ 2 đến n,ta thấy có (n− 1) bn−2 hoán vị của dạng(b).Vì vậy bn = (n− 1) (bn−1 + bn−2)

Từ a1 = b1 = 0 và a2 = b2 = 1, an = bn với n ≥ 1 như yêu cầu chứngminh.

.4.29.Một câu lạc bộ bóng bàn muốn tổ chức 1 giải đấu đôi,một loạt những

trận đấu mà trong mỗi trận đấu một cặp người chơi sẽ thi đấu với một

cặp khác.Gọi số trận đấu của một người chơi trong một giải đấu là số

trận đấu mà anh(cô) ta tham gia.

Cho dãy số

A = {a1, a2, ..., ak}

phân biệt,nguyên dương,chia hết cho 6.

Xác định số lượng người chơi tối thiểu để có thể thiết lập một giải đấu

đôi mà :

vnmath.com

Page 36: OLYMPIC TOÁN NĂM 2000 52 ĐỀ THI VÀ LỜI …»i nói đầu Để thử gói lệnh lamdethi.sty tôi biên soạn một số đề toán thi Olympic, mà các học trò của

36 Nguyễn Hữu Điển, ĐHKHTN Hà Nội

(i) mỗi người tham gia nhiều nhất là 2 cặp đấu.

(ii) bất kì 2 cặp khác nhau có nhiều nhất 1 trận đấu gặp nhau.

(iii) nếu 2 người chơi cùng một cặp,họ không bao giờ phải thi đấu với

nhau.

(iv) Số lượng các trận đấu của người tham gia được thiết lập là A.

Lời giải: Bổ đề.

Giả sử rằng :k ≥ 1 và 1 ≤ b1 < b2 < · · · < bk.Như thế tồn tại mộtđồ thị có bk + 1 đỉnh trong đó bộ số {b1, b2, ..., bk} là số độ của các đỉnhtrong bk + 1 đỉnh.Chứng minh:

Ta chứng minh bổ đề bằng phương pháp quy nạp theo k.Nếu k = 1,toàn bộ đồ thị gồm những đỉnh b1 thỏa mãn.Nếu k = 2,lấy b2 + 1 đỉnh, phân biệt các đỉnh này với đỉnh b1 và nối2 đỉnh bằng một đường thẳng khi và chỉ khi một trong số các đỉnh làphân biệt.Ta cần chứng minh bổ đề đúng với k = i ≥ 3 và giả sử nó đúng khik < i .Ta dựng đồ thị G của bi +1 đỉnh,tạo thành các đường thẳng giữahai điểm và từ đó thay đổi độ của các đỉnh trong mỗi đỉnh.Chọn nhữngđiểm có độ bi + 1 đỉnh,và chia chúng ,và chia chúng thành 3 bộ giá trịS1, S2, S3 với |S1|=b1, |S2| = bi−1 − b1 + 1, và |S3| = bi − (bi−1 + 1).Theo giả thiết quy nạp,ta có thể dựng các đường thẳng giữa cácđỉnh trong S1 trong đó độ của các đỉnh được xác định từ tập hợp{b2 − b1, ..., bi−1 − b1}Ngoài ra dựng các đường thẳng có đỉnh trong S1 là điểm cuối.Mỗi đỉnhtrong S1 bây giờ có độ bi, mỗi đỉnh trong S3 có độ b1, và độ của cácđỉnh trong S2được xác định từ tập hợp {b2, ..., bi−1}.Từ đó kết hợp lại tất cả các độ của bi + 1 đỉnh trong đồ thị G được xácđịnh từ tập {b1, b2, ..., bi}.Điều này hoàn tất bước quy nạp và có điều phải chứng minh.• Giả sử rằng ta có 1 giải đấu đôi trong đó n người chơi thỏa mãnđiều kiện đưa ra.Có 1 nhất 1 người chơi,ta gọi là X,có số trận đấu làmax(A).Gọi m là số cặp khác anh(cô) ấy phải thi đấu.Mỗi cặp này có 2 người

vnmath.com

Page 37: OLYMPIC TOÁN NĂM 2000 52 ĐỀ THI VÀ LỜI …»i nói đầu Để thử gói lệnh lamdethi.sty tôi biên soạn một số đề toán thi Olympic, mà các học trò của

Đề thi olympic Trung Quốc 37

chơi và được tính là 2m.Bất kì người chơi nào được tính nhiều nhất 2lần theo cách cấu thành này bởi vì mỗi người chơi thuộc nhiều nhất 2cặp .Do đó,người chơi X sẽ phải đấu với ít nhất m người chơi khác.Nếu X ởtrong j cặp (với j = 1 hoặc 2),sẽ có nhiều nhất tổng số m + j + 1 ngườichơi.Ngoài ra X chơi nhiều nhất jm trận,kéo theo jm ≥ max (A) .Do đó n ≥ m + j + 1 ≥ max (A) /j + j + 1 ≥min {max (A) + 2,max (A) /2 + 3}.Vì max (A) ≥ 6 ,ta có max(A) + 2 > max(A)/2 + 3,kéo theon ≥ max (A) /2 + 3

• Ta cần chứng minh n = max(A)/2 + 3 là số trận nhiều nhất có thểđạt được.Từ bổ đề,ta có thể dựng đồ thị của max(A)

6+ 1 đỉnh trong đó độ được

xác định từ tập hợp {a16, a2

6, ..., ak

6}.

• Chia n người chơi trong max(A)6

+ 1 thành ba phần ,và để 2 người chơicùng trong 1 đội khi và chỉ khi họ cùng nằm trong một phần của baphần trên .Gán cho mỗi phần (và cùng một thời điểm,hình thành 3 cặpngười chơi với những cầu thủ chơi tương ứng) ứng với các đỉnh của đồthị G,và 2 đội được xác định thi đấu khi và chỉ khi các đỉnh tương ứnglà liền kề.Giả sử rằng ta có 1 đội được sắp xếp ở đỉnh v độ ai

6.Với mỗi độ ai

6có

đỉnh w liền kề với v,đó là đội phải đấu trong một phần ba được sắp xếpcho đỉnh w tổng cộng ai

2trận.

Mỗi người chơi được xếp trong v là trong 2 đội,từ đó có số trận đấu là2.ai

2= ai .

Vì vậy số lượng các trận đấu của người tham gia chơi là {a1, a2, ..., ak},khi cần thiết.

.4.30.Cho số nguyên n ≥ 2 . Đối với bất kì tập hợp n số của dãy số thực

A = (a1, a2, ..., an)

Cho lợi điểm của A là số k ∈ {1, 2, ..., n} trong đó ak > aj với mọi giá

trị 1 ≤ j < k .

vnmath.com

Page 38: OLYMPIC TOÁN NĂM 2000 52 ĐỀ THI VÀ LỜI …»i nói đầu Để thử gói lệnh lamdethi.sty tôi biên soạn một số đề toán thi Olympic, mà các học trò của

38 Nguyễn Hữu Điển, ĐHKHTN Hà Nội

Xét tất cả các hoán vị A = (a1, a2, ..., an) của (1, 2, . . . , n) với điểm

lợi điểm là 2.Xác định và chứng minh ý nghĩa số học của phần tử đầu

tiên a1 trong hoán vị này?.

Lời giải: Với mỗi tập hợp n số của dãy số thực

A = (a1, a2, ..., an)

Nếu ak > aj với mọi giá trị 1 ≤ j < k, ta gọi ak là 1 lợi điểm.Nếu một hoán vị

A = (a1, a2, ..., an)

của (1, 2, . . . , n) có số lợi điểm là 2 thì 2 lợi điểm này phải là a1 vàn,trong đó n = ak đối với những giá trị k thỏa mãn 2 ≤ k ≤ n.Cố định m trong dãy số {1, 2, ..., n − 1}.Ta gọi các số m+ 1, m+ 2, ...n

là các số lớn, và 1, 2, ..., m− 1 là các số bé.Trong một hoán vị với 2 lợiđiểm ak = m, n sẽ phải xuất hiện trong hoán vị trước tất cả các số lớnkhác.Vì vậy,để xác định tất cả các hoán vị này, ta chọn n – m vị trí đầutiên là các số lớn,đặt n tại vị trí đầu và sắp xếp n – m – 1 các số lớnkhác vào phần còn lại của các vị trí đã chọn.Sau đó sắp xếp tất cả cácsố bé vào m – 1 vị trí còn lại.• Vì thế,ta có xm =

(n−1n−m

)(n−m− 1)! (m− 1)! = (n−1)!

n−m hoán vịVì vậy ý nghĩa số học của phần tử đầu tiên a1 có trung bình cộng mongmuốn là:

∑n−1m=1 mxm∑n−1m=1 xm

=(n− 1)!

∑n−1m=1

mn−m

(n− 1)!∑n−1

m=11

n−m=

∑n−1m=1

mn−m∑n−1

m=11

n−m

=

∑n−1m=1

nm−

∑n−1m=1

mm∑n−1

m=11m

= n− n− 1

1 + 12

+ ... + 1n

.4.31.Tìm tất cả các số nguyên dương n trong đó

n1, n2, ..., nk > 3

với

n = n1n2...nk = 21

2k(n1−1)(n2−1)...(nk−1) − 1

vnmath.com

Page 39: OLYMPIC TOÁN NĂM 2000 52 ĐỀ THI VÀ LỜI …»i nói đầu Để thử gói lệnh lamdethi.sty tôi biên soạn một số đề toán thi Olympic, mà các học trò của

Đề thi olympic Trung Quốc 39

Lời giải: Nếu một số nguyên dương n thỏa mãn điều kiện đưa ra,thìn = 2m − 1 với m là số nguyên dương.Dễ dàng kiểm tra thấy 3 là sốnguyên m duy nhất nhỏ hơn 10 làm cho n = 2m− 1 thỏa mãn điều kiệnđưa ra.Cho m ≥ 10,ta phải chứng minh 2m − 1 không thỏa mãn điều kiện đưara.Giả sử,để chỉ ra sự mâu thuẫn, lập phương trình biểu diễn một số k vàn1, n2, ..., nk:

m =1

2k(n1 − 1) (n2 − 1) ... (nk − 1) ≥ 10

Với ` ≥ 10 ,ta có(`+1`

)3<

(54

)3< 2

Sử dụng kết quả này,ta dễ dàng chứng minh được bằng phương phápquy nạp 2` − 1 > `3 với các số ngyên ` ≥ 10

Vì vậy

2m − 1 > m3 =

(n1 − 1

2

)3(n2 − 1

2

)3

...

(nk − 1

2

)3

. (1)

Vì n = 2m − 1 là lẻ, ni là chẵn,và với mỗi ni > 3, ni nhỏ nhất phải là5.Do đó (

ni − 1

2

)3

≥ 4ni − 1

2> ni (2)

Cho i = 1, 2, . . . , k. Đặt (1) và (2) cùng nhau,ta thu được

n = 2m − 1 > n1n2...nk = n

điều này mâu thuẫn.Như vậy giả thiết đưa ra là sai và n = 23 − 1 = 7 là đáp án duy nhất.

.4.32.Một bài thi bao gồm 5 câu hỏi nhiều lựa chọn,mỗi câu có 4 lựa chọn

khác nhau.Có 2000 sinh viên làm bài thi,và mỗi sinh viên chỉ được chọn

đúng 1 đáp án trong mỗi câu hỏi.

Tìm giá trị nhỏ nhất của n về những bài thi của sinh viên thỏa mãn điều

kiện: trong bất kì n bài thi,tồn tại 4 bài thi trong đó bất kì 2 bài nào có

nhiều nhất 3 đáp án giống nhau.

vnmath.com

Page 40: OLYMPIC TOÁN NĂM 2000 52 ĐỀ THI VÀ LỜI …»i nói đầu Để thử gói lệnh lamdethi.sty tôi biên soạn một số đề toán thi Olympic, mà các học trò của

40 Nguyễn Hữu Điển, ĐHKHTN Hà Nội

Lời giải: Trước tiên,ta chứng minh n ≥ 25 .Gọi 1,2,3,4 là các lựa chọncủa từng câu hỏi. Thể hiện mỗi câu trả lời của sinh viên bằng một chuỗicó trật tự bao gồm 5 phần tử

(a1, a2, a3, a4, a5), ai ∈ {1, 2, 3, 4}

Trong đó đáp án của sinh viên trong câu hỏi i là ai.Ta nói rằng 2 bài thilà cùng loại nếu chuỗi số 5 phần tử của nó thuộc vào một bộ của dạngsau

{ (k, a2, a3, a4, a5) |k ∈ {1, 2, 3, 4} },

Trong đóa2, a3, a4, a5 ∈ {1, 2, 3, 4} .

Và vì có 256 bộ,và 2000 = 256x7 + 208,nên có ít nhất 8 câu bài thi làcùng loại theo nguyên lý Dirichlet. Trong 1992 bài thi còn lại,có 8 bàinữa cùng loại.Cuối cùng,trong 1984 bài còn lại,thêm 8 bài cùng loại nữa.Xét tập A của 24 bài thi này.Cho 2 bài thi bất kì trong tập A,chúng phảicùng loại,nghĩa là,đáp án cho 4 bài thi cuối cùng là giống nhau.Điều nàymâu thuẫn với giả thiết có 4 bài thi trong tập A,trong đó có 2 bài bấtkỳ có nhiều nhất là 3 câu trả lời giống nhau.Do đó n ≥ 25

Bây giờ chúng ta chỉ ra rằng n=25 là kết quả chấp nhận được.Xác địnhbộ số

S = { (a1, a2, a3, a4, a5) |∑5

i=1ai ≡ 0 (mod4) , ai ∈ {1, 2, 3, 4}}.

Khi đó |S| = 44 = 256 và bất kì 2 bài thi có nhiều nhất 3 đáp án giốngnhau nếu 5 chuỗi phần tử tương ứng của chúng là các yếu tố phân biệtcủa S.Chọn bất kì 250 yếu tố của S,và giả định rằng chính xác có 8 bài thitương ứng với mỗi chuỗi 5 phần tử trong 250 yếu tố này.Vì 25 > 3 nêncó 8 bài thi,có 4 bài và có 5 chuỗi tương ứng là các yếu phân biệt củaS,và thỏa mãn điều kiện đưa ra.Vì vậy, n = 25.

vnmath.com

Page 41: OLYMPIC TOÁN NĂM 2000 52 ĐỀ THI VÀ LỜI …»i nói đầu Để thử gói lệnh lamdethi.sty tôi biên soạn một số đề toán thi Olympic, mà các học trò của

Chương 5

Đề thi olympic Tiệp khắc

.5.33.Chứng minh rằng:

3

√a

b+

3

√b

a≤ 3

√2(a+ b)

(1

a+

1

b

)

với mọi số thực a, b. Dấu "=" xảy ra khi nào ?

Lời giải: Nhân 2 vế của bất đẳng thức với 3√ab ta được bất đẳng thức

tương đương

3√a2 +

3√b2 ≤ 3

√2(a+ b)2

Đặt 3√a = x, 3

√b = y, chúng ta thấy rằng điều đó thỏa mãn chứng tỏ

rằngx2 + y2 ≤ 3

√2(x3 + y3)2(∗)

với x, y > 0.Áp dụng bất đẳng thức trung bình ta được

3x4y2 ≤ x6 + x3y3 + x3y3

Dấu "=" xảy ra nếu và chỉ nếu x6 = x3y3 = y6 hoặc x=y.Cộng 2 vế của 2 bất đẳng thức với nhau và cộng x6 + y6 vào 2 vế củabất đẳng thức ta có:

x6 + y6 + 3x2y2(x2 + y2) ≤ 2(x6 + y6 + 2x3y3)

Bất đẳng thức (*) xảy ra khi x=y hoặc a=b.

vnmath.com

Page 42: OLYMPIC TOÁN NĂM 2000 52 ĐỀ THI VÀ LỜI …»i nói đầu Để thử gói lệnh lamdethi.sty tôi biên soạn một số đề toán thi Olympic, mà các học trò của

42 Nguyễn Hữu Điển, ĐHKHTN Hà Nội

.5.34.Tìm tất cả tứ giác lồi ABCD mà tồn tại một điểm E nằm bên trong tứ

giác thỏa mãn điều kiện sau đây:

Bất kỳ đường thẳng nào qua E và cắt các cạnh AB và CD đều chia tứ

giác lồi ABCD thành 2 phần bằng nhau.

Lời giải: Giả sử rằng tứ giác lồi ABCD có tính chất đó. Lấy X1,X2,X3

là 3 điểm nằm trên cạnh ABvới AX1 < AX2< AX3. Như vậy cạnh XkE cặt đoạn CD tại Yk vớik=1,2,3.Do tứ giác ABCD lồi và CY1 < CY2 < CY3 nên ta có:0 = 1

2[ABCD] − 1

2[ABCD] = [AX1Y1D] − [AX2Y2D]

= [EY1Y2] − [EX1X2] = 12sin Y1EY2(EY1.EY2 − EX1.EX2)

Suy ra EX1.EX2= EY1.EY2

Tương tự có EX2.EX3= EY2.EY3

Do đó EX1/EY1= EX3.EY3 và ∆Y1EY3 ∼ ∆X1EX3

Cho nên X1X3 ‖ Y1Y3. Điều đó có nghĩa là AB ‖ CD

Mặt khác ta có ABCD là tứ giác lồi với AB ‖ CD. Có E là trung điểmcủa đoạn M1M2. Điểm M1 và M2 tương ứng là trung điểm của đoạn ABvà CD.Giả sử một cạnh đó đi qua E và cắt cạnh AB tại X và cắt cạnh CD tạiY. Kết quả là nó đi qua M là giao điểm của AB và CD.Và XM1 với YM2

Điều đó ta có XM1 = YM2 và AX +DY = BX + CY

Khi đó mỗi tứ giác AXYD và BXYD là hình thang hoặc hình bình hànhcó cùng đường cao và cùng chiều dài cạnh đáy.Bởi vậy chúng có diện tích bằng nhau.(Đpcm)

.5.35.Bài 3: Cho tam giác đều ABC với cạnh đáy AB và đường cao CD. Điểm

P nằm trên cạnh CD. Điểm E là giao điểm của AP với BC và F là giao

điểm của BP với AC. Giả sử rằng đường tròn nội tiếp tam giác ABP và

tứ giác PECF bằng nhau. CMR đường tròn nội tiếp tam giác ADP và

BCP cũng bằng nhau.

Lời giải: Cho ω1 và ω2 là đường tròn nội tiếp tứ giác CEPF và tam giácABP, và hai điểm I1 và I2 lần lượt là tâm của của 2 đường tròn ω1 và

vnmath.com

Page 43: OLYMPIC TOÁN NĂM 2000 52 ĐỀ THI VÀ LỜI …»i nói đầu Để thử gói lệnh lamdethi.sty tôi biên soạn một số đề toán thi Olympic, mà các học trò của

Đề thi olympic Tiệp khắc 43

ω2.Bởi vì chúng đối xứng qua CD, I1 và I2 là 2 điểm trên đoạn CD với Pnằm giữa 2 điểm đó.Bởi vì ω1 và ω2 là hai đường tròn bằng nhau và vẽ nội tiếp trong gócđối đỉnh, chúng đối xứng qua mỗi điểm P.Do đó PI1 = PI2

Do tứ giác ADBP và BDP đồng dạng, chúng ta chỉ cần chứng minhrằng bán kính đường tròn nội tiếp r1 của tam giác BCD bằng bán kínhđường tròn nội tiếp BDP.Gọi X và Y tương ứng là giao điểm của 3 đường phân giác trong củatam giác BCP và BDP.Nhận xét rằng I1 nằm trong tam giác CBF, như vậy I1 nằm trên đườngphân giác của góc CBF, tức là góc CBP.Như vậy X nằm trên đoạn BI1 và tương tự Y nằm trên BI2Do PI1 = PI2, [BI1P ] = [BI2P ] nênr1(PI1 + BP ) = 2 ([I1PX] + [XPB]) = 2 [I1BP ] = 2 [PI2B] =

2 ([PI2Y ] + [PIB])

= r2(PI2 +BP )

Do đó r1 = r2

.5.36.Cho 2000 điểm trong tam giác của một bề mặt thuộc một mặt phẳng.

Với mỗi ảnh của hình tam giác qua một phép tịnh tiến là 1 tam giác bao

gồm trọng tâm.

Chứng minh diện tích của các tam giác này nhỏ hơn 229.

Lời giải: Định hướng của mỗi hình trong bài toán của 2000 điểm đãcho trong một tam giác có 1 cạnh cùng với đỉnh đối diện ở trên.Để cho tam giác ABC có 1 tam giác với AB nằm ngang và A nằm bêntrái B. Như vậy không có 1 điểm nào khác trong 1999 hình tam giác cóbề mặt nằm ngang bên dưới cạnh AB.Chúng ta bắt đầu bởi khái niệm về khoảng cách từ khoảng cách Oclitvà chúng ta định nghĩa chính thức mô tả một vài mối liên hệ giữa 2000tam giác.Xác định một điểm hay kẻ một đường thẳng song song với BC. Để chokhoảng cách dα từ một điểm α khác tồn tại khoảng cách giữa hai đường

vnmath.com

Page 44: OLYMPIC TOÁN NĂM 2000 52 ĐỀ THI VÀ LỜI …»i nói đầu Để thử gói lệnh lamdethi.sty tôi biên soạn một số đề toán thi Olympic, mà các học trò của

44 Nguyễn Hữu Điển, ĐHKHTN Hà Nội

thẳng song song đến BC đi qua tam giác đó.Chọn a′ = dα(BC,A)

Định nghĩa tương tự với β, γ và khoảng cách giữa CA và AB với :b′ = dβ(CA,B) > 0, c′ = dγ(AB,C) > 0 .Chúng ta biết rằng sự tịnh tiến ảnh qua 1 khoảng cách nào đó cho ảnhgiống nhau với khoảng cách giữa β và γ.Giả sử rằng ∆XY Z và ∆X0Y0Z0 là 2 hình tam giác trong số 2000 tamgiác đó với XY ‖ X ′Y ′ ‖ AB và Y Z ‖ Y ′Z ′ ‖ BC.Phép tịnh tiến biến ∆XY Z thành ∆X0Y0Z0.Bởi vì trọng tâm của ∆XY Z là điểm ở trên phía trái của Y ′Z ′ = T (Y Z),dα(Y Z, T (Y Z)) ≤ 1

3a′ do đó:

dα(X,X′) = dα(X, T (X)) ≤ 1

3a′ suy ra

dα(Y Z, X ′) ≤ 4

3a′

Tương tự dβ(ZX, Y ′) ≤ 43b′, dγ(XY, Z ′) ≤ 4

3c′.

Phép tịnh tiến biến ảnh tam giác ABC ở dưới mặt phẳng chứa C với tỉsố 1

3. Như thế [T ] = 1

9. Phép tịnh tiến T của u1 và v1 của hai điểm tương

ứng C, A và phép tịnh tiến u1 và v2 của 2 điểm tương ứng C và B.Với T1, T2 là 2 phép tịnh tiến với T1[u1] = v1, T2[u2] = v2.Để cho bao lồi của 4 tam giác chứa F bị chặn bởi l1 bên phải và l2 bêntrái, l3 ở trên đỉnh và ở dưới đáy AB. Quan sát rằng F là 1 đỉnh củahình thang với diện tích 24

9.

Bởi vì dα(AB, l3) = 43c′, những điểm bao gồm trọng tâm của tam giác

ABC nằm ở trên hay ở dưới l3. Tương tự dβ(l2, B) = 43b′, dγ(l3, C) = 4

3c′

theo thứ tự trong tam giác ABC bao gồm trọng tâm của 1 tam giácnhững điểm của tam giác phải nằm trên hoặc ở bên trái l2 hoặc nằmbên trái của l1.Tổ hợp với cực trị được xác định của tam giác ABC kéo theo hệ quả làmiền < bị phủ bởi 2000 tam giác là sai với hình thang F được xác địnhcủa đường kẻ qua mặt song song của 2000 tam giác.Do đó:

[<] ≤ [F ] − 1

9− 1

9− k =

22

9− k <

22

9

vnmath.com

Page 45: OLYMPIC TOÁN NĂM 2000 52 ĐỀ THI VÀ LỜI …»i nói đầu Để thử gói lệnh lamdethi.sty tôi biên soạn một số đề toán thi Olympic, mà các học trò của

Đề thi olympic Tiệp khắc 45

(đpcm)

vnmath.com

Page 46: OLYMPIC TOÁN NĂM 2000 52 ĐỀ THI VÀ LỜI …»i nói đầu Để thử gói lệnh lamdethi.sty tôi biên soạn một số đề toán thi Olympic, mà các học trò của

Chương 6

Đề thi olympic Estonia

.6.37.Cho 5 số thực, chọn 3 số bất kì thì hiệu của tổng 3 số đó và tổng hai số

còn lại sẽ là một số dương. Chứng minh rằng tích của tất cả 10 hiệu số

(tùy theo số khả năng 3 số được chọn) nhỏ hơn hoặc bằng tích của bình

phương 5 số đó.

Lời giải: Cho 5 số là x1, x2, x3, x4, x5 ta có 5 hiệu số tương ứng làa1, a2, a3, a4, a5 và b1, b2, b3, b4, b5, Ở đây:

ai = −xi−2 + xi−1 + xi + xi+1 − xi+2

bi = xi−2 − xi−1 + xi − xi+1 + xi+2

với i = 1, 2, 3, 4, 5. Với mỗi i, chúng ta cóx2i − aibi = (ai+bi

2)2 − aibi = (ai−bi

2)2 > 0

hoặc x2i > aibi vì aibi > 0 cho mỗi i, chúng ta có thể nhân 5 bất đẳng

thức aibi > x2i với 1 6 i 6 5 thì thu được∏5

i=1 x2i >

∏5i=1 aibi

.6.38.Chứng minh rằng không thể chia một tập bất kì gồm 18 số nguyên dương

liên tiếp thành hai tập A và B, với tích của các phần tử trong A bằng

tích của các phần tử trong B.

Lời giải: Chứng minh phản chứng. Giả sử, chúng ta chia được một tậpS = n, n+ 1, · · · , n+ 17 của 18 số nguyên dương liên tiếp thành tâp A,B sao cho

∏a∈A a =

∏b∈B b vì tích của các phần tử trong A bằng tích

của các phần tử trong B, nếu 1 tập chứa một bội số của 19, thì tập cònlại cũng phải như vậy. Do vậy, S không chứa bội số nào của 19 hoặc chứa

vnmath.com

Page 47: OLYMPIC TOÁN NĂM 2000 52 ĐỀ THI VÀ LỜI …»i nói đầu Để thử gói lệnh lamdethi.sty tôi biên soạn một số đề toán thi Olympic, mà các học trò của

Đề thi olympic Estonia 47

ít nhất hai bội số của 19. Vì có duy nhất 1 trong 18 số nguyên dươngliên tiếp có thể là bội số của 19, S phải không chứa bội số nào. Bởi vậy,n, n+ 1, · · · , n+ 17 lần lượt đồng dư với 1, 2, 3, · · · , 18 mod 19( chia lấydư). Do vậy, theo qui tắc Wilson∏

a∈A a×∏

b∈B b = n(n+ 1) · · · (n + 17) ≡ 18! ≡ −1 (mod 19)Tuy nhiên, hai tích của bên trái là bằng nhau, điều này không có khảnăng vì -1 không là bình phương của phép mod 19. Bởi vậy, không tồntại hai tập A và B.

.6.39.Cho M, N và K là các điểm tiếp xúc của đường tròn nội tiếp tam giác

ABC với các cạnh của tam giác, gọi Q là tâm đường tròn đi qua trung

các đoạn thẳng MN, NK, KM. Chứng minh rằng tâm đường tròn nội

tiếp và ngoại tiếp tam giác ABC thẳng hàng với Q.

Lời giải: Để chứng minh, giả sử M, N và K nằm trên các cạnh BC, CA,AB và X, Y, Z là trung điểm các đoạn thẳng NK, KM, MN. Theo đềbài, Q là tâm đường tròn ngoại tiếp tam giác XYZ. Đường thẳng AX làtrung tuyến đoạn thẳng KN của tam giác cân AKN, vừa là đường phângiác và là đường cao của tam giác. Do vậy, A, X và I là thẳng hàng,và AXZ = π

2. Do đó, Tam giác vuông AXK và AKI là đồng dạng, và

IA.IX = IK2. Bởi vậy, X la hình chiếu của A dưới phép nghịch đảo quađường tròn nộ tiếp của tam giác ABC. Tương tự, Y là hình chiếu củaB và Z là hình chiếu của C với cùng phép nghịch đảo. Dẫn đến, nghịchđảo ánh xạ đường tròn ngoại tiếp tam giác ABC đến đường tròn ngoạitiếp XYZ, nên tâm của của các đường tròn này là thẳng hàng với I. Nóimột cách khác, Q, I và tâm đường tròn ngoại tiếp của tam giác ABC làthẳng hàng.

.6.40.Tìm tất cả các hàm f: N −→ N sao cho:

f(f(f(n))) + (f(f(n)) + f(n) = 3n với ∀x ∈ N .

Lời giải: Nhận xét rằng nếu f(a) = f(b), thì với n = a và n = b chobiểu thức 3a = 3b hay a = b. Bởi vậy, f là duy nhất. Chúng ta chứngminh bởi qui nạp với n ∈ Z∗ thì f(n) = n. Giả thiết rằng với ∀n < n0,f(n) = n Chúng ta chứng minh rằng f(n0) = n0( Mệnh đề áp dụng vớin = 1). Vì f là duy nhất, nếun > n0 > k thì f(n) 6= f(k) = k, vì vậy

vnmath.com

Page 48: OLYMPIC TOÁN NĂM 2000 52 ĐỀ THI VÀ LỜI …»i nói đầu Để thử gói lệnh lamdethi.sty tôi biên soạn một số đề toán thi Olympic, mà các học trò của

48 Nguyễn Hữu Điển, ĐHKHTN Hà Nội

f(n) > n0 (*) với ∀n > n0 .Đặc biệt, (*) áp dụng với n = f(n0 và tươngtự với f(f(n0)) Thay n = n0 trong hàm đã cho, chúng ta thấy:

3n0 = f(f(f(n0))) + f(f(n0)) + f(n0) > n0 + n0 + n0

Dấu = xảy ra khi f(n0) = n0

.6.41.Trong tam giác ABC chúng ta có AC 6= BC . Lấy điểm X bên trong

tam giác và gọi α = A, β = B, γ = C và ψ = BCX. Chứng tỏ rằng:sinαsinβ

sin(α−β)= sinγsinψ

sin(γ−ψ)

Nếu và chỉ nếu X nằm trên trung tuyến của tam giác ABC kẻ từ điểm

C.

Lời giải: Lấy M là trung điểm của AB đặt γ′

= ACM và ψ′

= MCB.Không giảm tính tổng quát, giả sửα > β và BC > AC, theo quy tắcđường phân giác, phân giác góc ACB gặp cạnh AB tại điểm gần A hơnB, ví dụ tại một điểm trên AM . Do đó γ′

> ψ′.

Chiếu B qua đoạn CX được điểm B’ và đặt điểm D trên đoạn CB’, trênphía đối diện đoạn thẳng AB tại C, rõ ràng ψ′

= BAD. Áp dụng điịnhlý Ceva trong tam giác ABC đối với các đoạn thẳng đồng qui AD, XDvà B’D 4BCB′ là tam giác cân với BC = B’C và 2ψ

= B′CB như vậyCBB′ = π

2− ψ

′ . Do đó ABB′ = CBB′ − CBA = π2− ψ

′ − β

vì MA = MB = MB’, ABB′ = π2

và tam giác MAB’ là tam giác cân, vìvậy, các góc của tam giác này là:

B′AM = B′AB = π2− ABB′ = ψ

′ − β

MB′A = ψ′

+ β

AMB′ = π − 2(ψ′

+ β) = (α− β) + (γ′ − ψ

)

Đẳng thức cuối cùng đúng vì α + β + γ′

+ ψ′

= π

vì MAB′ > ψ′ = MAD, D nằm giữa bán kính AM và AB’ và do đó nằm

trong tam giác AXB’. Tương tự MAD = ψ′=BCM = MCB′ = MCD

dẫn đến tứ giác MCAD là ngoại tiếp. Chúng ta tính các góc các đoạnAD,MD và B’D tạo với các cạnh của tam giác AMB’. Đầu tiên chúngta có MAD = ψ

′ , DMA = DCA = γ′ − ψ

′ và DB′M = CB′M =

DB′M = α Kết hợp các biểu thức của các góc tam giác AM’B, ta cóbiểu thức

DAB′ = MAB′ − MAD = β

vnmath.com

Page 49: OLYMPIC TOÁN NĂM 2000 52 ĐỀ THI VÀ LỜI …»i nói đầu Để thử gói lệnh lamdethi.sty tôi biên soạn một số đề toán thi Olympic, mà các học trò của

Đề thi olympic Estonia 49

B′MD = B′MA− DMA = α− β

và AB′D = AB′M − DB′M = ψ′

Áp dụng định lí Ceva và định lí sin, chúng ta thấy:1 = sinMADsinAB′DsinB′MD

sinDAB′sinDB′MsinDMA

= sinψ′

sinβ× sinψ

sinβ× sin(α−β)

sin(γ′−ψ′

)

=MBMC

× sinψ′

sinβ× sin(α−β)

sin(γ′−ψ′ )

=MAMC

× sinψ′

sinβ× sin(α−β)

sin(γ′−ψ′ )

= sinγ′

sinα× sinψ

sinβ× sin(α−β)

sin(γ′−ψ′

)

Nếu X nằm trên đoạn CM, thì (γ, ψ) = (γ′

, ψ′

) và biểu thức bằng nhauở trên dẫn đến biểu thức đã cho. Ngược lại, giả sử có biểu thức đãcho. Đặt ϕ = BCA và đặt f(θ) = sinθ.sin(ϕ−θ)

sin(θ−(ϕ−θ)) . Chúng được cho rằngf(γ) = f(γ

), giá tri chung này 6= 0 (nonzero) vì 0 < γ, γ′

< ϕ vì vậy1

f(θ)được định nghĩa và nhận giá trị tại θ = γ và θ = γ

′ . Tuy nhiên:

1

f(θ)=sinθ.cos(ϕ− θ) − cosθ.sin(ϕ− θ)

sinθ.sin(ϕ− θ)= cotθ − cot(ϕ− θ)

đây là một hàm đồng biến nghịch với 0 ∈ (0, ϕ). Bởi vậy γ = gamma′

và X phải nằm trên đoạn thẳng CM. Mệnh đề đã được chứng minh.

.6.42.Chúng ta gọi một tập vô hạn dãy các số nguyên dương là F - sequence

nếu mọi số hạng của tập(Bắt đầu từ số hạng thứ 3 ) bằng tổng của 2 số

hạng trước đó. Có thể tách tập tất cả các số nguyên dương thành

a) Hữu hạn

b) Vô hạn

số lượng của F - sequence không có các phần tử chung?

Lời giải: a) Giả sử để chứng minh phản chứng tồn tại m tập F đượcphân chia từ các số nguyên dương. Đặt dãy ith là F

(i)1 , F

(i)2 , · · · ,, vì

F(i)n+2, F

(i)n+1 tăng với n > 2 nên tồn tại Ni sao cho F

(i)n+2, F

(i)n+1 > m với

mọi n > Ni.Đặt N = max{N1, N2, · · · , Ni}, và chọn một số nguyên dương k vượtquá số hạng N đầu tiên của mỗi dãy số. Theo nguyên lí Pigeonhole,hai trong số các số k, k + 1, · · · , k +m tồn tại trong cùng dãy F các sốnguyên này khác nhau tại mọi m, đây là mâu thuẫn. Vì vậy câu trả lờiđối với phần a) là không.

vnmath.com

Page 50: OLYMPIC TOÁN NĂM 2000 52 ĐỀ THI VÀ LỜI …»i nói đầu Để thử gói lệnh lamdethi.sty tôi biên soạn một số đề toán thi Olympic, mà các học trò của

50 Nguyễn Hữu Điển, ĐHKHTN Hà Nội

b) Định nghĩa dãy Fibonasi {Fn} với F0 = F1 = 0 và đệ quy Fn+1 =

Fn + Fn−1 với n > 1 nó có thể hiển thị bằng cách quy nạp j có duynhất biểu diễn Zeckendorp ak.ak−1 · · · .k1 với các tính chất ak = 1, mỗiak = i hoặc 1 không có 2 số liên tiếp bằng 1 và j =

∑ki=1 ak.Fk. Có vô

hạn các số nguyên dương m có biểu diễn Zeckendorpf kết thúc với a =1. Với mỗi m, định nghĩa một dãy Fm như sau: đặt số hạng thứ n là số∑n

k=1 ak.Fk+n−1 biểu diễn Zeckendorpf là ak.ak−1 · · · .k1 tiếp theo n - 1các số 0. Thì tổng của số hạng thứ n và n+ 1 là∑n

k=1 ak.Fk+n−1 +∑n

k=1 ak.Fk+n

=∑n

k=1 ak.(Fk+n−1 + Fk + n)

=∑n

k=1 ak.Fk+n+1

Là số hạng thứ n+2. Do đó, Fm là một dãy F, bất kỳ số nguyên dươngj tồn tại trong F đúng với một số nguyên dương m, một biểu diễnZeckendorpf biểu diễn là giống j, trừ bất kì các số 0. Do đó, các dãy sốnguyên dương, dẫn đến câu trả lời cho phần b) là có.

vnmath.com

Page 51: OLYMPIC TOÁN NĂM 2000 52 ĐỀ THI VÀ LỜI …»i nói đầu Để thử gói lệnh lamdethi.sty tôi biên soạn một số đề toán thi Olympic, mà các học trò của

Chương 7

Đề thi olympic Hungary

.7.43.Tìm tất cả các giá trị dương của p biết rằng tồn tại các số nguyên dương

n, x,y thoả mãn pn = x3 + y3

Lời giải: p=2 và p=3 ta có: 21 = 13+13 và 32 = 13+23 Giả sử rằng p>3và không xảy ra mâu thuẫn trong phươngtrình gồm 3 số thực dương.Chọn n, x, y sao cho n nhỏ nhất Vì p 6= 2 ta có (x, y) 6= (1, 1) . Do đóx2 − xy+ y2 = (x− y)2 + xy > 1 như là (x + y). Vì cả 2 giá trị này đềuchia hết cho x3+y3, chúng phải là những bội số của p. Do đó p phântích thành (x + y)2 − (x2 − xy + y2) = 3xy.Vì p không chia hết cho3,p chia hết cho ít nhất đ 1 số hoặc x hoặc y. Hơn nữa p không thể chiahết chỉ là một trong x và y vì p/(x+y). Theo đó pn = x′3 + y′3. Tại đó(n’,x’,y’) +(n-3, x/3, y/3). Nhưng n’ < n trái với giả thiêt n nhỏ nhất.Vậy chỉ có p=2 và p=3 thỏa mãn yêu cầu bài toán.

.7.44.Có hay không một đa thức f bậc 1999 với hệ số là f(n) , f(f(n)) , f(f(f(n))),

. . . là các cặp giá trị tương đối với bất kì số nguyên n nào?

Lời giải: Cho g(x) là đa thức bất kì bậc 1997 với những hệ số nguyênvà chof(x) =x(x-1)g(x) +1 . Ta chứng minh ràng nếu f thoả mãn yêu cầu bàitoán thì mệnh đề trên đúng.Tức la phải chỉ ra rằng với n là số nguyên bất kì và p là 1 giá trị phântích thành f(n), vậy thì p - fk(n) với bất kì số nguyên dương k>1.Đặc biệt hơn, ta chứng minh rằng fk(n) ≡ 1(modp)với mọi k> 1. Ta

vnmath.com

Page 52: OLYMPIC TOÁN NĂM 2000 52 ĐỀ THI VÀ LỜI …»i nói đầu Để thử gói lệnh lamdethi.sty tôi biên soạn một số đề toán thi Olympic, mà các học trò của

52 Nguyễn Hữu Điển, ĐHKHTN Hà Nội

đặt k để chứng minh bài toán. Biết rằng cho một đa thức h với các hệsố nguyên a ≡ b(modc) tức là h(a) ≡ h(b)(modc). Xét trường hợp k=2,f(n) ≡ 0(modp) suy ra f(f(n)) ≡ f(0) ≡ 1(modp).Vậy thì f(fk(n)) ≡ f(1) ≡ 1(modp). Điều phải chứng minh.

.7.45.Chân đường các đường phân giác của các góc tam giác ABC là X, Y và

Z. Đương tròn ngoại tiếp tam giác XYZ cắt 3 đoạn thẳng AB, BC và

CA. Chứng minh rằng tổng độ dài của hai trong những đoạn thẳng này

phụ thuộc vào độ dài cạnh thứ 3.

Lời giải: Ta đặt các kí hiệu khoảng cách trong suốt bài toán tại đóa=BC ,b=CA , c=AB là dương. Đồng thời, cho đường tròn cắt BC tạiX và P, CA tại Y và Q, và AB tại Z và R, đặt x = PX, y=QY, z=RZ.Theo định lí đường phân giác ta có: YA=bc/(c+a) , AZ=bc/(a+b). Dođó, QA=bc/(c+a)+y , AR=bc/(a+b)-z. Áp dụng định lí về phương tíchcủa 1 điểm cho điểm A ta được

bcc+a

( bcc+a

+ y) = bcb+a

( bca−b − z)

Sau khi nhân các vế với a/bc và sắp xếp lại ta có

cc+a

y + ab+a

z = abc(b+a)2

− abc(a+c)2

Tương tự ta có

bb+a

z + bb+cx = abc

(b+c)2− abc

(a+b)2

cb+cx+ c

a+cy = abc

(a+c)2− abc

(c+b)2

Khi cộng 3 phương trình lại ta nhận ra điều đơn giản làx+ y +z = 0.Theo đó hai trong các số hạng x,y,z phải cùng dấu và số hạng thứ 3phải trái dấu hai số còn lại. Do đó tổng giá trị tuyệt đối của hai số hạngtrước bằng trị tuyệt đối của số hạng sau, đó là điều phải chứng minh.

.7.46.Cho kvà t là các số nguyên tố cùng nhau lớn hơn 1. Bắt đầu từ phép

hoán vị (1,2,. . . .. n) của dãy số 1,2 . . . . . . ..n, ta co thể đổi 2 số nếu

chúng không là k hoặc t . Chứng minh rằng ta có thể thực hiện bất kì

phép hóan vị nào của 1,2,. . . n khi và chỉ khi n > k + t− 1

Lời giải: Dựng đồ thị G mà các đỉnh của nó là các số nguyên 1,2,. . . .nvới hiệu ứng biên giữa a và b khi và chỉ khi |a− b| ∈ {k, t} Ta thấy điều

vnmath.com

Page 53: OLYMPIC TOÁN NĂM 2000 52 ĐỀ THI VÀ LỜI …»i nói đầu Để thử gói lệnh lamdethi.sty tôi biên soạn một số đề toán thi Olympic, mà các học trò của

Đề thi olympic Hungary 53

kiện trvừa nêu tương đương với (i) mọi phép hoán vị có thể thực hiệnđược, (ii) G là đường liền; (iii) n > k + t− 1 (i) ⇒ (ii) Vì mỗi bưới traođổi 2 số trong cùng 1 nhánh của G, Theo đó không số nào có thể về vịtrí cũ đã được thay thế bởi 1 số ở một bộ phận khác. Vậy thì , ta khôngthể thực hiện được mọi phép hoán vị trừ khi tất cả các số đều nằm trênmột nhánh.(i) ⇒ (ii): Ta chứng minh bằng phép quy nạp trên m rằng xác địnhmột đồ thị liền với các số nguyên m., bất kì phép hoán vị nào của các sốnguyên này có thể có được từ phép hoán vị khác bật kì tao bởi sự traođổi liên tiếp của cặp (a,b), tại đó a và b là các đỉnh liền kề của đồ thị.Yêu cầu rõ ràng khi m = 1. Mặt khác, chọn một đỉnh a sao cho phầnđồ thị còn lại là đường liền sau khi bỏ a – ví du, ta có thể cho điểmA là điểm cuối của các đỉnh. Một số quỹ đạo của các đỉnh phân biệta0a1 . . . ar nối liền a0 = π−1(a) và ar = a Bằng cách chuyển vị trí liêntiếp (a0a1), (a1a2) . . . (ar−1ar) ta có thể di chuyển a tới vị trí ban đầuchiếm chỗ bởi π−1(a) Bằng giả thiết quy nạp, các biến số thay thế a cóthể hoán vị khi cần, do đó đạt được hoán vị . Áp dụng bổ đề này vớim=n và đồ thì G chứng minh rằng (ii) ⇒ (i)

(ii) ⇒ (iii): Nếu ít nhất k là n thì mọi cạnh biên sẽ nối 2 biến số đồngdư theo môđun t. Vậy sẽ không có đường nào giữa 1 và 2 (mâu thuẫn). Do đó, ta phải có k <n; tương tự , t<n . Vậy có n – k cạnh biên củadạng thức {a, a+ k} và n-t của cạnh biên của dạng thức{a, a+ t} Sựliên thông yêu cầu ita nhất n -1 cạnh biên, do đó (a − k) + (n − t) >

n− 1 ⇒ n > k + t− 1

(iii) ⇒ (ii): Tất nhiên trong trường hợp này k,t < n. chú ý rằng 2 sốmà đồng dư mô đun k sẽ được nối với nhau ( các cạnh biên ngang quacủa dạng {a, a+ k} ) do đó đủ để chỉ ra rằng tất cả các số 1,2,. . . .,kđược nối lẫn nhau. Vì t là gía trị tương đương với k nên 1, 2t, 3t, . . . , kt

biểu diễn bởi tất cả các đồng dư môđun k. Do đó, ta có thể sắp xếp lại1, 2, . . . ., k theo thứ tự b1, b2, . . . , bk tại đó bi ≡ it(modk). Chú ý rằngk ≡ 0 ≡ kt(modk) do đó bk = k Vậy khi k − 1 > i > 1 ta có k − 1 > bi

và vì thế nên n > k + t− 1 > bi + t Do đó tồn tại đỉnh bi + t và đượcnối bởi một cạnh biên tới bi Hơn nữa, bi + t ≡ bi+1(modk) nên bi + t

vnmath.com

Page 54: OLYMPIC TOÁN NĂM 2000 52 ĐỀ THI VÀ LỜI …»i nói đầu Để thử gói lệnh lamdethi.sty tôi biên soạn một số đề toán thi Olympic, mà các học trò của

54 Nguyễn Hữu Điển, ĐHKHTN Hà Nội

được nối tới bi+t. Vì thế cho nênbi, được nối tới bi+1 trong G với mỗii = 1, 2, . . . , k − 1. Vậy các số này đươc nối liền lẫn nhau và bài toánđược giải quyết.

.7.47.Cho số nguyên dương bất kì k, cho e(k) là số ước dương chẵn của k. và

cho o(k) là số ước dương lẻ của k. Với n>1, chứng minh rằng :∑n

k=1 e(k)

và∑n

k=1 o(k) khác nhau với hầu hết n.

Lời giải: Số nguyên có thể chia được bởi d nằm chính giữa dãy số1,2,. . . ,n là |n/d| Do đó, tổng của o(k) (rerp.e(k)) đến khoảng k=1,2. . . ,n bằng với tổng của |n/d| đến khoảng tất cả các số nguyên dươngchẵn d. Vì [n

d] > [ n

d+1] vớì số nguyên dương a và n, ta có

∑n

k=i o(k) −∑n

k=i e(k) =∑∞

k=i([n

2i+1] − [ n

2i])

Tại đó tổng hạn được xác định vì các số hạng bằng 0 khi i> [n2]. Tương

tự:∑n

k=i o(k) −∑n

k=i e(k) = [n1] −

∑∞k=i([

n2i

] − [ n2i+1

]) 6 n

.7.48.Cho một tam giác trong không gian, hãy nêu cách dựng điểm P nằm bên

trong tam giác thoả mãn điều kiện sau: nếu hạ một đường thẳng vuông

góc qua P tới các ạnh của tam giác. Chân của các đường vuông góc tạo

thành một tam giác nhận P là trọng tâm.

Lời giải: a: Gọi tam giác là ABC với độ dài các cạnh là a=BC, b= CA,c=AB. Gọi P là một điểm nằm trong tam giác, vị trí xác định sau. Gọi X,Y, Z là chân các đường vuông góc kẻ tới các cạnh BC, CA, AB, theo thứtự đó, x =PX, y= PY, z=PZ. Chuý ý rằng sin Y PZ = sin(π− BAC) =

sin BAC. Tương tự sin ZPX = sin CBA và sin XPY = sin ACB.Tương đương với các điều sau• P là trọng tâm tam giác XYZ• Các tam giác YPZ,ZPX, XPY có diện tích bằng nhau• yz sin Y PZ = zx sin ZPX = xy sin XPY

• sin<BAC/x = sin<CBA/y = sin<ACB/z• a/x = b/y = c/z (định lý hàm Sin)Dựng đường thẳng song song với BC, cách BC một khoảng là a nằm

vnmath.com

Page 55: OLYMPIC TOÁN NĂM 2000 52 ĐỀ THI VÀ LỜI …»i nói đầu Để thử gói lệnh lamdethi.sty tôi biên soạn một số đề toán thi Olympic, mà các học trò của

Đề thi olympic Hungary 55

cùng phía với A. Tiếp tục dựng đườn thẳng song song với CA, cáchCA một khoảng là b, nằm cùng phía với B. Đặt Q là giao điểm củachúng,chú ý rằng tia CQ đi qua miền trong tam giác Lấy P’ bất kỳ trênCQ, xét tỉ số khoảng cách tới BC với khoảng cách tới AB. Nếu P’=Qtỉ số này bằng a/b ;bởi vì mọi điểm P’ là hình ảnh đồng dạng của nhauvới C,nên tỉ số này không phụ thuộc vào P’ và luôn bằng a/b. Hơn nữata có thể dựng 1 tia từ A hướng vào tam giác mà với mọi P’ trên tiathì tỉ số k/c từ AB với k/c từ CA bằng c/b. Hai tia này giao nhau tạiđiểm P nào đó ở trong tam giác . Nếu ta đặt P là giao điểm của chúng,ta thu được a/x = b/y và b/y = c/z ,và bài toán có lời giải.

vnmath.com

Page 56: OLYMPIC TOÁN NĂM 2000 52 ĐỀ THI VÀ LỜI …»i nói đầu Để thử gói lệnh lamdethi.sty tôi biên soạn một số đề toán thi Olympic, mà các học trò của

Chương 8

Đề thi olympic India

.8.49.Cho 4 ABC không đều. Gọi P là một điểm nằm trong tam giác đó. Các

đoạn thẳng qua P và nối đỉnh với các cạnh đối diện của tam giác đó

có độ dài bằng nhau và bằngλ thỏa mãn λ < min(AB,BC,CA). Chứng

minh rằng có điểm P , 6= P thỏa mãn các tính chất giống như P.

Lời giải: Xét 3 đoạn là AD, BE, và CF , gọi các đường cao của tamgiác là AHa, BHb, và CHc.Lấy đối xứng các đoạn thẳng ở trên qua các đường cao tương ứng tathu được các đoạn AD

,, BE,, và CF ,. Nếu AD, không nằm trong tam

giác thì ta suy ra được là AB hoặc AC phải nằm trong 4ADD,. Thếnhưng điều này không xảy ra vì AD = AD, = λ < min(AB,AC).

Do vậy AD, phải nằm trong 4 ABC, tương tự như vậy BE

,, và CF ,

cũng phải nằm ở trong 4 ABC.Bây giờ ta dùng độ dài hình học. Chú ý rằng

BD.BD, = (BHa+HaD)(BHa−HaD) = BH2a = (AB2−AH2

a)−(AD2−AH2a)

= AB2 − AD2 = AB2 − λ2 (8.1)

Tương tự như vậy ta có EA.EA, = AB2 − λ2

Thật vậy, EA.EA, = BD.B,D. Như vậy FB.F ,B = CE.CE, vàDC.D,C = AF.A,F

vnmath.com

Page 57: OLYMPIC TOÁN NĂM 2000 52 ĐỀ THI VÀ LỜI …»i nói đầu Để thử gói lệnh lamdethi.sty tôi biên soạn một số đề toán thi Olympic, mà các học trò của

Đề thi olympic India 57

Bây giờ áp dụng định lý Ceva cho 3 đương đồng quy AD, BE, và CFta có BD

DC.CEEA.AFFB

= 1

Do vậy

BD,

D,C.CE,

E,A.AF ,

F ,B= (

BD,BD

D,C.DC).(CE,CE

E,A.EA).(AF ,.AF

F ,B.FB)

= (BD,.BD

E,A.EA)(CE,CE

F ,B.FB)(AF ,.AF

D,C.DC) = 1 (8.2)

Nhưng theo định lý Ceva AD,, BE,, và CF , đòng quy tại điểm P , nằmngoài 4 ABC. Nếu P trùng với P , thì P sẽ là trực tâm của tam giác,nhưng nếu vậy thì theo giả thiết 3 đường cao của 4 ABC có độ dàibằng nhau và bằng λ.. Điều này là vô lý, vì 4 ABC không đều. Do đóP , 6= P , và các đoạn thẳng qua P , đó có độ dài bằng nhau và bằng λ.

.8.50.Cho m, n là các số nguyên dương thỏa mãn m ≤ n2

4và mọi ước nguyên

tố của m đều nhỏ hơn hoặc bằng n. Hãy chứng minh rằng m là ước của

n!

Lời giải: Ta chỉ chỉ cần chứng mỉnh rằng với mọi ước nguyên tố p củam và với mọi số nguyên k ≥ 1 mà P k|m thì pk|n! (1)Thật vậy nếu k = 1 thấy ngay (1)đúng vì p ≤ n. và do vậy p|n! . Mặtkhác vì m ≤ n2

4nên ta có pk ≤ n2

4. hay n ≥ 2

√pk.

Ta thấy, nếu n ≥ kp thì ít nhất k số thuộc vào tập 1, 2, ....., n là bộicủa p, kéo theo pk|n!.

Để chứng minh n ≥ kp ta đi chứng minh 2

√pk

p≥ k ⇔ p

k−2

2 ≥ k2

(*) (Vìkhi đó n ≥ 2

√pk ≥ kp)

Đi chứng minh (*): Với k = 2 thì (*) ⇔ 1 ≥ 1 (đúng). Nếu k ≥ 4 thì ápdụng BĐT Bernoulli ta có p

k−2

2 = (1+(p−1))k−2

2 ≥ 1+ k−22.(p−1) ≥ k

2.

Cuối cùng, nếu k = 3 thì (*) luôn đúng ngoại trừ p = 2. Nếu k = 3 vàp = 2 thì m ≥ 8;n ≥ 5 và n! thực sự chia hết cho 8 (8 = 23), do vậy tacó pk là ước của n! trong mọi trường hợp, bài toán được chứng minh.

.8.51.Cho G là một hình với n đỉnh với n ≥ 4 và m cạnh. Chứng minh rằng

nếu m > n(√

4n− 3 + 1)/4 thì G chứa một 4- Chu trình.

Lời giải: Chúng ta đếm số các bộ 3 các khoảng cách giữa các đỉnh (v, a,b) sao cho v là cạnh kề cả a và b. Số các bộ như thế với mỗi v cố định là

vnmath.com

Page 58: OLYMPIC TOÁN NĂM 2000 52 ĐỀ THI VÀ LỜI …»i nói đầu Để thử gói lệnh lamdethi.sty tôi biên soạn một số đề toán thi Olympic, mà các học trò của

58 Nguyễn Hữu Điển, ĐHKHTN Hà Nội

deg(v).(deg(v)-1). Vì tổng các deg(v) lấy theo tất cả các v là 2m, và x(x-1) là hàm lồi theo x, bất đẳng thức Jensen cho ta tổng deg(v)(deg(v)-1)lấy theo tất cả các v bé nhất là n.(2m/n)((2m/n)−1) = 2m, (2m/n−1).Nếu G không có 4-chu trình nào, thì với bất kỳ a, b cố định nào có nhiềunhất một đỉnh kề với cả a và b. Kéo theo có nhiều nhất n(n-1) bộ 3 đãnói ở trên. Do vậy, có một 4- chu trình nếu 2m, (2m/n−1) > n(n−1) ⇔4m2 − (2n)m− n2(n− 1) > 0. Bất đẳng thức này luôn đúng đối với mlớn hơn nghiệm lớn của phương trình 4x2 − (2n)x− n2(n− 1) Điều nàycó nghĩa là G có 4-chu trình nếu như m lớn hơn nghiệm lớn của phươngtrình bậc 2 trên, hay m > 2n+

√4n2+16n3−16n2

8⇔ m > n.

√4n−3+1

4. Từ giả

thiết cho ta điều phải chứng minh.

.8.52.Cho hàm f : Q→ {0, 1} thỏa mãn f(x) = f(y) thì f(x) = f(x+y2

) = f(y)

với ∀ x, y ∈ Q,

Nếu f(0)=0 và f(1)=1 hãy chứng minh f(q) = 1 với ∀q ∈ Q

Lời giải: Trước tiên ra chứng minh bổ đề sau: Cho a, b là các sô hữu

tỷ. Nếu f(a) 6= f(b) thì f(n(b - a) + a) = f(b) với mọi n nguyên dương

(1)Thật vậy, ta sẽ chứng minh bổ đề trên bằng quy nạp.

• Với n = 1 (1) hiển nhiên đúng• Giả sử mệnh đề đúng với n ≤ k.

Đặt (x1, y1, x2, y2) = (b, k(b − a) + a, a, (k + 1)(b − a) + a). Theogiả thiết quy nạp, f(x1) = f(y1). Ta cần cần chứng minh f(x2) 6=f(y2). Thật vậy giả swr f(x2) = f(y2) khi đó xét (x, y) = (x1, y1) và(x, y) = (x2, y2) vào trong điều kiện đề bài, ta có f(b) = f(x1+y1

2),

và f(a) = f(x2+y22

) . Tuy nhiên điều này không xảy ra vì x1 + y1 =

x2 + y2. Do đó f(y2) 6= f(x2) hay f(y2) 6= f(a).Vậy f(y2) = f(b). chứng minh xong bổ đề.Áp dụng bổ đề trên với a = 0 và b = 1 ta thấy rằng f(n)=1 vớimọi n nguyên dương. Tiếp theo ta thấy f(1 + r

s) 6= 0 ∀r, s ∈ N ,

vì nếu ngược lại, áp dụng bổ đề với a = 1, b =1 + rs

và n = s thìf(1+r) = 0 mâu thuẫn với f(n) = 1 ở trên. Do vậy, f(q) = 1 với mọisố hữu tỷ q ≥ 1.

vnmath.com

Page 59: OLYMPIC TOÁN NĂM 2000 52 ĐỀ THI VÀ LỜI …»i nói đầu Để thử gói lệnh lamdethi.sty tôi biên soạn một số đề toán thi Olympic, mà các học trò của

Tài liệu tham khảo

[1] Titu Andreescu, Zuming Feng, and George Lee, Jr. Mathematical

Olympiads 2000–2001, Problems and Solutions From Around the World,The Mathematical Association of America, 2002.

[2] Nguyễn Hữu Điển, Phương pháp Đirichle và ứng dụng, NXBKHKT, 1999.

[3] Nguyễn Hữu Điển, Phương pháp Quy nạp toán học, NXBGD, 2000.

[4] Nguyễn Hữu Điển, Những phương pháp điển hình trong giải toán phổ

thông, NXBGD, 2001.

[5] Nguyễn Hữu Điển, Những phương pháp giải bài toán cực trị trong hình

học, NXBKHKT, 2001.

[6] Nguyễn Hữu Điển, Sáng tạo trong giải toán phổ thông, NXBGD, 2002.

[7] Nguyễn Hữu Điển, Đa thức và ứng dụng, NXBGD, 2003.

[8] Nguyễn Hữu Điển, Giải phương trình vô định nghiệm nguyên,NXBĐHQG, 2004.

[9] Nguyễn Hữu Điển, Giải toán bằng phương pháp đại lượng bất biến,NXBGD, 2004.

vnmath.com

Page 60: OLYMPIC TOÁN NĂM 2000 52 ĐỀ THI VÀ LỜI …»i nói đầu Để thử gói lệnh lamdethi.sty tôi biên soạn một số đề toán thi Olympic, mà các học trò của

Nguyễn Hữu Điển

OLYMPIC TOÁN NĂM 2000

49 ĐỀ THI VÀ LỜI GIẢI(Tập 2)

NHÀ XUẤT BẢN GIÁO DỤC

vnmath.com

Page 61: OLYMPIC TOÁN NĂM 2000 52 ĐỀ THI VÀ LỜI …»i nói đầu Để thử gói lệnh lamdethi.sty tôi biên soạn một số đề toán thi Olympic, mà các học trò của

2

vnmath.com

Page 62: OLYMPIC TOÁN NĂM 2000 52 ĐỀ THI VÀ LỜI …»i nói đầu Để thử gói lệnh lamdethi.sty tôi biên soạn một số đề toán thi Olympic, mà các học trò của

Lời nói đầu

Để thử gói lệnh lamdethi.sty tôi biên soạn một số đề toán thi Olympic,mà các học trò của tôi đã làm bài tập khi học tập LATEX. Để phụ vụ các bạnham học toán tôi thu thập và gom lại thành các sách điện tử, các bạn có thểtham khảo. Mỗi tập tôi sẽ gom khoảng 50 bài với lời giải. Tập này có sự đónggóp của Trịnh Quang Anh, Nguyễn Thị Bình, Nguyễn Thị Thanh Bình, Đàothị Kim Cúc, Nguyễn Hoàng Cương, Giáp Thị Thùy Dung, Mai Xuân Đông,Hoàng Hà, Nguyễn Thị Thanh Hà.

Rất nhiều bài toán dịch không được chuẩn, nhiều điểm không hoàn toànchính xác vậy mong bạn đọc tự ngẫm nghĩ và tìm hiểu lấy. Nhưng đây là nguồntài liệu tiếng Việt về chủ đề này, tôi đã có xem qua và người dịch là chuyên vềngành Toán phổ thông. Bạn có thể tham khảo lại trong [1].

Rất nhiều đoạn vì mới học TeX nên cấu trúc và bố trí còn xấu, tôi khôngcó thời gian sửa lại, mong các bạn thông cảm.

Hà Nội, ngày 2 tháng 1 năm 2010

Nguyễn Hữu Điển

51GD-05

89/176-05 Mã số: 8I092M5

vnmath.com

Page 63: OLYMPIC TOÁN NĂM 2000 52 ĐỀ THI VÀ LỜI …»i nói đầu Để thử gói lệnh lamdethi.sty tôi biên soạn một số đề toán thi Olympic, mà các học trò của

Mục lục

Lời nói đầu . . . . . . . . . . . . . . . . . . . . . . . . . . . . . . . . . . . . . . . . . . . . 3

Mục lục . . . . . . . . . . . . . . . . . . . . . . . . . . . . . . . . . . . . . . . . . . . . . . . 4

Chương 1. Đề thi olympic Israel. . . . . . . . . . . . . . . . . . . . . . . . . . . . 5

Chương 2. Đề thi olympic Italy . . . . . . . . . . . . . . . . . . . . . . . . . . . . 9

Chương 3. Đề thi olympic Nhật Bản . . . . . . . . . . . . . . . . . . . . . . 14

Chương 4. Đề thi olympic Korea . . . . . . . . . . . . . . . . . . . . . . . . . . 18

Chương 5. Đề thi olympic Mông cổ . . . . . . . . . . . . . . . . . . . . . . . 24

Chương 6. Đề thi olympic Rumani . . . . . . . . . . . . . . . . . . . . . . . . 32

Chương 7. Đề thi olympic Nước Nga . . . . . . . . . . . . . . . . . . . . . . 39

Chương 8. Đề thi olympic Đài Loan . . . . . . . . . . . . . . . . . . . . . . . 45

Chương 9. Đề thi olympic Thổ Nhĩ Kỳ. . . . . . . . . . . . . . . . . . . . 50

vnmath.com

Page 64: OLYMPIC TOÁN NĂM 2000 52 ĐỀ THI VÀ LỜI …»i nói đầu Để thử gói lệnh lamdethi.sty tôi biên soạn một số đề toán thi Olympic, mà các học trò của

Chương 1

Đề thi olympic Israel

.1.1. Định nghĩa f(n) = n!. Cho

a = 0.f(1)f(2)f(3)....

Nói cách khác, để thu được sự biểu diễn phần thập phân của a viết các

biểu diễn thập phân của f(1), f(2)., ... trong một hàng, a có phải là số

hữu tỷ không?

Lời giải: Nếu a là số hữu tỷ thì các con số trong phần thập phân phảixuất hiện một cách tuần hoàn. Vì f(n) luôn bao gồm một số khác không,nên phần tuần hoàn của phần thập phân không thể chỉ bao gồm toànsố không. Tuy nhiên, n đủ lớn, số các số 0 chưa trong f(n) tiến tới vôcùng, vì vậy phần tuần hoàn của phần thập phân phải chứa toàn số 0 –mâu thuẫn. Vì vậy a không là số hữu tỷ.

.1.2. . ∆ ABC đỉnh là những điểm nguyên. Hai trong ba cạnh có độ dài thuộc

tập√

17,√

1999,√

2000 . Tìm giá trị lớn nhất có thể của diện tích

∆ABC.

Lời giải: Không mất tổng quát, giả sử cạnh AB, BC có độ dài thuộc√17,

√1999,

√2000 thì

SABC = 12AB.BC sin BCA≤1

2

√2000

√2000 sin π

2= 1000.

vnmath.com

Page 65: OLYMPIC TOÁN NĂM 2000 52 ĐỀ THI VÀ LỜI …»i nói đầu Để thử gói lệnh lamdethi.sty tôi biên soạn một số đề toán thi Olympic, mà các học trò của

6 Nguyễn Hữu Điển, ĐHKHTN Hà Nội

Đẳng thức có thể xảy ra, chẳng hạn trong ∆ mà đỉnh là (0,0); (44,8) và(-8, 44) chính xác 2 cạnh dài

√2000 vì 442 + 82 = 2000 và góc giữa 2

cạnh là π2. Từ đó, diện tích lớn nhất của ∆ là 1000.

.1.3. Bài toán 3.Các điểm A, B, C, D, E, F nằm trên 1 đường tròn và các

đường thẳng AD, BE, CF đồng quy. Lấy P, Q, R là các trung điểm cạnh

AD, BC, CF tương ứng. 2 đoạn (dây cung) AG, AH được vẽ sao cho AG

// BE và AH//CF chứng minh rằng ∆ PQR và ∆ DGH đồng dạng.

Lời giải: Các góc định hướng môđun π. Giả sử đoạn thẳng AD, BE,CF đồng quy (cắt nhau) tại X và O là tâm đường tròn cho ở bài.Hiển nhiên OPX = OQX = ORX = π

2, suy ra O, P, Q, R và X cùng

thuộc 1 đường tròn.Vì vậy DGH = DAH = DXC = π− CXP = π− RXP = PQR Tươngtự DGH = PRQ, từ đó suy ra ∆PQR ∼ ∆DGH.

.1.4. Một hình vuông ABCD cho trước, một phép đạc tam giác của hình vuông

là 1 sự phân chia hình và thành các tam giác sao cho bất kỳ 2 tam giác

đều được tách rời, chỉ chung 1 đỉnh hoặc chung nhau chỉ 1 cạnh cụ thể.

Không đỉnh nào của 1 tam giác có thể nằm ở phần trong của cạnh tam

giác khác). Một “phép đạc tam giác tốt” của 1 hình vuông là phép đạc

trong đó mọi tam giác đều nhọn.

a. Cho 1 ví dụ về phép đạc tam giác tốt của hình vuông.

b. Tìm số nhỏ nhất của các tam giác cần để có một phép đạc tam giác

tốt?

Lời giải: Ta đưa ra 1 ví dụ về phép đạc tam giác tốt với 8 tam giác.Đặt hướng hình vuông sao cho đoạn AB đặt nằm ngang và A là đỉnhtrên bên trái. Lấy M và N là các trung điểm cạnh AB và CD tương ứng,và P là 1 điểm trung đoạn MN khác trung điểm MN. Các góc MPA,APD và DPN và các góc phản xạ của chúng qua MN - tất cả đều làcác góc nhọn. Ta chọn Q, R trên đường thẳng nằm ngang qua P sao choQ, P, R nằm theo thứ tự từ trái qua phải và QP, PR có độ dài rất nhỏ(không đáng kể) chia hình vuông thành các ∆ bằng cách vẽ đoạn QA,QM, QN, QD, RB, RM, RN, RC và QR. Nếu ta chọn Q sao cho PQ đủnhỏ thì số đo các góc MQA, AQD, DQN sẽ gần bằng số đo góc MPA,

vnmath.com

Page 66: OLYMPIC TOÁN NĂM 2000 52 ĐỀ THI VÀ LỜI …»i nói đầu Để thử gói lệnh lamdethi.sty tôi biên soạn một số đề toán thi Olympic, mà các học trò của

Đề thi olympic Israel 7

APD, DPN , vì vậy những tam giác này sẽ nhọn.Tương tự, nếu chọn R sao cho PR đủ nhỏ thì MRB, BRC, CRN sẽcùng nhọn. Dễ kiểm tra rằng các góc trong sự phân chia trên là nhọnnhư yêu cầu.b.Ta sẽ chứng minh số nhỏ nhất là 8. Ta đã chỉ ra rằng 8 là giá trị cóthể thực hiện được. Vì vậy, chỉ cần chỉ ra những phép đạc tam giác tốtnào với ít hơn 8 tam giác. Nhận xét rằng trong 1 phép đạc tam giác tốt,mỗi đỉnh của ABCD là đỉnh của ít nhất 2 tam giác bởi vì góc vuông đóphải được chia thành các góc nhọn. Như vậy, bất kỳ đỉnh nào nằm trêncạnh ABCD phải là đỉnh của ít nhất 3 tam giác và bất kỳ đỉnh nằm ởphần trong phải là đỉnh của ít nhất 5 tam giác.Tóm lại, ta có thể chứng minh một kết quả mạnh hơn về mỗi góc củahình vuông ABCD. Phải có một tam giác mà cạnh bắt đầu từ đỉnh hìnhvuông và điểm cuối nằm trọn ở phần trong hình vuông ABCD. Khôngmất tổng quát, giả sử góc (đỉnh) đó là A. Cạnh AX nào đó của tam giácchia góc vuông tại A ra.Giả sử phản chứng rằng X không nằm ở phầntrong hình vuông ABCD, không mất tổng quát, giả sử X thuộc đoạnBC (không trùng B). Bằng định nghĩa của phép đạc tam giác : không cóđỉnh khác của một tam giác trong phép đạc tam giác nằm trên đoạn AX.Vì vậy, có 1 điểm Y trong ∆ABX sao cho ∆AXY là một thành phần Cphần tử của phép đạc tam giác tốt. Nhưng nếu vậy AY X ≥ ABX = π

2

: mâu thuẫn.Ta xét 1 phép đạc tam giác tốt bất kỳ của ABCD. Lấy i là số của “cácđỉnh trong” – các đỉnh trong phép đạc tam giác mà nằm bên trong hìnhvuông ABCD. Theo trên i ≥ 1. Trước tiên giả sử rằng có một đỉnh trongP. Kết quả của đoạn trước cho ta: đoạn PA, PB, PC, PD phải là cáccạnh của các tam giác trong phép đạc tam giác. Một trong góc APB,BPC, CPD, DPA phải lớn hơn π

2giả sử là APB. Góc này phải được

chia ra trong phép đạc tam giác này bằng cạnh PQ nào đó, với Q làđiểm thuộc đoạn AB. Nhưng cả AQP và BQP có số đo ít nhất là π

2nên

Q phải nằm trong cạnh của tam giác nào đó mà không nằm trong đoạnQA, QB hoặc QP. Tuy nhiên không thể tạo được một cạnh mà khôngcắt AP hoặc BP và cạnh đó không kết thúc ở một đỉnh trong thứ hai.

vnmath.com

Page 67: OLYMPIC TOÁN NĂM 2000 52 ĐỀ THI VÀ LỜI …»i nói đầu Để thử gói lệnh lamdethi.sty tôi biên soạn một số đề toán thi Olympic, mà các học trò của

8 Nguyễn Hữu Điển, ĐHKHTN Hà Nội

Giả sử tiếp i ≥ 2. Với mỗi một n các tam giác, ta có thể đếm 3 cạnhđể có tổng 3n; mỗi cạnh nằm trên biên hình vuông được đếm 1 lần, cáccạnh khác được đếm hai lần. Nếu i = 2 thì với mỗi 2 điểm trong ít nhất5 cạnh tam giác nhận điểm đó làm điểm cuối, nhiều nhất 1 cạnh tamgiác chứa cả hai đỉnh trong, nên ít nhất 9 cạnh tam giác không nằm ởbiên của hình vuông. Nếu i ≥ 3, lấy bất kỳ 3 đỉnh trong. Mỗi đỉnh thuộcít nhất 5 cạnh tam giác và nhiều nhất 3 cạnh tam giác chứa 2 trong 3đỉnh đó. Vì vậy ít nhất 3 x 5 – 3 = 12 cạnh tam giác. Không thuộc biênhình vuông. Trong cả hai trường hợp đều có ít nhất 9 cạnh tam giáckhông thuộc biên hình vuông, và hơn nữa lại có 4 cạnh tam giác thuộcbiên hình vuông. Vì vậy 3n≥9 x 2 + 4 = 22 hay n ≥ 8. Vì vậy trongmọi trường hợp phải có ít nhất 8 tam giác thoả mãn yêu cầu.

vnmath.com

Page 68: OLYMPIC TOÁN NĂM 2000 52 ĐỀ THI VÀ LỜI …»i nói đầu Để thử gói lệnh lamdethi.sty tôi biên soạn một số đề toán thi Olympic, mà các học trò của

Chương 2

Đề thi olympic Italy

.2.5. Giả sử ABCD là một tứ giác lồi, với α = ∠DAB; β = ∠ACB; δ =

∠DBC; và ε = ∠DBA. Giả thiết rằng α < π/2, β+γ = π/2 và δ+2ε =

π, chứng minh rằng

(DB + BC)2 = AD2 + AC2.

A D

B

D′

C

α β

γδε

ε

β

Lời giải: Giả sử D′ là điểm đối xứng của D qua đường thẳng AB. Tacó ∠D′BA = ∠DBA = ε, nên ∠D′BC = ∠D′BA+∠ABD +∠DBC =

2ε + δ = π. Vậy, D′, B, C là thẳng hàng. Cũng có ∠AD′C + ∠ACD′ =

∠ADB + ∠ACB = β + γ = π/2, nên ∠D′AC = π/2 và tam giác A′AC

vuông. Theo định lí Pythagorean, D′C = AD′2 + AC2, kéo theo

(DB + BC)2 = (D′B + BC)2 = D′C2 = AD′2 + AC2

= AD2 + AC2,

vnmath.com

Page 69: OLYMPIC TOÁN NĂM 2000 52 ĐỀ THI VÀ LỜI …»i nói đầu Để thử gói lệnh lamdethi.sty tôi biên soạn một số đề toán thi Olympic, mà các học trò của

10 Nguyễn Hữu Điển, ĐHKHTN Hà Nội

được điều phải chứng minh.

.2.6. Cho số nguyên cố định n > 1, Alberto và Barbara chơi trò chơi sau, bắt

đầu với bước đầu tiên và sau đó xen kẽ giữa lần thứ hai và lần thứ ba :

• Alberto chọn một số nguyên dương.

• Barbara chọn một số nguyên lớn hơn 1 là một bội hoặc ước của số

nguyên của Alberto, có thể chọn đúng là số nguyên của Alberto.

• Alberto cộng hoặc trừ 1 từ số của Barbara.

Barbara chiến thắng nếu cô ấy chọn ra n với 50 lần chơi. Với giá trị nào

của n cô ấy là người thắng cuộc.

Lời giải: Mục đích của chúng ta là Barbara là người thắng cuộc nếu vàchỉ nếu ít nhất là một điều kiện sau được thỏa mãn :

• n = 2;• 4| n ;• có số nguyên m > 1, sao cho (m2 − 1)| n.

Đầu tiên chúng ta chỉ ra rằng khi và chỉ khi ba điều kiện này là đúng,thì Barbara là người chiến thắng. Nếu Barbara chọn lần đầu tiên a làmột số chẵn thì Barbara có thể chọn 2 trong lần đàu tiên. Nếu thay a

bằng một số lẻ, thì Barbara có thể chọn chính là số a là tốt nhất. Nếua = n, cô ấy chiến thắng; nói cách khác, lần chọn thứ hai của Albertophải là số chẵn, và Barbara có thể chọn số 2 trong lần chọn thứ hai. Giảsử a1, b1, a2, b2, . . . là các số được chọn sau khi Barbara chọn 2 cho lầnchọn đầu tiên.Trường hợp 1 :

(a) n = 2, trong trường hợp này Barbara thực sự chiến thắng.(b) 4| n. Nếu a1 = 1, thì Barbara có thể chọn b1 = n và chiến thắng.

Nói cách khác, a1 = 3, Barbara có thể chọn b1 = 3, a2 bằng 2 hoặc4, và Barbara có thể chọn b2 = n.

(c) Có số nguyên m > 1, (m2 − 1)| n. Như trường hợp 2, Alberto phảichọn a1 = 3 để ngăn Barbara thắng cuộc. Thực tế, có đúng mộtsố nguyên trong các số m − 1, m và m + 1 chia hết cho 3, nghĩalà hoặc 3 chia hết m hoặc 3 chia hết m2 − 1và vì vị 3 chia hết n.Trong trường hợp đầu tiên, Barbara có thể chọn b1 = m, bắt buộca2 = m ± 1 và kéo theo Barbara chọn b2 = n. trong trường hợp

vnmath.com

Page 70: OLYMPIC TOÁN NĂM 2000 52 ĐỀ THI VÀ LỜI …»i nói đầu Để thử gói lệnh lamdethi.sty tôi biên soạn một số đề toán thi Olympic, mà các học trò của

Đề thi olympic Italy 11

sau, Barbara có thể chọn b1 = n.

Bây giờ chúng ta thấy rằng Barbara có một chiến thuật chiến thắng nếuít nhất một trong các điều kiện là đúng. Bây giờ chúng ta giả thiết rằngkhông điều kiện nào là đúng với n > 1 và chứng minh rằng Alberto cóthể luôn luôn ngăn cản Barbara tiến đến chiến thắng. Bởi vì điều kiệnthứ nhất và thứ hai là không đúng và bởi vì điều kiện thứ hai là sai vớim = 2, chúng ta có n 6= 2, 3, 4,. Vậy n > 4.Gọi một số nguyên dương n là số hy vọng nếu a|n và n|a. Chúng tachứng minh rằng với số nguyên b > 1, tồn tại a ∈ {b− 1, b + 1} sao choa là số hy vọng. Đêìu này kéo theo rằng Alberto có thể bắt đầu chọnmột vài số hy vọng và cũng chọn một vài số hy vọng theo sau để ngăncản Barbara tiến đến chiến thắng sau 50 lần.Giả sử vì điều kiện là mục đích trên là sai với số nguyên b > 1. Nếub > n, thì b− 1 và b + 1 phải là bội của n. Do đó n chia hết hiệu, tức là2, mâu thuẫn.Nói cách khác, b ≤ n. Bởi vì n không chia hết n+1 hoặc n+2 với n > 2,chúng ta phải có (b − 1)|n và (b + 1)|n. Nếu b − 1 và b + 1 là chẵn, thìmột trong chúng phải chia hết cho 4 - nhưng 4| n, mâu thuẫn. Vậy, b−1

và b+1 là lẻ. Điều này kéo theo chúng nguyên tố cùng nhau và tích củachúng b2 − 1 chia hết n, mâu thuẫn với giả thiết điều kiện thứ ba là sai.

.2.7. Giả sử p(x) là một đa thức với hệ số nguyên sao cho p(0) = 0 và 0 ≤p(1) ≤ 107, và sao cho tồn tại các số nguyên a, b thỏa mãn p(a) = 1999

và p(b) = 2001. Xác định các giá trị có thể của p(1).

Lời giải: Nếu p(x) = 2000x2 − x, thì p(0) = 0, p(1) = 1999, và p(−1) =

2001. Nếu p(x) = 2000x2+x, thì p(0) = 0, p(1) = 2001, và p(−1) = 1999.Do đó, có thể p(1) = 1999 hoặc 2001.Bây giờ giả sử rằng p(1) 6= 1999, 2001. Thì a, b 6= 1. Bởi vì p(0) = 0,chúng ta viết được p(x) = xq(x) với đa thức q(x) có hệ số nguyên. Bởivì q có hệ số nguyên, q(a) là một số nguyên, và có thể viết q(x)− q(a) =

(x−a)r(x) với đa thức r(x) có hệ số nguyên. Và bởi vì r có hệ số nguyên,r(b) là một số nguyên, và chúng ta có thể viết r(x)− r(b) = (x− b)s(x)

vnmath.com

Page 71: OLYMPIC TOÁN NĂM 2000 52 ĐỀ THI VÀ LỜI …»i nói đầu Để thử gói lệnh lamdethi.sty tôi biên soạn một số đề toán thi Olympic, mà các học trò của

12 Nguyễn Hữu Điển, ĐHKHTN Hà Nội

với đa thức s có hệ số nguyên. Do đó :

p(x) = xq(x) = xq(a) + x(x − a)r(x) (*)

= xq(a) + x(x − a)r(b) + x(x − a)(x − b)s(x).

Đặc biệt, khi cho x = a và x = b, chúng ta tìm được

1999 = aq(a)

2001 = bq(a) + b(b − a)r(b).

Bởi vì p(0), p(a) và p(b) là các số phân biệt, vì vậy 0, a và b cũng phânbiệt. Do đó, chúng ta có thể giải hai phương trình trên để tìm được

q(a) =1999

a(*)

r(b) =2001 − bq(a)

b(b − a).

Bởi vì a 6= b, chúng ta có |a − b| chi hết p(a)− p(b). Vì vậy |a − b| bằng1 hoặc 2. Cũng vậy, với mọi x ∈ Z, chúng ta có p(x) = xq(x) và vì vậyx| p(x). Đặc biệt, a| 1999, cho nên

|a| ∈ {1, 1999}.

Với hạn chế này, kết hợp với điều kiện |a − b| ∈ {1, 2}, b|2001, a 6= 1, vàb 6= 1, kéo theo rằng (a, b) bằng một trong các cặp sau :

(−1999,−2001), (−1,−3), (1999, 2001).

Cố định (a, b) là chung cho ba cặp trên. Từ (*) chúng ta biết rằng q(a)

phải bằng q =1999

avà r(b) phải bằng r =

2001 − bq

b(b − a). Cho x = 1 và (*)

để tìm p(1) :

(a, b) q(a) r(a) p(1)

(−1999,−2001) −1 0 −1 + (2000.2002)s(1)

(−1,−3) −1999 −666 −3331 + 8s(1)

(1999, 2001) 1 0 1 + (1998.2000)s(1).

vnmath.com

Page 72: OLYMPIC TOÁN NĂM 2000 52 ĐỀ THI VÀ LỜI …»i nói đầu Để thử gói lệnh lamdethi.sty tôi biên soạn một số đề toán thi Olympic, mà các học trò của

Đề thi olympic Italy 13

Vì vậy, p(1) có dạng m + ns(1) với số nguyên cố định m, n. Thật vậy,giả sử rằng có một số dạng m + ns giữa 0 và 107, ở đât s là số nguyên.Chúng ta có

p(x) = qx + rx(x − a) + sx(x − a)(x − b),

chúng ta có p(0) = 0, p(a) = 1999, p(b) = 2001, và p(1) = m + ns.Do đó, các giá trị có thể của p(1) là 1999 và 2001, và các số giữa 0

và 107 đồng dư với −1 (mod 2000.2002), −3331 ≡ 5 (mod 8), hoặc1 (mod 1998.2000).

vnmath.com

Page 73: OLYMPIC TOÁN NĂM 2000 52 ĐỀ THI VÀ LỜI …»i nói đầu Để thử gói lệnh lamdethi.sty tôi biên soạn một số đề toán thi Olympic, mà các học trò của

Chương 3

Đề thi olympic Nhật Bản

.3.8. Ta tráo một loạt các lá bài đánh số a1, a2, . . . , a3n từ trái qua phải bằng

việc sắp xếp các lá bài theo thứ tự mới:

a3, a6, . . . , a3n, a2, a5, . . . a3n−1, a1, a4, . . . , a3n−2

Ví dụ nếu 6 lá bài được đánh số 1, 2, . . . , 6 từ trái qua phải thì việc tráo

chúng 2 lần sẽ thay đổi trật tự của chúng như sau:

1, 2, 3, 4, 5, 6 −→ 3, 6, 2, 5, 1, 4 −→ 2, 4, 6, 1, 3, 5

Bắt đầu với 192 quân bài dánh số 1, 2, . . . , 192 từ trái qua phải, liệu ta

có được trật tự 192, 191, . . . , 1 sau số lần tráo hữu hạn?

Lời giải: Với mỗi n, cho f(n) là vị trí trong chuỗi các quân bài ở đóquân bài đi vị trí thứ n sau mỗi lần tráo. Ta thấy rằng sau k lần tráo,fk(n) ở vị trí thứ n. Ta đã được biết rằng f(1), . . . , f(192) bằng 3, 6,. . . , 192, 2, 5, . . . , 191, 1, 4, . . . , 190. Trong trật tự này, sự khác biệtgiữa bất kỳ số hạng nào với số hạng đứng trước nó là đồng dư từ 3 tớimodul 193. Vì f(1) ≡ 3 (mod 193) ta có f(n) ≡ 3n (mod 193) với mỗin.Trong trật tự (33)20, (33)21, . . . , (33)26, với mỗi số hạng là bình phươngcủa số hạng trước nó. Ít nhất một số hạng trước nó (số hạng đầu tiên 2t)không đồng dư với một modul 193; giả sử N = 3d (ở đó d là số nguyên

vnmath.com

Page 74: OLYMPIC TOÁN NĂM 2000 52 ĐỀ THI VÀ LỜI …»i nói đầu Để thử gói lệnh lamdethi.sty tôi biên soạn một số đề toán thi Olympic, mà các học trò của

Đề thi olympic Nhật Bản 15

dương) là giá trị lớn nhất với thuộc tính của nó, vì 193 là số nguyêntố, theo định lý Fermat có: (33)26 ≡ (33)192 ≡ 1 (mod 193), do vậy 3d

không phải là số hạng cuối cùng trong trật tự này. Do vậy, N2 số hạngtiếp theo của N trong trật tự là đồng dư với 1 modul 193. Vì 193 chiađược cho N2 − 1 nhưng không chia được cho N - 1, nó phải chia hết cho(N2 − 1)(N − 1) = N + 1 = 3d + 1, có nghĩa là 3d ≡ −1 (mod 193) .Vớin = 1, 2, . . . , 193 ta có fd(n) ≡ 3dn ≡ −n (mod 193). Do vậyfd(n) = 193 − n, có nghĩa là trật tự 192, 191, . . . , 1 xuất hiện sau dlần tráo.Chú ý: Giá trị d tìm thấy ở trên thực tế là bằng 24. Số nguyên dươngk nhỏ nhất thỏa mãn 3k ≡ −1 (mod 193) là 8, có nghĩa là trật tự192, 191, . . . , 1 xuất hiện lần đầu tiên sau 8 lần tráo bài.

.3.9. Trong mặt phẳng cho các điểm phân biệt A, B, C, P, Q, không có 3

điểm nào thẳng hàng. Chứng minh rằng:

AB + BC + CA + PQ < AP + AQ + BP + BQ + CP + CQ.

Lời giải: Trong bài giải này, khi gọi một đa giác V1 . . . Vn là lồi nếuV1, . . . , Vn tạo thành một đa giác lồi trong trật tự đó. (Ví dụ nếu ta nóihình vuông ABCD là lồi thì ta không nói rằng ACBD là lồi.)Ta nói rằng điều kiện (a) cố định nếu tứ giác XYPQ là lồi với X, Y ∈{A, B, C}. Trong trường hợp này ta chứng minh bất đẳng thức cầnchứng minh là cố định. Không mất tính tổng quát, ta có thể giả thiếtrằng tứ giác ABPQ là lồi. Nếu AP giao với BQ tại O, thì bất đẳng thứctam giác cho ta AB ≤ AO + BO và PQ ≤ PO + QO. Cộng 2 bất đẳngthức này ta có:

AB ≤ AO + BO + OP + OQ = AP + BQ

Vì không có 3 trong số các điểm đã cho nào thẳng hàng nên bất đẳngthức tam giác cũng chỉ ra rằng BC < BP + PC và CA < CQ + QA

Cộng 3 bất đẳng thức cuối cùng này ta có kết quả cần chứng minh.Tiếp đến ta nói tới điều kiện (b) cố định, nếu xem X nằm trong tam giácYZM với sự hoán vị (X, Y, Z) của (A, B, C) và với M ∈ {P, Q}. Ta chứng

vnmath.com

Page 75: OLYMPIC TOÁN NĂM 2000 52 ĐỀ THI VÀ LỜI …»i nói đầu Để thử gói lệnh lamdethi.sty tôi biên soạn một số đề toán thi Olympic, mà các học trò của

16 Nguyễn Hữu Điển, ĐHKHTN Hà Nội

minh rằng bất đẳng thức cần chứng minh cố định trong trường hợp này.Không mất tính tổng quát, giả sử A nằm trong tam giác BCQ. Sơ đồchuyển điểm P tùy ý tới mỗi cạnh PB, PC là các hàm lồi ngặt, có nghĩa làP −→ PB +PC cũng là một hàm lồi ngặt. Do vậy, trên tất cả các điểmP hoặc trong tam giác BCQ hàm này chỉ đạt cực đại khi P trùng với B,C hoặc Q. Vậy nên: AB+AC < max{BB+BC, CB+CC, QB+QC} =

QB + QC cộng bất đẳng thức này với bất đẳng thức BC < BP + PCvà PQ < PA + QA, đã có ở bất đẳng thức tam giác ta được kết quảcần chứng minh.Do việc đổi tên các điểm, bao lồi của 5 điểm đã cho hoặc phải là tamgiácBC, hoặc ABP, hoặc APQ, hoặc tứ giác lồi ABCD, hoặc ABPQ,hoặc APBQ, hoặc ngũ giác lồi ABCPQ hoặc ABPCQ.Nếu tam giác ABC là bao lồi thì Q phải nằm phía trong một trong cáctam giác APB, BPC, CPA. Không mất đi tính khái quát giả thiết rằngQ nằm trong tam giác APB. Vì C không nằm bên trong tam giác APBnhưng nằm cùng phía đường AB so với Q, do vậy QC phải giao với mộttrong 2 đoạn thẳng AP và PB. Nếu QC giao với AP , thì tứ giác ACPQlà lồi và điều kiện (a) cố định; tương tự điều kiện (a) cố định nếu QC

giao với PB.Nếu tam giác ABP là bao lồi thì C phải nằm trong tam giác ABP vàđiều kiện (b) cố định.Nếu tam giác APQ là bao lồi thì ta có thể giả thiết C không gần hơnPQ so với B mà không mất di tính tổng quát. Vậy nên điều kiện (b) cốđịnh.Nếu tứ giác ABCP là bao lồi thì Q nằm trong tam giác APB hoặc trongCPB. Trong trường hợp đầu tứ giác BCPQ là lồi và trong trường hợpthứ hai tứ giác BAPQ là lồi. Vậy nên điều kiện (a) cố định.Nếu tứ giác lồi ABPQ, ngũ giác lồi ABCPQ hay ngũ giác lồi ABPCQlà bao lồi thì tứ giác ABPQ là lồi và điều kiện (a) cố định.Cuối cùng, nếu tứ giác lồi APBQ là bao lồi thì C hoặc nằm trong tamgiác ABP hoặc ABQ; cả trong 2 trường hợp điều kiện (b) cố định.Do vậy, trong tất cả các trường hợp, hoặc điều kiện (a) hoặc điều kiện(b) cố định, từ đó suy ra bất đẳng thức cần chứng minh là đúng.

vnmath.com

Page 76: OLYMPIC TOÁN NĂM 2000 52 ĐỀ THI VÀ LỜI …»i nói đầu Để thử gói lệnh lamdethi.sty tôi biên soạn một số đề toán thi Olympic, mà các học trò của

Đề thi olympic Nhật Bản 17

.3.10.Cho 1 số tự nhiên n ≥ 3, chứng minh rằng tồn tại 1 tập hợp An với 2

thuộc tính sau:

(i) An bao gồm n số tự nhiên riêng biệt.

(j) Với bất kỳ a ∈ An, tích số của tất cả các phần tử khác trong An có

số dư là 1 khi được chia bởi a.

Lời giải:Giả sử a1, a2, . . . , ak(vik ≥ 2) là các số nguyên riêng biệt lớn hơn 1 thỏamãn a1a2 . . . ak ≡ (mod ai) khi 1 ≤ i ≤ k. Giả sử ε ∈ {−1, 1} và xácđịnh aa+1 = a1a2 . . . ak−ε. Vì ak+1 ≥ 2ak − 1 > ak với tất cả các k, cácsố nguyên a1, a2, . . . , ak+1 vẫn là các số nguyên riêng biệt lớn hơn 1.Xem xét biểu thức

a1a2 . . . ai−1ai+1 . . . ak+1 ≡ ε (mod ai)

rõ ràng nó không đổi với i = k + 1. Với i < k nó không đổi vì

(a1a2 . . . ai−1ai+1 . . . ak)ak+1 ≡ (−1)(−ε) ≡ ε (mod ai)

Bắt đầu với các số a1 = 2, a2 = 3, ta áp dụng cách này n-3 lần tập hợpε = −1 và một lần tập hợp ε = 1. Tập hợp An bao gồm các số kết quảlà a1, a2, . . . , an do đó thỏa mãn điều kiện đầu bài.

vnmath.com

Page 77: OLYMPIC TOÁN NĂM 2000 52 ĐỀ THI VÀ LỜI …»i nói đầu Để thử gói lệnh lamdethi.sty tôi biên soạn một số đề toán thi Olympic, mà các học trò của

Chương 4

Đề thi olympic Korea

.4.11.Chỉ ra rằng với mọi số nguyên tố cho trước p thì tồn tại những số tự

nhiên x, y, z, ω thoả mãn x2 + y2 + z2 − ω.p = 0 và 0 < ω < p.

Lời giải: Với trường hợp p = 2, ta có thể lấy x = 0, y = z = ω = 1.Bây giờ ta xét trường hợp p > 2. Trước tiên ta xét trường hợp −1 là đồngdư bình phương modun p, khi đó tồn tại một số tự nhiên a, 0 < a < p−1

sao cho a2 ≡ −1(modp). Bộ (x, y, z) = (0, 1, a). Vì x2 + y2 + z2 = a2 + 1

chia hết cho p nhưng 1 + (p − 1)2 < p2 nên tồn tại ω ∈ {1, 2, ..., p − 1}sao cho x2 + y2 + z2 − ω.p = 0 .Tiếp theo, giả sử (−1) không là đồng dư bình phương modunp. Ta phảitìm một số k nào đó để cả k và p − k − 1 đều là đồng dư bình phương.Nếu p−1

2là đồng dư bình phương thì chọn k = p−1

2. Nếu ngược lại, thì

mỗi đồng dư trong số p−12

các đồng dư bình phương khác không sẽ rơivào trong các cặp {1, p − 2} , {2, p − 3} , ...,

{p−32

},{

p+12

}. Theo nguyên

lý Pigeonhole Principle sẽ có một cặp (k, p − k − 1) mà cả hai số k và(p − k − 1) đều là đồng dư bình phương như ta đã định tìm.Vì vậy, ta có thể chọn x, y ∈

{0, 1, ..., p−1

2

}sao cho x2 ≡ k(modp) và

y2 ≡ p− k − 1(modp). Cho z = −1, ta có x2 + y2 + z2 chia hết cho p vàx2 + y2 + z2 < p2. Giá trị ω sẽ được xác định như ở trường hợp trước.

vnmath.com

Page 78: OLYMPIC TOÁN NĂM 2000 52 ĐỀ THI VÀ LỜI …»i nói đầu Để thử gói lệnh lamdethi.sty tôi biên soạn một số đề toán thi Olympic, mà các học trò của

Đề thi olympic Korea 19

.4.12.Tìm tất cả các hàm f : R → R thoả mãn

f(x2 − y2) = (x − y) [f(x) + f(y)]

với mọi x, y ∈ R.

Lời giải: Cho x = y, ta được f(0) = 0.Cho x = −1, y = 0 ta được f(1) = −f(−1).Cho x = a, y = 1, sau đó cho x = a, y = −1 ta có:

f(a2 − 1) = (a − 1) [f(a) + f(1)]

f(a2 − 1) = (a + 1) [f(a) − f(1)]

Cho các vế phải của các phương trình đó bằng nhau và giải phương trìnhđối với f(a) ta được f(a) = f(1).a với mọi a.Như vậy, mọi hàm số nào thoả mãn ràng buộc đã cho phải có dạngf(x) = kx với hằng số k nào đó. Ngược lại, bất kỳ hàm số nào có dạngf(x) = kx với hằng số k nào đó rõ ràng đều thoả mãn yêu cầu bài toán.

.4.13.Cho tứ giác lồi ABCD là tứ giác nội tiếp. Gọi P, Q, R, S lần lượt là các

giao điểm của hai đường phân giác ngoài các góc ABD và ADB, DAB

và DBA, ACD và ADC, DAC và DCA tương ứng. Chứng minh rằng

bốn điểm P, Q, R, S cùng nằm trên một đường tròn.

A

B

C

D

P

vnmath.com

Page 79: OLYMPIC TOÁN NĂM 2000 52 ĐỀ THI VÀ LỜI …»i nói đầu Để thử gói lệnh lamdethi.sty tôi biên soạn một số đề toán thi Olympic, mà các học trò của

20 Nguyễn Hữu Điển, ĐHKHTN Hà Nội

Lời giải: Các góc xét đến đều là các góc định hướng ngoại trừ cáctrường hợp nói khác đi.Giả sử chúng ta có một tam giác tuỳ ý XY Z với tâm đường tròn nộitiếp là điểm I và tâm đường tròn bàng tiếp IX đối diện với góc X. Suyra X, I, IX thẳng hàng. Ta có IY IX=π

2= IZIX vì vậy tứ giác IY IXZ là

nội tiếp được và XIXY =IIXY =IZY hay Y IXX=Y ZI.

X

Y Z

IX

I

Gọi I1, I2 lần lượt là tâm đường tròn nội tiếp các tam giác ABD và tamgiác ACD.Từ giả thiết ta suy ra P , Q là các tâm đường tròn bàng tiếp của tamgiác ABD đối diện với góc A và góc D, tương tự R, S là các tâm đườngtròn bàng tiếp của tam giác ACD đối diện với góc A và góc D.Áp dụng kết quả của phần trên với (X, Y, Z, IX) là (A, D, B, P ),(D, A, B, Q), (A, D, C, R) và (D, A, C, S) ta được, APD=I1BD,AQD=ABI1, ARD=I2CD, và ASD=ACI2.Khi coi các góc sau là không định hướng, ta thấy I1BD, ABI1, I2CD

và ACI2 đều bằng AQD

2= ACD

2.

Hơn nữa, các góc trên đều cùng một hướng, nên nếu coi chúng là nhữnggóc định hướng, chúng sẽ bằng nhau. Như vậy (trở lại với những góc địnhhướng) ta có: APD = AQD = ARD = ASD và bốn điểm P, Q, R, S

cùng nằm trên cung tròn trương bởi A, D.

.4.14.Cho p là một số nguyên tố sao cho p ≡ 1 (mod4). Hãy tính

p−1∑

k=1

([2k2

p

]

− 2

[k2

p

])

vnmath.com

Page 80: OLYMPIC TOÁN NĂM 2000 52 ĐỀ THI VÀ LỜI …»i nói đầu Để thử gói lệnh lamdethi.sty tôi biên soạn một số đề toán thi Olympic, mà các học trò của

Đề thi olympic Korea 21

Lời giải: Với mỗi số thực x, đặt

{x} = x − [x] ∈ [0, 1).

Ta có [2k2

p

]

=2k2

p−

{2k2

p

}

và [k2

p

]

=k2

p−

{k2

p

}

Ta được[2k2

p

]

− 2

[k2

p

]

= 2

{k2

p

}

−{

2k2

p

}

Nếu {x} < 12

thì 2 {x} − {2x} = {x} − 2 {x} = 0

Nếu {x} ≥ 12

thì 2 {x} − {2x} = 2{x} − (2 {x} − 1) = 1

Như vậy, tổng cần tính trong bài ra sẽ bằng α là số các phần tử k trong[1, p − 1] sao cho

{k2

p

}

≥ 12, hay bằng với số đồng dư k khác không mà

k2 là đồng dư mô đun p với một số nào đó trong[

p+12

, p − 1].

Vì p là số nguyên tố đồng dư với 1 mô đun p, ta đã biết −1 ≡ d2(modp),với d là một số nào đó. Phân chia các đồng dư mô đun p khác khôngthành p−1

2cặp dạng {a, da} sao cho a2 ≡ −(da)2(modp).

Vì vậy, có đúng một đồng dư trong mỗi cặp mà bình phương của nóđồng dư với một số nào đó trong

[p−12

, p − 1], và có tất cả p−1

2đồng dư

như thế.Từ đó suy ra tổng đã cho bằng p−1

2.

.4.15.Xét những hình L sau đây, mỗi hình được tạo bởi bốn hình vuông đơn

vị ghép lại.

vnmath.com

Page 81: OLYMPIC TOÁN NĂM 2000 52 ĐỀ THI VÀ LỜI …»i nói đầu Để thử gói lệnh lamdethi.sty tôi biên soạn một số đề toán thi Olympic, mà các học trò của

22 Nguyễn Hữu Điển, ĐHKHTN Hà Nội

Cho m và n là các số tự nhiên lớn hơn 1. Chứng minh rằng một hình

chữ nhật kích thước mxn sẽ được xếp bởi các hình đã cho khi và chỉ khi

m.n là bội số của 8.

Lời giải: Trước tiên ta chứng minh rằng nếu 8\mn, thì hình chữ nhậtmxn có thể được xếp bởi các hình đã cho.Trường hợp 1: Cả m và n đều là số chẵn. Không mất tính tổng quát tagiả sử rằng 4\m, 2\n. Hai hình đã cho có thể ghép được một hình chữnhật kích thước 4x2, và m.n/8 hình chữ nhật như vậy sẽ ghép thànhmột hình chữ nhật kích thước mxn (gồm n/2 hàng và m/4 cột).Trường hợp 2: Hoặc m hoặc n lẻ. Không mất tính tổng quát, ta giả sửrằng m là số lẻ. Khi đó 8\n. Vì m > 1 nên m ≥ 3. Ta có thể ghép đượcmột hình kích thước 3x8 như hình vẽ sau:

Những hình 3x8 như vậy có thể ghép thành hình chữ nhật (3xn).Nếu m = 3, ta đã ghép xong. Trong trườg hợp ngược lại, m > 3, thìphần còn lại (m−3)xn có thể ghép như trong trường hợp 1 vì 2\(m−3).Bây giờ ta sẽ chứng minh rằng: nếu hình chữ nhật có kích thước (mxn)

được ghép bởi các hình trên thì 8\m.n. Vì mỗi một hình L có diện tíchlà 4 nên 4\(m.n). Không mất tính tổng quát, giả sử 2\n, và tô m hàngtrong hình chữ nhật mxn thành hai màu đen trắng cạnh nhau. Mỗimảnh hình chữ L trong hình chữ nhật được ghép sẽ gồm một số lẻ ô đenhình vuông. Vì có tất cả 1 số chẵn (nx

[m2

]) ô vuông màu đen, nên hình

chữ nhật được ghép chứa 1 số chẵn các hình chữ L, mà ta đặt số đó là2k. Như vậy m.n = 8k, hay 8\mn.

.4.16.Cho những số thực a, b, c, x, y, z thoả mãn a ≥ b ≥ c > 0 và x ≥ y ≥z > 0.

Chứng minh rằng:

a2x2

(by + cz)(bz + cy)+

b2y2

(cz + ax)(cx + az)+

c2z2

(ax + by)(ay + bx)≥ 3

4

vnmath.com

Page 82: OLYMPIC TOÁN NĂM 2000 52 ĐỀ THI VÀ LỜI …»i nói đầu Để thử gói lệnh lamdethi.sty tôi biên soạn một số đề toán thi Olympic, mà các học trò của

Đề thi olympic Korea 23

Lời giải: Đặt vế trái của bất đẳng thức là S. Vì a ≥ b ≥ c > 0 vàx ≥ y ≥ z > 0 nên ta có bz + cy ≥ by + cz suy ra

(by + cz)(bz + cy) ≤ (by + cz)2 ≤ 2[(by)2 + (cz)2

]

Đặt α = (ax)2; β = (by)2; γ = (cz)2, khi đó ta có:

a2x2

(by + cz)(bz + cy)≥ a2x2

2 [(by)2 + (cz)2]=

α

β + γ

Áp dụng tương tự cho hai bất đẳng thức, ta có

S ≥ 1

2(

α

β + γ+

β

γ + α+

γ

α + β)

Sử dụng bất đẳng thức Cauchy-Schwarz, ta có

β + γ+

β

γ + α+

γ

α + β)(α(β + γ)+β(α+ γ)+ γ(α+β))≥(α+β + γ)2

mà vế phải bằng

1

2

[(α − β)2 + (β − γ)2 + (γ − α)2 + 3(αβ + αβ + γα)

]≥ 3

2(2αβ+2βγ+2γα)

Do đó,

S ≥ 1

2(

α

β + γ+

β

γ + α+

γ

α + β) ≥ 1

2

(α + β + γ)2

(2αβ + 2βγ + 2γα)≥ 3

4

Vậy bài toán đã được chứng minh.vnmath.com

Page 83: OLYMPIC TOÁN NĂM 2000 52 ĐỀ THI VÀ LỜI …»i nói đầu Để thử gói lệnh lamdethi.sty tôi biên soạn một số đề toán thi Olympic, mà các học trò của

Chương 5

Đề thi olympic Mông cổ

.5.17.Đặt rad (1) = 1, với k > 1, đặt rad (k) là tích các số nguyên tố của k.

Một dãy các số tự nhiên a1, a2, ... với số hạng đầu a1 được xác định bởi

mối quan hệ: an+1 = an + rad (an). Hãy chỉ ra với mỗi nguyên dương

N , dãy an+1 = an + rad (an) gồm N số hạng liên tiếp trong một cấp số

cộng.

Lời giải: *) Bổ đề 1: Trong dãy rad (a1) , rad (a2) , ... mỗi số hạng làước của số hạng tiếp sau nó.Chứng minh:Vì rad (an) là ước của cả an và rad (an)

nên rad (an) là ước của an + rad (an) = an+1

⇒ mọi thừa số nguyên tố của rad (an) là ước của an+1

Vì rad (an) và rad (an+1) là tích của các ước số nguyên tốTừ đó cho ta kết quả rad (an) là ước của rad (an+1)

*) Với mỗi số nguyên dương n đặt bn = an

rad(an)và zn = rad(an+1)

rad(an)

Vì rad (an) là ước của rad (an+1) nên bn là một số nguyên dương ∀n

Do bổ đề 1, ta cũng có kết quả tương tự với zn

Mặt khác zn và rad (an) là nguyên tố cùng nhau do rad (an+1) là tíchcủa các ước số nguyên tố.

vnmath.com

Page 84: OLYMPIC TOÁN NĂM 2000 52 ĐỀ THI VÀ LỜI …»i nói đầu Để thử gói lệnh lamdethi.sty tôi biên soạn một số đề toán thi Olympic, mà các học trò của

Đề thi olympic Mông cổ 25

Vì vậy ta có:

bn+1 =an+1

rad (an+1)=

an+rad(an)rad(an)

rad(an+1)rad(an)

=bn+1 + 1

zn

*) Bổ đề 2: Với mỗi N , tồn tại một số nguyên dương M thỏa mãn:

zM = zM+1 = ... = zM+N−2 = 1

Chứng minh:Có vài số nguyên tố p nhỏ hơn 2N thỏa mãn điều kiện tồn tại một số n

sao cho p là ước của an

Áp dụng bổ đề 1, tồn tại một số nguyên m đủ lớn sao cho am chia hếtcho mọi số nguyên tố.Gọi M là số lớn hơn m sao cho bM là nhỏ nhất. Ta cần chứng minh M

thỏa mãn điều kiện của bổ đề này.Thật vậy, giả sử kết quả trên là không đúng, khi đó ta cần chỉ ra số k

dương nhỏ nhất thỏa mãn zM+k−1 6= 1

Mặt khác k ≤ N − 1 và zM = zM+1 = ... = zM+N−2 = 1

do đó bM+k−1 = bM + k − 1

Ta cần chỉ ra không có số nguyên tố nào nhỏ hơn 2N có thể chia hếtcho zM+k−1. Điều này là đúng vì zM+k−1 là tích các số nguyên tố chiahết cho aM+k nhưng không chia hết cho aM+k−1 và do aM+k−1 chia hếtcho rad (aM) và rad (aM) chia hết cho mọi số nguyên tố nhỏ hơn 2N .Do đó chia hết cho mọi an. Từ đó suy ra zM+k−1 ≥ 2N .Vì vậy:

bM+k =bM+k−1 + 1

zM+k−1=

bM + k

zM+k−1≤ bM + k

2N≤ bM + N − 1

2N< bM

Điều này mâu thuẫn với giả thiết M là số tự nhiên lớn hơn m và bM làsố tự nhiên nhỏ nhất. Vậy bổ đề 2 được chứng minh.*) Áp dụng bổ đề 2, với mỗi N , tồn tại một số tự nhiên M thỏa mãn:

rad (aM) = rad (aM+1) = ... = rad (aM+N−1)

Vậy aM , aM+1, ..., aM+N−1 là các số hạng liên tiếp trong một cấp sốcộng.

vnmath.com

Page 85: OLYMPIC TOÁN NĂM 2000 52 ĐỀ THI VÀ LỜI …»i nói đầu Để thử gói lệnh lamdethi.sty tôi biên soạn một số đề toán thi Olympic, mà các học trò của

26 Nguyễn Hữu Điển, ĐHKHTN Hà Nội

.5.18.Trong mặt phẳng, cho ba đường tròn ω1, ω2, ω3 đôi mọt tiếp xúc ngoài

nhau. Gọi P1 là tiếp điểm của ω1, ω3, P2 là tiếp điểm của ω2, ω3. A, B

là hai điểm trên đường tròn ω3 khác P1, P2 sao cho AB là đường kính

của đường tròn ω3. Đường thẳng AP1 cắt lại đường tròn ω1 tại X, đường

thẳng BP2 cắt lại đường tròn ω2 tại Y . Các đường thẳng AP2, BP1 cắt

nhau tại Z. Chứng minh rằng X, Y, Z thẳng hàng.

Lời giải: Xét các góc là có hướng theo modulo π

Gọi P3 là tiếp điểm của hai đường tròn ω1, ω2 và O1, O2, O3 là tâm củaba đường tròn ω1, ω2, ω3 tương ứng.Gọi ω4 là đường tròn ngoại tiếp tam giác P1P2P3 và O4 là tâm đẳngphương của ba đường tròn ω1, ω2, ω3 thì ta có: O4P1 = O4P2 = O4P3.Do đó O4 là tâm của đường tròn ω4.Vì O4O1⊥O1O3 nên O1O3 là tiếp tuyến của ω4.Chứng minh tương tự ta cũng có O1O2, O2O3 là tiếp tuyến của ω4.Vì O3 ∈ AB nên ta có: P2P1Z = P2AO3 = O3P2A.Nếu gọi Z ′ là giao điểm thứ hai của AP2 và ω4 thì do O3P2 là tiếp tuyếncủa ω4 nên ta có:

O3P2A = O3P2Z ′ = P2P1Z ′

Do đó P2P1Z = P2P1Z ′ và Z ′ thuộc đường thẳng BZ

Vì Z và Z ′ cùng thuộc AP2, AP2 6= BZ nên suy ra Z ≡ Z ′. Do đóZ ∈ ω4.Vì O4P1O3 và XP1Z cùng vuông (do AB là đường kính) nên ta có:

ZP1O3 = ZP1O4 + O4P1O3 = XP1Z + ZP1O4 = XP1O4

(1) Vì P1O4 là tiếp tuyến của ω1 nên ta có: XP1O4 = XP3P1 (2)Từ (1) và (2) suy ra ZP1O3 = XP3P1

Gọi l là đường thẳng ZP3 nếu Z ≡ P3 hoặc đường thẳng tiếp xúc vớiω4 tại P . Khi đó: (l, P3P1) = ZP1O3 (vì O3P1 là tiếp tuyến của ω4.Kết hợp điều này với kết quả ở trên, ta suy ra: (l, P3P1) = XP3P1 ⇒X ∈ l

Chứng minh tương tự ta có Y ∈ l .Vì Z ∈ l nên ta suy ra ba điểm X, Y, Z thẳng hàng.

vnmath.com

Page 86: OLYMPIC TOÁN NĂM 2000 52 ĐỀ THI VÀ LỜI …»i nói đầu Để thử gói lệnh lamdethi.sty tôi biên soạn một số đề toán thi Olympic, mà các học trò của

Đề thi olympic Mông cổ 27

.5.19.Một hàm số f : R → R thỏa mãn các điều kiện sau:

1) |f (a) − f (b)| 6 |a − b| ∀a, b ∈ R

2) f (f (f (0))) = 0

Chứng minh rằng f (0) = 0.

Lời giải: Ta sử dụng nhận xét sau: fk (x) = f(f(...f(x)︸ ︷︷ ︸

k lần f

...))

Từ (1) ta có:

|f (0)| = |f (0) − 0| >∣∣f 2 (0) − f (0)

∣∣ >

∣∣f 3 (0) − f 2 (0)

∣∣ =

∣∣f 2 (0)

∣∣

và |f 2 (0)| = |f 2 (0) − 0| > |f 3 (0) − f (0)| = |f (0)|ta suy ra |f (0)| = |f 2 (0)|*) Trường hợp 1: f (0) = f 2 (0)

Khi đó: f (0) = f 2 (0) = f 3 (0) = 0

*) Trường hợp 2: f (0) = −f 2 (0)

Ta có: |f (0)| = |f (0) − 0| > |f 2 (0) − f (0)| = 2 |f (0)|⇒ |f (0)| 6 0 ⇒ f (0) = 0

Vậy f (0) = 0.

.5.20.Đường phân giác của các góc A, B, C của tam giác ABC cắt các cạnh

của tam giác tại các điểm A1, B1, C1. Chứng minh rằng:

BC

AC + AB=

AC

BA + BC=

AB

CA + CB

Lời giải: Giả sử đường tròn ω ngoại tiếp tứ giác BA1B1C1 cắt lạiđường thẳng AC tại X. Ta cần chứng minh X phải thuộc đoạn AC.Trước hết, do A nằm trên đường BC1 nhưng không thuộc đoạn BC1

nên A phải nằm ngoài đường tròn ω.Tương tự, C nằm ngoài đường tròn ω.Ta có mọi điểm nằm trong đoạn B1X đều nằm trong đường tròn ω, dođó B1X không chứa A cũng không chứa C.Vì B1 nằm trên cạnh AC do đó X nằm trên cạnh AC.Đặt BC = a; CA = b; AB = c. Áp dụng phương tích của điểm A đốivới đường tròn ω ta có: AB.AC1 = AX.AB1

vnmath.com

Page 87: OLYMPIC TOÁN NĂM 2000 52 ĐỀ THI VÀ LỜI …»i nói đầu Để thử gói lệnh lamdethi.sty tôi biên soạn một số đề toán thi Olympic, mà các học trò của

28 Nguyễn Hữu Điển, ĐHKHTN Hà Nội

Từ định lý về đường phân giác trong tam giác ta có:

AC1 =bc

a + b; AB1 =

bc

a + c

Do đó ta có:

AX =AC1.AB

AB1=

bc

a + b.c.

a + c

bc=

(a + c) c

a + b

Chứng minh tương tự ta cũng có: CX = (a+c)ab+c

Hơn nữa, do X thuộc cạnh AC nên:

b = AC = AX + CX = (a + c)

(c

a + b+

a

b + c

)

⇔ b

a + c=

c

a + b+

a

b + c⇔ a

b + c=

b

a + c− c

a + b

hayBC

AC + AB=

AC

BA + BC=

AB

CA + CB

.5.21.Những số nguyên nào có thể biểu diễn được dưới dạng

(x + y + z)2

xyz

với x, y, z là các số nguyên dương.

Lời giải: *) Nhận xét: Ta có các số sau thỏa mãn yêu cầu bài toán:

1 =(9 + 9 + 9)2

9.9.9; 2 =

(4 + 4 + 8)2

4.4.8; 3 =

(3 + 3 + 3)2

3.3.3

4 =(2 + 2 + 4)2

2.2.4; 5 =

(1 + 4 + 5)2

1.4.5; 6 =

(1 + 2 + 3)2

1.2.3

8 =(1 + 1 + 2)2

1.1.2; 9 =

(1 + 1 + 1)2

1.1.1

Ta sẽ chứng minh không có các kết quả khác thỏa mãn bổ đề sau:Bổ đề: Nếu n có thể biếu diễn được dưới dạng

(x + y + z)2

xyz

vnmath.com

Page 88: OLYMPIC TOÁN NĂM 2000 52 ĐỀ THI VÀ LỜI …»i nói đầu Để thử gói lệnh lamdethi.sty tôi biên soạn một số đề toán thi Olympic, mà các học trò của

Đề thi olympic Mông cổ 29

thì n có thể viết dưới dạng:

(x′ + y′ + z′)2

x′y′z′

với x′ 6 y′ + z′ ; y′ 6 x′ + z′ ; z′ 6 x′ + y′

Chứng minh bổ đề:

Gọi x, y, z là các số nguyên dương thỏa mãn n = (x+y+z)2

xyzvà x + y + z

là nhỏ nhất.Vì n là một số nguyên nên x là ước của (x + y + z)2 do đó x là ước của(y + z)2

Đặt x′ = (y+z)2

x, khi đó ta có

(x′ + y + z)2

x′yz=

(y + z)2(y+z

x+ 1

)2

(y+z)2

xyz

=x(

y+z

x+ 1

)2

yz=

(x + y + z)2

xyz= n

Do x+y+z là nhỏ nhất nên x+y+z 6 x′+y+z Suy ra x 6 x′ = (y+z)2

x⇒

x 6 y + z Chứng minh tương tự thì ta cũng có: y 6 x + z ; z 6 x + y

(Bổ đề được chứng minh)Giả sử n = (x+y+z)2

xyz. Áp dụng bổ đề, không mất tính tổng quát, ta có

thể giả sử y + z > x > y > z

Ta xét các trường hợp sau:*) Trường hợp 1: x = y > z = 1

Khi đó n = (2x+1)2

x2 ⇒ x là ước của 2x + 1 ⇒ x = 1 và n = 9

*) Trường hợp 2: x = y + 1 > z = 1

Khi đó n = (2x)2

x(x−1)= 4x

x−1⇒ x − 1 là ước của 4x ⇒ x − 1 là ước của 4

⇒ x ∈ {2; 3; 5} ⇒ n ∈ {8; 6; 5}*) Trường hợp 3: y + z > x > y > z > 1

Khi đó: yz − (y + z) = (y − 1) (z − 1) − 1 > 0

⇒ yz > y + z > x

Do x > y > z ta có: xy > z ; xz > y

Do đó

n =(x + y + z)2

xyz= 2

(1

x+

1

y+

1

z

)

+x

yz+

y

xz+

z

xy6 2.

3

2+1+1+1 = 6

⇒ n ∈ {1; 2; 3; 4; 5; 6}Vậy các kết quả của n cần tìm là n ∈ {1; 2; 3; 4; 5; 6; 8; 9}

vnmath.com

Page 89: OLYMPIC TOÁN NĂM 2000 52 ĐỀ THI VÀ LỜI …»i nói đầu Để thử gói lệnh lamdethi.sty tôi biên soạn một số đề toán thi Olympic, mà các học trò của

30 Nguyễn Hữu Điển, ĐHKHTN Hà Nội

.5.22.Một đất nước có n thành phố. Tổng chi phí của chuyến đi từ thành phố i

đến thành phố j là xij. Giả sử rằng tổng chi phí của tuyến đường qua mỗi

thành phố đúng một lần và kết thúc tại điểm bắt đầu không phụ thuộc

vào việc chọn tuyến đường. Chứng minh rằng tồn tại các số a1, a2, ..., an

và b1, b2, ..., bn sao cho xij = ai + bj với mọi số nguyên dương i, j thỏa

mãn 1 6 i < j 6 n.

Lời giải: Đặt f (a, b) = xa1 + x1b − xab với a, b và 1 là ba số phân biệt

*) Bổ đề: f (a, b) không phụ thuộc vào a, b

Chứng minh bổ đề:

+) Với n 6 2, điều này là tầm thường vì khi đó f được xác định khôngcó a và b

+) Với n = 3 ta cần chỉ ra f (2, 3) = f (3, 2)

hay x21 + x13 + x32 = x31 + x12 + x23

Nhưng những đẳng thức này là tổng các chi phí của 2 tuyến đường màmỗi tuyến đều đi qua mọi thành phố đúng 1 lần, do đó chúng bằng nhau.+) Với n > 4, tuyến đường:

a, 1, b, c, 2, 3, ..., a − 1, a + 1, ..., b − 1, b + 1, ..., c − 1, c + 1, ..., n

và tuyến đường

a, b, 1, c, 2, 3, ..., a − 1, a + 1, ..., b − 1, b + 1, ..., c − 1, c + 1, ..., n

phải có tổng toàn bộ chi phí bằng nhau. Các tuyến đường này gần đồngnhất, cho phép tađễ dàng tìm được sự khác nhau của tổng chi phí của2 tuyến đường đó là:

(xa1 + x1b + xbc) − (xab + xb1 + x1c)

Do đó, f (a, b) = f (b, c) với mọi a, b, c đôi một khác nhau và khác 1Hơn nữa, tổng của toàn bộ chi phí của 3 tuyến đường:

1, a, b, 2, ..., n ; b, 1, a, 2, ..., n ; a, 1, b, 2, ..., n

phải bằng tổng của toàn bộ chi phí của 3 tuyến đường:

1, b, a, 2, ..., n ; a, 1, b, 2, ..., n ; b, 1, a, 2, ..., n

vnmath.com

Page 90: OLYMPIC TOÁN NĂM 2000 52 ĐỀ THI VÀ LỜI …»i nói đầu Để thử gói lệnh lamdethi.sty tôi biên soạn một số đề toán thi Olympic, mà các học trò của

Đề thi olympic Mông cổ 31

Do đó 2 (x1a + xab + xb1) = 2 (x1b + xba + xa1)

Từ đó suy ra f (a, b) = f (b, a)

Với c, d không bằng a, b ta có: f (a, b) = f (b, c) = f (c, d) ;f (a, b) = f (b, c) = f (c, b) ;f (a, b) = f (b, a) = f (a, c) = f (c, a)

Điều này đã chứng minh được kết quả của bổ đề.*) Với mọi a, b phân biệt và khác 1 ta có: f (a, b) = F với F là hằng sốCho a1 = 0 ; b1 = F và đặt bk = x1k ; ak = xk1 − F

Với mọi i, j không đồng thời bằng 1 ta có:

xij = xi1 − xi1 − x1j + xij + x1j = xi1 − F + xij = ai + bj

tức là tồn tại các số a1, a2, ..., an và b1, b2, ..., bn sao cho xij = ai + bj

với mọi số nguyên dương i, j thỏa mãn 1 6 i < j 6 n

vnmath.com

Page 91: OLYMPIC TOÁN NĂM 2000 52 ĐỀ THI VÀ LỜI …»i nói đầu Để thử gói lệnh lamdethi.sty tôi biên soạn một số đề toán thi Olympic, mà các học trò của

Chương 6

Đề thi olympic Rumani

.6.23.Hàm f : R2 → R được gọi là olympic nếu nó thỏa mãn tính chất: với

n ≥ 3 các điểm rời rạc A1, A2, · · · , An ∈ Rn, nếu f(A1) = f(A2) = · · · =

f(An) thì các điểm A1, A2, · · · , An được gọi là các đỉnh của đa giác lồi.

Cho P ∈ C[X] khác đa thức hằng. Chứng mình rằng hàm f : R2 → R

được cho bởi f(x, y) = |P (x + iy)|, là olympic khi và chỉ khi tất cả các

nghiệm của P là bằng nhau.

Lời giải: Trước hết ta giả sử rằng tất cả các nghiệm của P là bằngnhau, khi đó ta viết được dưới dạng:P (x) = a(z − z0)

n với a, z0 ∈ C và n ∈ N. Nếu A1, A2, · · · , An làcác điểm rời rạc trong R2 sao cho f(A1) = f(A2) = · · · = f(An) thìA1, A2, · · · , An nằm trên đường tròn với tâm là (Re(z0), Im(z0)) và bánkính là n

√α

|f(A1)| , suy ra các điểm đó là các đỉnh của một đa giác lồi.

Ngược lại, ta giả sử rằng không phải tất cả các nghiệm của P là bằngnhau, khi đó P (x) có dạng:P (x) = (z − z1)(z − z2)Q(z) với z1 và z2 là 2 nghiệm phân biệt củaP (x) sao cho |z1 − z2| là nhỏ nhất. Gọi l là đường thẳng đi qua haiđiểm Z1 và Z2 với Z1 = (Re(z1), Im(z2)),Z2 = (Re(z2), Im(z2)), và đặtz3 = 1

2(z1 + z2) sao cho Z3 = (Re(z3), Im(z3)) là trung điểm của Z1Z2.

Ký hiệu s1, s2 lần lượt là các tia Z3Z1, Z3Z2, và r = f(Z3) ≥ 0. Ta phảicó r ≥ 0, bởi vì nếu ngược lại ta có z3 là một nghiệm của P sao cho:

vnmath.com

Page 92: OLYMPIC TOÁN NĂM 2000 52 ĐỀ THI VÀ LỜI …»i nói đầu Để thử gói lệnh lamdethi.sty tôi biên soạn một số đề toán thi Olympic, mà các học trò của

Đề thi olympic Rumani 33

|z1 − z3| ≤ |z1 − z2|, điều này là mâu thẫu với |z1 − z2| là nhỏ nhất.Do

limZZ3→∞

Z∈s1

f(Z) = limZZ3→∞

Z∈s1

f(Z) = +∞.

và f liên tục, tồn tại Z4 ∈ s1 và Z5 ∈ s2 sao cho f(Z4) = f(Z5) = r. Dovậy f(Z3) = f(Z4) = f(Z5) và Z3, Z4, Z5 không phải là các đỉnh của đagiác lồi. Do vậy, f không phải là olympic.

.6.24.Với n ≥ 2 là số nguyên dương. Tìm số các hàm f : {1, 2, · · · , n} →{1, 2, 3, 4, 5} thỏa mãn tính chất: |f(k + 1) − f(k)| ≥ 3 với k =

1, 2, · · · , n − 1

Lời giải: Ta có n ≥ 2 bất kỳ và tìm số các hàm tương ứng. Nếuf : {1, 2, · · · , n} → {1, 2, 3, 4, 5} phải thỏa mãn đã cho thì f(n) 6= 3

bởi nếu ngược lại thì f(n − 1) ≤ 0 hoặc f(n − 1) ≥ 6, vô lý. Kýhiệu an, bn, dn, en là số các hàm f : {1, 2, · · · , n} → {1, 2, 3, 4, 5} thỏamãn tính chất đã cho sao cho f(n) tương ứng bằng 1, 2, 4, 5. Khi đóa2 = e2 = 2 và b1 = d2 = 1, và do vậy với n ≥ 2 :

an+1 = en + dn, bn+1 = en

en+1 = an + bn, dn+1 = an

Ta cần tìm an + bn + dn + en với ∀n ≥ 2. Ta có: a2 = e2 và b2 = d2;bằng quy nạp ta có an = en và bn = dn ∀n ≥ 2. Do vậy với ∀n, ta có:

an+2 = en+1 + dn+1 = an+1 + bn+1 = an+1 + en = an+1 + an

do vậy, {an}n≥2 thỏa mãn như dãy Fibonaci {Fn}n≥0, với các chỉ số đượcchọn sao cho: F1 = 0 và F1 = 1. Bởi vì a2 = 2 = F2 và a3 = e2 + d2 =

3 = F3, vậy suy ra an = Fn với ∀n. Do đó,an+bn+dn+en = 2(an+bn) =

2en+1 = 2an+1 = 2Fn+1 với ∀n ≥ 2 và 2Fn+1 thỏa mãn tính chất đã cho.

.6.25.Cho n ≥ 1 là một số nguyên dương và x1, x2, · · · , xn là các số thực sao

cho: |xk+1 − xk| leq1 với k = 1, 2, · · · , n − 1. Chứng mình rằng:

n∑

k=1

|xk| −∣∣∣∣∣

n∑

k=1

xk

∣∣∣∣∣≤ n2 − 1

4

vnmath.com

Page 93: OLYMPIC TOÁN NĂM 2000 52 ĐỀ THI VÀ LỜI …»i nói đầu Để thử gói lệnh lamdethi.sty tôi biên soạn một số đề toán thi Olympic, mà các học trò của

34 Nguyễn Hữu Điển, ĐHKHTN Hà Nội

Lời giải: Nếu số các số xk âm lớn hơn số các số xk dương thì (a1, · · · , an)

là một hoán vì của (−x1, · · · ,−xn)(tương ứng là (x1, · · · , xn)) sao choa1, · · · , an là một dãy không giảm. Do cách xây dựng, lực lượng P cácsố dương ak không nhiều phần tử hơn lực lượng N các số âmak, và do

vậy |P | ≤ n − 1

2. Vì N là khác rỗng và a1, · · · , an là không giảm, các

phần tử của P là ak0+1 < ak0+2 < · · · < ak0+l với k0 > 0

Giả sử rằng 1 ≤ i ≤ n − 1. Trong dãy x1, · · · , xn phải có hai phần tửkề nhau xj và xk sao cho xj ≤ ai và xk ≥ ai+1 suy ra 0 ≤ ai+1 − ai ≤xk − xj ≤ 1. Do vậy, ak0+1 ≤ ak0

+ 1 ≤ 1,ak0+2 ≤ ak0+1 + 1 ≤ 2.Ký hiệu σP và σN lần lượt là tổng của các số trong P và N. Mặt khácta có:

|σP − σN | − |−σP − σN | ≤ |2σP |

≤ 2(1 + 2 + · · · +⌊

n − 1

2

)

≤ n − 1

2.(

n − 1

2+ 1) =

n2 − 1

4

.6.26.Cho n, k là các số nguyên dương tùy ý.Chứng minh rằng tồn tại các số

nguyên dương a1 > a2 > a3 > a4 > a5 > ksao cho:

n = ±C3a1± C3

a2± C3

a3± C3

a4± C3

a5

ở đó

(a

3

)

=a(a − 1)(a − 2)

6

Lời giải: Ta thấy rằng: n + C3m > 2m + 1 với ∀m lớn hơn giá trị N, bởi

vì vế trái là bậc 3 với hệ số cao nhất dương trong khi vế phải là tuyếntính với m.

Nếu m ≡ 0(mod4), thì C3m = m(m−1)(m−2)

6là chẵn bởi vì tử số chia hết

cho 4 còn mẫu số thì không.Nếu m ≡ 3(mod4) thì C3m = m(m−1)(m−2)

6là

lẻ bởi vì cả tử số và mẫu số đều chia hết cho 2 nhưng không chia hếtcho 4. Do vậy ta chọn m > maxk, N sao cho n + C3

m là số lẻ.

vnmath.com

Page 94: OLYMPIC TOÁN NĂM 2000 52 ĐỀ THI VÀ LỜI …»i nói đầu Để thử gói lệnh lamdethi.sty tôi biên soạn một số đề toán thi Olympic, mà các học trò của

Đề thi olympic Rumani 35

Ta viết: 2a+1 = n+C3m > 2m+1. Ta thấy rằng:(C3

a+3−C3a+2)−(C3

a+1−C3

a) = C2a+2 − C2

a = 2a + 1. Do vậy

n = (2a + 1) −(

m

3

)

=

(a + 3

3

)

−(

a + 2

3

)

−(

a + 1

3

)

+

(a

3

)

=

(m

3

)

thỏa mãn yêu cầu bài toán vì a + 3 > a + 2 > a + 1 > a > m > k

.6.27.Cho P1P2 · · ·Pn là một đa giác lồi trong mặt phẳng. Giả sử rằng với cặp

đỉnh Pi, Pj, tồn tại đỉnh V của đa giác sao cho ∠PiV Pj = π3. Chứng

minh rằng n = 3

Lời giải: Trong lời giải này ta sử dụng kết quả sau:Cho tam giác XYZ sao cho ∠XY Z ≤ π

3thì tam giác đó là đều hoặc

max{Y X, Y Z} > XZ. Tương tự nếu ∠XY Z ≥ π3

thì tam giác XYZđều hoặc min{Y X, Y Z} < XZ

Chúng ta chỉ ra rằng tồn tại các đỉnh A, B, C và A1, B1, C1 sao cho:(i)tam giác ABC và A1B1C1 là tam giác đều và (ii) AB(tướng ứng là A1B1)là khoảng cách nhỏ nhất (lớn nhất) khác 0 giữa 2 đỉnh. Hơn nữa, A, B

là 2 đỉnh phân biệt sao cho AB có độ dài nhỏ nhất, và C là đỉnh saocho ∠ACB = π

3. Khi đó max{AC, CB} ≤ AC, để tam giác ABC phải

là tam giác đều. Tuơng tự, ta chọn A1, B1 sao cho A1B1 có độ dài lớnnhất, và đỉnh C1 sao cho ∠A1C1B1 = π

3, khi đó tam giác A1B1C1 là tam

giác đều.Ta chỉ ra rằng 4ABC ∼= 4A1B1C1. Các đường thẳng AB,BC, CA chiamặt phẳng thành 7 phần. Gọi DA gồm các phần do tam giác chia mànhận BC làm biên và các phần được tạo ra tại phần tạo ra ở các đỉnhB và C. Tương tự ta định nghĩa cho DB và DC . Bởi vì đa giác đã cholà lồi, nên mỗi hoặc nằm trong 1 phần hoặc trùng với A, B, C.Nếu 2 điểm bất kỳ trong A1, B1, C1 , giả sử là A1, B1 nằm trong miềnDX , thì ∠A1XB1 < π

3. Do vậy, max{A1X, XB1} > A1B1, mâu thuẫn

với A1B1 là lớn nhất.Hơn nữa, không có hai điểm trong A1, B1, C1 ở trong cùng 1 phần. Bâygiờ ta giả sử rằng một trong các điểm A1, B1, C1 ( giả sử là A1) nằmtrên 1 phần(giả sử đó là DA). Bởi vì min{A1B, A1C} ≥ BC, ta có∠BA1C ≤ π

3. Ta có B1 không nằm trong DA. Bởi vì đa giác đã cho là

vnmath.com

Page 95: OLYMPIC TOÁN NĂM 2000 52 ĐỀ THI VÀ LỜI …»i nói đầu Để thử gói lệnh lamdethi.sty tôi biên soạn một số đề toán thi Olympic, mà các học trò của

36 Nguyễn Hữu Điển, ĐHKHTN Hà Nội

lồi, B không nằm trong tam giác AA1B1, và tương tự C không nằm trongtam giác AA1B1. Từ đó có B1nằm trên miền đóng có biên là các tia A1B

và A1C. Tương tự, với C1. Hơn nữa, π3

= ∠B1A1C1 ≤ ∠BA1C = π3, dấu

bằng xảy ra khi B1 và C1 lần lượt nằm trên tia A1B và A1C . Bởi vì đagiác đã cho là lồi,nên điều này chỉ xảy ra khi B1 và C1 lần lượt bằng B

và C -trong trường hợp BC = B1C1, ta có tam giác ABC và A1B1C1 làbằng nhau.Mặt khác, không có điểm nào trong A1, B1, C1 nằm trên DA ∪DB ∪DC ,do đó chúng lần lượt trùng với A, B, C. Trong trường hợp này, tam giácABC và A1B1C1 là trùng nhau.Do vậy hai đỉnh bất kỳ của đa giác có khoảng cách giống nhau, nhưAB = A1B1. Điều này là không thể xảy ra nếu có hơn 3 điểm trong mặtphẳng thì hoàn toàn không có tính chất này. Do vậy n=3

.6.28.Chứng minh rằng tồn tại vô hạn bộ gồm 4 số nguyên dương(x, y, z, t)

sao cho ước chung lớn nhất của 4 số là 1.và thỏa mãn:

x3 + y3 + z2 = t4

Lời giải: Đặt a = k3 với k là số chẵn và k > 0 ta có:

(a + 1)4 − (a − 1)4 = 8a3 + 8a

tức là(2k3)3 + (2k3)3 + [(k3 − 1)2]2 = (k3 + 1)4

Bởi vì k3 + 1 là số lẻ,nên (2k3, k3 + 1) = (k3, k3 + 1) = 1. Do vậy, ta cóvô hạn bộ bốn số dạng (x, y, z, t) = (2k3, 2k, (k3 − 1)2, k3 + 1) với k > 0

là số chẵn, thỏa mãn các điều kiện của bài toán.

.6.29.Biểu diễn nhị phân của một số nguyên dương lẻ a, được xác định bằng

thuật toán đơn giản sau: xác định một số nguyên dương nhỏ nhất n sao

cho 22000 là ước của an − 1

Lời giải: Bởi vì a là số lẻ nên (a, 2k) = 1 với ∀k ≥ 0. Do vậy, theo địnhlý Euler, ta có a2k−1 ≡ aϕ(2k) ≡ 1(mod2k) với ∀k. Do vậy bậc n của amodulo 22000 chia cho 22000 − 1 = 21999

vnmath.com

Page 96: OLYMPIC TOÁN NĂM 2000 52 ĐỀ THI VÀ LỜI …»i nói đầu Để thử gói lệnh lamdethi.sty tôi biên soạn một số đề toán thi Olympic, mà các học trò của

Đề thi olympic Rumani 37

Nếu a ≡ 1(mod22000) suy ra n=1. Ta giả sử rằng a 6= 1(mod22000) . Vớimọi m ≥ 1 , ta viết:

a2m − 1 = (a − 1)(a + 1) (a2 + 1)(a22

+ 1) · · · (a2m−1

+ 1)︸ ︷︷ ︸

(∗).

Biểu diễn nhị phân của a có kết thúc là 2 chữ số 01 hoặc 11. Ta cóa ≡ ±1(mod4) và do vậy a2k ≡ 1(mod4) với ∀k ≥ 1. Do vậy phân tích(*)với m cố định (m ≥ 1), 21 là số mũ cao nhất của 2 mà chia hết chom − 1 biểu thức ở mỗi ngoặc đơn phía trên †.Nếu a ≡ 1(mod4), khi đó a 6= 1, biểu diễn nhị phân của a kết thúc là:

1 00 · · ·01︸ ︷︷ ︸

s chữ số

với s là số nguyên lớn nhất sao cho 2s|(a− 1). Trong trường hợp này, sốmũ cao nhất của 2 chia cho a − 1 là 2s trong khi số mũ cao nhất của 2chia hết cho a + 1 là 2Nếu thay thế là a ≡ −1(mod4) , khi đó bởi vì a 6= 1 , biểu diễn nhị phâncủa a kết thúc là:

1 011 · · ·1︸ ︷︷ ︸

s chữ số

với s là số ngueyen lớn nhất sao cho 2s|(a + 1). Trong trường hợp này,số mũ cao nhất của 2 chia cho (a + 1) được 2s trong khi đó số mũ caonhất của 2 chia cho (a − 1) được 2.Trong mỗi trường hợp, ta sử dụng (*) và kết quả số mũ cao nhát của 2chia cho (a2m − 1 được 2s+m. Từ đó có m ≥ 1 nhỏ nhất sao cho a2m − 1

chia hết cho 22000 là 2000 − s (nếu s<2000) hoặc 1 ( nếu s ≥ 2000).Trong các trường hợp tương ứng ta có n = 21999−s hoặc n = 2. Bởi vì tacó thể dễ dàng sử dụng biểu diễn nhị phân của a để suy ra hai trườnghợp và giá trị của s là gì, ta có thể sử dụng biểu diễn nhị phân của a đểtìm n.

.6.30.Cho tam giác nhọn ABC và điểm M là trung điểm của BC. Tồn tại duy

nhất một điểm trong N sao cho ∠ABN = ∠BAM và ∠ACN = ∠CAM .

Chứng minh rằng ∠BAN = ∠CAM .

vnmath.com

Page 97: OLYMPIC TOÁN NĂM 2000 52 ĐỀ THI VÀ LỜI …»i nói đầu Để thử gói lệnh lamdethi.sty tôi biên soạn một số đề toán thi Olympic, mà các học trò của

38 Nguyễn Hữu Điển, ĐHKHTN Hà Nội

Lời giải: Cho B′ là điểm nằm trên tia AC sao cho ∠ABB

= ∠BAM ,cho C

′ là điểm nằm trên tia AB sao cho ∠ACC′

= ∠CAM . Khi đó Nlà giao điểm của hai đường thẳng BB

′ và CC′

Đường thẳng đối xứng với AM qua đường phân giác của góc BAC, cắtBB

′ tại P. Gọi D là điểm đối xứng của A qua M, để tứ giác ABCDlà hình bình hành. Bởi vì ∠PAB = ∠CAM = ∠CAD và ∠ABP =

∠MAB = ∠DAB = ∠ADC, tam giác ABP và ADC là đồng dạng. Do

vậy,AB

AD=

AP

AC. Bởi vì ∠BAD = ∠PAC, nên tam giác BAD và tam

giác PAC đồng dạng. Hơn nữa, ∠ACP = ∠ADB = ∠CAM . Từ đó suyra P nằm trên đường thẳng CC

′ cũng như nằm trên BB′ , và do vậy

N ≡ P . Hơn nữa có ∠BAN = ∠BAP = ∠CAM như yêu cầu bài toán.vnmath.com

Page 98: OLYMPIC TOÁN NĂM 2000 52 ĐỀ THI VÀ LỜI …»i nói đầu Để thử gói lệnh lamdethi.sty tôi biên soạn một số đề toán thi Olympic, mà các học trò của

Chương 7

Đề thi olympic Nước Nga

.7.31.Sasha thử xác định vài số nguyên dương x 6 100. Anh ta chọn hai số

nguyên dương bất kì M và N mà nhỏ hơn 100 và có câu hỏi "Số nào

lớn nhất trong các ước số chung của x + M và N?" Chứng minh rằng

Sasha có thể xác định được giá trị của x sau 7 câu hỏi.

Lời giải: +) Với n = 0, 1, 2, ..., 6, đặt an là số nguyên duy nhất trong[0; 2n) thỏa mãn 2n |(x − an) . Rõ ràng a0 = 0.+) Với n 6 5, an+1 bằng an hoặc an + 2n mà khi đó kết quả cũ vẫn cónếu và chỉ nếu gcd (x + 2n − an , 2n+1) = 2n

Vì 2n − an < 2n+1 < 100 ta suy ra nếu Sasha biết được giá trị của an,anh ta có thể xác định được an+1 với một câu hỏi điều kiện bằng cáchđặt (M, N) = (2n − an , 2n+1). Do đó sau 6 câu hỏi, Sasha có thể xácđịnh được a1, a2, ..., a6 và kết luận x bằng a6 hoặc a6 + 64

Bởi vì a6 6= a6 + 64 (mod 3), Sasha có thể xác định được x nếu anh taphát hiện ra liệu có hay không x ≡ a6 (mod 3) với các câu hỏi của anhta.Thật vậy, anh ta có thể nếu đặt N = 3 và M ∈ {1, 2, 3} nên 3 |(a6 + M) ,anh ta sẽ thu được câu trả lời "3" nếu và chỉ nếu x ≡ a6 (mod 3).

.7.32.Cho O là tâm đường tròn ω ngoại tiếp tam giác nhọn ABC. Đường tròn

ω1 với tâm K đi qua các điểm A, O, C mà cắt các cạnh bên AB và

BC tại M và N . Đặt L là điểm đối xứng với K qua đường thẳng MN .

vnmath.com

Page 99: OLYMPIC TOÁN NĂM 2000 52 ĐỀ THI VÀ LỜI …»i nói đầu Để thử gói lệnh lamdethi.sty tôi biên soạn một số đề toán thi Olympic, mà các học trò của

40 Nguyễn Hữu Điển, ĐHKHTN Hà Nội

Chứng minh rằng BL⊥AC.

Lời giải: Gọi α, β, γ là các góc A, B, C của tam giác ABC. Vì tứ giácACNM nội tiếp, BNM = α, BMN = γ nên MKC = 2α ; NKA = 2γ

Vì đường thẳng AC là trục đẳng phương của đường tròn ω và ω1 nênnó vuông góc với OK. Khi đó ta có:

AOK = COK = OAK = OCK = β

Do đó AKC = 2π − 4β

Kết hợp các điều trên, ta có:

MKN = 2α + 2γ − (2π − 4β) = 2β

Vì L là điểm đối xứng của K qua đường thẳng MN nên ta có: MLN =

2β và LM = LN . Do đó ∆LMN ∼ ∆OCA

Mặt khác ta cũng có: ∆MBN ∼ ∆CBA bởi vì tứ giác ACNM nội tiếp.Do đó vì O là tâm đường tròn ngoại tiếp tam giác ABC và L là tâmđường tròn ngoại tiếp tam giác MBN nên ta suy ra MLB = α

Và khi đó: MBL = π2−α. Bởi vì BAC = α, ta dễ dàng suy ra BL⊥AC.

.7.33.Có vài thành phố trong một quốc gia và một cách đặt tên đường. Nơi

mà mỗi con đường nối hai thành phố và không có hai con đường nào nối

hai thành phố có tên giống nhau. Nó được hiểu rằng trong mọi thành

phố đều có tối thiểu 3 con đường để đi ra. Chứng minh rằng tồn tại một

con đường tuần hoàn (có nghĩa là nơi kết thúc là nơi bắt đầu) như thế

số con đường trong những con đường đó không thể chia hết cho 3.

Lời giải: Ta sử dụng sự dịch chuyển trong lý thuyết đồ thị. Trong mộtđồ thị, tất cả mọi đỉnh đều có bậc ít nhất là 3. Ta chứng minh tồn tạimột chu trình mà độ dài của mó không chia hết cho 3.Thực hiện thuật toán sau:+) Cố định một điểm đầu v1 sau đó cho v1, v2, ..., vi nếu tồn tại mộtđiểm riêng từ i đỉnh và tới đỉnh gần kề thì vi+1 là một đỉnh. Bởi vì đồthị được giới hạn và tất cả các đỉnh đạt được bằng thuật toán này làrõ ràng.Quá trình này kết thúc tại vài đỉnh vn. Ta biết mọi đỉnh có ít nhất bậc

vnmath.com

Page 100: OLYMPIC TOÁN NĂM 2000 52 ĐỀ THI VÀ LỜI …»i nói đầu Để thử gói lệnh lamdethi.sty tôi biên soạn một số đề toán thi Olympic, mà các học trò của

Đề thi olympic Nước Nga 41

3 và bởi sự thừa nhận mọi đỉnh gần kề với vn lập thành một dãy. Nhưvậy, vn là đỉnh kề với va, vb, vn−1 với a < b < n − 1.Ta có 3 chu trình:

va → va+1 → va+2 → ... → vn−1 → vn → va

vb → vb+1 → vb+2 → ... → vn−1 → vn → vb

va → va+1 → va+2 → ... → vb−1 → vb → vn → va

Những chu trình trên có độ dài n − a + 1 ; n − b + 1 ; b − a + 2 theothứ tự. Bởi vì (n − a + 1) − (n − b + 1) − (b − a + 2) = −2 không thểchia hết cho 3Do đó tồn tại 1 trong 3 chu trình trên có độ dài không chia hết cho 3.

.7.34.Cho x1, x2, ..., xn là các số thực (n > 2) thỏa mãn điều kiện −1 < x1 <

x2 < ... < xn < 1 và x131 + x13

2 + ... + x13n = x1 + x2 + ... + xn. Chứng

minh rằng: x131 y1 + x13

2 y2 + ... + x13n yn < x1y1 + x2y2 + ... + xnyn với mọi

số y1 < y2 < ... < yn.

Lời giải: Với −1 < x < 1, đặt f (x) = x − x13

Ta phải có x1 < 0 vì nếu không f (x1) > 0 và f (x2) > 0, f (x3) >

0, ..., f (xn) > 0.Điều này vô lý vì f (x1) + f (x2) + ... + f (xn) = 0 được cho bởi phươngtrình trên.Chứng minh tương tự ta cũng suy ra xn > 0

Giả sử rằng 2 6 i 6 n

+) Nếu xi 6 0 thì khi đó ta có

x1 < x2 < ... < xi−1 < xi 6 0 vàn∑

j=1

f (xj) = −i−1∑

j=1

f (xj) > 0

+) Nếu thay thế xi > 0 thì khi đó ta có: 0 < xi < xi+1 < ... < xn và ta

cũng chứng minh đượcn∑

j=1

f (xj) > 0

+) Sử dụng công thức lấy tổng Abel và kết quả trên ta có:

vnmath.com

Page 101: OLYMPIC TOÁN NĂM 2000 52 ĐỀ THI VÀ LỜI …»i nói đầu Để thử gói lệnh lamdethi.sty tôi biên soạn một số đề toán thi Olympic, mà các học trò của

42 Nguyễn Hữu Điển, ĐHKHTN Hà Nội

n∑

i=1

xiyi −n∑

i=1

x13i yi =

n∑

i=1

yif (xi)

= y1

n∑

i=1

f (xi) +n∑

i=2

(yi − yi−1) (f (xi) + f (xi+1) + ... + f (xn))

=n∑

i=2

(yi − yi−1) (f (xi) + f (xi+1) + ... + f (xn)) > 0

Từ đó ta dễ dàng suy ra điều phải chứng minh.

.7.35.Gọi AA1, CC1 là các đường cao của tam giác nhọn ABC. Đường phân

giác của góc nhọn giữa hai đường thẳng AA1, CC1 cắt các cạnh AB và

BC tại P, Q tương ứng. Gọi H là trực tâm tam giác ABC và M là

trung điểm của cạnh AC, đường phân giác của ABC cắt đoạn HM tại

R. Chứng minh rằng tứ giác PBQR nội tiếp được một đường tròn.

Lời giải: Hạ đường vuông góc với cạnh AB và BC tại P , Q tương ứng,chúng cắt nhau tại R′′. Gọi S là giao điểm của R′P và HA, T là giaođiểm của R′Q và HC.Hạ đường vuông góc từ M tới AB, cắt HA tại U và hạ đường vuônggóc từ M tới BC cắt HC tại V .Vì ∆PSH, ∆HTQ có các cạnh tương ứng song song và P SH = HTQ

Do PQ là đường phân giác của góc nhọn tạo bởi hai đường thẳngAA1, CC1 nên PHS = QHT .Do đó ∆PHS ∼ ∆QHT

Mặt khác, vì HAP = π2− ABC = QCH và PHA = QHC nên

∆PHA ∼ ∆QHC

Do đó:HT

HS=

HP

HQ=

HC

HA=

2MU

2MV=

MU

MV=

HV

HU

Khi đó, phép vị tự tâm H biến đường thẳng PS thành đường thẳngMU và cũng biến đường thẳng QT thành đường thẳng MV . Do đó nóbiến R′ = PS ∩ QT thành M = MU ∩ MV . Vì thế 3 điểm H, R′, M

thẳng hàngTa lại sử dụng giả thiết ∆PHA ∼ ∆QHC, ta có HPB, HQB là đồngdư vì chúng cùng phụ với hai góc HPA, HQC.Như vậy, BP = BQ và ∆BR′P ∼ ∆BR′Q nên PBR′ = QBR′.

vnmath.com

Page 102: OLYMPIC TOÁN NĂM 2000 52 ĐỀ THI VÀ LỜI …»i nói đầu Để thử gói lệnh lamdethi.sty tôi biên soạn một số đề toán thi Olympic, mà các học trò của

Đề thi olympic Nước Nga 43

Do đó R′ nằm trên cả hai đường thẳng HM và đường phân giác củaABC, suy ra R′ ≡ R

Từ đó dễ dàng suy ra tứ giác PBQR nội tiếp vì BPR = π2

= BQR

.7.36.Có 5 viên ngọc có trọng lượng khác nhau. Oleg biết được trọng lượng

của từng viên. Với mỗi viên ngọc x kí hiệu m (x) là trọng lượng của

nó. Dmitrii cố gắng xác định trọng lượng lớn nhất của các viên ngọc

đó. Anh ta được phép chọn 3 viên A, B, C và hỏi Oleg rằng: "Có phải

m (A) < m (B) < m (C) không?". Oleg chỉ trả lời "Đúng" hoặc "Sai".

Hỏi Dimitrii có thể xác định được trọng lượng lớn nhất sau 9 câu hỏi

hay không?

Lời giải: Chúng ta sẽ chỉ ra rằng Dimitrii sẽ không thể xác định đượckhối lượng viên ngọc lớn nhất sau 9 câu hỏi.Giả sử Dimitrii có một phương pháp để xác định được viên ngọc có khốilượng lớn nhất sau 9 câu hỏi hoặc ít hơn. Giả sử rằng sau khi Oleg trả lờicâu hỏi thứ i của Dimitrii có chính xác xi viên ngọc có trọng lượng lớnnhất thỏa mãn câu hỏi thứ i. Ta chỉ ra rằng xi+1 > max

{xi − 20, 1

2xi

}

với i = 1, 2, ..., 8

Để ý rằng có 5! = 120 cách để lấy được viên ngọc có trọng lượng lớnnhất. Sau đó với 3 viên ngọc A, B, C bất kì, đúng 1

6viên ngọc có thể có

m (A) < m (B) < m (C).Như vậy, nếu Dimitrii hỏi liệu m (A) < m (B) < m (C) và Oleg trả lời"Sai" thì Dimitrii có thể loại bỏ tối đa 20 trong số 120 khả năng có thể.Trong trường hợp xi+1 > xi −20 với mỗi i. Với mỗi xi mà phù hợp i câuhỏi đầu tiên, một phần s1 của những khả năng được loại bỏ nếu Olegtrả lời "Đúng" tới câu hỏi thứ i + 1; trong khi phần bù của một phầncủa s2 sẽ được loại bỏ nếu Oleg trả lời "Sai".Nếu |S1| 6 xi

2và Oleg trả lời "Đúng" thì ta có:

xi+1 = xi − |S1| >xi

2

Mặt khác ta có: |S2| 6 xi

2

Nếu Oleg trả lời "Sai" ta lại có xi+1 > xi

2

Do đó, nếu x1 = 120; x2 > 80; x3 > 60; x4 > 40; x5 > 20;

x6 > 10; x7 > 5; x8 > 3; x9 > 2

vnmath.com

Page 103: OLYMPIC TOÁN NĂM 2000 52 ĐỀ THI VÀ LỜI …»i nói đầu Để thử gói lệnh lamdethi.sty tôi biên soạn một số đề toán thi Olympic, mà các học trò của

44 Nguyễn Hữu Điển, ĐHKHTN Hà Nội

Từ đó, Dimitrii không thể chắc chắn rằng anh ta tìm thấy được kết quảsau 9 câu hỏi.

vnmath.com

Page 104: OLYMPIC TOÁN NĂM 2000 52 ĐỀ THI VÀ LỜI …»i nói đầu Để thử gói lệnh lamdethi.sty tôi biên soạn một số đề toán thi Olympic, mà các học trò của

Chương 8

Đề thi olympic Đài Loan

.8.37.Cho tam giác nhọn ABC, AC > BC và M là trung điểm AB. Các đường

cao AP và BQ gặp nhau ở H, đường thẳng AB và BQ cắt nhau ở R.

CMR: RH ⊥ CM.

Lời giải: Gọi S là chân đường cao hạ từ C xuống AB và X là chânđường vuông góc từ H xuống CM.Vì HPC = HQC =HXC = π

2nên H, P, Q, X và C cùng nằm trên một

đường tròn.Tương tự, vì HXM = HSM = π

2nên các điểm H, X, S và M cùng

thuộc một đường tròn.Hơn nữa, P, Q, S và M thuộc cùng một đường tròn vì chúng nằm trênđường tròn 9 điểm của tam giác ABC. Theo tính chất đối xứng, haitrong các trục đối xứng của 3 đường tròn này là AB, PQ, HX phảitrùng nhau. Vì R = AB ∩ PQ nếu R phải thẳng hàng với H và X.

Do đó: RH ⊥ CM.

.8.38.Gọi φ(h) là số các số nguyên dương n thoả mãn UCLN ( n, k) = 1 và

n ≤ k . Giả sử φ(5m−1) = 5n−1 với m, n nguyên dương nào đó. CMR:

UCLN (m, n) > 1.

Lời giải: Trong lời giải này, chúng ta sử dụng những lý thuyết về hàmφ sau:

vnmath.com

Page 105: OLYMPIC TOÁN NĂM 2000 52 ĐỀ THI VÀ LỜI …»i nói đầu Để thử gói lệnh lamdethi.sty tôi biên soạn một số đề toán thi Olympic, mà các học trò của

46 Nguyễn Hữu Điển, ĐHKHTN Hà Nội

φ(ab) = φ(a) · φ(b)(1)

φ(pα) = pα − pα−1(2)

(1) Với a, b nguyên tố cùng nhau(2)Nếu p nguyên tố và α là số nguyên dương

Giả sử phản chứng rằng UCLN (m, n) = 1Đầu tiên ta chỉ ra m là số lẻTa có: 5n ≡ 1(mod8) nếu x là chẵn

Nếu m là chẵn thì 5m − 1...8 nhưng 5m − 1 6= 8

Vì 5m − 1 6= 8, 5n − 1 = φ(5m − 1)...φ(16) = 8

hoặc φ(8).φ(P α) = 8φ(P α) ≡ 0(mod8)

Với P α > 1 là luỹ thừa của số nguyên tố lẻ P Do đó n phải chẵn, UCLN( m,n) = 1 theo như giả thiết phản chứng. Tiếp theo ta giả sử rằngP 2/(5m−1) với P nguyên tố lẻ Rõ ràng PX5, vì 5 có............... modulop,gọi d là ..................... nên ta có d/m

Lại có :d/φ(p2) và φ(5m − 1) = 5n − 1 do đó d/n.Nhưng d>1 vì 5 6= 1(modp) vàUCLN(m,n) 6=1-a (theo giả thiết phảnchứng)Do đó ,5m − 1 = 4

p∈S

p với S là tập các số nguyên tố lẻ.

Cho P là phần tử bất kỳ thuộc S. Vì 1 = (p

)= (5m

p) = ( 5

m)n và m là lẻ,

(5p) = 1

Lại có, theo luật tương hỗ : (5p)(p

5) = (−1) (5−1)(p−1)

4= 1

dẫn đến(p

5) = 1mà P ≡ 1 hoặc 4 (mod5).

Hơn nữa, ta không thể có P≡ 1(mod5) vì 5 chia hết ϕ(5m −1) = 5n−1vàP − 1 = φ(p) , điều này không xảy ra. Do đó P≡ 4 (mod 5)Suy ra −1 ≡ 5m − 1 = 4

p∈S

p ≡ 4 · 4|s|(mod5) và−1 = 5n − 1 =

ϕ(4).∏

p∈S

ϕ(p) = 2∏

p∈S

(p − 1) = 2 · 3|s|(mod5)

Từ phương trình đầu này ta có |S| phải chẵn. Nhưng từ phương trìnhthứ hai ta có:|S| ≡ 3(mod4)điều này mâu thuẫnDo đó gt của ta là sai, vậy UCLN (m, n) > 1.

.8.39.Cho A ={1, 2,......, n} . Với n ∈ N . Một tập hợp con của A được gọi là

“đã kết nối” nếu nó là số nguyên lớn nhất sao cho A chứa k tập đôi một

vnmath.com

Page 106: OLYMPIC TOÁN NĂM 2000 52 ĐỀ THI VÀ LỜI …»i nói đầu Để thử gói lệnh lamdethi.sty tôi biên soạn một số đề toán thi Olympic, mà các học trò của

Đề thi olympic Đài Loan 47

khác nhau: Sao cho giao của hai tập bất kỳ Ai và Aj là một tập đã kết

nối”

Lời giải: Gọi A1, A2, ...Ak là các tập con khác nhau của A trong giảthiếtĐặt

m = max1≤i≤n

(min Ai)

và giả thiết rằng min Ai0 = m

Mọi tập Ai có phần tử bé nhất nhỏ hoặc bằng m, do cách xác định m vàmọi tập Ai có phần tử lớn nhất lớn hơn hoặc bằng m hoặcAi ∩Ai0 = ∅

là tập không “kết nối”.Do đó, mọi cặp k(min Ai, max Ai) bằng một trong m(n+1−m)cặp

( r,s) mà 1 ≤ n ≤ m ≤ s ≤ n với mỗi cặp (r,s) ta chỉ ra rằng ít nhấtmột tập Ai có (MinAi, MaxAi) = (r, s)

Nếu có hai tập khác nhau thì giao của chúng là một tập kết nối chứa rvà s và chứa cả r, r+1,. . . ,sĐiều đó chỉ ra rằng cả hai tập đều bằng {r, r + 1, ..., s } điều này mâuthuẫn.Do đó k lớn nhất là bằng k = m(n + 1 − m) ≤ (n + 1)

⌊n2

⌋2 −⌊

n2

⌋2=

⌊n2+2n

4

.Giá trị lớn nhất này đạt được nếu Ai là tập con kết nối của A

chứa m0 mà m0 =

⌊n2

⌈n2

.8.40.Cho hàm F : N → N∗ thoả mãn

f(1) = 0

f(n) = max0≤j≤bn

2 c{f(j) + f(n − j) + j} ∀n ≥ 2

Tính f(2000).

Lời giải: Với mỗi n ∈ Z+ ta chú ý đến biểu diễn của n trong hệ nhịphân Chú ý rằng cơ sở của biểu diễn là sự thay đổi ít nhất 1 ký tự bêntrái của biểu diễn đó, vì thế cơ sở được bắt đầu bởi a1

Chúng ta gọi giá trị thập phân của cơ sở này là giá trị “đuôi” của n vớimỗi một xuất hiện trong biểu diễn nhị phân của n, nếu nó đại diện chosố 2k, 2k − k

2là một giá trị “place” của n.

vnmath.com

Page 107: OLYMPIC TOÁN NĂM 2000 52 ĐỀ THI VÀ LỜI …»i nói đầu Để thử gói lệnh lamdethi.sty tôi biên soạn một số đề toán thi Olympic, mà các học trò của

48 Nguyễn Hữu Điển, ĐHKHTN Hà Nội

Đặt g(n) là tổng giá trị “tail” và “place” của nTa CMR: F(n) = g(n)Đặt g(0) = 0 rõ ràng g(1) =0. . .Đầu tiên ta CMR: g(n) ≥ g(j)+g(n−j)+j∀n, j thoả mãn 0 ≤ j ≤

⌊n2

Điều này là hiển nhiên với j=0 vì g(0) =0Bây giờ ta bổ sung số kí tự của n-jVới trường hợp cơ bản ( khi n-j có ký tự 1) ta chỉ có thê thay n-j. Trongtrường hợp (n,j) = (2,1) hoặc ( n, j ) = (1;0)Trường hợp (1) dễ có là đúng.TH1: n-j và j có cùng số các ký tự là k +1Đặt a và b lần lượt là số các chữ số 1 ở ngoài cùng bên trái của n-j và j.ta sẽ chỉ ra rằng g(n) = g(a+ b)−2k+1 ≥ g(2k +a)+ g(2k + b)+ (2k + b)

Để ý rằng: g(a + b) ≥ g(a) + g(b) + b( điều này đúng do giả thiết)Do đó đủ để ta có: g(a+ b+2k+1)− g(a+ b) ≥ g(2k + a)− g(a)+ g(2k +

b) − g(b) + 2k (2)

Ở vế phải ta có

g(2k + a) = g(a) + 2k · k2

+ a

g(2k + b) = g(b) + 2k · k2

+ b

Do đó vế phải bằng: 2k · k2

+ a + 2k · k2

+ b + 2k = 2k+1 · k+12

+ a + b

Còn đối với vế trái: vì a < 2k, b < 2k

Biểu diễn nhị phân của a + b + 2k+1 giống như biểu diễn nhị phâncủaa+b với việc thêm 1 vào 2k+1 vị tríDo đó g(a + b + 2k+1)bằng g(a+b)cộngvới giá trị “đuôi” của a+b vàcộng2k+1 · k+1

2.

Vì thế g(a + b + 2k+1) − g(a + b) bằng vế phải, ta đã chứng minh được(2)

TH2: n-j có số ký tự nhiều hơn jGiả sử n-j có k +1 kí tự vì a = n − j − 2k . Ta cần CM rằng: g(a +

j + 2k) ≥ g(a + 2k) + g(j) + j. Ta đã biết theo giả thiết:g(a + j) ≥g(a) + g(j) + min {a, j} .Do đó ta cần chứng minh điều kiện đủ là:g?(a + j + 2k) − g(a + j) ≥g(a + 2k) − g(a) + j − min {a, i} (3)Theo TH1 vế phải: g(a + 2k) − g(a) = 2k · k

2+ a

Do đó ,vế phải bằng: 2k · k2+a+ j−min {a, j} = 2k · k

2+max {a, j}

vnmath.com

Page 108: OLYMPIC TOÁN NĂM 2000 52 ĐỀ THI VÀ LỜI …»i nói đầu Để thử gói lệnh lamdethi.sty tôi biên soạn một số đề toán thi Olympic, mà các học trò của

Đề thi olympic Đài Loan 49

Với vế trái của (3): nếu a + j < 2k

chẳng hạn như 2k ký tự không có trong tổng a + j + 2k

thì g = (a + j + 2k) = g(a + j) + 2k · k2

+ a + j

do đó VT(3)≥ VP(3)Mặt khác nếu có2k kí tự không có trong tổng a + j + 2k thì g(a +

j + 2k) = g(a + j) + 2k+1 · k+12

− 2k · k2

Vì thế, vế trái = 2k+1 · k+12

− 2k · k2

= 2k · k2

+ 2k > 2k · k2

+ max {a, j}Do đó(3) đúng.Giả thiết được CM hoàn toànTa CMR: Dấu bằng xảy ra khi g(n) = g(j) + g(n − j) + jvới 1 số

giá trị jĐặt 2k là luỹ thừa lớn nhất của 2 nhỏ hơn n và đặt j = n − 2k thìg(n) = g(n − 2k) + g(2k) = g(n − j) + n − 2k = j

Điều đó cho thấy f(n) = g(n)∀n

Vậy với việc tìm giá trị “place” và giá trị “đuôi” của 2000 (với biểu diễnnhị phân 11111010000) ta có:f (2000) = 10864.

vnmath.com

Page 109: OLYMPIC TOÁN NĂM 2000 52 ĐỀ THI VÀ LỜI …»i nói đầu Để thử gói lệnh lamdethi.sty tôi biên soạn một số đề toán thi Olympic, mà các học trò của

Chương 9

Đề thi olympic Thổ Nhĩ Kỳ

.9.41.Tìm các bộ 4 số xếp theo thứ tự (x, y, z, w) của các số nguyên với 0 ≤ x,

y, z, w ≤ 36 để

x2 + y2 ≡ z3 + w3 (mod 37)

Lời giải: Tất cả các đồng dư sẽ là mod 37. Với mỗi k trong khoảng 0đến 36 ta tìm được các cặp số nguyên (x, y) với 0 ≤ x, y ≤ 36 thỏa mãnx2 + y2 ≡ k. Chú ý rằng điều này tương đương với (x− 6y)(x+6y) ≡ k.

Trước hết ta xem xét trường hợp k = 0. Với mỗi y ∈ {0, 1, . . . , 36} tacó (x− 6y)(x + 6y) ≡ 0 nếu và chỉ nếu x ≡ ±6y.. Vì vậy có một cặp (x,y) với y = 0 để x2 + y2 ≡ 0 (đó là (x, y) = (0, 0)), và với bất kỳ y nàokhác có 2 cặp (x, y) như vậy. Do vậy có tổng cộng 2.36 + 1 = 73 cặp(x, y) để x2 + y2 ≡ 0.

Giờ ta xét trường hợp k 6= 0. Để a ≡ x + 6y, b ≡ x − 6y. Với bất kỳgiá trị a ∈ {1, 2, . . . , 36} có chính xác một giá trị b ∈ {1, 2, . . . , 36} đểab ≡ k. Với mỗi cặp (a, b) trong 36 cặp tương ứng với một nghiệm (x, y)duy nhất vì ta phải có x ≡ (a + b)2−1, y ≡ (a + b)12−1. Do vậy phươngtrình (x − 6y)(x + 6y) ≡ k có chính xác là 36 nghiệm (x, y) khi k 6= 0.

Ta xem xét số cặp 4 (x, y, z, w) để x2 + y2 ≡ z3 + w3 ≡ 0. Có 3căn bậc 3 r1, r2, r3 của 1 mod 37, đó là: Nếu ta để g là một phần tửnguyên thủy mod 37 thì căn bậc 3 là 1, g12, g24. Với bất kỳ z nào, ta

vnmath.com

Page 110: OLYMPIC TOÁN NĂM 2000 52 ĐỀ THI VÀ LỜI …»i nói đầu Để thử gói lệnh lamdethi.sty tôi biên soạn một số đề toán thi Olympic, mà các học trò của

Đề thi olympic Thổ Nhĩ Kỳ 51

có z3 + w3 ≡ 0 nếu và chỉ nếu w bằng −r1z,−r2z hay −r3z. Do vậy có109 cặp (z, w) để z3 + w3 ≡ 0, mỗi cặp để z = 0 và 3 cặp để z = z0

đối với mỗiz0 ∈ {1, 2, . . . , 36}. Ở trên ta đã tỉm ra rằng có chính xác73 cặp (x, y) để x2 + y2 ≡ 0.. Do vậy có 109.73 bộ 4 (x, y, z, w)đểx2 + y2 ≡ z3 + w3 ≡ 0.Với mỗi cặp trong 372 − 109 cặp (z, w) để z3 + w3 ≡ 0 có chính xác 36cặp (x, y) để x2 + y2 ≡ z3 + w3. Vậy nên có (372 − 109).36 bộ 4 (x, y, z,w) để x2 + y2 ≡ z3 + w3 ≡ 0. Vì vậy, có 109.73+ (372 − 109).36 = 53317

bộ 4 (x, y, z, w) để x2 + y2 ≡ z3 + w3.

.9.42.Cho một vòng tròn tâm O, 2 đường tiệm cận xuất phát từ điểm S nằm

bên ngoài đường tròn có tiếp điểm là P, Q. Đường thẳng SO giao với

đường tròn tại A, B với B gần S hơn A. Cho X là một điểm nằm trong

cung nhỏ PB và đường SO giao với các đường QX và PX lần lượt tại C,

D. Chứng minh rằng:1

AC+

1

AD=

2

AB

Lời giải: Kéo dài tia PC cho cắt với cung QB tại Y. Bằng phép đốixứng cung BX và BY, nó chỉ ra rằng CPB = Y PB = BPX = BPD.Do vậy, PB là phân giác bên trong của CPD. Áp dụng định lý đườngphân giác trong và phân giác ngoài ta tìm ra được:

BC

BD=

PC

PD=

AC

AD

Thay BC = AB - AC, BD = AD - AB và chia nửa bên trái, bên phảibởi đường AB, ta có

AB − AC

AB.AC=

AD − AB

AD.AB

Điều này có nghĩa là

1

AC− 1

AB=

1

AB− 1

AD

Điều này tương đương với đẳng thức cần chứng minh.

vnmath.com

Page 111: OLYMPIC TOÁN NĂM 2000 52 ĐỀ THI VÀ LỜI …»i nói đầu Để thử gói lệnh lamdethi.sty tôi biên soạn một số đề toán thi Olympic, mà các học trò của

52 Nguyễn Hữu Điển, ĐHKHTN Hà Nội

.9.43.Với 2 số nguyên dương bất kỳ n, p. Hãy chứng minh rằng có chính xác

(p + 1)n+1 − pn+1 hàm số

f : {1, 2, . . . , n} −→ {−p,−p + 1, . . . , p}

để |f(i) − f(j)| ≤ p với tất cả i, j ∈ {1, 2, . . . , n}.

Lời giải: Với m ∈ {−p,−p + 1, . . . , p}, có (min{p + 1, p − m + 1})n làhàm số thỏa mãn điều kiện đã cho bao gồm những giá trị chỉ nằm trong{m, . . . , m+ p}. Dĩ nhiên, (min{p+1, p−m+1})n là hàm số có các giátrị chỉ nằm trong {m + 1, . . . , m + p}. Do vậy, chính xác

(min{p + 1, p − m + 1})n − (min{p, p − m})n

là hàm số thỏa mãn điều kiện đã cho với giá trị m nhỏ nhất.Biểu thức này bằng với (p+1)n − pn đối với mỗi giá trị của p+1, m ≤ 0

và bằng (p + 1 −m)n − (p −m)n khi m > 0. Do đó, tổng của biểu thứckhi m ≤ 0 là

(p + 1)((p + 1)n − p)n

trong khi tổng biểu thức khi m > 0 là tổng

p∑

m=1

((p + 1 − m)n − (p − m)n) = pn

Cộng 2 tổng này lại ta được tổng các hàm số thỏa mãn điều kiện đã cholà (p + 1)n+1 − pn+1.

.9.44.Trong tam giác nhọn ABC có bán kính đường trong ngoại tiếp R. Đường

cao AD, BE, CF lần lượt có độ dài là h1, h2, h3. Nếu t1, t2, t3 lần lượt là

chiều dài các tiếp tuyến từ A, B, C tới đường tròn ngoại tiếp của tam

giác DEF. Hãy chứng minh:

3∑

i=1

(ti√hi

)2 ≤ 3

2R

.

Lời giải: Cho H là trực tâm của tam giác ABC và X, Y, Z lần lượt làtrung điểm của AH, BH, CH. Bởi đường tròn ngoại tiếp của tam giác

vnmath.com

Page 112: OLYMPIC TOÁN NĂM 2000 52 ĐỀ THI VÀ LỜI …»i nói đầu Để thử gói lệnh lamdethi.sty tôi biên soạn một số đề toán thi Olympic, mà các học trò của

Đề thi olympic Thổ Nhĩ Kỳ 53

DEF là đường tròn 9 điểm của tam giác ABC, nó qua các điểm X, Yvà Z. Do đó t21 = AX.AD = AX.h1 hoặc ( t1√

h1)2 = AX. Ta có thể tìm

được các biểu thức tương tự với BX và CX. Vậy nên bất dăbgr thức cầnchứng minh tương đương với

AX + BX + CX ≤ 3

2R

(nhân mỗi vế với 2):

AH + BH + CH ≤ 3R.

Cho A = α, B = β, C = ϕ ta có:

AH =AF

sin β=

AC. cos α

sin β= 2R cos α

Tương tự, BH = 2R cos β và CH = 2R cos ϕ, bất đẳng thức cần chứngminh tương đương với

cos α + cos β + cos ϕ ≤ 3

2.

Nhớ rằng tam giác ABC là tam giác nhọn và hàm t −→ cos t là lõmtrong khoảng (0, π

2). Do đó, theo bất đẳng thức Jensen ta có vế trái của

bất đẳng thức cuối cùng này đạt cực đại khi 3 góc đều bằng π3, trong

trường hợp vế trái bằng 32. Vậy bất dẳng thức cuối cùng này là đúng

đồng thời bất đẳng thức cần chứng minh cũng là đúng.

.9.45.(a) Chứng minh rằng với mỗi số nguyên dương n, số cặp của số nguyên

xếp theo thứ tự thỏa mãn x2 − xy + y2 = n là hữu hạn và chia hết cho

6.] [(b) Tìm tất cả các cặp số nguyên (x, y) xếp theo thứ tự thỏa mãn

x2 − xy + y2 = 727.

Lời giải: (a) Bất kỳ nghiệm (x, y) đều phải thỏa mãn bất đẳng thức:

n = x2 − xy + y2 =(x − y)2

2+

x2 + y2

2≥ x2 + y2

2

và rất nhiều cặp hữu hạn (x, y) thỏa mãn được điều kiện này. Do vậycó rất nhiều nghiệm hữu hạn.

vnmath.com

Page 113: OLYMPIC TOÁN NĂM 2000 52 ĐỀ THI VÀ LỜI …»i nói đầu Để thử gói lệnh lamdethi.sty tôi biên soạn một số đề toán thi Olympic, mà các học trò của

54 Nguyễn Hữu Điển, ĐHKHTN Hà Nội

Tiếp đến ta chứng minh rằng số nghiệm chia hết cho 6. Nếu (x, y) lànghiệm thì (y, y − x) cũng là nghiệm. Phép biến đổi tuyến tính này làkhả nghịch, do đó nó hoán vị tất cả các nghiệm và ta có thể chia cácnghiệm ra thành các lớp với mỗi lớp đó ở dưới dạng:

(x, y), (y, y − x), (y − x,−x), (−x,−y), (−y, x − y), (x− y, x)

đối với một số nghiệm(x, y) ban đầu. Rõ ràng để chứng minh không có2 trong 6 nghiệm ở mỗi lớp bằng nhau trừ khi x = y = 0 là không thể.Do vậy mỗi lớp có 6 phần tử riêng biệt và kết quả là được chứng minh.(b) Bất kỳ nghiệm nào với x2−xy+y2 = 727 ta có thể áp dụng được phépbiến đổi (x, y) −→ (y, y − x) như ở phần (a) và có thể (x, y) 7−→ (y, x),để có được nghiệm (x, y) khác với y ≤ 0 ≤ x ≤ |y|. Giờ ta tìm tất cảcác nghiệm như vậy với y ≤ 0 ≤ x ≤ |y|. Sắp xếp lại đẳng thức cần tìmta được x2 − xy + y2 − 727 = 0. Xem xét đẳng thức này là một toànphương trong y, ta có thể áp dụng phương trình bậc 2 để tìm ra rằng:

y =x ±

√2908 − 3x2

2

Do vậy, 2908 − 3x2 phải là số chính phương, và nó không thể chia hếtcho 3. Do 3x2 ≤ x2 − xy + y2 = 727 ta biết được thêm rằng 2181 ≤2908 − 3x2 ≤ 2908 với 46 <

√2908 − 3x2 < 54. Kiểm chứng những khả

năng ta thấy rằng chỉ√

2908 − 3x2 = 49 có nghiệm nguyên x, kết quảlà ta có nghiệm duy nhất(13,−18) của phương trình.Do đó bất kỳ nghiệm nào cũng có thể biến đổi thành (13,−18) bằngviệc áp dụng 2 sơ đồ mô tả ở trên. Như vậy bất kỳ nghiệm nào trong(13,−18) hoặc (−18, 13) dưới biến đổi (x, y) 7−→ (y, y−x), có nghĩa tấtcả các nghiệm tới x2 − xy + y2 = 727 là:

(13,−18), (−18,−31), (−31,−13), (−13, 18), (18, 31), (31, 13),

(−18, 13), (13, 31), (31, 18), (18,−13), (−13,−31), (−31,−18).

.9.46.Cho tam giác ABC, các đường phân giác trong và ngoài của góc A lần

lượt cắt đường thẳng BC tại D và E. Cho F là một điểm ( khác điểm

A) ở đó đường thẳng AC tiếp xúc với đường tròn ω có đương kính DE.

vnmath.com

Page 114: OLYMPIC TOÁN NĂM 2000 52 ĐỀ THI VÀ LỜI …»i nói đầu Để thử gói lệnh lamdethi.sty tôi biên soạn một số đề toán thi Olympic, mà các học trò của

Đề thi olympic Thổ Nhĩ Kỳ 55

Vẽ tiếp tuyến tại A với đường tròn ngoại tiếp của tam giác ABF và giao

với đường tròn ω tại A và G. Chứng minh rằng AF = AG.

Lời giải: Ta chứng minh cho trường hợp C, B và E thẳng hàng. Theothứ tự đó ta chứng minh cho các trường hợp khác tương tự. Gọi O làtâm của ω. Theo định lý về đường phân giác trong và ngoài của tamgiác, ta có:

CD

DB=

CA

AB=

CE

BE

Do vậy CD(CE −CB) = CD.BE = CE.DB = CE(CB −CD) hoặc (cộng CD (CE + CB) cho cả 2 vế).

2CD.CE = CB(CD + CE)

Bởi vì CD + CE = 2CO, ta có: CD.CE = CB.CO Mặt khác, theođịnh lý tích điểm áp dụng với C và ω ta có: CD.CE = CA.CF , suy raCB.CO = CA.CF . Do vậy, theo định lý tích điểm với các điểm A, B, O,F nằm trên đường tròn ω1 nào đó. Ta thực hiện phép nghịch đảo A vớitia AO. ω là một vòng tròn qua A, điểm trực giao với AO và bao gồm 1điểm P nằm trên AO vowis AP = 2AO. Do vậy, ảnh của nó dưới phépnghịch đảo là một đường trực giao với AO và bao gồm 1 điểm P’ trêntia AO với AP ′ = AO/2. Nói cách khác, ảnh l1 của ω là đường trungtrực của AO. Kế tiếp, phép nghịch đảo đưa ω1 (vòng tròn đi qua A, quaO và tiếp xúc với AG) tới l2 không đi qua A, qua O và song song vớiAG. Suy ra phép nghịch đảo đưa F, giao giữa ω và ω1, tới giao F’ của l1

và l2; hơn nữa phép nghịch đảo (phép biến đổi ngược) đưa G, giao củaω1 và AG, tới giao của l1 và AG. Phép dối xứng qua trung điểm của AO

đưa l1 tới chính nó và l2 tới AG; Do vậy, sự phản chiếu này đưa OF ′ tớiAG′, có nghĩa là OF’ = AG’. Bởi F’ nằm trên trung trực của AO, tacũng có OF’ = AG’. Vậy nên AF’ = AG’, điều này đòng nghĩa với AF= AG. Kết quả được chứng minh.

.9.47.Hãy chỉ ra rằng có thể cắt bất kỳ lăng trụ tam giác có chiều dài vô hạn

bởi 1 mặt phẳng cho ra kết quả thiết diện là một tam giác đều.

vnmath.com

Page 115: OLYMPIC TOÁN NĂM 2000 52 ĐỀ THI VÀ LỜI …»i nói đầu Để thử gói lệnh lamdethi.sty tôi biên soạn một số đề toán thi Olympic, mà các học trò của

56 Nguyễn Hữu Điển, ĐHKHTN Hà Nội

Lời giải: Giả sử rằng một mặt phẳng trực giao với các cạnh của lăngtrụ tại A, B, C và đặt a = BC, b = CA, c = AB và không mất tínhtổng quát giả sử a ≤ b ≤ c. Với t ≥ 0, xác định:

f(t) =

a2 + (t +√

c2 − b2 + t2)2 −√

c2 + t2

Khi đó:f(0) =

√a2 + c2 − b2 −

√c2 ≤ 0

Mặt khác, ta có f(b) =√

a2 + (b + c)2 −√

c2 + b2 > 0. Bởi f là liên tụcnên tồn tại t0 sao cho f(t0) = 0. Bây gờ ta cho B’ nằm trên 1 cạnh vớiB, cách B 1 khoảng là t. C’ nằm cùng cạnh với C và cách C 1 khoảng là√

c2 − b2 + t20 và nằm trên giá đối diện của mặt phẳng (ABC) tính từB’. Theo định lý Pitago ta có:

AB′ =√

c2 + t20; AC ′ =√

b2 + (c2 − b2 + t20) =√

c2 + t20;

B′C ′ =

a2 + (t0 +√

c2 − b2 + t20)2 =

c2 + t20

Vì vậy mặt phẳng (AB’C’) đáp ứng được yêu cầu đề bài.

.9.48.Cho hình vuông ABCD, các điểm M, N, K, L lần lượt nằm trên các cạnh

AB, BC, CD, DA sao cho MN song song với LK và khoảng cách giữa

MN và LK bằng AB. Hãy chỉ ra rằng các đường tròn ngoại tiếp của tam

giác ALM và NCK là giao nhau còn các đường tròn ngoại tiếp của tam

giác LDK và MBN thì không giao nhau.

Lời giải: Hướng tứ giác sao cho AB nằm ngang và nằm phía trên CD,ở đó A nằm phía tây so với B. Trước hết ta cho là AL > BN, nói cáchkhác N nằm phía bắc (mặc dù không nhất thiết phải chính bắc) so vớiL. Giả sử ngược lại thì có 1 đoạn nằm ngang với điểm cuối trái L vàđiểm cuối phải trên MN với độ dài ≤ AB. Mặt khác, độ dài của đoạnnày lớn hơn khoảng cách giữa LK và MN , điều này đã được giả thiếtlà AB. Do vậy ta có sự trái ngược nhau và AL > BN. Tương tự ta cũngcó thể rút ra được AM > DK.Dựng P và Q sao cho các tứ giác BMPN và DKQL là các hình chữ nhật.Ta biết từ phần trên rằng P nằm ở đông bắc so với Q. Dựng R và S sao

vnmath.com

Page 116: OLYMPIC TOÁN NĂM 2000 52 ĐỀ THI VÀ LỜI …»i nói đầu Để thử gói lệnh lamdethi.sty tôi biên soạn một số đề toán thi Olympic, mà các học trò của

Đề thi olympic Thổ Nhĩ Kỳ 57

cho R ở phía đông nam so với Q và sao cho tứ giác PRQS là hình chữnhật có các cạch song song với các cạch của hình vuông ABCD.Để chỉ ra rằng các đường tròn ngoại tiếp của tam giác ALM và NCK làgiao nhau, ta thấy rằng các đường tròn bị chặn bởi các đường tròn ngoạitiếp tam giác ALM và NCK lần lượt bao hàm các hình chữ nhật ALRMvà CKSN. Vì vậy các đường tròn này đều chứa hình chữ nhật PRQS. Docác miền trong của các đường tròn ngoại tiếp tam giác ALM và NCKlà giao nhau vậy nên các đường tròn ngoại tiếp cũng phải giao nhau.Giờ ta chứng minh rằng đường tròn ngoại tiếp ω1 của tam giác MBN vàđường tròn ngoại tiếp ω2 của tam giác LDK là không giao nhau. Chúý rằng chúng cũng lần lượt là đường tròn ngoại tiếp của hình chữ nhậtBMPN và DKQL. Cho l1 là tiếp tuyến với ω1 tại P, l2 là tiếp tuyến vớiω2 tại Q. Vì MN song song với LK do vậy BP song song với QD. Dol1 vuông góc với BP và l2 vuông góc với QD nên ta có l1 song song vớil2. Vậy nên mỗi điểm của ω1 nằm trên hoặc giá bên phải của l1 cũngnằm phía bên phải của l2; mặt khác mỗi điểm trên ω2 nằm trên hoặcbên trái của l2. Vậy nên ω1 và ω2 không thể giao nhau.

.9.49.Cho f : R −→ R là một hàm thỏa mãn :

|f(x + y) − f(x) − f(y)| ≤ 1

vittcx, y ∈ R. Chứng minh rằng tồn tại một hàm g :R −→ R thỏa mãn

|f(x) − g(x)| ≤ 1 với mọi x ∈ R, và g(x + y) = g(x) + g(y) với mọi

x, y ∈ R.

Lời giải: Ta cho hàm

g(x) = limn→∞

f(2nx)

2n

thỏa mãn điều kiện đầu bài. Việc trước tiên cần làm là chỉ ra rằng tồntại với mọi x. Thực tế ta có thể chứng minh được điều này và đồng thờicũng chứng minh được

|f(x) − g(x)| ≤ 1

vnmath.com

Page 117: OLYMPIC TOÁN NĂM 2000 52 ĐỀ THI VÀ LỜI …»i nói đầu Để thử gói lệnh lamdethi.sty tôi biên soạn một số đề toán thi Olympic, mà các học trò của

58 Nguyễn Hữu Điển, ĐHKHTN Hà Nội

đối với tất cả x. Trước hết thấy rằng đặt x = y = 2mx0 trong bất đẳngthức đã cho của f ta có:

∣∣f(2m+1x0) − 2f(2mx0)

∣∣ ≤ 1

Chia cho 2m+1 ta có:∣∣∣∣

f(2m+1x0)

2m+1− f(2mx0)

2m

∣∣∣∣≤ 1

2m+1

Với bất kỳ x xác định nào, xem xét tổng vô hạn:∞∑

m=0

(f(2m+1x)

2m+1− f(2mx)

2m

)

Vì các trị tuyệt đối của các số hạng bị chặn bởi một dãy 12, 1

4, . . ., tổng

của chúng tiến tới 1, tổng này hội tụ tuyệt đối và cũng bị chặn bởi 1.Mặt khác, theo định nghĩa tổng vô hạn bằng

limn→∞

n∑

m=0

(f(2m+1x)

2m+1− f(2mx)

2m

)

Tổng vô hạn trong giới hạn bằng (f(2n+1x)2n+1 − f(x)), có nghĩa là

limn→∞

(f(2n+1x)

2n+1− f(x)

)

Giờ ta có thể lấy ra hằng số f(x) để đạt(

limn→∞

f(2n+1x)

2n+1

)

−f(x)

Suy ra giới hạn trong biểu thức cuối cùng này hội tụ và ngẫu nhiên nócũng chính là giới hạn ta muốn dùng để xác định g(x). Hơn nữa, ta thấyở phần trên rằng lượng cuối cùng lớn nhất bằng 1, vì vậy ta có:

|g(x) − f(x)| ≤ 1

Tiếp tục thấy rằng g(x + y) = g(x) + g(y)với mọi x, y. Thấy rằng:

g(x + y) − g(x) − g(y) = limn→∞

f(2n(x + y))

2n− lim

n→∞

f(2nx)

2n− lim

n→∞

2ny

2n

= limn→∞

f(2n(x + y)) − f(2nx) − f(2ny)

2n

vnmath.com

Page 118: OLYMPIC TOÁN NĂM 2000 52 ĐỀ THI VÀ LỜI …»i nói đầu Để thử gói lệnh lamdethi.sty tôi biên soạn một số đề toán thi Olympic, mà các học trò của

Đề thi olympic Thổ Nhĩ Kỳ 59

Theo điều kiện đã cho,

|f(2n(x + y)) − f(2nx) − f(2ny)| ≤ 1

với n bất kỳ, hằng số trong gới hạn của biểu thức cuối cùng nằm trongkhoảng − 1

2n và 12n . Vì

limn→∞

1

2n= 0

suy ra giới hạn trong biểu thức cuối là bằng 0. Do vậy g(x + y) =

g(x) + g(y).

vnmath.com

Page 119: OLYMPIC TOÁN NĂM 2000 52 ĐỀ THI VÀ LỜI …»i nói đầu Để thử gói lệnh lamdethi.sty tôi biên soạn một số đề toán thi Olympic, mà các học trò của

Nguyễn Hữu Điển

OLYMPIC TOÁN NĂM 2000

33 ĐỀ THI VÀ LỜI GIẢI(Tập 3)

NHÀ XUẤT BẢN GIÁO DỤC

vnmath.com

Page 120: OLYMPIC TOÁN NĂM 2000 52 ĐỀ THI VÀ LỜI …»i nói đầu Để thử gói lệnh lamdethi.sty tôi biên soạn một số đề toán thi Olympic, mà các học trò của

2

vnmath.com

Page 121: OLYMPIC TOÁN NĂM 2000 52 ĐỀ THI VÀ LỜI …»i nói đầu Để thử gói lệnh lamdethi.sty tôi biên soạn một số đề toán thi Olympic, mà các học trò của

Lời nói đầu

Để thử gói lệnh lamdethi.sty tôi biên soạn một số đề toán thi Olympic, mà

các học trò của tôi đã làm bài tập khi học tập LATEX. Để phụ vụ các bạn ham

học toán tôi thu thập và gom lại thành các sách điện tử, các bạn có thể tham

khảo. Mỗi tập tôi sẽ gom khoảng 30 bài với lời giải. Tập này có sự đóng góp

của Nguyễn Văn Hậu, Lê Thị Thu Hiền, Nguyễn Trung Hiếu, Nguyễn Thị Mai

Hoa, Nguyễn Văn Huy, Nguyễn Thương Huyền

Rất nhiều bài toán dịch không được chuẩn, nhiều điểm không hoàn toàn

chính xác vậy mong bạn đọc tự ngẫm nghĩ và tìm hiểu lấy. Nhưng đây là nguồn

tài liệu tiếng Việt về chủ đề này, tôi đã có xem qua và người dịch là chuyên về

ngành Toán phổ thông. Bạn có thể tham khảo lại trong [1].

Rất nhiều đoạn vì mới học TeX nên cấu trúc và bố trí còn xấu, tôi không

có thời gian sửa lại, mong các bạn thông cảm.

Hà Nội, ngày 2 tháng 1 năm 2010

Nguyễn Hữu Điển

51GD-05

89/176-05 Mã số: 8I092M5

vnmath.com

Page 122: OLYMPIC TOÁN NĂM 2000 52 ĐỀ THI VÀ LỜI …»i nói đầu Để thử gói lệnh lamdethi.sty tôi biên soạn một số đề toán thi Olympic, mà các học trò của

Mục lục

Lời nói đầu . . . . . . . . . . . . . . . . . . . . . . . . . . . . . . . . . . . . . . . . . . . . 3

Mục lục . . . . . . . . . . . . . . . . . . . . . . . . . . . . . . . . . . . . . . . . . . . . . . . 4

Chương 1. Đề thi olympic Hoa Kỳ . . . . . . . . . . . . . . . . . . . . . . . . . 5

Chương 2. Đề thi olympic Việt Nam . . . . . . . . . . . . . . . . . . . . . . 11

Chương 3. Đề thi olympic Châu Á Thái Bình Dương . . . . . . 20

Chương 4. Đề thi olympic Áo - Balan . . . . . . . . . . . . . . . . . . . . . 26

Chương 5. Đề thi olympic Địa Trung Hải . . . . . . . . . . . . . . . . . 30

Chương 6. Đề thi olympic Petecbua . . . . . . . . . . . . . . . . . . . . . . . 34

Chương 7. Đề thi olympic Anh. . . . . . . . . . . . . . . . . . . . . . . . . . . . 40

Tài liệu tham khảo. . . . . . . . . . . . . . . . . . . . . . . . . . . . . . . . . . . . 42vnmath.com

Page 123: OLYMPIC TOÁN NĂM 2000 52 ĐỀ THI VÀ LỜI …»i nói đầu Để thử gói lệnh lamdethi.sty tôi biên soạn một số đề toán thi Olympic, mà các học trò của

Chương 1

Đề thi olympic Hoa Kỳ

.1.1. Một bộ bài có R quân đỏ, W quân trắng và B quân xanh. Một người chơi

thực hiện việc rút các quân bài ra khỏi bộ bài. Với mỗi lượt, anh ta chỉ

được phép rút đúng 1 lá bài, và phải chịu một số tiền phạt cho lượt rút

bài đó: - Nếu lá bài được rút có màu xanh, tiền phạt bằng số quân trắng

còn lại trong bộ bài - Nếu là bài được rút có màu trắng, tiền phạt bằng

hai lần số quân đỏ còn lại trong bộ bài - Nếu lá bài được rút có màu đỏ,

tiền phạt bằng ba lần số quân xanh còn lại trong bộ bài Hãy xác định

tổng số tiền phạt tối thiểu mà người chơi phải trả (phụ thuộc vào R, W,

B) và tìm tất cả các cách chơi để có thể đạt được số tiền phạt đó

Lời giải: Ta sẽ chứng minh số tiền phạt tối thiểu phải trả là min(BW,2

WR,3RB) Dĩ nhiên số tiền phạt này là đạt được, tương ứng với 1 trong

3 cách rút bài sau:(bb..bbrr..rr ww.. ww);(rr..rr ww.. wbb. . . )( ww..

wwbb..rr)Với mỗi một cách rút bài, ta định nghĩa chuỗi xanh là một

đoạn liên tiếp các quân bài màu xanh được rút ra khỏi bộ bài (tức trong

một số lượt lien tiếp, ta chỉ rút quân xanh ra). Tương tự, ta có định

nghĩa chuỗi đỏ, chuỗi trắng.

Bây giờ ta sẽ chứng minh 3 bổ đề:

Bổ đề 1: Với mỗi cách rút bài cho trước, ta có thể thực hiện 1 cách

rút bài khác, trong đó 2 chuỗi cùng màu được “gộp” vào nhau mà không

làm tăng số tiền phạt

vnmath.com

Page 124: OLYMPIC TOÁN NĂM 2000 52 ĐỀ THI VÀ LỜI …»i nói đầu Để thử gói lệnh lamdethi.sty tôi biên soạn một số đề toán thi Olympic, mà các học trò của

6 Nguyễn Hữu Điển, ĐHKHTN Hà Nội

Ta sẽ chứng minh trong trường hợp gộp 2 chuỗi đỏ, các trường hợp khác

hoàn toàn tương tự. Giả sử giữa 2 chuỗi đỏ có w quân trắng và b quân

xanh. Bây giờ, nếu ta chuyển một quân đỏ từ chuỗi thứ nhất sang chuỗi

thứ 2, số tiền phạt sẽ tăng them 2w – 3b (do ở mỗi lượt rút quân trắng

phải tăng them tiền phạt là 2 bởi sự xuất hiện của 1 quân đỏ mới, và

quân đỏ được chuyển đi nằm sau b quân xanh nên không phải chịu 3b

tiền phạt).

Nếu , ta chỉ việc chuyển tất cả các quân đỏ từ chuỗi 1 sang chuỗi 2.

Ngược lại, ta sẽ chuyển tất cả các quân đỏ từ chuỗi 2 sang chuỗi 1.

Trong cả 2 trường hợp, 2 chuỗi đỏ đã được gộp vào nhau và số tiền phạt

không bị tăng thêm.

Bổ đề 2: Cách chơi tối ưu không tồn tại chuỗi (tức không xảy ra trường

hợp rút 1 quân đỏ ngay sau 1 quân trắng)

Điều này là hiển nhiên, vì nếu xuất hiện lượt rút bài như vậy ta thay

đổi chuỗi bằng chuỗi , ta thu được 1 cách chơi mới có số tiền phạt nhỏ

hơn

Bổ đề 3: Cách chơi tối ưu (ít tiền phạt nhất) sẽ có ít hơn 5 chuỗi

Giả sử tồn tại 1 cách chơi tối ưu có 5 chuỗi trở lên. Giả thiết rằng quân

bài đầu tiên được rút có màu đỏ (các trường hợp khác chứng minh tương

tự). Bây giờ, ta giả sử rằng các chuỗi được rút có giá trị (theo thứ tự,

chứng minh tương tự trong các trường hợp khác).

Theo bổ đề 1, ta có thể gộp 2 chuỗi đỏ, hoặc 2 chuỗi trắng lại với nhau

mà không làm tăng số tiền phạt. Nhưng cách chơi hiện tại là tối ưu, do

vậy ta phải có: (1)

Gộp 2 chuỗi trắng lại và ta được 1 cách rút bài cũng có số tiền phạt tối

thiểu: . Theo bổ đề 1, ta có thể gộp 2 chuỗi đỏ với nhau mà số tiền phạt

không tăng them. Nhưng do cách chơi này là tối ưu, nên ta phải có ,

mâu thuẫn với (1). Vậy điều giả sử là sai, bổ đề 2 được chứng minh.

Tức là bất kì cách chơi tối ưu nào cũng chỉ có tối đa 4 chuỗi. Kết hợp

với bổ đề 2, cách chơi tối ưu nếu phải rút quân đỏ đầu tiên là

Cách chơi này là tối ưu khi và chỉ khi và , Tương tự, ta cũng có cách

chơi giống như trên nếu quân đầu tiên đuợc rút là quân trắng hoặc quân

xanh./.

vnmath.com

Page 125: OLYMPIC TOÁN NĂM 2000 52 ĐỀ THI VÀ LỜI …»i nói đầu Để thử gói lệnh lamdethi.sty tôi biên soạn một số đề toán thi Olympic, mà các học trò của

Đề thi olympic Hoa Kỳ 7

cả các số thực m để phương trình

(

x2 − 2mx − 4(m2 + 1)) (

x2 − 4x − 2m(m2 + 1))

= 0

có đúng ba nghiệm phân biệt.

Lời giải: Đáp án: m = 3.

Cho hai thừa số ở vế trái của phương trình bằng 0 ta nhận được hai

phương trình đa thức. Ít nhất một trong các phương trình này phải

nghiệm đúng với giá trị x nào đó để x là nghiệm của phương trình ban

đầu. Những phương trình này có thể viết dưới dạng (x−m)2 = 5m2 +4

(1)và (x − 2)2 = 2(m3 + m + 2) (2). Ta có ba trường hợp mà phương

trình ban đầu có thể có 3 nghiệm phân biệt: Phương trình (1) có nghiệm

kép hoặc phương trình (2) có nghiệm kép hoặc hai phương trình có một

nghiệm chung. Tuy nhiên, trường hợp thứ nhất không xảy ra vì hiển

nhiên 5m2 + 4 = 0 không thể thỏa mãn với mọi giá trị thực m.

Trong trường hợp thứ hai, ta phải có 2(m3 + m + 2) = 0; m3 + m + 2

phân tích thành (m+1)(m2 −m+2) và thừa số thứ hai luôn dương với

mọi giá trị thực m. Vì vậy ta phải có m=-1 để trường hợp này xảy ra.

Khi đó nghiệm duy nhất của phương trình này là x=2 và phương trình

(1) trở thành (x + 1)2 = 9, tức là x=2, -4. Nhưng điều này có nghĩa là

phương trình ban đầu của ta chỉ có nghiệm là 2 và -4, trái với yêu cầu

của bài toán.

Xét trường hợp thứ ba, gọi r là nghiệm của phương trình thì x−r là một

thừa số của cả hai biểu thức x2−2mx−4(m2+1) và x2−4x−2m(m2+1).

Trừ hai biểu thức này cho nhau ta nhận được x − r là một thừa số của

(2m−4)x−(2m3−4m2 +2m−4), hay (2m−4)r = (2m−4)(m2 +1). Vì

vậy m = 2 hoặc r = m2+1. Tuy nhiên, trong trường hợp thứ nhất thì cả

hai phương trình bậc hai của ta trở thành (x−2)2 = 24, và vì vậy, ta chỉ

thu được hai nghiệm phân biệt. Vậy ta phải có r = m2 +1. Khi đó, thay

vào đẳng thức (r−2)2 = 2(m3+m+2), ta được (m2−1)2 = 2(m3+m+2)

hay (m + 1)(m − 3)(m2 + 1) = 0. Do đó m = −1 hoặc 3. Trường hợp

m=-1 đã được chỉ ra không thỏa mãn. Vì vậy, ta chỉ có m=3. Khi đó

các phương trình của ta trở thành (x−3)2 = 49 và (x−2)2 = 64, chúng

có các nghiệm là x=-6, -4, 10, thỏa mãn yêu cầu của bài toán.

vnmath.com

Page 126: OLYMPIC TOÁN NĂM 2000 52 ĐỀ THI VÀ LỜI …»i nói đầu Để thử gói lệnh lamdethi.sty tôi biên soạn một số đề toán thi Olympic, mà các học trò của

8 Nguyễn Hữu Điển, ĐHKHTN Hà Nội

.1.2. Cho ABC là tam giác đều có diện tích bằng 7. Gọi M, N tương ứng là

các điểm trên cạnh AB, AC sao cho AN=BM. Gọi O là giao điểm của

BN và CM. Biết tam giác BOC có diện tích bằng 2.

(a) Chứng minh rằng MBAB

hoặc bằng 13

hoặc bằng 23.

(b) Tính góc AOB.

Lời giải: (a) Lấy điểm L trên BC sao cho CL=AN và gọi P, Q lần lượt là

giao điểm của CM và AL, AL và BN. Phép quay với góc quay 120o quanh

tâm của tam giác ABC biến A thành B, B thành C, C thành A; phép

quay này cũng biến M thành L, L thành N, N thành M và biến O thành

P, P thành Q, Q thành O. Do đó OPQ và MLN là các tam giác đều đồng

tâm với tam giác ABC. Suy ra BOC=π-MOC=2π3. Vì vậy, O nằm trên

đường tròn đối xứng với đường tròn ngoại tiếp tam giác ABC qua BC. Có

nhiều nhất hai điểm O trên đường tròn này và nằm trong tam giác ABC

để tỉ lệ khoảng cách từ O tới BC và từ A tới BC bằng 27, tỉ lệ này cũng

là tỉ lệ diện tích của các tam giác OBC và ABC. Vì vậy ta đã chỉ ra rằngMBAB

=13

hoặc 23

tương ứng với các vị trí của điểm O, và không có tỉ lệ nào

khác (tức là không có hai điểm M cho cùng một điểm O. Nếu MBAB

= 13

thì ANAC

= 13, áp dụng định lí Menelaus cho tam giác ABN và đường thẳng

CM, ta được BOON

=34, do đó [BOC]

[BNC]= BO

BN=3

7. Suy ra [BOC]

[ABC]=3

7CNCA

=27

và ta

có điều phải chứng minh. Tương tự, nếu MBAB

=23, theo định lí Menelaus

ta có BOBN

=67, do đó [BOC]

[BNC]= BO

BN=6

7. Suy ra [BOC]

[ABC]=6

7CNCA

=27. (b) MB

AB=1

3thì

MONA là một tứ giác nội tiếp do A=pi3

và O = π − POQ = 2π3. Do

đó AOB=AOM+MOB=ANM+POQ = ANM + π3. Nhưng MB

AB=1

3và

ANAC

=13

nên dễ dàng thấy được N là hình chiếu của M trên AC. Vì vậy

ANM = π2

và AOB = 5π6

. Lập luận tương tự đối với trường hợp còn lại,

ta được ANM = π6

và AOB = π2.

.1.3. Cho f(x) = x2 −2ax−a2 − 34. Tìm tất cả các giá trị của a để |f(x)| ≤ 1

với mọi x ∈ [0; 1].

Lời giải: Đáp án: −12≤ a ≤

√2

4.

Đồ thị của f(x) là một parabol có điểm cực tiểu (có nghĩa là hệ số a

âm) và đỉnh là (a;f(a)). Từ f(0) = −a2 − 34

ta có |a| ≤ 12

để f(0) ≥ −1.

Giả sử a ≤ 0 thì parabol của ta tăng nghiêm ngặt trong khoảng từ 0

vnmath.com

Page 127: OLYMPIC TOÁN NĂM 2000 52 ĐỀ THI VÀ LỜI …»i nói đầu Để thử gói lệnh lamdethi.sty tôi biên soạn một số đề toán thi Olympic, mà các học trò của

Đề thi olympic Hoa Kỳ 9

đến 1, do đó f(1) ≤ 1. Nhưng ta có 12≤ a + 1 ≤ 1, 1

4≤ (a + 1)2 ≤ 1,

14≤ 5

4− (a+1)2 ≤ 1. Từ 5

4− (a+1)2 = f(1), ta có f thỏa mãn điều kiện

của bài ra khi −12≤ a ≤ 0. Với a > 0, f giảm với 0 ≤ x ≤ a và tăng

với a ≤ x ≤ 1. Vì vậy ta cần chỉ ra giá trị nhỏ nhất của f(a) nằm trong

phạm vi theo yêu cầu của bài toán, tức là f(1) nằm trong giới hạn này.

Từ a ≤ 12

ta có 1 < (a+1)2 ≤ 94

và vì vậy f(x) = −1 ≤ 54− (a+1)2 < 1

4.

Mặt khác, f(a) = −2a2 − 34

nên ta phải có a ≤√

24

để f(a) ≥ −1. Ngược

lại, đánh giá f(0), f(a), f(1) ta chỉ ra được f thỏa mãn điều kiện của bài

ra khi 0 < a ≤√

24

.

.1.4. Kí hiệu u(k) là ước lẻ lớn nhất của số tự nhiên k. Chứng minh rằng

1

2n·

2n∑

k=1

u(k)

k≤ 2

3.

Lời giải: Đặt v(k) là ước lớn nhất của k có dạng lũy thừa của 2, nên

u(k)v(k) = k. Trong {1, 2, ..., 2n} có 2n−i−1 giá trị của k sao cho v(k) = 2i

với i ≤ n − 1, và một giá trị sao cho v(k) = 2n. Do đó, vế trái bằng

1

2n·

2n∑

k=1

u(k)

k=

1

4n+

n−1∑

i=0

2n−i−1

2n+i.

Từ tổng của chuỗi hình học ta có

1

2n·

2n∑

k=1

u(k)

k= 4−n +

2

3(1 − 4−n) >

2

3.

.1.5. Tìm tất cả các số thực thỏa mãn hệ

x3 = 2y − 1

y3 = 2z − 1

z3 = 2x − 1.

Lời giải: Trước hết ta chỉ ra rằng x = y = z. Giả sử trái lại rằng x 6= y.

Nếu x > y, thì y = (x3+1)2

> (y3+1)2

= z, nên y > z, và tương tự z > x,

vnmath.com

Page 128: OLYMPIC TOÁN NĂM 2000 52 ĐỀ THI VÀ LỜI …»i nói đầu Để thử gói lệnh lamdethi.sty tôi biên soạn một số đề toán thi Olympic, mà các học trò của

10 Nguyễn Hữu Điển, ĐHKHTN Hà Nội

mâu thuẫn. Tương tự, nếu x < y thì y < z và z < x, mẫu thuẫn. Nên

các nghiệm của hệ phương trình có dạng x = y = z = t với t là nghiệm

của phương trình t3 = 2t − 1. Vậy, nghiệm của hệ phương trình là

x = y = z = t, t ∈{

1,−1 +

√5

2,−1 −

√5

2

}

.

.1.6. Tìm số tự nhiên a nhỏ nhất để phương trình sau có một nghiệm thực:

cos2π(a − x) − 2 cosπ(a − x) + cos3πx2a

cos(πx

2a+

π

3

)

+ 2 = 0

Lời giải: Lời giải: Giá trị nhỏ nhất của a là 6. Phương trình thỏa mãn

khi a=6, x=8. Để chứng minh a là giá trị nhỏ nhất, ta viết phương trình

dưới dạng

(cosπ(a-x)-1)2 +

(

cos

(

3πx2a

)

cos(πx

2a+

π

3

)

+ 1

)

= 0

Do cả hai số hạng ở vế trái đều không âm nên để đẳng thức xảy ra

thì chúng phải cùng bằng 0. Từ cosπ(a-x)-1 = 0 ta có x phải là một

số nguyên đồng dư với a trong phép chia cho 2. Từ số hạng thứ hai

bằng 0, ta thấy các giá trị cosin phải nhận giá trị bằng 1 và -1. Nếu

cos(

πx

2a+ π

3

)

= 1 thì πx

2a+ π

3= 2kπ với giá trị k nguyên và nhân hai vế

với 6aπ

ta được 3x ≡ 4a (mod12a). Khi đó thì nếu cos(

πx

2a+ π

3

)

= −1 thìπx

2a+ π

3= (2k + 1)π và nhân hai vế với 6a

πta được 3x ≡ 4a (mod12a).

Trong cả hai trường hợp ta đều có 3x chia hết cho 2, vì vậy x phải chia

hết cho 2 và a cũng phải thỏa mãn điều đó. Hơn nữa, ở cả hai trường

hợp ta cũng đều có -2a và 4a cùng phải chia hết cho 3, vì thế a phải chia

hết cho 3. Tóm lại ta có 6 phải là ước của a và a=6 là giá trị nhỏ nhất

cần tìm.

vnmath.com

Page 129: OLYMPIC TOÁN NĂM 2000 52 ĐỀ THI VÀ LỜI …»i nói đầu Để thử gói lệnh lamdethi.sty tôi biên soạn một số đề toán thi Olympic, mà các học trò của

Chương 2

Đề thi olympic Việt Nam

.2.7. Trên mặt phẳng cho hai đường tròn ω1, ω1 theo thứ tự có tâm là O1 và

O2. Cho M′

1 và M′

2là hai điểm lần lượt nằm trên ω1, ω1 sao cho O1M′

1

và O2M′

2 cắt nhau. Cho M1và M2 lần lượt là hai điểm trên ω1, ω1 sao

cho khi quay theo chiều kim đồng hồ số đo của góc M ′1OM1 và M ′

2OM2

là bằng nhau.

(a) Xác định quĩ tích trung điểm của đoạn thẳng M1M2

(b) Gọi P là giao điểm của các đường thẳng O1M′

1 và O2M′

2. Đường

tròn ngoại tiếp ∆M1PM2cắt đường tròn ngoại tiếp ∆O1PO2 tại P và một

điểm khác là Q.Chứng minh rằng Q là điểm cố định không phụ thuộc

vào vị trí của M1 và M2.

Lời giải:

vnmath.com

Page 130: OLYMPIC TOÁN NĂM 2000 52 ĐỀ THI VÀ LỜI …»i nói đầu Để thử gói lệnh lamdethi.sty tôi biên soạn một số đề toán thi Olympic, mà các học trò của

12 Nguyễn Hữu Điển, ĐHKHTN Hà Nội

P

O1

O2

Q

M1

M2

M ′1

M ′2

M ′

O

M

(a)Chúng ta dùng các số phức, mỗi điểm có kí hiệu là chữ in hoa ta đặt

tương ứng với một số phức có kí hiệu là chữ in thường. Gọi M ′, M và

O lần lượt là trung điểm của các đoạn thẳng: M′

1M′

2, M1M2 và O1O2.

Ta cũng đặt z = m1−o1

m′

1−o1= m2−o2

m′

2−o2sao cho phép nhân bởi z là phép quay

quanh 1 điểm qua một số góc. Khi đó:

m =m1 + m2

2=

1

2(o1 +z(m′

1−o1))+1

2(o2 +z(m′

2−o2)) = o+z(m′−o)

từ đó suy ra quỹ tích điểm M là đường tròn tâm O bán kính OM ′.

(b)Chúng ta sử dụng trực tiếp các góc có modun π. Chú ý rằng:

∠QM1M2 = ∠QPM2 = ∠QPO2 = ∠QO1O2

Tương tự: ∠QM2M1 = ∠QO2O1, suy ra tam giác QM1M2 đồng dạng

với tam giác QO1O2. Do đó:q − o1

q − o2=

q − m1

q − m2

Hay tương đương:

q − o1

q − o2=

(q − m1) − (q − o1)

(q − m2) − (q − o2)=

o1 − m1

o2 − m2=

o1 − m′1

o2 − m′2

Vì hai đường thẳng O1M′

1 vàO2M′

2 cắt nhau, o1 − m′1 6= o2 − m′

2 và ta

có thể giải phương trình này để tìm được giá trị duy nhất của q, suy ra

Q là điểm cố định không phụ thuộc vào vị trí của M1 và M2.

vnmath.com

Page 131: OLYMPIC TOÁN NĂM 2000 52 ĐỀ THI VÀ LỜI …»i nói đầu Để thử gói lệnh lamdethi.sty tôi biên soạn một số đề toán thi Olympic, mà các học trò của

Đề thi olympic Việt Nam 13

.2.8. Giả sử rằng tất cả đường tròn ngoại tiếp của bốn mặt của một tứ diện

có bán kính bằng nhau. Hãy chỉ ra rằng hai cạnh đối bất kì của một tứ

diện là bằng nhau.

Lời giải: Trước hết ta chứng minh rằng với bốn điểm không đồng phẳng

bất kì X, Y , Z, W ta có:

∠XY Z + ∠Y ZW + ∠ZWX + ∠WXY < 2π

X

Y

Z

W

Thật vậy: Áp dụng bất đẳng thức tam giác cho các góc ta thấy rằng:

∠XY Z + ∠Y ZW + ∠ZWX + ∠WXY

< (∠ZY W+∠WY X)+∠Y ZW+(∠XWY +∠Y WZ)+∠WXY

= (∠ZY W+∠Y WZ+∠Y ZW )+(∠XWY +∠WY X+∠WXY )

= π + π = 2π.

Gọi R là bán kính chung của các đường tròn ngoại tiếp bốn mặt tứ diện.

Ta định nghĩa rằng hai góc của tứ diện ABCD gọi là đối diện với cùng

một cạnh là hai góc ví dụ như ∠ABCvà ∠ADC, ta có:

sin∠ABC =AC

2R= sin∠ADC

( do định lí hàm sin).

Do đó hai góc bất kì đối diện với cùng một cạnh hoặc bằng nhau hoặc

bù nhau.

Hơn nữa, để ý rằng nếu XZ và Y W là hai cạnh đối của tứ diện XYZW

thì suy ra (∠XY Z + ∠ZWX + (∠Y ZW∠WXY < 2π)) nên không

thể có trường hợp hai góc đối diện với cùng một cạnh XZ và với cạnh

vnmath.com

Page 132: OLYMPIC TOÁN NĂM 2000 52 ĐỀ THI VÀ LỜI …»i nói đầu Để thử gói lệnh lamdethi.sty tôi biên soạn một số đề toán thi Olympic, mà các học trò của

14 Nguyễn Hữu Điển, ĐHKHTN Hà Nội

Y W đều là bù nhau. Nói cách khác, nếu các góc đối diện của cạnh

XZ là bù nhau thì hai góc đối diện với cùng một cạnh Y W là bằng nhau.

A

B

C

D

Bây giờ ta giả sử ngược lại rằng có hai góc đối diện với cùng một cạnh

ví dụ như ∠ABCvà ∠CDA là bù nhau còn tất cả các cặp góc đồi diện

với cùng một cạnh khác là bằng nhau, ta có:

∠BCD + ∠DAB = (π − ∠CDB − ∠DBC) + (π − ∠ADB − ∠DBA)

= ( pi − ∠CAB − ∠DAC) + (π − ∠ACB − ∠DCA)

= (π − ∠CAB − ∠ACB) + (π − ∠DAC − ∠DCA)

= ∠ABC + ∠CDA

= π

Điều này trái với giả sử. Do đó bên cạnh các góc đối diện với cạnh AC

thì còn một số cặp góc đối diện với cùng một cạnh khác là bù nhau.Theo

phần lí luận trên thì cặp góc đối diện với cạnh BD không thể bù nhau

nên chúng phải bằng nhau. Do đó cặp góc đối diện với cùng một cạnh

trong số các cạnh AB, AD, CB và CD là bù nhau. Không giảm tính

tổng quát, giả sử cặp góc đối diện với cạnh AB là bù nhau suy ra cặp

vnmath.com

Page 133: OLYMPIC TOÁN NĂM 2000 52 ĐỀ THI VÀ LỜI …»i nói đầu Để thử gói lệnh lamdethi.sty tôi biên soạn một số đề toán thi Olympic, mà các học trò của

Đề thi olympic Việt Nam 15

góc đối diện với cạnh CD là bằng nhau. Hơn nữa:

∠CDB = π − ∠DCB − ∠DBC

= π − ∠DAB − ∠DAC

= π − (π − ∠ABD − ∠ADB) − (π − ∠ACD − ∠ADC)

= ∠ABD + ∠ACD + ∠ADB + ∠ADC − π

= ∠ABD + ∠ACD + (π − ∠ACB) + (π − ∠ABC) − π

= ∠ABD + ∠ACD + (π − ∠ACB − ∠ABC)

= ∠ABD + ∠ACD + ∠CAB

Suy ra ∠CDB < ∠CAB. Vì các ∠CDB và ∠CAB là không

bằng nhau nên chúng phài bù nhau. Bây giờ ta lại có các góc

∠ADB, ∠BDC và ∠CDA là các góc ở đỉnh của một tứ diện nên

(∠ADB + ∠BDC) + ∠CDA < ∠ADC + ∠CDA < 2π. Nhưng:

∠ADB + ∠BDC + ∠CDA = (π − ∠ACB) + (π − ∠BCA) + (π − ∠CBA)

= 3π − π

= 2π.

Điều này vô lí. Do đó các góc đối diện với cùng một cạnh của tứ diện là

bằng nhau. Như chúng ta lí luận ở trên thì trong trường hợp này ta có:

∠BCD + ∠DAB = ∠ABC + ∠CDA

Suy ra 2∠DAB = 2∠ABC hay ∠DAB = ∠ABC.

Cho nên, DB = 2Rsin∠DAB = 2Rsin∠ABC = AC. Tương tự ta cũng

có: DA = BC, DC = BA.

Đó là điều phải chứng minh.

.2.9. Cho hai đường tròn (C1), (C2) cắt nhau tại P và Q. Tiếp tuyến

chung của hai đường tròn( gần P hơn Q) tiếp xúc với (C1), (C2) lần

lượt tại A và B. Tiếp tuyến của(C1) tại P cắt (C2) tại E ( khác P ),

tiếp tuyến của (C2) tại P cắt (C1) tại F ( khác P ). Gọi H, K lần lượt là

vnmath.com

Page 134: OLYMPIC TOÁN NĂM 2000 52 ĐỀ THI VÀ LỜI …»i nói đầu Để thử gói lệnh lamdethi.sty tôi biên soạn một số đề toán thi Olympic, mà các học trò của

16 Nguyễn Hữu Điển, ĐHKHTN Hà Nội

hai điểm nằm trên hai tia AF và BE sao cho AH = AP , BK = BP .

Chứng minh rằng năm điểm A, H, Q, K, B nằm trên cùng một đường

tròn.

Lời giải:

AT

B

P

QF

E

R

H

Vì điều kiện có tính chất đối xứng nên ta chỉ cần chứng minh ABKQ

là nội tiếp đường tròn.

Chúng ta sử dụng góc có hướng modulo π. Gọi R là giao điểm của

đường thẳng AP và tia BE. Đường thẳng AB và BE cắt nhau tại T .

Sử dụng tính chất của tiếp tuyến và tứ giác nội tiếp ta có:

∠QAR = ∠QAP = ∠QPC = ∠QBC = ∠QBR

Nên tứ giác ABRQ là nội tiếp đường tròn. Chúng ta sẽ chứng minh

rằng K = R. Thật vậy:

Sử dụng góc ngoài tam giác ABP và CPR; tiếp tuyến AB và

PT ; tứ giác nội tiếp đường tròn ta có:

∠BPR = ∠BAP + ∠PBA = ∠AQP + ∠PQB

= ∠APT + ∠PEB = ∠RPE + ∠PER

= ∠PRB

Do đó tam giác BPR là tam giác cân với BP = PR suy ra R = K.Đó

là điều chúng ta cần chứng minh.

.2.10.Cho a, b, c là các số nguyên dương đôi một ngyên tố cùng nhau. Một

số nguyên n được gọi là ”stubborn” nếu nó không được biểu diễn dưới

vnmath.com

Page 135: OLYMPIC TOÁN NĂM 2000 52 ĐỀ THI VÀ LỜI …»i nói đầu Để thử gói lệnh lamdethi.sty tôi biên soạn một số đề toán thi Olympic, mà các học trò của

Đề thi olympic Việt Nam 17

dạng:

n = bcx + cay + abz

với x, y, z là các số ngyuên dương bất kì. Hãy xác định qua hàm số của

a, b và c số lượng các số nguyên ”stubborn” .

Lời giải: Chúng ta có thể khẳng định được rằng bất kì số ngyuên n nào

đều có thể biểu diễn dưới dạng n = bcx + cay + abz trong đó x, y, z là

các số nguyên và 0 < y ≤ b; 0 < z ≤ c còn x có thể âm. Thật vậy,vì a

và bc nguyên tố cùng nhau nên ta có thể viết: n = an′ + bcx0 với n′, x0

là các số nguyên. vì b và c là nguyên tố cùng nhau nên n′ = cy0 + bz0 với

y0, z0 là các số nguyên. Do đó n = bcx0 = cay0 + abz0. Chọn số nguyên

β, γ sao cho 0 < y0 + βb ≤ b và 0 < z0 + γc ≤ c, khi đó ta có :

n = bc(x0 − βa − γa) + ca(y0 + βb) + ab(z0 + γc)

đây chính là dạng biểu diễn của n mà ta muốn.

Chú ý rằng bất kì số nguyên dương nào nhỏ hơn bc + ca + ab đều là

”stubborn”. Mặt khác ta cũng khẳng định rằng mọi số nguyên n > 2abc

không là ”stubborn”. Chẳng han, biểu diễn n = bcx + cay + abz với các

số nguyên và 0 < y ≤ b, 0 < z ≤ c. Khi đó:

2abc < bcx + cay + abz ≤ bcx + cab + abc = bcx + 2abc

Suy ra x > 0 như vậy n không là ”stubborn” khi n > 2abc.

Tiếp theo chúng ta chứng minh rằng đúng một nửa các số nguyên

dương trong S = [bc + ca + ab; 2abc]. Để làm được điều này ta đi chứng

minh n ∈ S là ”stubborn” khi và chỉ khi f(n) = (2abc+ bc+ ca+ab)−n

không là ”stubborn”.

Điều kiện cần: Giả sử rằng n là ”stubborn” và biểu diễn f(n) =

bcx + cay + abz với 0 < y ≤ b, 0 < z ≤ c. Nếu x không dương thì chúng

ta có thể viết n = bc(1 − x) + ca(b + 1 − y) + ab(c + 1 − z), với 1 − x0,

b+1− y0 và c+1− z0 là các số nguyên dương, nhưng điều này là không

thể vì n là ”stubborn”. Do đó, x > 0 và f(n) không là ”stubborn”.

Điều kiện đủ: Giả sử ngược lại f(n) không là ”stubborn” và n

cũng không là ”stubborn”. Biểu diễn f(n) = bcx0 + cay0 + abz0 và n =

vnmath.com

Page 136: OLYMPIC TOÁN NĂM 2000 52 ĐỀ THI VÀ LỜI …»i nói đầu Để thử gói lệnh lamdethi.sty tôi biên soạn một số đề toán thi Olympic, mà các học trò của

18 Nguyễn Hữu Điển, ĐHKHTN Hà Nội

bcx1 + cay1 + abz1 với xi, yi, zi là các số nguyên dương. Khi đó:

2abc = bc(x0 + x1 − 1) + ca(y0 + y1 − 1) + ab(z0 + z1 − 1)

Đặt x = x0 + x1 − 1 và cũng đặty,z tương tự. Từ đẳng thức trên chứng

tỏ 0 ≡ bcx(moda). Vì bc nguyên tố cùng nhau với a nên x phải chia hết

cho a suy ra x ≥ a.

Tương tự y ≥ b, z ≥ c, khi đó 2abc = bcx + cay + abz ≥ 3abc ( vô lý).

Tóm lại: có bc+ca+ab−1 các số nguyên dương nhỏ hơn bc+ca+ab

là ”stubborn”, mọi số nguyên lớn hơn 2abc không là ”stubborn”, và một

nửa của 2abc−(bc+ca+ab)+1 các số nguyên dương còn lại là ”stubborn”.

Kết quả của bài là tổng của:

bc + ca + ab − 1 +2abc − (bc + ca + ab) + 1

2=

2abc + bc + ca + ab − 1

2

các số nguyên dương ”stubborn”.

.2.11. Gọi R+ là tập các số thực dương và a, r > 1 là các số thực. Giả

sử rằng f : R+ → R là một hàm số sao cho: (f(x))2 ≤ axrf(xa) với mọi

x > 0.

(a) Nếu f(x) < 22000 với mọi x < 122000 , chứng minh rằng f(x) ≤

xra1−r với mọi x > 0.

(b) Xây dựng một hàm f : R+ → R ( không cần thoả mãn điều

kiện trong câu (a)) sao chof(x) > xra1−rvới mọi x > 0.

Lời giải: Chú ý rằng ta có thể viết lại bất đẳng thức dưới dạng:(

f(x)

xra1−r

)2

≤ f(x/a)

(x/a)ra1−r(*)

Giả sử ngược lại tức là tồn tại x0 sao cho f(x0) > xora1−r. Đặt xn = x0

an

và λn = f(xn)xr

na1−r với n ≤ 0, suy ra λ0 > 1. Từ (*) ta có:λn+1 ≥ λ2n với

n ≥ 0, và bằng quy nạp ta chứng minh được rằng: λn ≥ λ2n

0 với n ≥ 0.

Chúng ta sẽ sử dụng điều này ngay sau đây, để ý rằng mỗi λn ≥ λ2n

0 là

một số dương và do đó f(xn) cũng là số dương. Chúng ta gán x = xn

vào bất đẳng thức và sắp xếp các bất đẳng thức lại ta được:

f(xn+1)

f(xn)≥ f(xn)

axrn

=λnx

nr a

1−r

axrn

=λn

ar

vnmath.com

Page 137: OLYMPIC TOÁN NĂM 2000 52 ĐỀ THI VÀ LỜI …»i nói đầu Để thử gói lệnh lamdethi.sty tôi biên soạn một số đề toán thi Olympic, mà các học trò của

Đề thi olympic Việt Nam 19

Với mọi n ≥ 0.

Ta thấy luôn tồn tại N sao cho 2ar < λ2n

0 ≤ λn với mọi n > N .

Với tất cả các giá trị n như vậy thì ta có: f(xn+1)f(xn)

≥ 2 hay tương đương (

do f(xn) là dương) f(xn+1) ≥ 2f(xn). Cho nên f(x) ≥ 22000 ( với n đủ

lớn), nhưng cũng với n đủ lớn thì ta lại có xn = x0

an < 122000 . Điều này trái

với giả thiết tức điều giả sử là sai. Do đó f(x) ≤ xra1−r với mọi x > 0.

(b) Với mỗi số thực xluôn tồn tại duy nhất một giá trị x0 ∈ (1; a]

sao cho x0

x= an với n là số nguyên. Gọi λ(x) = x2n

0 và đặt f(x) =

λ(x)xra1−r. Bằng phép co lại, chúng ta cũng có λ(x)2 = λ(x/a) với mọi

x, nói cách khác (*) cũng đúng với mọi x. Ta cũng có λ(x) > 1 với mọi

x hay nói cách khác f(x) > xra1−r với mọi x > 0.

Đó là điều cân chứng minh.

vnmath.com

Page 138: OLYMPIC TOÁN NĂM 2000 52 ĐỀ THI VÀ LỜI …»i nói đầu Để thử gói lệnh lamdethi.sty tôi biên soạn một số đề toán thi Olympic, mà các học trò của

Chương 3

Đề thi olympic Châu Á Thái

Bình Dương

.3.12.Tính tổng S =100∑

i=0

x3i

1−3xi+3x2i

với xi = i100

(

i = 1, 101)

Lời giải: Vì 1 − 3x + 3x2 = x3 − (x − 1)3 6= 0 ∀x

Ta có thể đặt f(x) = x3

1−3x+3x2 = x3

x3+(1−x)3∀x

Cho x = xi, x = 1 − xi = x101−i và thêm 2 phương trình hệ quả ta tìm

ra:f(xi) = f(x101−i) = 1. Vì thế

S =

101∑

i=0

f(xi) =

50∑

i=0

(f(xi) + f(x101−i)) = 51

.3.13.Cho một sự bố trí vòng tròn quanh ba cạnh một tam giác, một vòng ở

mỗi góc, hai vòng ở mỗi cạnh, mỗi số từ 1 đến 9 được viết vào một trong

những vòng tròn này sao cho

i. Tổng của 4 số ở mỗi cạnh tam giác là bằng nhau.

ii. Tổng của bình phương của 4 số trên mỗi cạnh của tam giác là bằng

nhau.

Tìm tất cả các cách thoả mãn yêu cầu này.

Lời giải: Lấy bất kì một sự bố trí các con số, gọi x, y, z là số ở trong

góc và S1, S2 lần lượt là tổng của bốn số, tổng của bình phương bốn số

vnmath.com

Page 139: OLYMPIC TOÁN NĂM 2000 52 ĐỀ THI VÀ LỜI …»i nói đầu Để thử gói lệnh lamdethi.sty tôi biên soạn một số đề toán thi Olympic, mà các học trò của

Đề thi olympic Châu Á Thái Bình Dương 21

trên một cạnh bất kì. Do điều kiện đã cho ta có:

3S1 = x + y + z +

9∑

k=1

k = x + y + z + 45

3S2 = x2 + y2 + z2 +

9∑

k=1

k2 = x2 + y2 + z2 + 285

Từ đẳng thức thứ hai ta suy ra x, y, z hoặc tất cả chia hết cho 3 hoặc

không có số nào chia hết cho 3. Bởi nguyên lý Pigeouhole có hai số là

đồng dư mod3. Lấy phương trình thứ nhất theo mod3 ta cũng suy ra

3| (x + y + z). Do đó x ≡ y ≡ z(mod3)

Nếu (x, y, z) = (3, 6, 9) hay (1, 4, 7) thì S2 = 137 hoặc 17. Nếu S2 = 137

thì S2 ≡ 1(mod3) suy ra chỉ có một số trên ba cạnh là lẻ. Điều này

không thể vì 5 > 3 số lẻ được viết trong mỗi khe

Vì thế, (x, y, z) = (2, 5, 8) và S2 = 126. Vì 92 + 82 > 126 nên 9 không

thể nằm cùng cạnh với 8, tức là nó nằm trên cạnh chứa 2 hoặc 5. Vì

Min{

72 + 92, 72 + 52 + 82}

> 126

nên số 7 phải nằm trên cạnh chứa số 2 hoặc 8. Như vậy 4 lần các số trên

3 cạnh phải là (2, 4, 9, 5); (5, 1, 6, 8); (8, 7, 3, 2) để cho tổng bình phương

các số trên mỗi cạnh là 126. Cuối cùng, ta thấy các bộ số trên đều thoả

mãn.

.3.14.Cho tam giác ∆ABC, trung tuyến AM và phân giác AN . Vẽ đường

vuông góc qua N cắt MA, BA tại P, Q. Gọi O là điểm mà đường vuông

góc qua P với BA cắt AN . Chứng minh QO⊥BC

Lời giải: Cách 1. Nếu AB = AC thì QO là trung trực của BC và yêu

cầu phải chứng minh được thoả mãn. Giả sử AB 6= AC, sử dụng toạ độ

Castesian:A(0; 0), N(1; 0). Đặt độ dốc của AB là m thì của AC là −1.

Viết B = (b; mb), C = (c;−mc), ở đó b 6= c và cả b, c là dương. Độ dốc

của BC là m(b+c)b−c

.

Vì PN⊥AN và x-toạ độ của P là 1, vì P thuộc đường thẳng AB(đường

thẳng y = mx) nên ta có P = (1, m). Do đó phương trình của OP

là y = −(x − 1)/m + m, suy ra x − interceptO là (m2 + 1, 0), M là

vnmath.com

Page 140: OLYMPIC TOÁN NĂM 2000 52 ĐỀ THI VÀ LỜI …»i nói đầu Để thử gói lệnh lamdethi.sty tôi biên soạn một số đề toán thi Olympic, mà các học trò của

22 Nguyễn Hữu Điển, ĐHKHTN Hà Nội

((b+ c)/2; m(b− c)/2) và nó là trung điểm của BC. Do đó phương trình

của đường thẳng AM là y = m(b−c)b+c

x. Bởi vì Q là giao điểm của AM và

PN , ta có Q(

1; m(b−c)b+c

)

. Vì thế độ dốc của PQ làm(b−c)

b+c

m2+1−1= b−c

m(b+c). Mà

−1 là độ dốc của BC nên QO⊥BC.

Cách 2. Gọi α, β, γ là số đo các góc ∠CAB, ∠ABC, ∠BCA và y =

∠BAM, z = ∠MAC, x = ∠MAN = |y−z|2

Nếu β = γ thì QO là trung trực của BC. Ngược lại, giả sử β 6= γ thì

y 6= z , với cách làm này, ta biến đổi phương trình bằng cách nhân và

chia với biểu thức lượng giác khác 0 vì β 6= γ, y 6= z; β, γ, y, z ∈ (0; π)

Sử dụng quan hệ lượng giác vào các ∆ABC, BNO, ONQ ta có

tan∠OQN. tan∠QAN =(

ON/QN

)(

QN/AN

)

=(

ON/BN

)(

BN/AN

)

= tan∠OBN

Suy ra rằng ∠BAN = α/2, ∠OBN = α/2, ∠QAN = x

Do đó: tan∠OQN.tanx = tan (α/2) tan (α/2) (∗)Áp dụng luật hàm sin cho ∆ABC, ACM , ta được:

sin y

sin z=

sin β.BM/AM

sin γ.CM/AM=

sin β

sin γ

Suy ra: sin y+sin zsin y−sin z

= sinβ−sin γsinβ+sin γ

. Cho u, v trong khoảng (0; π/2), chú ý rằng

tan(u + v)

tan(u − v)=

sin(u + v)cos(u − v)

sin(u − v)cos(u + v)=

sin(2u) + sin(2v)

sin(2u) − sin(2v)

Cho (u, v) = (y/2,z/2) ; (u, v) =

(

β/2,γ/2

)

trong đẳng thức này ta tìm

ra:

tan (α/2)

tan (y/2 − z/2)=

tan (y/2 + z/2)

tan (y/2 − z/2)=

tan(

β/2 + γ/2

)

tan(

β/2 − γ/2

) =cot (α/2)

tan(

β/2 − γ/2

)

Nếu β > γ thì x = y/2 − z/2 so sánh biểu thức cuối cùng với

(*), ta có: tan ∠OQN = tan(

π/2 −(

β/2 − γ/2

))

. ta có: ∠OQN, π/2 −(

β/2 − γ/2

)

∈ (0; π/2) và t 7→ tan t là đơn ánh, vì thế ∠OQN =

π/2 −(

β/2 − γ/2

)

⇒ OQ⊥AB .

Chứng minh tương tự nếu β > γ

vnmath.com

Page 141: OLYMPIC TOÁN NĂM 2000 52 ĐỀ THI VÀ LỜI …»i nói đầu Để thử gói lệnh lamdethi.sty tôi biên soạn một số đề toán thi Olympic, mà các học trò của

Đề thi olympic Châu Á Thái Bình Dương 23

.3.15.Cho n, k là các số nguyên dương, n > k. Chứng minh rằng:

1

n + 1.

nn

kk (n − k)n−k<

n!

k! (n − k)!<

nn

kk (n − k)n−k

Lời giải: Sử dụng khai triển nhị thức,ta viết: nn = (k + (n − k))n ở

dạngn∑

m=0

am, với am =

n

m

km (n − k)n−m > 0 với mỗi m

Bất đẳng thức đã cho tương đương với:

nn

n + 1< ak < nn

Bất đẳng thức bên phải thoả mãn do nn =n∑

m=0

am > ak. Để chứng minh

bất đẳng thức còn lại ta chỉ cần chỉ ra ak > a0, .....am−1, am+1, .....am.

Bởi vì

nn =n∑

m=0

am <n∑

k=0

ak = (n + 1) ak

Thật vậy, ta chứng minh rằng am tăng với m ≤ k, giảm với m ≥ k, chú

ý rằng

n

m

= m+1n−m

n

m + 1

. Vì thế:

am

am+1=

n

m

km (n − k)n−m

n

m + 1

km+1 (n − k)n−m−1

=n − k

n − m.m + 1

k

Biểu thức này nhỏ hơn 1 khi m < k và lớn hơn hay bằng 1 khi m ≥ k.

Do đó ta có điều phải chứng minh.

.3.16.Cho một hoán vị (a0, a1, .....an) của dãy 0, 1, ....., n. Một chuyển vị của

ai, aj gọi là hợp pháp nếu ai = 0, i > 0 và ai−1 + 1 = aj . Hoán vị

(a0, a1, .....an) gọi là chính quy nếu sau hữu hạn các bước chuyển vị hợp

pháp nó trở thành (1, 2, ...., n, 0). Với n nào thì (1, n, n− 1, .....3, 2, 0) là

chính quy ?.

vnmath.com

Page 142: OLYMPIC TOÁN NĂM 2000 52 ĐỀ THI VÀ LỜI …»i nói đầu Để thử gói lệnh lamdethi.sty tôi biên soạn một số đề toán thi Olympic, mà các học trò của

24 Nguyễn Hữu Điển, ĐHKHTN Hà Nội

Lời giải: Với n cố định, π0 và π1 là cái hoán vị (1, n, n − 1, .....3, 2, 0)

và (1, 2, .....n, 0). Ta nói π0 là chấm dứt trong hoán vị π,1 nếu sau một

số chuyển vị hợp pháp của π0 ta thu được π,1 và nếu không một chuyển

vị hợp pháp nào có thể áp dụng cho chuyển vị π,1. Vì không có chuyển

vị hợp pháp nào có thể được áp dụng cho π1, nếu π0 chính quy thì nó

chấm dứt trong π1.Như khi áp dụng chuyển vị hợp pháp cho π0 ở nhiều

nhất một chuyển vị hợp pháp có thể áp dụng cho mỗi hoán vị thu được.

Vì thế π0 chấm dứt trong nhiều nhất một hoán vị.

Nếu n = 1, 2, dễ dàng kiểm tra (1, n, n− 1, .....3, 2, 0) là chính quy. Nếu

n > 2, chẵn, ta đòi hỏi rằng π0 không chấm dứt trong π1 và vì thế không

chính quy. Cho k ∈[

0; n−22

]

áp dụng k chuyển vị hợp pháp cho π0 suy

ra hoán vị mà bắt đầu với 1, n, n − 1, ....2k + 2, 0. Do đó π0 chấm dứt

trong mọi hoán vị bất đầu bởi 1, n, 0 thu được sau n−22

chuyển vị hợp

pháp.

Bây giờ ta giả sử rằng n > 2, để xét trường hợp này ta sẽ đưa ra vài kí

hiệu. Với mọi số nguyên s > 0, t ≥ 0 sao cho s + t chia hết cho n + 1,

ta xây dựng phép hoán vị gọi là(s, t)_bậc thang tại một thời điểm như

sau: Áp dụng (1) một lần và lặp lại (2) và (3) một cách xen kẽ:

(1) Cho s số đầu tiên 1, 2, ....s − 1, 0

(2) Cho t số tiếp theo là t số lớn nhất trong 1, 2, ..., n chưa phân định

vào mục nào, sắp xếp theo trật tự tăng.

(3) Cho s số tiếp theo là s số lớn nhất trong 1, 2, ..., n chưa phân định

vào mục nào, sắp xếp theo trật tự tăng.

Nếu s + 1|n + 1 và t > 0 thì áp dụng n/(s + t) chuyển vị hợp pháp cho

(s, t)_bậc thang, quá trình ta ám chỉ như sự lựa chọn bậc thang

Tiếp theo ta giả sử rằng s|(n + 1). Nếu 2s không chia hết cho n + 1 thì

áp dụng n/(s− 2) chuyển vị hợp pháp cho (s, 0)_bậc thang suy ra một

phép chuyển vị khác π1 mà không có thêm chuyển vị hợp pháp nào có

thể được áp dụng. Nếu thay vì 2s|(n + 1) thì (s, 0)_bậc thang thực sự

là (s, s)_bậc thang cái mà có thể chọn từ (2s, 0)_bậc thang

Giờ ta chứng minh rằng nếu n > 2 và n lẻ thì π0 chính quy nếu và chỉ

nếu n + 1 là luỹ thừa của 2. Vì n + 1 chẵn nên ta viết n + 1 = 2qr, ở đó

q là số tự nhiên và r là số tự nhiên lẻ. Áp dụng (n− 1)/2 chuyển vị hợp

vnmath.com

Page 143: OLYMPIC TOÁN NĂM 2000 52 ĐỀ THI VÀ LỜI …»i nói đầu Để thử gói lệnh lamdethi.sty tôi biên soạn một số đề toán thi Olympic, mà các học trò của

Đề thi olympic Châu Á Thái Bình Dương 25

pháp cho π0 dẫn đến (2, 0)_bậc thang

Nếu 2q > 2 thì 2s|(n + 1) với s = 21, ......2q−1 ta có thể lặp lại để dẫn

đến (2q, 0)_bậc thang

Nếu r = 1 suy ra ta thu được π1 và π0 chính quy. Ngược lại, áp dụng

r − 2 chuyển vị hợp pháp dẫn đến một hoán vị ở đó 0 là lân cận trái

của n vì thế không có một chuyển vị hợp pháp nào là có thể. Tuy nhiên

hoán vị cuối bắt đầu bởi 1, 2, ....2q hơn 1, n. Suy ra π0 không chấm dứt

trong π1 vì thế π0 không chính quy

Vậy π0 là chính quy khi và chỉ khi n = 2 hoặc n + 1 là luỹ thừa của 2.

vnmath.com

Page 144: OLYMPIC TOÁN NĂM 2000 52 ĐỀ THI VÀ LỜI …»i nói đầu Để thử gói lệnh lamdethi.sty tôi biên soạn một số đề toán thi Olympic, mà các học trò của

Chương 4

Đề thi olympic Áo - Balan

.4.17.Tìm tất cả các số nguyên dương N sao cho số đó chỉ chia hết cho 2 và

5 và N + 25 là số chính phương.

Lời giải: Ta có thể biểu diễn N dưới dạng là 2a.5b,với a và b là các

số mũ nguyên. Với số nguyên x > 5,ta có x2 = N + 25,điều này tương

đương với (x + 5)(x − 5) = N . Vì vậy N được biểu diễn bằng tích của

hai số tự nhiên hơn kém nhau 10. Ta xét hai trường hợp:

Trường hợp 1: b = 0. Khi đó với 2a = (x + 5)(x− 5) ta có x +5 và x− 5

là ước của 2. Nhưng không có hai số là ước của 2 mà hai số đó lại hơn

kém nhau 10, do đó không có số thoả mãn trường hợp này.

Trường hợp 2: b ≥ 1 .Trong trường hợp này, x2 chia hết cho 5, vì vậy

x2 cũng chia hết cho 25. Điều đó chứng tỏ b ≥ 2. Lấy x = 5y, cho

y > 1 và (y− 1)(y + 1) = 2a.5b−2. Ta có y− 1 và y + 1 là các số chẵn, và

p = 12.(y−1) và q = 1

2.(y+1) là các số nguyên dương mà p.q = 2a−2.5b−2.

Vì vậy p và q bằng 2m và 5n với m, n là các số nguyên. Ta xét hai trường

hợp nhỏ như sau:

1) 5n − 2m = 1; vì 5n, 2m 6= 0(mod3),ta có : 2m ≡ 1(mod3) và 5n ≡2(mod3). Vì vậy n là số tự nhiên và 5n ≡ 5(mod8). Ta có 2m ≡ (5m−1) ≡4(mod8),từ đó suy ra m = 2. Từ đó ta có N = 2000.

2) 2m − 5n = 1. Ta có 2m = 5n + 1 ≡ 2(mod4) từ đó suy ra m = 1 và

n = 0. Từ đó ta có N = 200. Vì vậy, tất cả các số thoả mãn bài toán là

vnmath.com

Page 145: OLYMPIC TOÁN NĂM 2000 52 ĐỀ THI VÀ LỜI …»i nói đầu Để thử gói lệnh lamdethi.sty tôi biên soạn một số đề toán thi Olympic, mà các học trò của

Đề thi olympic Áo - Balan 27

N = 200 ; 2000.

.4.18.Tìm các số nguyên n ≥ 5,sao cho ta có thể sử dụng màu tô đỉnh đa giác

n-đỉnh bởi 6 màu mà 5 đỉnh liên tiếp nhau có màu khác nhau ?

Lời giải: Ta gọi các màu là a, b, c, d, e, f . Biểu thị S1 bởi dãy a, b, c, d, e

và S2 bởi dãy a, b, c, d, e, f . Nếu n > 0, ta có thể biểu diễn dưới dạng

5x + 6y với x; y ≥ 0, khi đó n thoả mãn các điều kiện sau: x liền với

dãy S1 kéo theo y liền với dãy S2 trên hình đa giác. Ta có: y có thể

bằng 0,1,2,3 hoặc 4. Khi đó n có thể bằng 1 trong các số có dạng 5x ;

5x+6; 5x+12; 5x+18 hoặc 5x+24. Tất cả các số lớn hơn 4 không thoả

mãn các dạng trên là 7,8,9,13,14,19.

Xét tất cả các số n, trừ các trường hợp bằng 7,8,9,13,14,19. Ta có tồn

tại số k sao cho 6k < n < 6(k + 1). Theo định lý Pigeonhole, có ít nhất

k +1 đỉnh của n cạnh có màu giống nhau. Giữa 2 hoặc 3 đỉnh có ít nhất

4 đỉnh khác nữa, bởi vì 5 đỉnh liên tiếp có màu khác nhau. Vì vậy, có ít

nhất 5k + 5 đỉnh, và n ≥ 5k + 5. Do vậy, ngoại trừ n = 7, 8, 9, 13, 14, 19,

các trường hợp còn lại đều thoả mãn bài toán.

Vậy tất cả các số n ≥ 5 trừ 7,8,9,13,14,19 thoả mãn bài toán.

.4.19.Trong không gian 3 chiều, cho hình lập phương đơn vị cùng với 6 hình

lập phương đơn vị khác, tạo thành 7 hình lập phương đơn vị với các

trọng tâm là (0, 0, 0), (±1, 0, 0), (0,±1, 0), (0, 0,±1). Chứng minh hoặc

phản chứng(trong không gian 3 chiều) không gian có thể chia thành 3

chiều sao cho không có 2 trong chúng có thể chia các điểm nằm bên

trong.

Lời giải: Ta có thể chia không gian thành các không gian 3 chiều, mà

trọng tâm của các hình lập phương đơn vị trùng với các lưới điểm. Xét

các điểm có toạ độ (x, y, z). Xét x + 2y + 3z. Ta gọi 2 lưới điểm gần kề

nhau nếu và chỉ nếu chúng khác nhau duy nhất 1 toạ độ. Ta xem xét

không gian 3 chiều chứa 7 hình lập phương đơn vị mà trọng tâm của

chúng đúng với các số từ 0 đến 6. Do vậy, không gian có thể chia thành

các không gian 3 chiều thoả mãn bài toán.

.4.20.Trong mặt phẳng, cho tam giác AoBoCo. Xét tất cả các tam giác ABC

thoả mãn các điều kiện sau:(i)Co; AovBo lần lượt nằm trên AB, BC và

vnmath.com

Page 146: OLYMPIC TOÁN NĂM 2000 52 ĐỀ THI VÀ LỜI …»i nói đầu Để thử gói lệnh lamdethi.sty tôi biên soạn một số đề toán thi Olympic, mà các học trò của

28 Nguyễn Hữu Điển, ĐHKHTN Hà Nội

CA.(ii)ABC = AoBoCo, BCA = BoCoAo và CAB = CoAoBo. Tìm quỹ

tích tâm đường tròn ngoại tiếp tam giác ABC.

Lời giải: Ta có ít nhất một tam giác ABC tồn tại, ví dụ ta xét tam

giác AoBoCo ở giữa tam giác ABC.

Gọi tam giác AoBoCo có đường tròn ngoại tiếp w, tâm O. Giả sử tam

giác ABC thoả mãn các điều kiện của bài toán. Vì A và Ao nằm ở 2

phía của cạnh BC, và CoAoBo = CAB. Vì vậy, đường tròn ngoại tiếp

tâm O1 của tam giác BoCoA phải đối xứng với O qua BoCo. Tương tự

ta xét với đường tròn ngoại tiếp tâm O2 và O3 của tam giác AoCoB và

AoBoC. Ta có tứ giác OBoO1Co là hình thoi sao cho O1 = bo + co.Tương

tự,ta có:O2 = ao + co

Lấy M là trung điểm của O1O2. Xét A(a, 0), Co(co, 0), B(b, 0). Ta có

hoành độ của điểm O1 và O2 là 12(a + co) và 1

2(b + co). Vì vậy điểm M

có hoành độ 12(a + b) + co. Vì vậy, lấy điểm H ′ có hoành độ 1

2(a + b) thì

có H ′ nằm trên AB. Chú ý rằng h′ = m + (m − co) = ao + bo + co. Ta

có H ′ nằm trên OG với G là trọng tâm ∆AoBoCo, mà OH ′ = 3OG.

Do vậy H ′ là trực tâm H của tam giác AoBoCo. Quỹ tích cần tìm là duy

nhất một điểm là trực tâm của tam giác AoBoCo.

.4.21.Cho 27 điểm phân biệt trên mặt phẳng không có 3 điểm nào thẳng hàng.

4 điểm trong chúng lập thành các đỉnh của hình vuông đơn vị; 23 đỉnh

còn lại nằm trong hình vuông trên. Chứng minh rằng tồn tại 3 điểm

riêng biệt X; Y ; Z sao cho [XY Z] ≤ 148

.

Lời giải: Ta chứng minh bằng quy nạp. Lấy n ≥ 1 điểm nằm trong hình

vuông (không kể 3 đường thẳng), hình vuông có thể chia thành 2n + 2

tam giác mà các đỉnh của các tam giác cũng là 1 trong n điểm hoặc các

tam giác nằm bên trong hình vuông.

Với n = 1, vi hình vuông là hình lồi nên ta có thể chia hình vuông thành

4 tam giác bởi các đường nằm trong hình vuông, do đó các tam giác đều

nằm trong hình vuông.

Giả sử bài toán đúng với n = k ≥ 1. Ta sẽ chứng minh bài toán đúng

với n = k + 1.

Thật vậy, với n = k, ta có thể chia hình vuông thành 2k + 2 tam

vnmath.com

Page 147: OLYMPIC TOÁN NĂM 2000 52 ĐỀ THI VÀ LỜI …»i nói đầu Để thử gói lệnh lamdethi.sty tôi biên soạn một số đề toán thi Olympic, mà các học trò của

Đề thi olympic Áo - Balan 29

giác mà các đỉnh của các tam giác cũng là một trong k điểm hoặc

các tam giác nằm bên trong hình vuông. Với n = k + 1, ta xét thêm

điểm P1. Vì không có 3 điểm nào thẳng hàng nên P nằm trong số

2n+2 tam giác, ví dụ ∆ABC. Như vậy ∆ABC chia thành các tam

giác nhỏ là APB; BPCvCPA. Như vậy hình vuông được chia thành

2(n+1)+2 = 2n+4 tam giác.Vậy bài toán được chứng minh bằng quy

nạp.

Trường hợp đặc biệt n = 23, hình vuông có thê chia thành 48 tam giác

với tổng diện tích bằng 1. Vậy 1 trong các tam giác trên có diện tích tối

đa là 148

thoả mãn yêu cầu bài toán.

vnmath.com

Page 148: OLYMPIC TOÁN NĂM 2000 52 ĐỀ THI VÀ LỜI …»i nói đầu Để thử gói lệnh lamdethi.sty tôi biên soạn một số đề toán thi Olympic, mà các học trò của

Chương 5

Đề thi olympic Địa Trung Hải

.5.22.Cho n số dương a1, a2, ..., an và tập hợp {σ1, σ2, ..., σn}, với mỗi σi ∈{−1, 1}. Chứng minh rằng tồn tại một phép hoán vị (b1, b2, ..., bn) của

a1, a2, ..., an và tập hợp {β1, β2, ..., βn} với mỗi βi ∈ {−1, 1} sao cho dấu

hiệu của∑i

j=1 βjbj bằng với dấu hiệu của σi, ∀1 ≤ i ≤ n.

Lời giải: Ta xây dựng một dãy các số khác không x1, x2, ..., xn với những

thuộc tính sau đây:

i) Với 1 ≤ i ≤ n, x1, x2, ..., xn có giá trị tuyệt đối khác nhau

ii) Khi được sắp xếp theo thứ tự giá trị tuyệt đối tăng dần, dấu hiệu

của chúng xen kẽ nhau

iii) Với 1 ≤ i ≤ n, dấu hiệu của các số trong x1, x2, ..., xn có giá trị tuyệt

đối lớn nhất bằng σi.

Để làm được như vậy, ta chỉ đơn giản xây dựng x1, x2, ..., xn trong trật

tự, tại mỗi bước chọn xi0 theo quy tắc dấu hiệu đúng với thuộc tính (ii),

với i = i0 và đặt hoặc |xi0 | > max {|x1| , |x2| , ..., |xi0−1|} hoặc

|xi0 | < min {|x1| , |x2| , ..., |xi0−1|}. Vì vậy thuộc tính (iii) đúng với i = i0.

Chọn bi và βi sao cho bj1 < bj2 ⇔ |xj1 | < |xj2 | và βjxj > 0, ∀j, j1, j2. Giả

thiết rằng 1 ≤ i ≤ n. Sắp xếp b1, b2, ..., bi theo bậc tăng dần để thu được

bk1 , bk2, ..., bki. Bằng cách xây dựng, dãy các dấu hiệu βk1, βk2, ..., βki

xen

kẽ nhau, và βki= σi. Vì vậy :

vnmath.com

Page 149: OLYMPIC TOÁN NĂM 2000 52 ĐỀ THI VÀ LỜI …»i nói đầu Để thử gói lệnh lamdethi.sty tôi biên soạn một số đề toán thi Olympic, mà các học trò của

Đề thi olympic Địa Trung Hải 31

i∑

j=1

βjbj = σi

(

bki− bki−1

+ bki−2− bki−3

+ ... ± bk1

)

.

Biểu thức trong ngoặc là tổng của bk/2c biểu thức dương của các hình

thức bkj+1− bkj

và có thể cộng thêm vào một số hạng bk1 . Vì vậy,∑i

j=1 βjbj có cùng dấu hiệu với σi với mỗi i, đpcm.

.5.23.Cho tứ giác lồi ABCD. Dựng ra phía ngoài các cạnh của tứ giác các

tam giác đều WAB, XBC, Y CD, ZDA với S1, S2, S3, S4 lần lượt là trọng

tâm. Chứng minh rằng S1S3⊥S2S4 ⇔ AC = BD.

Lời giải: Chọn O là một điểm bất kì. Gọi a, b, c, d theo thứ tự biểu thị

cho các vectơ từ O đến A, B, C, và D. M1, M2, M3, M4 theo thứ tự là

trung điểm của AB, BC, CD, DA, và s′

i biểu thị cho các vectơ từ Mi tới

Si với i = 1, 2, 3, 4.

Có 2 vectơ x và y, cho ∠ (x, y) là góc giữa chúng thuận theo chiều kim

đồng hồ. (Tất cả các góc đều mod 2π). Không mất tính tổng quát, giả

sử rằng ABCD là định hướng theo chiều kim đồng hồ, và cho ϕ là phép

biến đổi mà quay bất kỳ vectơ nào π/2 ngược chiều kim đồng hồ và

nhân độ lớn của nó lên bởi 12√

3. Khi đó :

ϕ (x) · ϕ (y) = |ϕ (x)| |ϕ (y)|∠ (ϕ (x) , ϕ (y))

=

( |x|2√

3

)( |y|2√

3

)

∠ (x, y)

=1

12x · y

Tích vô hướng của vectơ−−→S3S1 với vectơ

−−→S4S2 bằng :

(

(b − d) + (a − c)

2+ s

1 − s′

3

)

·(

(b − d) − (a − c)

2+ s

2 − s′

4

)

,

và bằng tổng của 4 biểu thức sau đây :

(

|b − d|2 − |a − c|24

)

,(

s′

1 − s′

3

)

·(

s′

2 − s′

4

)

,

vnmath.com

Page 150: OLYMPIC TOÁN NĂM 2000 52 ĐỀ THI VÀ LỜI …»i nói đầu Để thử gói lệnh lamdethi.sty tôi biên soạn một số đề toán thi Olympic, mà các học trò của

32 Nguyễn Hữu Điển, ĐHKHTN Hà Nội

1

2

[

s′

1 · (b − a) − s′

3 · (c − d) + (b − c) · s′

2 − (a − d) · s′

4

]

,

1

2

[

s′

1 · (c − d) − s′

3 · (b − a) + (a − d) · s′

2 − (b − c) · s′

4

]

.

Biểu thức đầu tiên bằng 14(BD2 − AC2). Bốn số hạng trong biểu thức

thứ 3 đều bằng 0: MS1⊥AB ngụ ý rằng s′

1 (b − a) = 0, v. v. . .

Trong biểu thức thứ 2, ta thấy :

s′

1 − s′

3 = ϕ ((b − a) − (d − c)) = ϕ ((c − a) + (b − d))

s′

2 − s′

4 = ϕ ((c − b) − (a − d)) = ϕ ((c − a) − (b − d))

Do đó, tích vô hướng của chúng bằng 1/12 của :

((c − a) + (b − d)) · ((c − a) − (b − d)) = |c − a|2 + |b − d|2,hay 1

12(CA2 − BD2).

Đối với biểu thức thứ tư,

s′

1 · (c − d) =(

AB/

2√

3)

(CD) cos (π/2 + ∠ (a − b, c − d))

trong khi

−s′

3·(b − a) = s′

3·(a − b) =(

CD/

2√

3)

(AB) cos (π/2 + ∠ (c − d, a − b)) .

Tổng của các argument của 2 côsin là :

π + (∠ (a − b, c − d) + ∠ (c − d, a − b)) = 3π,

Với ngụ ý rằng giá trị của mỗi côsin là phủ định của cái khác. Do đó, s′

1

và s′

3 trong biểu thức 4 triệt tiêu lẫn nhau ở bên ngoài. Tương tự, cũng

làm như vậy với s′

2 và s′

4.

Vì thế, toàn bộ tích vô hướng bằng(

14− 1

12

)

(BD2 − AC2). Vì

S1S3⊥S2S4 khi và chỉ khi tích vô hướng này bằng 0, S1S3⊥S2S4 ⇔BD = AC, đpcm.

vnmath.com

Page 151: OLYMPIC TOÁN NĂM 2000 52 ĐỀ THI VÀ LỜI …»i nói đầu Để thử gói lệnh lamdethi.sty tôi biên soạn một số đề toán thi Olympic, mà các học trò của

Đề thi olympic Địa Trung Hải 33

.5.24.P, Q, R, S theo thứ tự là trung điểm của các cạnh BC, CD, DA, AB của

tứ giác lồi ABCD. Chứng minh rằng :

4(

AP 2 + BQ2 + CR2 + DS2)

≤ 5(

AB2 + BC2 + CD2 + DA2)

Lời giải: Ta đã biết công thức : XM là trung tuyến của tam giác

XY Z, thì XM2 = 12XY 2 + 1

2XZ2 − 1

4Y Z2. Ta thay (X, Y, Z, M) bằng

(A, B, C, P ), (B, C, D, Q), (C, D, A, R) và (D, A, B, S) vào trong công

thức này và cộng 4 công thức lại với nhau để thu được 1 công thức thứ

5. Nhân cả 2 vế của công thức thứ 5 với 4, ta tìm thấy vế trái của bất

đẳng thức sẽ bằng

AB2 + BC2 + CD2 + DA2 + 4(

AC2 + BD2)

Do đó, ta chỉ cần chứng minh AC2 +BD2 ≤ AB2 +BC2 +CD2 +DA2.

Đây là công thức “bất đẳng thức hình bình hành”. Để chứng minh nó,

gọi O là một điểm bất kỳ trong mặt phẳng và với mỗi điểm X, gọi x

biểu thị vectơ từ O tới X. Ta có thể khai triển toàn bộ các số hạng trong

AB2 + BC2 + CD2 + DA2 − AC2 − BD2, ví dụ viết AB2 = |a − b|2 =

|a|2 − 2a · b + |b|2, để thấy rằng biểu thức này bằng

|a|2 + |b|2 + |c|2 + |d|2 + 2 (a · b + b · c + c · d + d · a − a · c − b · d)

= |a + c − b − d|2 ≥ 0

với dấu “=” xảy ra khi và chỉ khi a+c = b+d(hay khi và chỉ khi ABCD

là hình bình hành). Điều phải chứng minh.

vnmath.com

Page 152: OLYMPIC TOÁN NĂM 2000 52 ĐỀ THI VÀ LỜI …»i nói đầu Để thử gói lệnh lamdethi.sty tôi biên soạn một số đề toán thi Olympic, mà các học trò của

Chương 6

Đề thi olympic Petecbua

.6.25.Cho AA1, BB1, CC1 là đường cao của tam giác nhọn ABC. Hai điểm

A2 và C2 nằm trên đường thẳng A1C1 sao cho đường thẳng CC1 chia

đôi đoạn thẳng A2B1 và đường thẳng AA1 chia đôi đoạn thẳng C2B1.

Đường thẳng A2B1 và AA1 gặp nhau tại điểm K, và đường thẳng C2B1

và CC1 gặp nhau tại điểm L. Chứng minh rằng đường thẳng KL và AC

song song với nhau.

Lời giải: Gọi điểm K1 và L1 là điểm giữa của C2B1 và A2B1 do đó K1

nằm trên đường AA1 và L1 nằm trên đường thẳng CC1. Dễ dàng chứng

minh được đường cao AA1 của tam giác ABC là đường phân giác của

tam giác A1B1C1 từ đó cho thấy rằng A1K1 vừa là đường phân giác

vừa là đường trung bình của tam giác A1C2B1, vì vậy A1C2 = A1B1 và

A1K1 cũng là đường cao của tam giác A1C2B1 do đó A1K1 vuông góc

với B1C2 tương tự C1L1 vuông góc với A2B1.

Từ đó suy ra đường thẳng KK1 và LL1 là đường cao của tam giác KLB1

đưa đến chúng đồng quy tại với đường cao l từ B1 trong tam giác này.

Từ đó đường thẳng KK1 và LL1 gặp nhau tại trực tâm H của tam giác

ABC, l phải đi qua điểm B1 và H. Do đó l vuông góc với AC. Bởi vì l là

đường cao trong tam giác KLB1 đi qua B1, nó cũng vuông góc với KL.

Chúng ta kết luận rằng KL//AC, như đề bài.

.6.26.Một trăm điểm được chọn trong mặt phẳng toạ độ. Hãy chỉ ra rằng tối

vnmath.com

Page 153: OLYMPIC TOÁN NĂM 2000 52 ĐỀ THI VÀ LỜI …»i nói đầu Để thử gói lệnh lamdethi.sty tôi biên soạn một số đề toán thi Olympic, mà các học trò của

Đề thi olympic Petecbua 35

đa 2025 = 452 các hình chữ nhật với các đỉnh trong số các điểm này có

cạnh song song với các trục.

Lời giải: Lời giải thứ nhất: Gọi O là một trong 100 điểm, và gọi hình

chữ nhật có giá trị nếu các đỉnh của nó là O và 03 điểm khác được lựa

chọn. Chúng ta khẳng định rằng có ít nhất 81 hình chữ nhật có giá trị.

Vẽ qua O đường thẳng I1 và I2 song song với trục toạ độ, tại đó m được

chọn là các điểm nằm trên I2 − {O}. Cho điểm cố định bất kỳ đã chọn

P không nằm trên I1 hoặc I2, tối đa một hình chữ nhật có giá trị có P

như là một đỉnh, hơn thế nữa, tất cả các hình chữ nhật có giá trị của

dạng này cho điểm P nào đó. Do đó có 99−m−n như điểm P, có tối đa

nhiều tam giác có giá trị kiểu như vậy. Nếu m+n > 17, chúng ta đã làm

được. Mặt khác, cho một cặp (P, Q) là các điểm đã chọn, ở đây P thuộc

I1 − {O} và Q thuộc I2 − {O}, ít nhất có một hình chữ nhật giá trị có

P và Q như là các đỉnh; hơn nữa tất cả các hình chữ nhật có giá trị của

dạng này đối với cặp (P, Q) như vậy. Vì có mn < m(17−m) < 8.9 = 72

các cặp như vậy, có ít nhất 72 < 8 hình chữ nhật có giá trị.

Chúng ta kết luận rằng trong bất kỳ trường hợp nào, có ít nhất 81 hình

chữ nhật mà giá trị của nó là O và có 03 điểm khác được lựa chọn. Biến

đổi O trên tất cả 100 điểm, đếm số hình chữ nhật cho mỗi O. Tổng cộng

số điểm ít nhất là 8100, và chúng ta tam giác bất kỳ mà các đỉnh của

nó được lựa chọn các điểm là 4 lần. Vì vậy, có ít nhất 8100/4 = 2025

hình chữ nhật, như đề bài đã yêu cầu.

Lời giải thứ hai: Gọi một hình chữ nhật thích hợp nếu 4 đỉnh của nó

được lựa chọn là các điểm. Vẽ tất cả các đường thẳng đứng I1....., In đi

qua ít nhất một trong những điểm đã lựa chọn. Giả thiêt rằng Ii chứa xi

là các điểm đã lựa chọn, để s :=∑n

i=1 xi = 100. Số tam giác thích hợp

với các cạnh bên trên đường thẳng ith và jth là tối đa min{C2xi

, C2xj}.

Quan sát thấy rằng min{C2xi

, C2xj} 6

2xy−x−xy4

đối với các số nguyên

dương là x và y, bởi vì nếu x 6 y thì cạnh bên tay trái tối đa làx(x−1)

26 1

4[x(y − 1) + y(x − 1)]. Do vậy số tam giác thích hợp tối đa

là∑

16i6j6n2xixj)

4−∑

16i6j6nxi+xj

4= 1

4(s2 −

∑ni=1 x2

i ) − 14(n − 1)s =

2525 − 14(∑n

i=1 x2i + 100n)

Áp dụng bất đẳng thức bình phương trung bình nghiệm và bất đẳng thức

vnmath.com

Page 154: OLYMPIC TOÁN NĂM 2000 52 ĐỀ THI VÀ LỜI …»i nói đầu Để thử gói lệnh lamdethi.sty tôi biên soạn một số đề toán thi Olympic, mà các học trò của

36 Nguyễn Hữu Điển, ĐHKHTN Hà Nội

trung bình hình số học, ta thấy rằng biểu thức đạt cuối cùng này tối đa

là bằng 2525− 14( s2

n+100n) = 2525−25(100

n+n) > 2525−25.2

(100n

n) =

2025 Như đề bài đã yêu cầu

.6.27.a, Tìm tất cả các cặp của các số nguyên khác biệt a, b để b2+aa2+b

và b2 + a

là luỹ thừa của một số nguyên tố.

b, Cho a và b là các số nguyên dương lớn hơn 1thoả mãn b2+a−1a2+b−1

. Chứng

minh rằng b2 + a − 1 có ít nhất hai thừa số nguyên tố khác nhau.

Lời giải:a, Chúng ta chứng minh rằng chỉ có cặp như vậy là (a, b) = (5, 2). Nếu

(b = 1), thì a+1a2+1

dẫn đến a+1a(a+1)−(a2+1)

= a − 1. Do vậy a=1 không đưa

ra lời giải khi a, b được đưa ra yêu cầu khác biệt. Vì vậy tổng b>1, và

viết b2 + a = pm khi đó p là nguyên tố và m > 1.

Quan sát thấy b(b3 +1) ≡ (b2)2 + b ≡ a2 + b ≡ 0(mod(b2 + a)), để b2 +a

chia cho b(b3 + 1) nhưng gcd(b, b3 + 1)và b2 + a là luỹ thừa của một

nguyên tố, vì vậy một trong số b hoặc b3 + 1 có thể chia hết cho b2 + a.

Trường hợp đầu rõ ràng là không thể được, trong trường hợp thứ 2,

chúng ta có b2+a(b+1)(b2−b+1)

. Mỗi b+1 và b2 − b + 1 ít hơn b2 + a, vì vậy cả

hai đều không chia hết cho b2 + a. Vì b2 + a = pm là luỹ thừa của p,

chúng ta kết luận rằng p chia hết cho cả b+1 và b2 − b + 1, thì nó cũng

phải chia hết cho (b2 − b + 1) − (b + 1)(b − 2) dẫn đến p=3.

Không có lời giải cho m=1. Nếu m=2, thì chúng ta có b2 + a = 9, lời giả

dễ dàng là (a,b)=(5,2). Mặt khác, giả sử rằng m > 3, một trong b+1 và

b2 − b + 1 chia hết cho 3 và chia hết cho 3m−1 nhưng b2 < b2 + a = 3m

dẫn đến rằng b+1 nhiều hơn 3m/2 +1 < 3m−1 và không thể chia hết cho

3m−1. Chúng ta kết luận rằng 3m−1 chia hết cho b2 − b + 1 và do vậy 9

chia hết cho 4.(b2 − b+1) = (2b−1).2+3. Điều này không thể vì không

có bình phương đồng dạng với 6 modun 9. Vì vậy không có lời giải ngoài

(5,2)

b, Giả thiết, vì sự mâu thuẫn, rằng b2−1+a là luỹ thừa của một nguyên

tố. Vì (b2 − 1)2 − a2 chia hết cho b2 − 1 + a, khi a2 + b − 1 là giả thiết

tổng của chúng là (b2 − 1)2 + b + 1 = b(b − 1)(b2 + b − 1). Quan sát

thấy rằng b, b-1, và b2 + b − 1 = b(b + 1) − 1 = (b + 2)(b − 1) + 1 là

nguyên tố cùng nhau từng cặp. Vì vậy, một trong b, b-1, và b2 + b − 1

vnmath.com

Page 155: OLYMPIC TOÁN NĂM 2000 52 ĐỀ THI VÀ LỜI …»i nói đầu Để thử gói lệnh lamdethi.sty tôi biên soạn một số đề toán thi Olympic, mà các học trò của

Đề thi olympic Petecbua 37

phải chia hết cho luỹ thừa nguyên tố b2 − 1 + a. Vì b và b-1 nhỏ hơn

b2 + a− 1 chúng ta phải có rằng b2 + a− 1 chia cho b2 + b− 1 và vì vậy

a 6 b vì a 6= b là giả thiết a<b. Mặt khác, vì b2+a−1a2+b−1

chúng ta phải có

0 6 (a2 + b− 1)− (b2 + a− 1) = (a − b)(a + b− 1). Vì vậy a > b như là

một mâu thuẫn .

.6.28.Trong một đất nước có 2000sân bay, ban đầu không có một chuyến bay

nào của hãng hàng không. Hai hãng hàng không lần lượt giới thiệu những

chuyến bay thẳng khứ hồi mới (giữa hai thành phố bất kỳ, chỉ có một

chuyến bay thẳng được giới thiệu). Cơ quan vận tải muốn đạt được mục

tiêu là nếu sân bay nào bị đóng cửa thì mọi người vẫn có thể du lịch giữa

hai sân bay bất kỳ khác, có thể bằng chuyển tải. Hãng hàng không tạo

ra mục đích để đạt được sẽ bị thua lỗ. Hãng hàng không nào sẽ chiến

thắng cuộc chơi một cách hoàn hảo?

Lời giải:Công ty hàng không thứ hai đã chiến thắng. Xem xét tình huống khi

mà mục tiêu không đạt được, nhưng việc bổ sung thêm chuyến bay đơn

bất kỳ để đạt được mục đích nêu ra. Do mục tiêu không đạt được, có

sân bay A bất kỳ khi đóng cửa đã chia các thành phố ra làm hai nhóm

không liên lạc được là G1 và G2. Khi đó hai thành phố bất kỳ nằm

trong G1 hoặc trong G2 thì phải gia nhập, bởi vì nếu không bổ sung

chuyến bay giữa hai thành phố đó thì mục tiêu đưa ra không đạt được.

Tương tự như vậy, tất cả các thành phố phải gia nhập vào A, nhưng

không có thành phố trong G1 có thể gia nhập với bất kỳ thành phố nào

của G2. Do đó nếu hệ số k thành phố trong G1, thì có k−12

chuyến bay

giữa hai thành phố trong G1, (1999−k)(1998−k)2

chuyến bay giữa hai thành

phố trong G2, và 1999 chuyến bay giữa A và thành phố khác. Như vậy

tổng số chuyến bay là k(k-1999)+1999000 là đều nhau. Nói cụ thể, chưa

bao giờ tới lượt của hãng hàng không thứ hai bổ sung thêm một chuyến

bay mới. Do vậy, công ty hàng không thứ hai luôn tránh được sự thua

lỗ.

.6.29.Chúng ta có phương trình bậc hai đa thức lồi, tất cả có cùng biệt số.

Tổng của hai đa thức bất kỳ có nghiệm thực riêng biệt. Chỉ ra rằng tổng

vnmath.com

Page 156: OLYMPIC TOÁN NĂM 2000 52 ĐỀ THI VÀ LỜI …»i nói đầu Để thử gói lệnh lamdethi.sty tôi biên soạn một số đề toán thi Olympic, mà các học trò của

38 Nguyễn Hữu Điển, ĐHKHTN Hà Nội

số của tất cả các đa thức cũng đều có nghịêm thực riêng biệt.

Lời giải:Biệt số chung phải là dương, bởi vì nếu không mỗi đa thức chỉ có giá trị

dương, vì vậy tổng của hai đa thức bất kkỳ sẽ không có nghiệm thực.

Cho biệt số chung là 4D, để mỗi đa thức là dạng (x− c)2 −D đối c bất

kỳ. Để mỗi đa thức, được xem là khoảng cách trong đó đa thức có chứa

giá trị âm, khoảng cách có độ dài là 2√

D . Nếu hai trong số khoảng

cách này (c1 −√

D, c1 +√

D và (c2 −√

D, c2 +√

D không giao nhau,

thì |c2−c1| >√

D và 12(c1+c2) không nằm trong một trong hai khoảng

cách. Do vậy cả hai đa thức đều không âm tại 12(c1 + c2) nhưng điểm

này khi tổng số đa thức p đạt được giá trị nhở nhất của nó - mâu thuẫn

với giả thiết là có p nghiệm thực phân biệt Do vậy, hai khoảng cách

giao nhau bất kỳ. Chọn một khoảng cách (c −√

D, c +√

D để c là cực

tiểu. Vì tất cả các khoảng cách giao nhau bất kỳ khác điều này, chúng

ta thấy rằng tất cả các khoảng cách đều chứa (c +√

D thuộc đối với

thuộc bất kỳ. Tại điểm này, tổng số của tất cả các đa thức chứa giá trị

âm a, do vậy tổng này phải có nghiệm thực riêng biệt.

.6.30.Trên một bàn cờ đam vô hạn đặt 111 góc không chồng nhau. Các hình

có chữ L được đặt làm 3 đơn vị hình vuông. Tập hợp có tính chất sau:

đối với góc bất kỳ, chứa 2.2 hình vuông bao phủ toàn bộ các góc. Chứng

minh rằng một góc có thể dịch chuyển giữa một và 110 của các góc để

tính chất được bảo vệ.

Lời giải: Nếu 2.3 hình chữ nhật bất kỳ được bao phủ bởi hai góc, thì

chúng ta có thể dị chuyển tất cả các góc ngoại trừ hai góc đó. Do vậy,

chúng ta có thể tổng kết rằng không có hình chữ nhật như vậy tồn tại.

Chúng ta xây dựng một đồ thị trực tiếp mà các đỉnh của nó là các góc,

như sau: đối với mỗi góc, vẽ 2.2 hình vuông có chứa góc đó, và thêm

một đường nối từ góc này tới góc kia bao phủ số dư của 2.2 hình vuông.

Nếu một góc không có điểm nối về phiá nó, chúng ta có thể dịch chuyển

góc đó, vì vậy chúng ta có thể tổng kết rằng, không có góc như vậy tồn

tại. Do đó, mỗi đường nối của đồ thị nằm trong chu trình nào đó. Nếu

có hơn một chu trình, thì chúng ta có thể dịch chuyển tất cả các góc

vnmath.com

Page 157: OLYMPIC TOÁN NĂM 2000 52 ĐỀ THI VÀ LỜI …»i nói đầu Để thử gói lệnh lamdethi.sty tôi biên soạn một số đề toán thi Olympic, mà các học trò của

Đề thi olympic Petecbua 39

ngoại trừ các góc nằm trong chu trình của chiều dài dài cực tiểu, và yêu

cầu đặc tính được tồn tại. Vì vậy, nó thoả mãn để chỉ ra rằng không thể

tồn tại một chu trình đơn bao gồm tất cả 111 đỉnh.

Theo điểm giữa của một góc chúng ta hướng theo điểm tại điểm giữa của

2.3 hình vuông chứa góc đó. Nhớ lại rằng chúng ta đã tổng kết không

có hai góc bao phủ 2.3 hình vuông, một điều dễ dàng kiểm tra là nếu

có một điểm nối từ một góc tới một góc khác, thì điểm giữa của những

góc này khác 1 trong cả hai toạ độ của chúng là x và y. Do đó, trong

chu trình bất kỳ, toạ độ x của các đỉnh nằn trong sự biến đổi của chu

trình, do đó số của các đỉnh nằm trong chu trình là chẵn. Do vậy, không

có một chu trình chứa tất cả 111đỉnh, như đề bài đã đưa ra.vnmath.com

Page 158: OLYMPIC TOÁN NĂM 2000 52 ĐỀ THI VÀ LỜI …»i nói đầu Để thử gói lệnh lamdethi.sty tôi biên soạn một số đề toán thi Olympic, mà các học trò của

Chương 7

Đề thi olympic Anh

.7.31.Cho hai đường tròn cắt nhau (C1) và (C2)có một tiếp tuyến chung tiếp

xúc (C1) tại P, tiếp xúc (C2) tại Q. hai đường tròn này cắt nhau tại M

và N. Chứng minh rằng tam giác MNP và tam giác MNQ có cùng diện

tích.

Lời giải: Gọi X là giao điểm của MN và PQ. Vì MN là trục đẳng

phương của (C1) và (C2), X có cùng mối liên quan này với hai đường

tròn vậy XP 2 = XQ2 hay XP = XQ Cũng vì PXM + MXQ = π

ta có sinPXM = sinMXQ cho nên [MNP ] = 12MN(XPsinPXM) =

12MN(XQsinMXQ) = MNQ. ta có điều phải chứng minh.

.7.32.Cho x, y, z là những số thực dương thoả mãn xyz = 32. Tìm giá trị nhỏ

nhất của x2 + 4xy + 4y2 + 2z2.

Lời giải: Áp dụng bất đẳng thức trung bình hai lần ta có:

x2 +4xy +4y2 +2z2 = (x2 +4y2)2 +4xy +2z2 ≥ 2√

x24y2 +4xy +2z2 =

4xy + 4xy + 2z2 ≥ 3 3√

4xy4xy2z2 = 3 3√

32(xyz)2 = 96.

Dấu "=" xảy ra khi và chỉ khi x2 = 4y2 và 4xy = 2z2, tức là

(x,y,z)=(4,2,4)

.7.33.a, Tìm một tập A của 10 số thực nguyên mà không có 6 phần tử phân

biệt nào của A có tổng chia hết cho 6

b, Có thể tìm được một tập nếu 10 được thay bởi 7 hay không?

vnmath.com

Page 159: OLYMPIC TOÁN NĂM 2000 52 ĐỀ THI VÀ LỜI …»i nói đầu Để thử gói lệnh lamdethi.sty tôi biên soạn một số đề toán thi Olympic, mà các học trò của

Đề thi olympic Anh 41

Lời giải: a, Một ví dụ của tập A là A = {6j + k|1 ≤ j ≤ 5, 1 ≤ k ≤ 2}Trong bất kỳ 6 phần tử trong tập con của A nếu có t số đồng dư 1

modun 6 thì t ∈ {1, 2, ..., 5}. những phần tử khác trong tập con là đồng

dư 0 môdun 6. Vì vậy tổng của những phần tử trong tập con là đồng

dư t 6= 0 (mođun6)

b, Không thể cho bất kỳ tập nào có 7 số thực nguyên, chúng ta có 6

phần tử phân biệt của tập này có tổng chia hết cho 6. Bởi vì có hơn 2

số nguyên trong tập này, chúng ta có thể chọn hơn 2 mà là chẵn. Cùng

cách này làm như thế ta có thể tìm thấy 5 tập con có 2 phần tử rời rạc

cái mà có tổng là đồng dư của 0, 2, 4 mođun 6. Nếu tất cả tổng đó xuất

hiện, 6 phần tử trong tập con tương ứng có tổng đồng dư 0 + 2 + 4 =

6 (mođun 6). Cách khác, chỉ tổng xuất hiện . Bởi Pizeônhle principle,

3 tập con sẽ có tổng như nhau. Do đó những phần tử trong 3 cặp sẽ có

tổng chia hết cho 6.

vnmath.com

Page 160: OLYMPIC TOÁN NĂM 2000 52 ĐỀ THI VÀ LỜI …»i nói đầu Để thử gói lệnh lamdethi.sty tôi biên soạn một số đề toán thi Olympic, mà các học trò của

Tài liệu tham khảo

[1] Titu Andreescu, Zuming Feng, and George Lee, Jr. Mathematical

Olympiads 2000–2001, Problems and Solutions From Around the World,

The Mathematical Association of America, 2002.

[2] Nguyễn Hữu Điển, Phương pháp Đirichle và ứng dụng, NXBKHKT, 1999.

[3] Nguyễn Hữu Điển, Phương pháp Quy nạp toán học, NXBGD, 2000.

[4] Nguyễn Hữu Điển, Những phương pháp điển hình trong giải toán phổ

thông, NXBGD, 2001.

[5] Nguyễn Hữu Điển, Những phương pháp giải bài toán cực trị trong hình

học, NXBKHKT, 2001.

[6] Nguyễn Hữu Điển, Sáng tạo trong giải toán phổ thông, NXBGD, 2002.

[7] Nguyễn Hữu Điển, Đa thức và ứng dụng, NXBGD, 2003.

[8] Nguyễn Hữu Điển, Giải phương trình vô định nghiệm nguyên,

NXBĐHQG, 2004.

[9] Nguyễn Hữu Điển, Giải toán bằng phương pháp đại lượng bất biến,

NXBGD, 2004.

vnmath.com

Page 161: OLYMPIC TOÁN NĂM 2000 52 ĐỀ THI VÀ LỜI …»i nói đầu Để thử gói lệnh lamdethi.sty tôi biên soạn một số đề toán thi Olympic, mà các học trò của

Nguyễn Hữu Điển

OLYMPIC TOÁN NĂM 1997-1998

51 ĐỀ THI VÀ LỜI GIẢI(Tập 4)

NHÀ XUẤT BẢN GIÁO DỤC

vnmath.com

Page 162: OLYMPIC TOÁN NĂM 2000 52 ĐỀ THI VÀ LỜI …»i nói đầu Để thử gói lệnh lamdethi.sty tôi biên soạn một số đề toán thi Olympic, mà các học trò của

2

vnmath.com

Page 163: OLYMPIC TOÁN NĂM 2000 52 ĐỀ THI VÀ LỜI …»i nói đầu Để thử gói lệnh lamdethi.sty tôi biên soạn một số đề toán thi Olympic, mà các học trò của

Lời nói đầu

Để thử gói lệnh lamdethi.sty tôi biên soạn một số đề toán thi Olympic, mà

các học trò của tôi đã làm bài tập khi học tập LATEX. Để phụ vụ các bạn ham

học toán tôi thu thập và gom lại thành các sách điện tử, các bạn có thể tham

khảo. Mỗi tập tôi sẽ gom khoảng 51 bài với lời giải.

Rất nhiều bài toán dịch không được chuẩn, nhiều điểm không hoàn toàn

chính xác vậy mong bạn đọc tự ngẫm nghĩ và tìm hiểu lấy. Nhưng đây là nguồn

tài liệu tiếng Việt về chủ đề này, tôi đã có xem qua và người dịch là chuyên về

ngành Toán phổ thông. Bạn có thể tham khảo lại trong [1].

Rất nhiều đoạn vì mới học TeX nên cấu trúc và bố trí còn xấu, tôi không

có thời gian sửa lại, mong các bạn thông cảm.

Hà Nội, ngày 2 tháng 1 năm 2010

Nguyễn Hữu Điển

51GD-05

89/176-05 Mã số: 8I092M5

vnmath.com

Page 164: OLYMPIC TOÁN NĂM 2000 52 ĐỀ THI VÀ LỜI …»i nói đầu Để thử gói lệnh lamdethi.sty tôi biên soạn một số đề toán thi Olympic, mà các học trò của

Mục lục

Lời nói đầu . . . . . . . . . . . . . . . . . . . . . . . . . . . . . . . . . . . . . . . . . . . . 3

Mục lục . . . . . . . . . . . . . . . . . . . . . . . . . . . . . . . . . . . . . . . . . . . . . . . 4

Chương 1. Đề thi olympic Austria. . . . . . . . . . . . . . . . . . . . . . . . . . 5

Chương 2. Đề thi olympic Bungari . . . . . . . . . . . . . . . . . . . . . . . . . 9

Chương 3. Đề thi olympic Canada . . . . . . . . . . . . . . . . . . . . . . . . 13

Chương 4. Đề thi olympic Chine . . . . . . . . . . . . . . . . . . . . . . . . . . 17

Chương 5. Đề thi olympic Colombia . . . . . . . . . . . . . . . . . . . . . . 21

Chương 6. Đề thi olympic Czech và Slovak Repubulick . . . . 24

Chương 7. Đề thi olympic Pháp. . . . . . . . . . . . . . . . . . . . . . . . . . . 28

Chương 8. Đề thi olympic Đức . . . . . . . . . . . . . . . . . . . . . . . . . . . . 31

Chương 9. Đề thi olympic Irland . . . . . . . . . . . . . . . . . . . . . . . . . . 37

vnmath.com

Page 165: OLYMPIC TOÁN NĂM 2000 52 ĐỀ THI VÀ LỜI …»i nói đầu Để thử gói lệnh lamdethi.sty tôi biên soạn một số đề toán thi Olympic, mà các học trò của

Chương 1

Đề thi olympic Austria

.1.1. Giải hệ phương trình với x, y là số thực

(x − 1)(y2 + 6) = y(x2 + 1),

(y − 1)(x2 + 6) = x(y2 + 1)

Lời giải: Ta cộng hai phương trình trên cho nhau. Sau khi rút gọn và đưa về

bình phương của một hiệu ta được phương trình sau

(x − 52)2 + (y − 5

2)2 = 1

2

Chúng ta lại trừ hai phương trình cho nhau, trừ phương trình thứ hai cho

phương trình thứ nhất và nhóm lại, ta có:

xy(y − x) + 6(x − y) + (x + y)(x − y) = xy(x − y) + (y − x)

(x − y)(−xy + 6 + (x + y) − xy + 1) = 0

(x − y)(x + y − 2xy + 7) = 0

Do vậy, hoặc x−y = 0 hoặc x+y−2xy+7 = 0. Cách duy nhất để có x−y = 0

là với x = y = 2 hoặc x = y = 3 (tìm được bằng cách giải phương trình (1))

với phép thế x = y

Bây giờ, ta xét trường hợp x 6= y sẽ được giải để x + y − 2xy + 7 = 0. Phương

trình này là tương đương với phương trình sau(được suy ra từ cách sắp xếp

vnmath.com

Page 166: OLYMPIC TOÁN NĂM 2000 52 ĐỀ THI VÀ LỜI …»i nói đầu Để thử gói lệnh lamdethi.sty tôi biên soạn một số đề toán thi Olympic, mà các học trò của

6 Nguyễn Hữu Điển, ĐHKHTN Hà Nội

lại các số hạng và thừa số)

(x − 12)(y − 1

2) = 15

4

Giả sử, chúng ta có thể giải phương trình (1) và (2) một cách đồng thời. Đặt

a = x − 52

và b = y − 52. Do đó, phương trình (1)) tương đương với

a2 + b2 = 12

và phương trình (2) tương đương với:

(a + 2)(b + 2) =15

4⇒ ab + 2(a + b) =

1

4→ 2ab + 4(a + b) =

−1

2

Cộng phương trình (4) và (3) chúng ta thấy:

(a + b)2 + 4(a + b) = 0 → a + b = 0,−4

Lấy phương trình (4) trừ (3) ta thấy:

(a − b)2 − 4(a + b) = 1

Nhưng bây giờ chúng ta thấy rằng ,nếu a + b = −4 thì phương trình (6) sẽ bị

sai; Do đó, a + b = 0. Thế a + b = 0 vào phương trình (6) chúng ta thu được:

(a − b)2 = 1 → a − b = ±1

Vì từ phương trình (5) chúng ta có a+b = 0,và cùng với phương trình (7)bây giờ

ta có thể tìm được tất cả các cặp có thứ tự (a, b). Chúng là (−12, 1

2) và (1

2,−1

2)

. Do vậy, các nghiệm (x, y)của hệ phương trình đã cho là(2, 2),(3, 3),(2, 3) và

(3, 2).

.1.2. Cho dãy số nguyên dương thỏa mãn an = a2n−1 + a2

n−2 + a2n−3 với n ≥ 3.

Chứng minh rằng nếu ak = 1997 thì k ≤ 3.

Lời giải: Chúng ta giải trực tiếp: Giả sử với k > 3, ak = 1997. Khi đó,

có ít nhất một số trong 4 số ak−1, ak−2, ak−3 vàak−4 phải tồn tại. Đặt w =

ak−1, x = ak−2, y = ak−3 và z = ak−4.Bây giờ, điều kiện của chúng ta là:

1997 = w2 + x2 + y2. Do đó, w ≤√

1997 < 45 và vì w là một số nguyên dương

nên w ≤ 44. Nhưng do x2 + y2 ≥ 1997 − 4462 = 61.

Bây giờ, (với) w = x2 + yx2 + z2 . Vì x2 + y2 > 61 và z2 > 0, x2 + y2 + z2 > 61.

Nhưng w 6 44. Do đó, chúng ta có mâu thuẫn và giả thiết của chúng ta là

không đúng.

Vậy, nếu ak = 1997 thì k 6 3.

.1.3. Cho k là một số nguyên dương. Dãy a − n được xác định bởi a − 1 = 1

và an là n− số nguyên dương lớn hơn an−1 là đồng dư n modulo k. Tìm an

trong dãy trên.

vnmath.com

Page 167: OLYMPIC TOÁN NĂM 2000 52 ĐỀ THI VÀ LỜI …»i nói đầu Để thử gói lệnh lamdethi.sty tôi biên soạn một số đề toán thi Olympic, mà các học trò của

Đề thi olympic Austria 7

Lời giải: Chúng ta có an = n(2+(n−1)k)2

. Nếu k = 2 thì an = n2. Trước tiên,

chú ý rằng a1 ≡ 1(modk). Do đó, với tất cả n, an ≡ n(modk), và số nguyên

đầu tiên lớn hơn an−1 mà là đồng dư n modulo k phải là an−1 + 1.

n - th số nguyên dương lớn hơn an−1là đồng dư n modul k là đơn giản (n - 1)k

hơn số nguyên dương đầu tiên lớn hơn an−1 mà thỏa mãn điều kiện đó. Do

vậy, an = an−1 + 1 + (n − 1)k. Lời giải bằng phép đệ quy này đưa ra câu trả

lời của bài toán trên.

.1.4. Cho hình bình hành ABCD, một đường tròn nội tiếp trong góc BAD và

nằm hoàn toàn trong hình bình hành. Tương tự, một đường tròn nội tiếp trong

góc BCD nằm hoàn toàn trong hình bình hành sao cho 2 đường tròn đó tiếp

xúc. Hãy tìm quỹ tích các tiếp điểm của 2 đường tròn đó khi chúng thay đổi.

Lời giải: Giả sử K1 là đường tròn lớn nhất nội tiếp trong góc BAD sao cho

nó nằm hoàn toàn trong hình bình hành. Nó cắt đường thẳng AC tại 2 điểm

và giả sử điểm ở xa A hơn là P1. Tương tự Giả sử K2 là đường tròn lớn nhất

nội tiếp trong góc BCD sao cho nó nằm hoàn toàn trong hình bình hành. Nó

cắt đường thẳng AC tại 2 điểm và giả sử điểm ở xa C hơn là P2. Khi đó, quỹ

tích là giao của 2 đoạn AP1 và AP2.

Chúng ta bắt đầu chứng minh điểm tiếp xúc phải nằm trên đường AC. Giả sử

I1 là tâm đường tròn nội tiếp góc BAD và I2 là tâm đường tròn nội tiếp góc

BCD. Giả sử X là điểm tiếp xúc của 2 đường tròn. Vì các đường tròn tâm I1

và I2 là nội tiếp trong các góc nên các tâm này phải nằm trên các đường phân

giác của các góc. Mặt khác vì AI1 và CI2 là các đường phân giác của các góc

đối hình bình hànhneen chúng song sonh với nhau. Do vậy I1I2 là đường nằm

ngang.

Giả sử T1 là chân đường vuông góc hạ từ I1 tới AB và T2 là chân đường vuông

góc hạ từ I2 tới CD. Chú ý rằng:

I1T1

AI1= sin I1AB = sin I2CD =

I2T2

CI2

Nhưng I1X = I1T1 và I2X = I2T2. Do vậy

I1X

AI1=

I2X

CT2

Vì thế tam giác CI2X và tam giác AI1X là đồng dạng và các góc vuông

I1XA, I2XC là bằng nhau. Vì các góc này bằng nhau nên các điểm A, X và C

vnmath.com

Page 168: OLYMPIC TOÁN NĂM 2000 52 ĐỀ THI VÀ LỜI …»i nói đầu Để thử gói lệnh lamdethi.sty tôi biên soạn một số đề toán thi Olympic, mà các học trò của

8 Nguyễn Hữu Điển, ĐHKHTN Hà Nội

phải cộng tuyến. Do vậy, điểm tiếp xúc X phải nằm trên đường chéo AC (đó

là điều phải chứng minh).

Như vậy, chúng ta biết rằng X sẽ luôn nằm trên AC, bây giờ ta sẽ chứng minh

bất kỳ điểm nào thuộc quỹ tích đó đều là điểm tiếp xúc. Cho X bất kỳ nằm

trên quỹ tích đó, giả sử I1 là đường tròn bé hơn đường tròn qua X, nội tiếp

trong góc BAD.

Nó sẽ nằm hoàn toàn bên trong hình bình hành bời vì X là điểm giữa A và P1.

Tương tự, ta vẽ một đường tròn tiếp xúc với đường tròn I1 và nội tiếp trong

góc BCD, từ chứng minh trên ta biết rằng nó phải tiếp xúc với đường tròn I1

tại X, hơn nữa nó sẽ hoàn toàn xác định bên trong hình bình hành bởi vì X

là điểm giữa của C và P2.

Vì vậy, bất cứ điểm nào thuộc quỹ tích sẽ chạy qua X. Để chứng minh rằng

bất kỳ điểm nào khác sẽ không chạy qua. Chú ý rằng bất kỳ điểm nào sẽ hoặc

không nằm trên đường thẳng AC hoặc sẽ không cho 1 trong 2 đường tròn I1

hoặc I2 được chứa bên trong hình bình hành. Do vậy, quỹ tích thực sự là giao

của các đoạn AP1 và CP2.

vnmath.com

Page 169: OLYMPIC TOÁN NĂM 2000 52 ĐỀ THI VÀ LỜI …»i nói đầu Để thử gói lệnh lamdethi.sty tôi biên soạn một số đề toán thi Olympic, mà các học trò của

Chương 2

Đề thi olympic Bungari

.2.5. Tìm tất cả các số thực m để phương trình

(x2 − 2mx − 4(m2 + 1)

) (x2 − 4x − 2m(m2 + 1)

)= 0

có đúng ba nghiệm phân biệt.

Lời giải: Đáp án: m = 3.

Cho hai thừa số ở vế trái của phương trình bằng 0 ta nhận được hai phương

trình đa thức. Ít nhất một trong các phương trình này phải nghiệm đúng với

giá trị x nào đó để x là nghiệm của phương trình ban đầu. Những phương trình

này có thể viết dưới dạng (x−m)2 = 5m2 + 4 (1)và (x− 2)2 = 2(m3 + m + 2)

(2). Ta có ba trường hợp mà phương trình ban đầu có thể có 3 nghiệm phân

biệt: Phương trình (1) có nghiệm kép hoặc phương trình (2) có nghiệm kép

hoặc hai phương trình có một nghiệm chung. Tuy nhiên, trường hợp thứ nhất

không xảy ra vì hiển nhiên 5m2 + 4 = 0 không thể thỏa mãn với mọi giá trị

thực m.

Trong trường hợp thứ hai, ta phải có 2(m3 + m + 2) = 0; m3 + m + 2 phân

tích thành (m + 1)(m2 −m + 2) và thừa số thứ hai luôn dương với mọi giá trị

thực m. Vì vậy ta phải có m=-1 để trường hợp này xảy ra. Khi đó nghiệm duy

nhất của phương trình này là x=2 và phương trình (1) trở thành (x+1)2 = 9,

tức là x=2, -4. Nhưng điều này có nghĩa là phương trình ban đầu của ta chỉ

có nghiệm là 2 và -4, trái với yêu cầu của bài toán.

vnmath.com

Page 170: OLYMPIC TOÁN NĂM 2000 52 ĐỀ THI VÀ LỜI …»i nói đầu Để thử gói lệnh lamdethi.sty tôi biên soạn một số đề toán thi Olympic, mà các học trò của

10 Nguyễn Hữu Điển, ĐHKHTN Hà Nội

Xét trường hợp thứ ba, gọi r là nghiệm của phương trình thì x − r là một

thừa số của cả hai biểu thức x2 − 2mx− 4(m2 + 1) và x2 − 4x− 2m(m2 + 1).

Trừ hai biểu thức này cho nhau ta nhận được x − r là một thừa số của

(2m − 4)x − (2m3 − 4m2 + 2m − 4), hay (2m − 4)r = (2m − 4)(m2 + 1). Vì

vậy m = 2 hoặc r = m2 + 1. Tuy nhiên, trong trường hợp thứ nhất thì cả

hai phương trình bậc hai của ta trở thành (x − 2)2 = 24, và vì vậy, ta chỉ

thu được hai nghiệm phân biệt. Vậy ta phải có r = m2 + 1. Khi đó, thay vào

đẳng thức (r − 2)2 = 2(m3 + m + 2), ta được (m2 − 1)2 = 2(m3 + m + 2) hay

(m + 1)(m − 3)(m2 + 1) = 0. Do đó m = −1 hoặc 3. Trường hợp m=-1 đã

được chỉ ra không thỏa mãn. Vì vậy, ta chỉ có m=3. Khi đó các phương trình

của ta trở thành (x− 3)2 = 49 và (x− 2)2 = 64, chúng có các nghiệm là x=-6,

-4, 10, thỏa mãn yêu cầu của bài toán.

.2.6. Cho ABC là tam giác đều có diện tích bằng 7. Gọi M, N tương ứng là

các điểm trên cạnh AB, AC sao cho AN=BM. Gọi O là giao điểm của BN và

CM. Biết tam giác BOC có diện tích bằng 2.

(a) Chứng minh rằng MBAB

hoặc bằng 13

hoặc bằng 23.

(b) Tính góc AOB

Lời giải: (a) Lấy điểm L trên BC sao cho CL=AN và gọi P, Q lần lượt là

giao điểm của CM và AL, AL và BN. Phép quay với góc quay 120o quanh tâm

của tam giác ABC biến A thành B, B thành C, C thành A; phép quay này

cũng biến M thành L, L thành N, N thành M và biến O thành P, P thành Q,

Q thành O. Do đó OPQ và MLN là các tam giác đều đồng tâm với tam giác

ABC. Suy ra BOC=π-MOC=2π3

. Vì vậy, O nằm trên đường tròn đối xứng

với đường tròn ngoại tiếp tam giác ABC qua BC. Có nhiều nhất hai điểm O

trên đường tròn này và nằm trong tam giác ABC để tỉ lệ khoảng cách từ O

tới BC và từ A tới BC bằng 27, tỉ lệ này cũng là tỉ lệ diện tích của các tam

giác OBC và ABC. Vì vậy ta đã chỉ ra rằng MBAB

=13

hoặc 23

tương ứng với các

vị trí của điểm O, và không có tỉ lệ nào khác (tức là không có hai điểm M cho

cùng một điểm O. Nếu MBAB

= 13

thì ANAC

= 13, áp dụng định lí Menelaus cho

tam giác ABN và đường thẳng CM, ta được BOON

=34, do đó [BOC]

[BNC]= BO

BN=3

7. Suy

ra [BOC][ABC]

=37

CNCA

=27

và ta có điều phải chứng minh. Tương tự, nếu MBAB

=23, theo

định lí Menelaus ta có BOBN

=67, do đó [BOC]

[BNC]= BO

BN=6

7. Suy ra [BOC]

[ABC]=6

7CNCA

=27. (b)

vnmath.com

Page 171: OLYMPIC TOÁN NĂM 2000 52 ĐỀ THI VÀ LỜI …»i nói đầu Để thử gói lệnh lamdethi.sty tôi biên soạn một số đề toán thi Olympic, mà các học trò của

Đề thi olympic Bungari 11

MBAB

=13

thì MONA là một tứ giác nội tiếp do A=pi3

và O = π− POQ = 2π3. Do

đó AOB=AOM+MOB=ANM+POQ = ANM + π3. Nhưng MB

AB=1

3và AN

AC=1

3

nên dễ dàng thấy được N là hình chiếu của M trên AC. Vì vậy ANM = π2

AOB = 5π6

. Lập luận tương tự đối với trường hợp còn lại, ta được ANM = π6

và AOB = π2.

.2.7. Cho f(x) = x2 −2ax−a2 − 34. Tìm tất cả các giá trị của a để |f(x)| ≤ 1

với mọi x ∈ [0; 1].

Lời giải: Đáp án: −12≤ a ≤

√2

4.

Đồ thị của f(x) là một parabol có điểm cực tiểu (có nghĩa là hệ số a âm) và

đỉnh là (a;f(a)). Từ f(0) = −a2 − 34

ta có |a| ≤ 12

để f(0) ≥ −1. Giả sử a ≤ 0

thì parabol của ta tăng nghiêm ngặt trong khoảng từ 0 đến 1, do đó f(1) ≤ 1.

Nhưng ta có 12≤ a + 1 ≤ 1, 1

4≤ (a + 1)2 ≤ 1, 1

4≤ 5

4− (a + 1)2 ≤ 1. Từ

54− (a + 1)2 = f(1), ta có f thỏa mãn điều kiện của bài ra khi −1

2≤ a ≤ 0.

Với a > 0, f giảm với 0 ≤ x ≤ a và tăng với a ≤ x ≤ 1. Vì vậy ta cần chỉ ra

giá trị nhỏ nhất của f(a) nằm trong phạm vi theo yêu cầu của bài toán, tức

là f(1) nằm trong giới hạn này. Từ a ≤ 12

ta có 1 < (a + 1)2 ≤ 94

và vì vậy

f(x) = −1 ≤ 54− (a + 1)2 < 1

4. Mặt khác, f(a) = −2a2 − 3

4nên ta phải có

a ≤√

24

để f(a) ≥ −1. Ngược lại, đánh giá f(0), f(a), f(1) ta chỉ ra được f thỏa

mãn điều kiện của bài ra khi 0 < a ≤√

24

.

.2.8. Kí hiệu u(k) là ước lẻ lớn nhất của số tự nhiên k. Chứng minh rằng

1

2n·

2n∑

k=1

u(k)

k≤ 2

3.

Lời giải: Đặt v(k) là ước lớn nhất của k có dạng lũy thừa của 2, nên u(k)v(k) =

k. Trong {1, 2, ..., 2n} có 2n−i−1 giá trị của k sao cho v(k) = 2i với i ≤ n − 1,

và một giá trị sao cho v(k) = 2n. Do đó, vế trái bằng

1

2n·

2n∑

k=1

u(k)

k=

1

4n+

n−1∑

i=0

2n−i−1

2n+i.

Từ tổng của chuỗi hình học ta có

1

2n·

2n∑

k=1

u(k)

k= 4−n +

2

3(1 − 4−n) >

2

3.

vnmath.com

Page 172: OLYMPIC TOÁN NĂM 2000 52 ĐỀ THI VÀ LỜI …»i nói đầu Để thử gói lệnh lamdethi.sty tôi biên soạn một số đề toán thi Olympic, mà các học trò của

12 Nguyễn Hữu Điển, ĐHKHTN Hà Nội

.2.9. Tìm tất cả các số thực thỏa mãn hệ

x3 = 2y − 1

y3 = 2z − 1

z3 = 2x − 1.

Lời giải: Trước hết ta chỉ ra rằng x = y = z. Giả sử trái lại rằng x 6= y.

Nếu x > y, thì y = (x3+1)2

> (y3+1)2

= z, nên y > z, và tương tự z > x, mâu

thuẫn. Tương tự, nếu x < y thì y < z và z < x, mẫu thuẫn. Nên các nghiệm

của hệ phương trình có dạng x = y = z = t với t là nghiệm của phương trình

t3 = 2t − 1. Vậy, nghiệm của hệ phương trình là

x = y = z = t, t ∈{

1,−1 +

√5

2,−1 −

√5

2

}.

.2.10.Tìm số tự nhiên a nhỏ nhất để phương trình sau có một nghiệm thực:

cos2π(a − x) − 2 cosπ(a − x) + cos3πx2a

cos(πx

2a+

π

3

)+ 2 = 0

Lời giải: Giá trị nhỏ nhất của a là 6. Phương trình thỏa mãn khi a=6, x=8.

Để chứng minh a là giá trị nhỏ nhất, ta viết phương trình dưới dạng

(cosπ(a-x)-1)2 +

(cos

(3πx2a

)cos

(πx2a

3

)+ 1

)= 0

Do cả hai số hạng ở vế trái đều không âm nên để đẳng thức xảy ra thì chúng

phải cùng bằng 0. Từ cosπ(a-x)-1 = 0 ta có x phải là một số nguyên đồng dư

với a trong phép chia cho 2. Từ số hạng thứ hai bằng 0, ta thấy các giá trị

cosin phải nhận giá trị bằng 1 và -1. Nếu cos(

πx

2a+ π

3

)= 1 thì πx

2a+ π

3= 2kπ

với giá trị k nguyên và nhân hai vế với 6aπ

ta được 3x ≡ 4a (mod12a). Khi đó

thì nếu cos(

πx

2a+ π

3

)= −1 thì πx

2a+ π

3= (2k + 1)π và nhân hai vế với 6a

πta

được 3x ≡ 4a (mod12a). Trong cả hai trường hợp ta đều có 3x chia hết cho

2, vì vậy x phải chia hết cho 2 và a cũng phải thỏa mãn điều đó. Hơn nữa, ở

cả hai trường hợp ta cũng đều có -2a và 4a cùng phải chia hết cho 3, vì thế a

phải chia hết cho 3. Tóm lại ta có 6 phải là ước của a và a=6 là giá trị nhỏ

nhất cần tìm.

vnmath.com

Page 173: OLYMPIC TOÁN NĂM 2000 52 ĐỀ THI VÀ LỜI …»i nói đầu Để thử gói lệnh lamdethi.sty tôi biên soạn một số đề toán thi Olympic, mà các học trò của

Chương 3

Đề thi olympic Canada

.3.11.Có bao nhiêu cặp số (x; y) nguyên dương với x ≤ y thoả mãn gcd(x, y) =

5! và lcm(x, y) = 50! ?

Lời giải: Trước hết, chú ý là có 15 số nguyên tố từ 1 đến 50:

(2, 3, 5, 7, 11, 13, 17, 19, 23, 29, 31, 37, 41, 43, 47).

Để làm cho bài toán đơn giản hơn, ta xác định f(a, b) là mũ lớn nhất của b

chia cho a. (Chú ý rằng g(50!, b) > g(5!, b) với mọi b < 50.)

Do đó, với mỗi số nguyên tố p, ta có

f (x, p) = f (5!, p)

f (y, p) = f (50!, p)

f (y, p) = f (5!, p)

f (x, p) = f (50!, p)

Vì ta có 15 số nguyên tố nên có 215 cặp, và trong bất kì cặp nào cũng hiển

nhiên có x 6= y ( do gcd và lcm khác nhau), do đó có 214 cặp với x ≤ y.

.3.12.Cho trước một số hữu hạn các khoảng đóng có độ dài bằng 1 sao cho hợp

của chúng là khoảng đóng [0, 50], chứng minh rằng tồn tại một tập con của

các khoảng đó không giao với tất cả các khoảng khác.

Lời giải: Xét

I1 = [1 + e, 2 + e] , I2 = [3 + 2e, 4 + 2e] , ..., I24 = [47 + 24e, 48 + 24e]

vnmath.com

Page 174: OLYMPIC TOÁN NĂM 2000 52 ĐỀ THI VÀ LỜI …»i nói đầu Để thử gói lệnh lamdethi.sty tôi biên soạn một số đề toán thi Olympic, mà các học trò của

14 Nguyễn Hữu Điển, ĐHKHTN Hà Nội

trong đó e đủ nhỏ để 48 +24e < 50. Để hợp các đoạn chứa 2k + ke, ta phải có

một đoạn mà phần tử nhỏ nhất nằm trong Ik. Tuy nhiên, sự khác nhau giữa

một phần tử trong tập Ik và Ik + 1 luôn lớn hơn 1, vì vậy các tập này không

chồng lên nhau. Từ 24 khoảng ban đầu và [0, 1] ( phải tồn tại vì hợp là [0, 50]

) ta có 25 khoảng rời nhau mà tổng độ dài tất nhiên bằng 25.

.3.13.Chứng minh rằng:1

1999< 1

2.34. ... .1997

1998< 1

44

Lời giải: Đặt p = 12.34. ... .1997

1998và q = 2

3.45. ... .1998

1999. Chú ý rằng p < q, vì vậy

p2 < pq = 12.23. ... .1998

1999= 1

1999. Do đó,

p <1

19991

2

<1

44

Lại có

p =1998!

(999!.2999)2 = 2−1998

1998

999

đồng thời

21998 =

1998

0

+ ... +

1998

1998

< 1999

1998

999

Do đó

p >1

1999

.

.3.14.Cho O là một điểm nằm trong tứ giác ABCD sao cho AOB+ COD = π.

Chứng minh rằng OBC = ODC.

Lời giải: Tịnh tiến ABCD theo vectơ−−→AD thì A’ và D như nhau, và vì vậy

B’ và C như nhau. Ta có COD + CO′D = COD + A′O′D′ = 1800 nên tứ giác

OC ′O′D′ nội tiếp. Do đó ODC = OO′C.

.3.15.Biểu diễn tổng saun∑

k=0

(−1)k

k3+9k2+26k+24

n

k

về dạng p(n)/q(n), trong đó

p, q là các đa thức với các hệ số nguyên.

vnmath.com

Page 175: OLYMPIC TOÁN NĂM 2000 52 ĐỀ THI VÀ LỜI …»i nói đầu Để thử gói lệnh lamdethi.sty tôi biên soạn một số đề toán thi Olympic, mà các học trò của

Đề thi olympic Canada 15

Lời giải: Ta có

n∑k=0

(−1)k

k3+9k2+26k+24

n

k

=n∑

k=0

(−1)k

(k+2)(k+3)(k+4)

n

k

=n∑

k=0

(−1)k k+1(n+1)(n+2)(n+3)(n+4)

n + 4

k + 4

= 1(n+1)(n+2)(n+3)(n+4)

n+4∑k=4

(−1)k (k − 3)

n + 4

k

n+4∑k=0

(−1)k (k − 3)

n + 4

k

=n+4∑k=0

(−1)k k

n + 4

k

− 3n+4∑k=0

(−1)k

n + 4

k

=n+4∑k=1

(−1)k k

n + 4

k

− 3 (1 − 1)n+4

= 1n+4

n+4∑k=1

(−1)k

n + 3

k − 1

= 1n+4

(1 − 1)n+3 = 0

Do đó

n+4∑k=4

(−1)k (k − 3)

n + 4

k

= −3∑

k=0

(−1)k (k − 3)

n + 4

k

= 3

n + 4

0

− 2

n + 4

1

+

n + 4

2

= (n+1)(n+2)2

vnmath.com

Page 176: OLYMPIC TOÁN NĂM 2000 52 ĐỀ THI VÀ LỜI …»i nói đầu Để thử gói lệnh lamdethi.sty tôi biên soạn một số đề toán thi Olympic, mà các học trò của

16 Nguyễn Hữu Điển, ĐHKHTN Hà Nội

và tổng đã cho bằng1

2 (n + 3) (n + 4)

vnmath.com

Page 177: OLYMPIC TOÁN NĂM 2000 52 ĐỀ THI VÀ LỜI …»i nói đầu Để thử gói lệnh lamdethi.sty tôi biên soạn một số đề toán thi Olympic, mà các học trò của

Chương 4

Đề thi olympic Chine

.4.16. Cho x1, x2, ..., x1997 là các số thực thỏa mãn điều kiện

(a) − 1√3≤ xi ≤

√3 với mọi i = 1; 2; ...; 1997

(b) x1 + x2 + ... + x1997 = x121997

Lời giải: Do x12 là một hàm lồi của x nên tổng các lũy thừa bậc 12 của xi là

cực đại nếu mỗi giá trị xi là đầu mút của khoảng quy định.

Giả sử có n giá trị xi bằng − 1√3, 1996 − n có giá trị bằng

√3 và các giá trị

cuối cùng bằng

−318√

3 +n√3−

√3(1996 − n)

Do giá trị cuối cùng này phải nằm trong miền[− 1√

3;√

3]

nên

−1 ≤ −318x3 + n − 3x(1996 − n) ≤ 3

tương đương với

−1 ≤ 4n − 6942 ≤ 3

có duy nhất một số nguyên n = 1736 thỏa mãn.

Khi đó, giá trị cuối cùng là 2√3

và giá trị lớn nhất cần tìm là:

1736x3−6 + 260x36 + (4

3)6

vnmath.com

Page 178: OLYMPIC TOÁN NĂM 2000 52 ĐỀ THI VÀ LỜI …»i nói đầu Để thử gói lệnh lamdethi.sty tôi biên soạn một số đề toán thi Olympic, mà các học trò của

18 Nguyễn Hữu Điển, ĐHKHTN Hà Nội

.4.17. Cho tứ giác lồi A1B1C1D1 và một điểm P nằm trong tứ giác lồi đó. Giả

sử các góc PA1B1 và PA1D1 là các góc nhọn, tương tự cho ba đỉnh còn lại.

Xác dịnh Ak, Bk, Ck, Dk là hình chiếu của P lên các đường thẳng Ak−1Bk−1,

Bk−1Ck−1, Ck−1Dk−1 (k = 2, 3, ...)

a) Trong các tứ giác AkBkCkDk (k = 1, 2, 3, ..., 12) thì tứ giác nào đồng dạng

với tứ giác thứ 1997

b) Giả sử rằng tứ giác thứ 1997 là nội tiếp. Hỏi trong 12 tứ giác đầu tiên thì

tứ giác nào cũng nội tiếp đường tròn

Lời giải: Ta có Ak chính là chân của các đường vuông góc từ P đến Ak−1Bk−1

và tương tự như vậy cho các điểm còn lại. Do các tứ giác nội tiếp với các đường

kính PAk, PBk, PCk, PDk ta có

PAkBk = PDk+1Ak+1 = PCk+2Dk+2

PBk+3Ck+3 = PAk+4Bk+4

Mặt khác, ta cũng có PBkAk= PBk+1Ak+1 và tương tự như vậy cho các góc

còn lại.

Do vậy, các tứ giác thứ 1, 5, 9 đồng dạng với tứ giác thứ 1997.

Nếu tứ giác thứ 1997 là nội tiếp thì các tứ giác thứ 3, 7 và 11 cũng vậy.

.4.18. Chỉ ra tồn tại vô số số nguyên dương n sao cho các số1, 2, 3, ..., 3n có thể

được gán cho

a1, a2, ..., an, b1, b2, ..., bn, c1, c2, ..., cn

theo thứ tự này thỏa mãn điều kiện sau:

a) a1 + b1 + c1 = an + bn + cn

b) a1 + a2 + ... + an = b1 + b2 + ... + bn = c1 + c2 + ... + cn là bội của 6

Lời giải: Tổng các số nguyên từ 1 đến 3n là 3n(3n+1)2

trong đó ta đòi hỏi phải

vừa là bội của 6n và 9.

Vì thế, n phải là bội của 3 đồng dư với 1 theo môddun4.

Ta sẽ chỉ ra rằng tồn tại sự sắp xếp cho n = 9m.

Với n = 9 ta có sự sắp xếp sau:

vnmath.com

Page 179: OLYMPIC TOÁN NĂM 2000 52 ĐỀ THI VÀ LỜI …»i nói đầu Để thử gói lệnh lamdethi.sty tôi biên soạn một số đề toán thi Olympic, mà các học trò của

Đề thi olympic Chine 19

8 1 16 17 10 15 26 19 24

21 23 25 3 5 7 12 14 16

13 18 11 22 27 20 4 9 2

(trong đó, dòng đầu tiên là a1, a2, ..., an và tiếp tục). Điều này chứng tỏ từ sự

sắp xếp cho m và n dẫn đến sự sắp xếp cho mn

a′′

i+(j−1)m = aj + (m − 1)a′

j(1 ≤ j ≤ m, 1 ≤ j ≤ n)

và tương tự cho bi, ci.

.4.19. Cho ABCD là một tứ giác nội tiếp. Các đường thẳng AB và CD cắt

nhau tại P . Các đường thẳng AD và BC cắt nhau tại Q. Gọi E và F là giao

điểm tiếp tuyến từ Q với đường tròn ngoại tiếp tứ giác ABCD. Chứng minh

rằng P, E, F thẳng hàng.

P

Q

A

D

C

B

F

E

Lời giải: Gọi X ′ là tiếp tuyến của đường tròn tại điểm X nằm trên đường

tròn.

Để chứng minh P, E, F thẳng hàng ta chứng minh các điểm cực của nó trùng

nhau.

Các tiếp tuyến E ′ và F ′ tại các điểm E và F cắt nhau tại Q. Do P là giao của

AB và CD nên điểm cực của P là đường thẳng đi qua giao điểm của A′ giao

với B′ và C ′ giao với D′.

vnmath.com

Page 180: OLYMPIC TOÁN NĂM 2000 52 ĐỀ THI VÀ LỜI …»i nói đầu Để thử gói lệnh lamdethi.sty tôi biên soạn một số đề toán thi Olympic, mà các học trò của

20 Nguyễn Hữu Điển, ĐHKHTN Hà Nội

Ta sẽ chứng minh những điểm này thẳng hàng với Q. Tuy nhiên theo định lý

Pascal cho lục giác suy biến AADBBC thì Q và giao điểm của AC với BD sẽ

cộng tuyến.

Tương tự, áp dụng định lý Pascal cho lục giác suy biến ADDBCC ta cũng có

kết quả tương tự.

.4.20. Cho A = {1, 2, ..., 17} và hàm f : A → A thoả mãn

Ký hiệu f [1] = f(x) và f [k+1](x) = f(f [k](x)) với k ∈ N

Tìm số tự nhiên lớn nhất M sao cho tồn tại song ánh f : A → A thỏa mãn

điều kiện sau:

a) Nếu m < M và 1 ≤ i ≤ 17 thì f [m](i + 1) − f [m](i) không đồng dư với ±1

theo môđun 17 b) Với 1 ≤ i ≤ 17 thì

f [m](i + 1)−f [m](i) ≡ ±1(mod17)

(ở đây f [k](18)) được xác định bằng f [k](1)))

Lời giải: Ánh xạ f(x) = 3x(mod17) thỏa mãn yêu cầu cho M = 8 và ta sẽ

chỉ ra rằng nó là giá trị lớn nhất.

Chú ý rằng bằng cách sắp xếp với chu trình chuyển ta có thể giả sử rằng

f(17) = 17, do đó M là số nguyên đầu tiên sao cho f [M ](1)) bằng 1 hoặc bằng

16. Cũng như vậy cho 16.

Nếu 1 và 16 cùng trên một quỹ đạo của hoán vị f thì quỹ đạo này có độ dài

lớn nhất là 16 và 1 hoặc 16 phải ánh xạ với những giá trị khác nhau sau 8

bước. Suy ra, M≤ 8.

Nếu có một quỹ đạo khác, một hoặc thậm chí hai quỹ đạo có độ dài lớn nhất

là 8 và như vậy M≤ 8.

.4.21. Cho a1, a2, ... là các số không âm thỏa mãn

am+n ≤ am+an (m, n≤N).

Chứng minh rằng: an ≤ ma1+ ( nm− 1)am với mọi n≥m.

Lời giải: Bằng phương pháp quy nạp với k

an ≤ kam+an−mk với k <mn

Đặt n = mk + r với r∈{1, 2, ..., m} thì

an ≤ kam+ar=n−rm

am+ar ≤ ma1

(Do am ≤ ma1 và ar ≤ ra1)

vnmath.com

Page 181: OLYMPIC TOÁN NĂM 2000 52 ĐỀ THI VÀ LỜI …»i nói đầu Để thử gói lệnh lamdethi.sty tôi biên soạn một số đề toán thi Olympic, mà các học trò của

Chương 5

Đề thi olympic Colombia

.5.22. Cho một bảng kẻ ô kích thước n × n và 3 màu. Ta sẽ tô màu mỗi đoạn

của lưới bởi một trong 3 màu trên sao cho mỗi ô vuông đơn vị có 2 cạnh cùng

màu và 2 cạnh còn lại cùng màu khác. Hỏi có bao nhiêu cách tô màu có thể?

Lời giải: Gọi 3 màu trên là A, B, C.

Gọi an là số cách tô màu của 1 × n ô ở dòng đàu tiên của bảng.

Với n = 1, giả sử WLOG đoạn trên cùng của bảng được tô màu A, khi đó có

3 cạnh để chọn đoạn khác được tô màu A và có 2 cách để chọn màu còn lại và

như vậy có tất cả a1 = 6 cách tô màu.

Bây giờ ta tìm an+1 từ an. Cho bất kỳ màu nào của dòng 1 × n, giả thiết

WLOG rằng đoạn bên phải nhất được tô màu A. Bây giờ ta tưởng tượng thêm

một ô vuông đơn vị kép vào cạnh phải của dòng để được dòng mới có kích

thước 1 × (n + 1), ở đó màu trên của ô vuông mới đã biết. Nếu đoạn trên

mới được tô màu A thì sẽ có 2 cách để chọn màu cho 2 đoạn còn lại. Do vậy:

a(n + 1) = 2an và an = 3.2n.

Trở lại yêu cầu ban đầu, có 3n cách để tô màu góc trên và 3.2n cách tô màu

mỗi dòng. Như vậy có tất cả 3m+n.2m.n cách tô màu.

.5.23. Ta chơi một trò chơi với tam giác đều của n.(n+1)2

đồng xu (với n đồng

xu trên mỗi cạnh). Đầu tiên, tất cả các đồng xu đều đặt sấp. Trong mỗi lần

lật ta có thể lật 3 đồng xu liên tiếp liền kề. Mục đích là tất cả các đồng xu bi

vnmath.com

Page 182: OLYMPIC TOÁN NĂM 2000 52 ĐỀ THI VÀ LỜI …»i nói đầu Để thử gói lệnh lamdethi.sty tôi biên soạn một số đề toán thi Olympic, mà các học trò của

22 Nguyễn Hữu Điển, ĐHKHTN Hà Nội

lật ngửa. Hỏi n bằng bao nhiêu để hoàn thành việc đó?

Lời giải: Trò chơi này sẽ được hoàn thành với các giá trị của n mà chia 3 dư

0 hoặc 2.

Rõ ràng thấy ngay ở trường hợp đơn giản nhất. Bài toán đúng với n = 2 và

n = 3 (mỗi trường hợp có bốn khả năng lật).

Với các giá trị n lớn hơn, chọn mỗi lần lật 3 đồng xu, số đồng xu còn dư được

lật một lần, và những đồng xu dọc theo các cạnh của tam giác có thể được

lật 3 lần. Vì vậy tất cả các đồng xu đều ngửa. Trong khi đó, mỗi đồng xu bên

trong tam giác được lật 6 lần, và chúng lập thành một tam giác có số đồng xu

trên mỗi cạnh là n − 3.

Theo phương pháp quy nạp, các giá trị n như trên đều thỏa mãn.

Nếu n chia 3 dư 1, ta tô các đồng xu bởi các màu vàng, đỏ và xanh sao cho

bất kỳ 3 đồng xu nào cạnh nhau cũng có màu khác nhau. Cũng vậy, 3 đồng

xu liên tiếp bất kỳ trên một hàng cũng có màu khác nhau.

Nếu các đồng xu ở góc đều có màu vàng thì số đồng xu màu vàng nhiều hơn

số đồng xu màu xanh hoặc màu đỏ là 1 đồng.

Lúc này tính chẵn, lẻ của số đồng xu ngửa màu vàng khác với tính chẵn, lẻ

của số đồng xu ngửa màu đỏ.

Từ sự khác nhau về tính chẵn, lẻ của số đồng xu ngửa của mỗi màu, chúng ta

không thể kết thúc nếu không có sự như nhau về tính chẵn, lẻ của số đồng xu

ngửa màu vàng và màu đỏ. Đó có thể là một trường hợp nếu tất cả các đồng

xu đều đã lật ngửa. Vậy các đồng xu không thể được sắp xếp.

.5.24. Cho ABCD là một hình vuông cố định. Xác định tất cả các vị trí có

thể của S để hình vuông PQRS với P và R nằm trên 2 cạnh khác nhau của

ABCD; Q nằm trên đường chéo của ABCD. Xác định tất cả các vị trí có thể

của điểm S.

Lời giải: Các vị trí tạo thành các hình vuông khác nhau, quay 450 với tâm là

giao của hai đường chéo của hình vuông.

Giả sử ta đưa ra các số phức sao cho A = 0; B = 1; C = 1 + i và D = i.

Trước tiên, giả sử P và R nằm trên 2 cạnh liền nhau của ABCD. Không mất

tính tổng quát, giả sử P nằm trên AB và R nằm trên BC. Trong trường hợp

này Q phải nằm trên AC. (Với bất cứ điểm nào nằm trên BD không trùng

vnmath.com

Page 183: OLYMPIC TOÁN NĂM 2000 52 ĐỀ THI VÀ LỜI …»i nói đầu Để thử gói lệnh lamdethi.sty tôi biên soạn một số đề toán thi Olympic, mà các học trò của

Đề thi olympic Colombia 23

với tâm của hình vuông, phép quay với góc quay 900 mà AB không trùng với

AD.) Nếu P = x; Q = y + yi thì R = (2y − x)i và S = (x − y) + (y − x)i,

trong đó các biến dọc theo hình vuông đã cho.

Nếu P và R nằm trên 2 cạnh đối diện của ABCD, không mất tính tổng quát,

ta giả sử P nằm trên AB, R nằm trên CD và Q nằm trên AC. Hơn nữa, ta giả

thiết Q = y + yi với 12≤ y ≤ 1. Quay cạnh AB một góc 900 ta có Q trùng với

một điểm duy nhất trên CD. Như vậy P = 2y−1; R = i và S = y−1+(1−y)i

với các biến dọc theo hình vuông đã cho.

.5.25. Chứng minh rằng tập hợp các số nguyên dương có thể chia thành vô hạn

các tập có vô hạn số A1, A2, . . . , (các tập rời nhau) sao cho nếu x, y, z, w thuộc

Ak với k nào đó, khi đó x − y và z − w cùng thuộc tập Ai (trong đó i không

nhất thiết bằng k) khi và chỉ khi xy

= zw.

A B

CD

P

RQ

Lời giải: Gọi Ak là tập bao gồm tất cả các số có dạng (2k−1)2n và cách phân

chia này sẽ thỏa mãn yêu cầu đề ra.

Thật vậy, giả sử x, y, z, w ∈ Ak với x > y và z > w Ta có:

x = (2k − 1)(2(a + b)), y = (2k − 1)2a, z = (2k − 1)2(c + d), w = (2k − 1)2c.

Khi đó

x − y = (2k − 1)(2b − 1)(2a), z − w = (2k − 1)(2d − 1)(2c)

Do xy

= 2b; zw

= 2d, ⇒ xy

= zw

khi và chỉ khi b = d khi và chỉ khi x− y và z−w

có ước số lẻ chung lớn nhất.

vnmath.com

Page 184: OLYMPIC TOÁN NĂM 2000 52 ĐỀ THI VÀ LỜI …»i nói đầu Để thử gói lệnh lamdethi.sty tôi biên soạn một số đề toán thi Olympic, mà các học trò của

Chương 6

Đề thi olympic Czech và Slovak

Repubulick

.6.26. Cho tam giác ABC có ba cạnh lần lượt là a, b, c và ba góc tương ứng

α, β, γ. Chứng minh rằng: nếu α = 3β thì (a2 − b2) (a − b) = bc2, xét xem

chiều ngược lại có đúng không.

Lời giải: Theo hệ quả của định lý Sin ta có a = 2R sin α, b = 2R sin β, c =

2R sin γ, với R là tâm đường tròn ngoại tiếp tam giác ABC. Vì vậy:

(a2 − b2

)(a − b) = 8R3

(sin2 α − sin2 β

)(sin α − sin β)

= 8R3(sin2 3β − sin2 β

)(sin 3β − sin β)

= 8R3 (sin 3β − sin β)2 (sin 3β + sin β)

= 8R3(8 cos2 2β sin2 β sin2 β cos β

)

= 8R3(sin2

(1800 − 4β

))(sin β)

= 8R3(sin2 γ

)(sin β)

= bc2

Nói chung, chiều ngược lại sai; ta có thể lấy α = 3β − 360o, ví dụ như α =

15o, β = 125o, γ = 40o

vnmath.com

Page 185: OLYMPIC TOÁN NĂM 2000 52 ĐỀ THI VÀ LỜI …»i nói đầu Để thử gói lệnh lamdethi.sty tôi biên soạn một số đề toán thi Olympic, mà các học trò của

Đề thi olympic Czech và Slovak Repubulick 25

.6.27. Mỗi cạnh và đường chéo của một n-giác đều (n ≥ 3) được tô màu đỏ

hoặc màu xanh. Ta chọn một đỉnh và thay đổi màu của các đoạn thẳng nhận

điểm đó làm đầu mút đó từ màu đỏ thành màu xanh và ngược lại. Chứng minh

rằng, với bất kỳ cách tô màu lúc đầu thế nào, ta vẫn có thể biến số cạnh màu

xanh xuất phát từ mỗi đỉnh là số chẵn. Chứng minh rằng, kết quả cuối cùng

của việc tô màu được quy định dựa trên cách tô màu ban đầu.

Lời giải: Nhận thấy, thứ tự chọn các đỉnh không ảnh hưởng đến kết quả tô

màu cuối cùng. Và việc chọn một đỉnh hai lần không ảnh hưởng đến kết quả tô

màu. Vì thế, việc chọn một tập hợp các đỉnh cũng cho kết quả như việc chọn

các đỉnh còn lại: Quá trình sau cũng tương tự như việc chọn một tập hợp các

đỉnh đầu tiên, sau đó chọn tất cả các đỉnh (ở đây, trong tập hợp các đỉnh còn

lại, những đỉnh ban đầu được chọn số lẻ lần bây giờ được chọn theo số chẵn

lần và ngược lại).

Đặt tên các đỉnh là 1, 2, ..., 2n + 1. Gọi ai là số các đoạn màu xanh xuất phát

từ đỉnh thứ i, gọi bi là số lần mỗi đỉnh được chọn và B =∑

bi. Khi chọn đỉnh

k thì ak trở thành 2n − a ≡ ak; mặt khác, mỗi đoạn từ đỉnh k tới một đỉnh

khác đổi màu nên ai còn lại thay đổi tính chẵn lẻ.

Tính tổng số ai thì cho ra kết quả là hai lần tổng số các đoạn màu xanh, vì

thế có một số chẵn các đỉnh với ai là số lẻ -gọi là 2x các đỉnh. Chọn các đỉnh

này. Tính chẵn lẻ của các số ai thay đổi 2x − 1 lần để thành số chẵn. Tính

chẵn lẻ của các số ai còn lại thì thay đổi 2x lần để giữ nguyên là số chẵn. Do

đó, tất cả các đỉnh đều có một số chẵn các đoạn màu xanh. Vậy ta đã chứng

minh được kết quả tô màu cuối cùng là duy nhất.

Ta xét một cách tô màu với kết quả như mong muốn. Cuối cùng, số đoạn màu

xanh ai xuất phát từ đỉnh thứ i là ai + B − bi (mod2). Khi đó, số đoạn màu

xanh xuất phát từ các đỉnh là bằng nhau, do đó, bj ≡ bk khi và chỉ khi lúc đầu

aj ≡ ak. Vì vậy, hoặc bi ≡ 1 khi và chỉ khi ai ≡ 1 hoặc là bi ≡ 1 khi và chỉ khi

ai ≡ 0, ta có kết quả tô màu như trên. Do đó kết quả tô màu là duy nhất.

Bài toán được chứng minh

Chú ý: với một 2n-giác (n ≥ 2), thì việc chọn một đỉnh sẽ làm thay đổi tính

chẵn lẻ của tất cả các ai. Vì thế, ta không thể có được kết quả tất cả các ai là

số chẵn, nếu các ai ban đầu không cùng tính chẵn lẻ. Và nếu có tất cả các ai

là số chẵn thì kết quả tô màu cuối cùng là không duy nhất.

vnmath.com

Page 186: OLYMPIC TOÁN NĂM 2000 52 ĐỀ THI VÀ LỜI …»i nói đầu Để thử gói lệnh lamdethi.sty tôi biên soạn một số đề toán thi Olympic, mà các học trò của

26 Nguyễn Hữu Điển, ĐHKHTN Hà Nội

.6.28. Cho tứ diện ABCD được chia thành 5 khối đa diện lồi sao cho mỗi mặt

của tứ diện ABCD là một mặt của khối đa diện (không có mặt nào bị chia),

và hai khối đa diện bất kỳ trong 5 khối đa diện hoặc có một đỉnh chung, hoặc

có một cạnh chung hoặc có một mặt chung. Hỏi 5 khối đa diện có tổng số mặt

ít nhất là bao nhiêu?

Lời giải: Tổng số mặt nhỏ nhất là 22. Không có khối đa diện nào có chung

hai mặt với tứ diện ABCD, nếu không, do tính lồi của khối đa diện nên nó sẽ

là ABCD. Do đó, có một khối đa diện P không có chung một mặt với ABCD

và các mặt của nó nằm bên trong tứ diện ABCD. Do đó, mỗi mặt của P phải

là mặt chung của P với một khối đa diện khác, có nghĩa là P có chung ít nhất

3 mặt với một trong những khối đa diện còn lại. Đồng thời, bất kỳ mặt nào

của khối đa diện không là một mặt của tứ diện ABCD thì phải là một mặt

của khối đa diện khác. Tức là, tổng số các mặt của 5 khối đa diện là số chẵn.

Do mỗi khối đa diện phải có ít nhất 4 mặt, nên tổng số mặt ít nhất là 20. Giả

thiết, đây là tổng. Khi đó mỗi khối đa diện là một tứ diện có 4 đỉnh và P có ít

nhất 2 đỉnh chung với ABCD. Và nếu nó có 2 điểm chung với ABCD, giả sử

là A và B, khi đó, nó sẽ có tối đa 2 đỉnh chung với tứ diện mà 3 trong 4 đỉnh

là A, C, D. Điều này là vô lý. Do đó, tổng của các mặt lơn hớn hoặc bằng 22.

Ta sẽ chỉ ra trường hợp để dấu bằng xảy ra. Lần lượt lấy P và Q gần với A và

B. Khi đó 5 khối đa diện APCD, PQCD, BQCD, ABDPQ và ABCPQ thỏa

mãn các điều kiện đề bài mà tổng số mặt của 5 khối đa diện này bằng 22.

.6.29. Chỉ ra rằng tồn tại một dãy các số tự nhiên tăng dần {an}∞n=1 với mọi

k ≥ 0, mà dãy {k + an} chỉ chứa một số hữu hạn số nguyên tố.

Lời giải: Lấy pk là số nguyên tố thứ k, k ≥ 1 . Chọn a1 = 2. Với n ≥ 1 ,

lấy an+1 là số nguyên nhỏ nhất lớn hơn an mà an+1 ≡ −p (modpk+1) với mọi

k ≤ n. Những số nguyên này tồn tại theo định lý Thặng dư Trung Hoa. Vì

vậy, với mọi k + an ≡ 0 (modpk+1) với n ≥ k + 1 . Do đó, trong dãy {k + an},giá trị lớn nhất trong k + 1 có thể là số nguyên tố; từ số hạng thứ k = 2 trở

đi, các số hạng là bội của pk+1 và phải là hợp số .Ta có điều phải chứng minh.

.6.30. Với mỗi số tự nhiên n ≥ 2, hãy tìm giá trị lớn nhất của biểu thứ sau:

Vn = sin x1 cos x2 + sin x2 cos x3 + · · · + sin xn cos x1

vnmath.com

Page 187: OLYMPIC TOÁN NĂM 2000 52 ĐỀ THI VÀ LỜI …»i nói đầu Để thử gói lệnh lamdethi.sty tôi biên soạn một số đề toán thi Olympic, mà các học trò của

Đề thi olympic Czech và Slovak Repubulick 27

với x1, x2, ·, xn là các số thực bất kỳ.

Lời giải: Áp dụng bất đẳng thức 2ab ≤ a2 + b2 ta có:

Vn 6sin2 x1 + cos2 x2

2+

sin2 x2 + cos2 x3

2+ · · · + sin2 xn + cos2 x1

2=

n

2

Dấu bằng xảy ra khi x1 = x2 = · · · = xn = π4

.6.31. Cho hình bình hành ABCD mà ABD là tam giác nhọn, và BAD = π4.

Trên các cạnh của hình bình hành, lấy các điểm K thuộc AB, L thuộc BC, M

thuộc CD, N thuộc DA sao cho KLMN là tứ giác nội tiếp có bán kính bằng

bán kính đường tròn ngoại tiếp các tam giác ANK và CLM . Tìm quỹ tích các

giao điểm của đường chéo của tứ giác KLMN .

Lời giải: Do các cung chứa các KLN, KMN, LKM, LNM trên đường tròn

ngoại tiếp tứ giác KLMN và các cung chứa KAN, LCM lần lượt trên đường

tròn ngoại tiếp tam giác AKN và CLM có cùng số đo, các góc đó đều

bằng nhau và có cùng số đo là 45o. Các tam giác SKL và SMN với S là

giao điểm của KM và NL, là các tam giác vuông cân tại S và đồng dạng

với nhau. Khi đó, qua phép đồng dạng, K biến thành M, L biến thành N,

AB biến thành CD và BC biến thành DA, vì vậy S nằm trên đoạn BD.

vnmath.com

Page 188: OLYMPIC TOÁN NĂM 2000 52 ĐỀ THI VÀ LỜI …»i nói đầu Để thử gói lệnh lamdethi.sty tôi biên soạn một số đề toán thi Olympic, mà các học trò của

Chương 7

Đề thi olympic Pháp

.7.32. Tại mỗi đỉnh của 1997- giác được gán một số nguyên, sao cho tổng của

chúng bằng 1. Bắt đầu từ một đỉnh nào đó, ta gán theo chiều ngược kim đồng

hồ quanh đa giác. Hỏi có thể chọn một đỉnh bắt đầu mà tổng của k số nguyên

đầu tiên là dương với k = 1, 2, .., 1997

Lời giải: Có. Gọi bk là tổng của k số nguyên đầu tiên, ta có b1997 = 1. Gọi x

là giá trị nhỏ nhất của bk ta tìm một số k lớn nhất mà bk−1 = x. Sau đó ta

bắt đầu từ k đỉnh đó thì mọi tổng số sẽ là số dương.

.7.33. Tìm thể tích lớn nhất của một hình trụ được chứa trong phần chung của

một hình cầu tâm O bán kính R và một hình nón đỉnh O cắt hình cầu theo

một đường tròn bán kính r, có cùng trục với hình nón.

Lời giải: Ta có hình trụ cắt hình cầu theo một đường tròn bán kính s < r.

Khoảng cách từ tâm của hình cầu đến mặt phẳng chứa đường tròn này là√R2 − s2. Lại có hình trụ cũng cắt hình nón theo một đường tròn bán kính

s, khoảng cách từ tâm hình cầu đến mặt phẳng chứa đường tròn đó bằng

s√

R2/r2 − 1. (Vì khoảng cách từ tâm của hình cầu đến mặt phẳng chứa

đường tròn là√

R2 − r2). Vì vậy, thể tích của hình trụ là:

πs2(√

R2 − s2 − s√

R2/r2 − 1)

Chúng ta tìm giá trị lớn nhất của biểu thức trên bằng cách cho đạo hàm theo

vnmath.com

Page 189: OLYMPIC TOÁN NĂM 2000 52 ĐỀ THI VÀ LỜI …»i nói đầu Để thử gói lệnh lamdethi.sty tôi biên soạn một số đề toán thi Olympic, mà các học trò của

Đề thi olympic Pháp 29

s bằng 0:

0 = 2s√

R2 − s2 − s3

√R2 − s2

− 3s2√

R2/r2 − 1

Chuyển vế và bình phương ta có:

s4 − 4R2s2 + 4R4

R2 − s2=

9s2R2 − s2r2

r2

Giải phương trình ta được:

s2 =3R2 + r2 +

√(9R2 − r2) (R2 − r2)

6

Và thay s2 vào công thức thể tích ở trên cho ta thể tích lớn nhất.

.7.34. Tìm diện tích lớn nhất của hình chiếu vuông góc của hình lập phương

đơn vị lên một mặt phẳng.

Lời giải: Nhận thấy hình chiếu của hình lập phương là tổng hình chiếu của

3 mặt của hình lập phương đôi một vuông góc với nhau. Diện tích hình chiếu

của mỗi mặt bằng giá trị tuyệt đối của tích hai vectơ đơn vị lần lượt vuông

góc với mặt đó và mặt phẳng chiếu.

Như vậy nếu các tích đó là x, y, z thì giá trị lớn nhất của diện tích hình của

chiếu hình lập phương bằng giá trị lớn nhất của tổng x+y+z với điều kiện x2+

y2+z2 = 1. Mặt khác, theo bất đẳng thức Cauchy-Shwarz√

3√

x2 + y2 + z2 ≥(x+ y + z). Dấu bằng xảy ra khi và chỉ khi x = y = z. Khi đó, giá trị lớn nhất

của diện tích bằng√

3

.7.35. Cho tam giác ABC với a, b, c là độ dài của các cạnh và m, n, p là độ dài

của các đường trung tuyến của nó. Với mọi số thực dương α, gọi λ (α) là số

thực được xác định bởi :

aα + bα + cα = λ (α)α (mα + nα + pα)

(a). Tính λ(2)

(b). Tính giới hạn của λ(α) khi α dần tới 0.

(c). Với điều kiện nào của tam giác ABC thì λ(α) không phụ thuộc vào α.

Lời giải: (a). Gọi m, n, p là độ dài của các đường trung tuyến tương ứng với

các cạnh a, b, c và giả sử a ≤ b ≤ c. Dễ dàng tính được m2 = (2b2 +2c2 − c2)/4

vnmath.com

Page 190: OLYMPIC TOÁN NĂM 2000 52 ĐỀ THI VÀ LỜI …»i nói đầu Để thử gói lệnh lamdethi.sty tôi biên soạn một số đề toán thi Olympic, mà các học trò của

30 Nguyễn Hữu Điển, ĐHKHTN Hà Nội

và tương tự với hai trung tuyến còn lại, vì vậy λ(2) = 2√3

(b). Nếu x ≤ y ≤ z và α → 0 thì

x 6 (xα + yα + zα)1/α6 31/αx

và do đó (xα + yα + zα)1/α dần tới x. Vậy chúng ta có limα→0

λ (α) = ap

(c). Để λ(α) không phụ thuộc vào α ta phải có a2

p2 = 43, dẫn đến a2 + c2 = 2b2.

Kết hợp với giả thiết ta có m = c√

32

, n = b√

32

, p = a√

32

.

Vậy λ(α) là hằng số khi tam giác ABC thỏa mãn điều kiện trên.

vnmath.com

Page 191: OLYMPIC TOÁN NĂM 2000 52 ĐỀ THI VÀ LỜI …»i nói đầu Để thử gói lệnh lamdethi.sty tôi biên soạn một số đề toán thi Olympic, mà các học trò của

Chương 8

Đề thi olympic Đức

.8.36. Xác định tất cả các số nguyên tố p sao cho hệ

p + 1 = 2x2

p2 + 1 = 2y2

có nghiệm x, y nguyên.

Lời giải: Chỉ có duy nhất số nguyên tố p = 7 thỏa mãn bài toán. Không mất

tính tổng quát có thể giả sử rằng x, y ≥ 0. Chú ý rằng p + 1 = 2x2 là chẵn,

nên p 6= 2. Hơn nữa, 2x2 ≡ 1 ≡ 2y2 (mod p) suy ra x ≡ ±y (mod p) vì p là lẻ.

Từ x < y < p, ta có x + y = p. Do đó

p2 + 1 = 2(p − x)2 = 2p2 − 4xp + p + 1,

nên p = 4x − 1, 2x2 = 4x, x là 0 hoặc 2 và p là −1 hoặc 7. Hiển nhiên −1

không là số nguyên tố, và p = 7, (x, y) = (2, 5) là nghiệm của bài toán.

.8.37. Một hình vuông Sa nội tiếp một tam giác nhọn ABC với hai đỉnh nằm

trên cạnh BC và một đỉnh nằm trên AB, một đỉnh nằm trên AC. Các hình

vuông Sb, Sc được xây dựng tương tự. Với những trường hợp nào của tam giác

ABC thì các hình vuông Sa, Sb, Sc là bằng nhau.

Lời giải:

vnmath.com

Page 192: OLYMPIC TOÁN NĂM 2000 52 ĐỀ THI VÀ LỜI …»i nói đầu Để thử gói lệnh lamdethi.sty tôi biên soạn một số đề toán thi Olympic, mà các học trò của

32 Nguyễn Hữu Điển, ĐHKHTN Hà Nội

A

B C

xa

α

β γ

Đặt R là bán kính đường tròn ngoại tiếp tam giác ABC và đặt xa, xb, xc

là độ dài các cạnh của Sa, Sb, Sc tương ứng. Kí hiệu α, β, γ là các góc

∠BAC, ∠CBA, ∠ACB.

Giả sử rằng Sa có các đỉnh P, Q nằm trên cạnh BC trong đó P gần B hơn.

Khi đó

2R sin α = BC = BP + PQ + QC

= xa cot β + xa + xa cot γ.

xa =2R sin α

1 + cotβ + cot γ

=2R sin α sin β sin γ

sin β sin γ + cos β sin γ + cos γ sin β

=2R sin α sin β sin γ

sin β sin γ + sin α

và tương tự cho xb, xc. Từ xa = xb suy ra

sin β sin γ + sin α = sin γ sin α + sin β

0 = (sin β − sin α)(sin γ − 1).

Từ tam giác ABC là nhọn, ta có sin β = sin α, suy ra α = β vì trong trường

hợp trái lại thì α + β = π là không thể xảy ra trong tam giác. Tương tự như

vậy β = γ, nên ABC là tam giác đều.

.8.38. Trong một công viên, 10000 cây được trồng theo một hình lưới vuông.

Xác định số lớn nhất các cây có thể đốn hạ mà sao cho từ mỗi gốc cây đã đốn,

bạn không thể nhìn thấy một gốc bất kì khác. (Giả sử rằng bán kính của các

cây là không đáng kể so với khoảng cách của các cây kề nhau.)

vnmath.com

Page 193: OLYMPIC TOÁN NĂM 2000 52 ĐỀ THI VÀ LỜI …»i nói đầu Để thử gói lệnh lamdethi.sty tôi biên soạn một số đề toán thi Olympic, mà các học trò của

Đề thi olympic Đức 33

Lời giải: Ta nhận thấy rằng trong một hình vuông gồm bốn cây kề nhau ta

chỉ bỏ đi được nhiều nhất một cây. Từ lưới 100 × 100 ta có thể chia 10000

đỉnh vào 2500 hình vuông rời nhau thực sự, do đó, có thể đốn hạ nhiều nhất

2500 cây.

Ta sẽ chỉ ra một cách đốn 2500 cây thỏa mãn bài toán. Đồng nhất các cây

với tọa độ (x, y) trên lưới, 0 ≤ x, y ≤ 99, và đốn hạ tất cả các cây có các tọa

độ chẵn. Xét hai gốc bất kì (a, b) và (c, d) với a, b, c, d là chẵn. Xét p/q là một

biểu diễn của (d − b)/(c − a) với các hạng tử là bé nhất (ở đây p, q có cùng

dấu với d− b, c− a, tương ứng), khi đó, ít nhất một trong hai số a+ p và b+ q

phải là lẻ, do đó, cây (a+p, b+q) sẽ chắn tầm nhìn giữa các cây (a, b) và (c, d).

.8.39. Cho một hình viên phân AMB với góc trung tâm ∠AMB nhỏ hơn 90o.

Từ một điểm P bất kì trên cung AB hạ các đường vuông góc PC và PD xuống

MA và MB (C ∈ MA, D ∈ MB). Chứng minh rằng độ dài đoạn thẳng CD

không phụ thuộc vào vị trí điểm P trên cung AB.

Lời giải:

M

A

B

P

C

D

Từ ∠PCM = ∠PDM = π/2, nên tứ giác PCMD nội tiếp đường tròn đường

kính MP . Do đó, áp dụng Định lí hàm số sin cho tam giác MCD, ta có,

CD = MP sin CMD là một hằng số.

.8.40. Trong một hình vuông ABCD xây dựng bốn cung tròn vuông, mỗi cung

tròn có tâm tương ứng là A, B, C, D và chứa hai đỉnh kề với tâm. Bốn cung

tròn này cắt nhau tại bốn điểm E, F, G, H nằm bên trong ABCD, các điểm

vnmath.com

Page 194: OLYMPIC TOÁN NĂM 2000 52 ĐỀ THI VÀ LỜI …»i nói đầu Để thử gói lệnh lamdethi.sty tôi biên soạn một số đề toán thi Olympic, mà các học trò của

34 Nguyễn Hữu Điển, ĐHKHTN Hà Nội

này cũng làm thành một hình vuông gọi là S. Gọi C là đường tròn tiếp xúc với

cả bốn cung tròn trên. Hãy so sánh diện tích của S và C.

Lời giải: Kí hiệu [C] là diện tích của cung tròn C, và [S] là diện tích của hình

vuông S. Không mất tính tổng quát có thể coi E là giao điểm của các cung

tròn mà gần AB nhất, và G là giao điểm gần CD nhất. Hạ các đường vuông

góc EE ′ xuống AB, và GG′ xuống CD. Theo tính chất đối xứng, E ′, E, G, G′

là thẳng hàng.

Từ AB = BG = AG ta có ABG là tam giác đều và đường cao GE ′ =√

3AB/2.

Tương tự, G′E =√

3AB/2. Do đó,√

3AB = GE ′ + G′E = AB + EG, nên

EG = (√

3 − 1)AB và [S] = EG2/2 = (2 −√

3)AB2.

Đặt I, K là các tiếp điểm của đường tròn C với các cung tròn tâm C và A,

tương ứng. Lại do tính đối xứng thì A, I, K, C thẳng hàng. Khi đó

2AB = AK + IC = AC + IK =√

2AB + IK,

nên, IK = (2 −√

2)AB. Do vậy

[C] =π

4IK2 =

(3 − 2√

2)π

2AB2 > (2 −

√3)AB2 = [S].

Vậy diện tích của đường tròn C lớn hơn diện tích của hình vuông S.

A

B C

D

E

F

G

H

I

K

E ′ G′

.8.41. Kí hiệu u(k) là ước lẻ lớn nhất của số tự nhiên k. Chứng minh rằng

1

2n·

2n∑

k=1

u(k)

k≤ 2

3.

vnmath.com

Page 195: OLYMPIC TOÁN NĂM 2000 52 ĐỀ THI VÀ LỜI …»i nói đầu Để thử gói lệnh lamdethi.sty tôi biên soạn một số đề toán thi Olympic, mà các học trò của

Đề thi olympic Đức 35

Lời giải: Đặt v(k) là ước lớn nhất của k có dạng lũy thừa của 2, nên u(k)v(k) =

k. Trong {1, 2, ..., 2n} có 2n−i−1 giá trị của k sao cho v(k) = 2i với i ≤ n − 1,

và một giá trị sao cho v(k) = 2n. Do đó, vế trái bằng

1

2n·

2n∑

k=1

u(k)

k=

1

4n+

n−1∑

i=0

2n−i−1

2n+i.

Từ tổng của chuỗi hình học ta có

1

2n·

2n∑

k=1

u(k)

k= 4−n +

2

3(1 − 4−n) >

2

3.

.8.42. Tìm tất cả các số thực thỏa mãn hệ

x3 = 2y − 1

y3 = 2z − 1

z3 = 2x − 1.

Lời giải: Trước hết ta chỉ ra rằng x = y = z. Giả sử trái lại rằng x 6= y. Nếu

x > y, thì y = (x3 +1)/2 > (y3 +1)/2 = z, nên y > z, và tương tự z > x, mâu

thuẫn. Tương tự, nếu x < y thì y < z và z < x, mẫu thuẫn. Nên các nghiệm

của hệ phương trình có dạng x = y = z = t với t là nghiệm của phương trình

t3 = 2t − 1. Vậy, nghiệm của hệ phương trình là

x = y = z = t, t ∈{

1,−1 +

√5

2,−1 −

√5

2

}.

.8.43. Định nghĩa các hàm số

f(x) = x5 + 5x4 + 5x3 + 5x2 + 1

g(x) = x5 + 5x4 + 3x3 − 5x2 − 1.

Tìm tất cả các số nguyên tố p mà tồn tại số tự nhiên 0 ≤ x < p, sao cho cả

f(x) và g(x) đều chia hết cho p, và với từng giá trị của p, hãy tìm tất cả các

giá trị của x tương ứng.

vnmath.com

Page 196: OLYMPIC TOÁN NĂM 2000 52 ĐỀ THI VÀ LỜI …»i nói đầu Để thử gói lệnh lamdethi.sty tôi biên soạn một số đề toán thi Olympic, mà các học trò của

36 Nguyễn Hữu Điển, ĐHKHTN Hà Nội

Lời giải: Chú ý rằng

f(x) + g(x) = 2x3(x + 1)(x + 4).

Nên nếu p là ước của f(x) và g(x) thì vì p là số nguyên tố nên p phải là ước

của ít nhất một trong các số sau 2, x, x + 1, x + 4. Từ f(0) = 1 và f(1) = 17

dẫn đến p 6= 2. Hơn nữa, p cũng không thể là ước của x, vì khi đó, f(x) ≡ 1

(mod p) là vô lí. Nếu p là ước của x + 1 thì f(x) ≡ 5 (mod p), kéo theo, p là

ước của 5 nên p = 5 và có ngay rằng x = 4. Trường hợp p là ước của x + 4 thì

f(x) ≡ 17 (mod p) nên p = 17 và dễ thấy x = 13 là thỏa mãn.

Vậy các lời giải của bài toán là p = 5, x = 4 và p = 17, x = 13.vnmath.com

Page 197: OLYMPIC TOÁN NĂM 2000 52 ĐỀ THI VÀ LỜI …»i nói đầu Để thử gói lệnh lamdethi.sty tôi biên soạn một số đề toán thi Olympic, mà các học trò của

Chương 9

Đề thi olympic Irland

.9.44. Tìm tất cả các cặp số nguyên (x, y) sao cho 1 + 1996x + 1998y = xy

Lời giải: Ta có: (x − 1998) (y − 1996) = xy− 1998y− 1996x+1996.1998 =

19972

Do 1997 là số nguyên tố, nên ta có: x − 1998 = ±1;±1997;±19972. Vậy có 6

giá trị (x, y) thỏa mãn là

(x, y) =(1999, 19972 + 1996

),(1997,−19972 + 1996

),

(3995, 3993) , (1,−1)(19972 + 1998, 1997

),(−19972 + 1998, 195

)

.9.45. Cho ∆ABC, M là điểm trong tam giác. Goi D,E,F lần lượt là hình

chiếu của M xuống BC, CA, AD. Tìm tập hợp tất cả các điểm M thỏa mãn

FDE = π2.

Lời giải: Từ các tứ giác nội tiếp MDBF và MDCE ta có MDE = MCE và

MDF = MBE do đó FDE = π2⇔ MCB + MBC = π

6hay BMC = 5π

6⇔

M nằm trên cung tròn đi qua B và C.

.9.46. Tìm tất cả các đa thức P (x) sao cho đối với mọi x ta có :

(x − 16) P (2x) = 16 (x − 1) P (x) .

vnmath.com

Page 198: OLYMPIC TOÁN NĂM 2000 52 ĐỀ THI VÀ LỜI …»i nói đầu Để thử gói lệnh lamdethi.sty tôi biên soạn một số đề toán thi Olympic, mà các học trò của

38 Nguyễn Hữu Điển, ĐHKHTN Hà Nội

Lời giải: Goi d = degP và a là hệ số của x trong P (x) với số mũ lớn nhất.

Khi đó hệ số của x mũ lớn nhất ở bên trái là 2da phải bằng 16a do đó d = 4

Do vế phải lúc này chia hết cho (x−1), nhưng trong trường hợp đó vế phải lại

chia hết cho (x − 2), tương tự là chia hết cho (x − 4) và (x − 8). Vậy đa thức

P (x) là bội của (x − 1)(x − 2)(x − 4)(x − 8) là tất cả các đa thức thỏa mãn.

.9.47. Cho a, b, c là các số thức không âm sao cho a + b + c ≥ abc. Chứng minh

rằng a2 + b2 + c2 ≥ abc.

Lời giải: Giả sử phản chứng rằng với a, b, c > 0 mà a2 + b2 + c2 < abc do đó

abc > a2 ⇒ a < bc. Làm tương tự ta cũng có b < ca, c < ab. Do đó abc ≥a2+b2+c2 ≥ ab+bc+ca. Theo bất đẳng thức AM-GM và ab+bc+ca > a+b+c

suy ra abc > a + b + c. Trái với giả thiết. Vậy bài toán được chứng minh.

.9.48. Cho tập hợp S = {3, 5, 7, ...}. Với mỗi x ∈ S ta đặt δ(x) là xác định một

số nguyên duy nhât sao cho: 2δ(x) < x < 2δ(x)+1

Đối với a, b ∈ S ta định nghĩa phép toán

a ∗ b = 2δ(a)−1 (b − 3) + a

a, Chứng minh rằng nếu a, b ∈ S thì a ∗ b ∈ S

b, Chứng minh rằng nếu a, b, c ∈ S thì (a ∗ b) ∗ c = a ∗ (b ∗ c).

Lời giải: a, Hiển nhiên

b, Nếu 2m < a < 2m+1, 2n < b < 2n+1 thì

a ∗ b = 2m−1 (b − 3) + a ≥ 2m−1 (2n − 2) + 2m + 1 = 2n+m−1 + 1

và a ∗ b ≤ 2m−1 (2n+1 − 4) + 2m+1 − 1 = 2m+n − 1. Vì vậy δ(a ∗ b) = m + n− 1

Nếu 2p < c < 2p+1 thì

(a ∗ b) ∗ c =(2m−1 (b − 3) + a

)∗ c = 2m+n−2 (c − 3) + 2m−1(b − 3) + a

a∗(b ∗ c) = a∗(2m−1 (c − 3) + b

)= 2m−1

(2n−1(c − 3) + b − 3

)+a = (a ∗ b)∗c.

.9.49. Cho tứ giác lồi ABCD có một đường tròn nội tiếp. Nếu

A = B =2π

3, D =

π

2, BC = 1

vnmath.com

Page 199: OLYMPIC TOÁN NĂM 2000 52 ĐỀ THI VÀ LỜI …»i nói đầu Để thử gói lệnh lamdethi.sty tôi biên soạn một số đề toán thi Olympic, mà các học trò của

Đề thi olympic Irland 39

Tìm độ dài AD

Lời giải: Goi I là tâm đường tròn nôi tiếp . Do ∆ABC là tam giác đều,

BIC = 1050, ICB = 150, AID = 750, IDA = 450 nên

AD =BI

BC

AD

AI=

sin 150

sin 1050

sin 750

sin 450=

√2 sin 150.

.9.50. Gọi A là tập con của {0, 1, 2, ..., 1997} gồm hơn 1000 phần tử. Chứng

minh rằng A chỉ gồm những lũy thừa của 2 hoặc hai phần tử phân biệt có tổng

là lũy thừa của 2.

Lời giải: Giả sử tập A không thỏa mãn bài toán. Khi đó A sẽ bao gồm hơn

nửa số nguyên từ 51 tới 1997 mà chúng được chia thành từng cặp có tổng là

2048 (V D : 51 + 1997 = 2048...). Tương tự như vậy, A bao gồm nhiều nhất

nửa số nguyên từ 14 tới 50, gồm nhiều nhất nửa số nguyên từ 3 tới 13, và có

thể cả số 0, do đó A có tổng cộng 937 + 18 + 5 + 1 = 997 số nguyên, trái với

giả thiết A gồm hơn 1000 số nguyên từ tập {0, 1, 2, ..., 1997}.

.9.51. Xác định số tự nhiên n thỏa mãn những điều kiện sau:

a, Khai triển thập phân của n gồm 1000 số

b, Tất cả các số trong khai triển là số lẻ.

c, Hai phần tử bất kỳ liền nhau trong khai triển của n hơn kém nhau 2 đơn vị

Lời giải: Đặt an, bn, cn, dn, en là số trong khai triển của n, đó là những số lẻ

và hai số liên tiếp khác nhau 2 đơn vị do đó tận cùng theo thứ tự là 1, 3, 5, 7, 9

do đó

0 1 0 0 0

1 0 1 0 0

0 1 0 1 0

0 0 1 0 1

0 0 0 0 1

an

bn

cn

dn

en

=

an+1

bn+1

cn+1

dn+1

en+1

Gọi A là ma trận vuông trong biểu thức đó. Ta tìm giá trị riêng của của A,

vnmath.com

Page 200: OLYMPIC TOÁN NĂM 2000 52 ĐỀ THI VÀ LỜI …»i nói đầu Để thử gói lệnh lamdethi.sty tôi biên soạn một số đề toán thi Olympic, mà các học trò của

40 Nguyễn Hữu Điển, ĐHKHTN Hà Nội

giả sử Av = λv với v = (v1, v2, v3, v4, v5). Do đó

v2 = λv1

v3 = λv2 − v1 = (λ2 − 1) v1

v4 = λv3 − v2 = (λ3 − 2λ) v1

v5 = λv4 − v3 = (λ4 − 3λ2 + 1) v1

và v4 = λv5, do đó λ5 − 3λ3 + λ = λ3 − 2λ. Giải pt này ta được λ = 0, λ =

±1, λ = ±√

3 tương ứng ta có các vectơ riêng x1, x2, x3, x4, x5 là

(1, 0,−1, 0, 1) , (1, 1, 0,−1,−1) , (1,−1, 0, 1,−1) ,(1,±

√3, 2,±

√3, 1

)

(1, 1, 1, 1, 1) =1

3x1

2 +√

3

6x4 +

2 −√

3

6x5

Vì vậy

(a1000,b1000, c1000, d1000, e1000) =

= 3999

2

2 +√

3

6

(1,√

3, 2,√

3, 1)− 2 −

√3

6

(1,−

√3, 2,−

√3, 1

)

=(3499, 2.3499, 2.3499, 2.3499, 3499

)

Vì thế kết quả của bài toán là 8.3499.

vnmath.com

Page 201: OLYMPIC TOÁN NĂM 2000 52 ĐỀ THI VÀ LỜI …»i nói đầu Để thử gói lệnh lamdethi.sty tôi biên soạn một số đề toán thi Olympic, mà các học trò của

Nguyễn Hữu Điển

OLYMPIC TOÁN NĂM 1997-1998

49 ĐỀ THI VÀ LỜI GIẢI(Tập 5)

NHÀ XUẤT BẢN GIÁO DỤC

vnmath.com

Page 202: OLYMPIC TOÁN NĂM 2000 52 ĐỀ THI VÀ LỜI …»i nói đầu Để thử gói lệnh lamdethi.sty tôi biên soạn một số đề toán thi Olympic, mà các học trò của

2

vnmath.com

Page 203: OLYMPIC TOÁN NĂM 2000 52 ĐỀ THI VÀ LỜI …»i nói đầu Để thử gói lệnh lamdethi.sty tôi biên soạn một số đề toán thi Olympic, mà các học trò của

Lời nói đầu

Để thử gói lệnh lamdethi.sty tôi biên soạn một số đề toán thi Olympic,mà các học trò của tôi đã làm bài tập khi học tập LATEX. Để phụ vụ các bạnham học toán tôi thu thập và gom lại thành các sách điện tử, các bạn có thểtham khảo. Mỗi tập tôi sẽ gom khoảng 51 bài với lời giải.

Rất nhiều bài toán dịch không được chuẩn, nhiều điểm không hoàn toànchính xác vậy mong bạn đọc tự ngẫm nghĩ và tìm hiểu lấy. Nhưng đây lànguồn tài liệu tiếng Việt về chủ đề này, tôi đã có xem qua và người dịch làchuyên về ngành Toán phổ thông. Bạn có thể tham khảo lại trong [1].

Rất nhiều đoạn vì mới học TeX nên cấu trúc và bố trí còn xấu, tôi khôngcó thời gian sửa lại, mong các bạn thông cảm.

Hà Nội, ngày 2 tháng 1 năm 2010

Nguyễn Hữu Điển

51GD-05

89/176-05 Mã số: 8I092M5

vnmath.com

Page 204: OLYMPIC TOÁN NĂM 2000 52 ĐỀ THI VÀ LỜI …»i nói đầu Để thử gói lệnh lamdethi.sty tôi biên soạn một số đề toán thi Olympic, mà các học trò của

Mục lục

Lời nói đầu . . . . . . . . . . . . . . . . . . . . . . . . . . . . . . . . . . . . . . . . . . . . . . . . . . . 3

Mục lục. . . . . . . . . . . . . . . . . . . . . . . . . . . . . . . . . . . . . . . . . . . . . . . . . . . . . . . 4

Chương 1. Đề thi olympic Hy Lạp . . . . . . . . . . . . . . . . . . . . . . . . . . . . . . . . 5

Chương 2. Đề thi olympic Hungary . . . . . . . . . . . . . . . . . . . . . . . . . . . . . . . 8

Chương 3. Đề thi olympic Iran . . . . . . . . . . . . . . . . . . . . . . . . . . . . . . . . . . . 15

Chương 4. Đề thi olympic Ireland . . . . . . . . . . . . . . . . . . . . . . . . . . . . . . . 18

Chương 5. Đề thi olympic Italy . . . . . . . . . . . . . . . . . . . . . . . . . . . . . . . . . . 22

Chương 6. Đề thi olympic Japan . . . . . . . . . . . . . . . . . . . . . . . . . . . . . . . . . 25

Chương 7. Đề thi olympic Korean . . . . . . . . . . . . . . . . . . . . . . . . . . . . . . . 30

Chương 8. Đề thi olympic Poland . . . . . . . . . . . . . . . . . . . . . . . . . . . . . . . . 38vnmath.com

Page 205: OLYMPIC TOÁN NĂM 2000 52 ĐỀ THI VÀ LỜI …»i nói đầu Để thử gói lệnh lamdethi.sty tôi biên soạn một số đề toán thi Olympic, mà các học trò của

Chương 1

Đề thi olympic Hy Lạp

.1.1. Cho P là một điểm nằm bên trong hay trên 1 cạnh bất kì của hình vuông

ABCD. Hãy xác định giá tri lớn nhất và giá trị nhỏ nhất có thể có của hàm số

f (P) = ABP + BCP + CDP + DAP

Lời giải:B

A D

C

P

Đặt các đỉnh của hình vuông tương ứng với các giá trị 1, i, -1, -i trong mặtphẳng và coi P là số phức z. Khi đó f(P) là argument của số phức z thoả mãnz − 1

i + 1

z − i

−1 − i

z + 1

−i + 1

z + 1

1 + i=

z4 − 1

4

Khi |P| ≤ 1,z4 − 1

4chạy trên miền phẳng được giới hạn bởi đường tròn bán

kính 1/4, tâm có toạ độ -1/4. Do đó giá trị lớn nhất của góc đạt được tại 1điểm trên biên của hình tròn trên, điều đó xảy ra khi P nằm trên cạnh củahình vuông. Do vai trò của các cạnh là như nhau, không mất tổng quát ta cóthể giả sử cạnh đó là AB.

vnmath.com

Page 206: OLYMPIC TOÁN NĂM 2000 52 ĐỀ THI VÀ LỜI …»i nói đầu Để thử gói lệnh lamdethi.sty tôi biên soạn một số đề toán thi Olympic, mà các học trò của

6 Nguyễn Hữu Điển, ĐHKHTN Hà Nội

Khi P chạy từ A đến B thì CDP giảm từπ

2đến

π

4; BCP giảm từ

π

4đến 0;

Hai góc còn lại nhận các giá trị làπ

2và 0.

Vậy ta có giá trị lớn nhất và nhỏ nhất của f (P) lần lượt là5π

4và

4

.1.2. Cho hàm f : (0; ∞) →R thoả mãn các điều kiện sau:

(a) f tăng nghiêm ngặt

(b) f(x)>−1

xvới mọi x>0

(c) f(x)f(f(x)+1

x)=1 với mọi x>0

Tính f(1).

Lời giải: Đặt k=f(x)+1

x. Vì k>0 nên f(k)f(f(k)+

1

k)=1

Mặt khác f(x)f(k)=1. Do đó f(x)=f(f(k)+1

k)=f(

1

f (x)+

1

f (x) +1

x

)

Do f tăng nghiêm ngặt nên ta có x=1

f (x)+

1

f (x) +1

x

Giải ra ta thu được f(x)=1 ±

√5

2x.

Dễ dàng kiểm tra được rằng chỉ có1 −

√5

2xthoả mãn các yêu cầu của đề bài.

Do đó f(1)=1 −

√5

2

.1.3. Tìm tất cả các số nguyên thoả mãn phương trình sau:

13

x2+

1996

y2=

z

1997

Lời giải: Đặt d=gcd(x,y), từ đó x=dx1, y=dy1

Khi đó phương trình đã cho tương đương với1997(13)y2

1+1997(1996)x21=d2zy2

1x21

Khi x1 và y1 nguyên tố cùng nhau, ta phải có x21|1997 × 13,

y21|1997 × 1996

Dễ dàng kiểm tra được rằng 1997 không phải số chính phương và rõ ràngnó nguyên tố cùng nhau với 13 và 1996. Hơn nữa 1996 = 22.499, và cũng dễdàng kiểm tra được rằng 499 không phải số chính phương.

vnmath.com

Page 207: OLYMPIC TOÁN NĂM 2000 52 ĐỀ THI VÀ LỜI …»i nói đầu Để thử gói lệnh lamdethi.sty tôi biên soạn một số đề toán thi Olympic, mà các học trò của

Đề thi olympic Hy Lạp 7

Khi đó (x1, y1) = (1, 1) hoặc (1,2)

Bài toán được chia thành 2 trường hợp:* Trường hợp 1: (x1, y1) = (1, 1). Khi đó

d2z = (13 + 1996)1997 = 1997.72.41

Khi 1997 nguyên tố cùng nhau với 7 và 41 thì d=1,7. Từ đó ta có kết quả lầnlượt là:

(x,y,z)=(1,1,4011973), (7,7,81877)* Trường hợp 2: (x1, y1) = (1, 2).Khi đó d2z = (13 + 499)1997 =

1997.29

Do đó d=1,2,4,8,16. Ta lại có các kết quả lần lượt là:(x,y,z)=(1,2,1022464),(2,4,255616),(4,8,63904), (8,16,15976), (16,32,3994)Đó là các kết quả thu được.

.1.4. Cho P là một đa thức với các hệ số nguyên có 13 nghiệm nguyên phân biệt.

Hãy chỉ ra rằng nếu n ∈ Z không phải là nghiệm của P thì |P(n)| ≥ 7(6!)2. Hãy

cho 1 ví dụ khi dấu bằng xảy ra.

Lời giải: Phân tích đa thức với các hệ số nguyên thành tích của các đa thứccũng có hệ số nguyên với bậc nhỏ hơn thì P(x) có thể viết dưới dạng

(x − r1)(x − r2)...(x − r13)Q(x)

trong đó rs là 1 trong 13 nghiệm phân biệt của đa thức đó.Do đó với tất mỗi số nguyên x, P(x) có giá trị bằng tích của 13 số nguyênphân biệt với 1 số nguyên khác.Rõ ràng giá trị tuyệt đối nhỏ nhất của kết quả trên là|(1)(−1)(2)(−2)...(6)(−6)(7)(1)| = 7(6!)2.Từ đó ta có điều phải chứng minh.Một ví dụ khi dấu bằng đạt được đó là khi x = 0

và P(x)=(x+1)(x-1)(x+2)(x-2)...(x+7)

vnmath.com

Page 208: OLYMPIC TOÁN NĂM 2000 52 ĐỀ THI VÀ LỜI …»i nói đầu Để thử gói lệnh lamdethi.sty tôi biên soạn một số đề toán thi Olympic, mà các học trò của

Chương 2

Đề thi olympic Hungary

.2.5. Mỗi thành viên trong hội đồng xếp hạng các ứng viên A, B, C theo thứ tự.

Điều đó chỉ ra rằng phần lớn các thứ hạng hội đồng A cao hơn nhiều so với B và

cũng có thể là phần lớn các thứ hạng B cao hơn nhiều so với C. Có phải mà theo đó

A cao hơn C.

Lời giải: Không. Giả sử giả hội đồng có ba thành viên, một trong nhữngngười xếp hạng A > B > C, một trong những người xếp hạng B > C > A,và là một trong những người xếp hạng C > A > B. Sau đó, thứ nhất và thứba cả hai thích A đến B, và thứ nhất và thứ hai thích cả hai B to C, nhưngchỉ là người đầu tiên thích A đến C.

.2.6. Cho phép a, b, c được các bên,

ma, mb, mc

là các độ cao, và

da, db, dc

là các khoảng cách từ đỉnh vào trong một trọng tâm tam giác. Chứng minh rằng.

mada + mbdb + mcdc =a2 + b2 + c2

2

vnmath.com

Page 209: OLYMPIC TOÁN NĂM 2000 52 ĐỀ THI VÀ LỜI …»i nói đầu Để thử gói lệnh lamdethi.sty tôi biên soạn một số đề toán thi Olympic, mà các học trò của

Đề thi olympic Hungary 9

Lời giải: Cho D, E, F là chân của chiều cao từ A, B, C tương ứng, và cho Hlà trực tâm của hình tam giác ABC, Sau đó hình tam giác ACD là giống vớihình tam giác AHE. Vậy

mada = AD · AH = CE · AE = AE · b.

Tương tự hình tam giác ABD là giống với hình tam giác AHF. Vậy

mada = AD · AB = AF · AE = AB · c.

Do đó

mada =AE · b + AF · c

2

Tương tự

mbdb =BF · c + BD · a

2

mcdc =CD · a + CE · b

2

Do đó

mada + mbdb + mcdc

=1

2(AE · b + AF · c + BF · c + BD · a + CD · a + CE · b)

=1

2((BD + CD) · a + (CE + AE) · b + (AF + BF) · c)

=a2 + b2 + c2

2

.2.7. Cho R là bán kính hình tam giác ABC và G, H là trọng tâm và trực tâm

tương ứng. Cho F là trung điểm của GH. Để

AF2 + BF2 + CF2 = 3R2

vnmath.com

Page 210: OLYMPIC TOÁN NĂM 2000 52 ĐỀ THI VÀ LỜI …»i nói đầu Để thử gói lệnh lamdethi.sty tôi biên soạn một số đề toán thi Olympic, mà các học trò của

10 Nguyễn Hữu Điển, ĐHKHTN Hà Nội

Lời giải: Chúng ta sử dụng Vector với gốc tọa độ tại tâm hình tam giácABC. Sau đó chúng ta có công thức H = A + B + CvG = H/3. Vậy F =

(G + H)/2 = 2H/3v2(A + B + C) = 3F.Do đó

AF2 + BF2 + CF2

=(A-F).(A-F) + (B-F).(B-F) + (C-F).(C-F)

= A · A + B · B + C · C − 2(A + B + C) · F + 3F · F

= 3R2 − F · (2(A + B + C) − 3F) = 3R2

.2.8. Một hộp chứa 4 quả bóng trắng và 4 quả bóng đỏ, chúng ta cần vẽ từ cái hộp

theo một số thứ tự mà không cần thay thế. Trước khi vẽ chúng ta cần đoán màu của

quả bóng sẽ vẽ. Con số được mong đợi của các dự đoán chính xác là bao nhiêu?

Lời giải: Con số được chờ đợi của các dự đoán chính xác là 373/70. Cho

i, j ≥ 0, aij

biểu thị con số mong đợi của các dự đoán chính xác khi có i quả bóng trắngvà j quả bóng đỏ. Giả sử i>j>=1. Sau đó dự đoán của chúng ta là chính xácvới xác suất i/(i+j), đưa ra con số mong muốn của các dự đoán chính xáccủa

1 + ai−1,j

và sai với xác suất j/(i+j), đưa ra con số mong muốn của

ai,j−1

Vậy

aij =i

i + j(1 + ai−1,j) +

i

i + jai,j−1

if

i > j

Cũng vậy, chúng ta có

aij = aji

vnmath.com

Page 211: OLYMPIC TOÁN NĂM 2000 52 ĐỀ THI VÀ LỜI …»i nói đầu Để thử gói lệnh lamdethi.sty tôi biên soạn một số đề toán thi Olympic, mà các học trò của

Đề thi olympic Hungary 11

choi, j ≥ 0

Nếui = j ≥ 1

sau đó chúng ta đoán với xác suất 1/2 và

aij =1

2(1 + ai−1,j) +

1

2ai,i−1 =

1

2+ ai,i−1

Nhưai,i−1 = ai−1,i

. Cuối cùng, điều kiện bắt đầu là:

ai0 = a0i = i

vớii ≥ 0

Chúng ta có thể sử dụng những phương trình này cho việc tính toán

a4,4 = 373/70

.2.9. Tìm tất cả các giải pháp cho những số nguyên của phương trình

x3 + (x + 1)3 + (x + 2)3 + ... + (x + 7)3 = y3

Lời giải: các giải pháp là: (-2,6), (-3,4), (-4,-4), (-5,-6) Cho

P(x) = x3 + (x + 1)3 + (x + 2)3 + ... + (x + 7)3 = 8x3 + 84x2 + 420x + 784.

Nếux ≥ 0

thì:

(2x + 7)3 = 8x3 + 84x2 + 294x + 343 < P(x) < 8x3 + 120x2 + 600x + 1000 = (2x + 10)3

vnmath.com

Page 212: OLYMPIC TOÁN NĂM 2000 52 ĐỀ THI VÀ LỜI …»i nói đầu Để thử gói lệnh lamdethi.sty tôi biên soạn một số đề toán thi Olympic, mà các học trò của

12 Nguyễn Hữu Điển, ĐHKHTN Hà Nội

Vậy 2x+7 < y < 2x100: do đó y là 2x + 8 hoặc 2x + 9 nhưng cả hai phươngtrình

P(x) − (2x + 8)3 = −122 + 36x + 272 = 0

P(x) − (2x + 9)3 = −24x2 + 66x + 55 = 0

có bất kỳ căn nguyên. Do vậy nên ko có giải pháp bào với

x ≥ 0

Tiếp theo, chú ý rằng P thỏa mãn P (-x-7) = -P(x), vậy (x.y) là 1 giải pháp nếu(-x-7,-y) là một giải pháp. Do vậy không có giải pháp nào với

x ≤ −7

Do vậy (x.y) là một giải pháp. Chúng ta phải có

−6 ≤ x ≤ −1

Cho−3 ≤ x ≤ −1

Chúng ta có P(-1) = 440không một lũy thừa 3.

P(−2) = 216 = 63

vàP(−3) = 64 = 43

vậy (-2,6) và (-3,4) và chỉ các giải pháp với

−3 ≤ x ≤ −1

Do đó (-4,-4) và (-5,-6) chỉ là giải pháp với

−6 ≤ x ≤ −4

Vậy đáp án chỉ có thể là (-2,6), (-3,4), (-4,-4) và (-5,-6).

vnmath.com

Page 213: OLYMPIC TOÁN NĂM 2000 52 ĐỀ THI VÀ LỜI …»i nói đầu Để thử gói lệnh lamdethi.sty tôi biên soạn một số đề toán thi Olympic, mà các học trò của

Đề thi olympic Hungary 13

.2.10. Chúng ta có 1997 số nguyên dương không trùng nhau, bất kỳ 10 trong số đó

có cùng ít nhất chung. Tìm số lớn nhất có thể của các số nguyên tố cùng nhau giữa

chúng.

Lời giải: số lớn nhất của từng đôi số nguyên tố trong tập hợp này là 9. Trướctiên, giả sử có 10 số nguyên tố..

n1, n2, ....n10

Sau đó ít nhất của 10 thành viên của tập hợp này là

lcm(n1, n2, ....n10) = (n1n2...n10)

Cá biệt, cho bất kỳ N khác trong tập hợp này

lcm(Nn2, ....n10)− (n1n2...n10)

là chia hết chon1

Nhưn1

có quan hệ vớinj

cho−2 ≤ j ≤ 10

chi cho N.Tương tự

n1

chia cho N với mỗii ∈ {2......10}

như vậyni

có quan hệ nguyên tố.n1n2...n10

vnmath.com

Page 214: OLYMPIC TOÁN NĂM 2000 52 ĐỀ THI VÀ LỜI …»i nói đầu Để thử gói lệnh lamdethi.sty tôi biên soạn một số đề toán thi Olympic, mà các học trò của

14 Nguyễn Hữu Điển, ĐHKHTN Hà Nội

chia cho N. Nhưng

N ≤ lcm(N.n2....n10) = n(1)n(2)...n(10)

Vậy chúng ta phải cóN = n(1)n(2)....n(10)

Từ đây nó lưu giữ mỗi thành phần của tập hợp của chúng ta hơn

n(1).......n(10)

Tập hợp của chúng ta chỉ có thể chứa 11 thành phần, một sự mâu thuẫn.Bây giờ chúng ta khởi tạo một ví dụ mà có 9 số nguyên tố.Cho

p(n)

biểu thị số nguyên tố thứ n và cho

S =

{p1p2...p1988

pj|1 ≤ j ≤ 1988

}⋃{n1, n2....n9}

khin(i) = p(i)

với1 ≤ i ≤ 8

n(9) = p(9)p(10)......p(1988)

Rõ ràng bất kỳ 2 thành phần của

n(1).........n(9)

là cặp nguyên tố.

vnmath.com

Page 215: OLYMPIC TOÁN NĂM 2000 52 ĐỀ THI VÀ LỜI …»i nói đầu Để thử gói lệnh lamdethi.sty tôi biên soạn một số đề toán thi Olympic, mà các học trò của

Chương 3

Đề thi olympic Iran

.3.11. Giả sử w1, ..., wk là những số thực phân biệt với tổng khác không.

CMR : tồn tại các số nguyên n1, ..., nk sao cho : n1w1 + ... + nkwk > 0 và một số

hoán vị π của {1, .. ., k} không đồng nhất bằng nhau. Ta có :

n1w1(1) + ... + nkwπ(k) < 0

Lời giải:Đầu tiên, ta “sắp xếp lại” bất đẳng thức :Nếu a1 < ... < an, b1 < ... < bn là những số thực, α = min {ai+1 − ai},β = min {bi+1 − bi} thì với 1 vài hoán vị không thông thường π của {1, ..., n}: ∑ biaπ(i) ≤ ∑ biai − αβ

Điều này là hiển nhiên vì nếu i < j nhưng π (i) > π (j) thì thay π bởinhững sự hợp thành đó với sự chuyển vế của i và j tăng lên thì tổng bằng(

aj − ai

) (bj − bi

)

Giả sử rằng w1 < ... < wk và s = |∑ wi|Đặt α = min {wi+1 − wi} và chọn 1 số tự nhiên N = s

α

Ta đặt (n1, n2, ..., nk) = (N, 2N, ..., kN) + p(1, ...., 1)

Ở đây p là số nguyên duy nhất sao cho ∑ niwi ∈ (0, s]

Đây là định lý bao hàm rằng π 6= 1

∑ niwπ(i) ≤ ∑ niwi − Nα ≤ s − Nα < 0

vnmath.com

Page 216: OLYMPIC TOÁN NĂM 2000 52 ĐỀ THI VÀ LỜI …»i nói đầu Để thử gói lệnh lamdethi.sty tôi biên soạn một số đề toán thi Olympic, mà các học trò của

16 Nguyễn Hữu Điển, ĐHKHTN Hà Nội

.

.3.12. Giả sử điểm P di động dọc theo cung BC của đường tròn ngoại tiếp ∆ABC,

và cho I1, I2 tương ứng là tâm đường tròn nội tiếp ∆PAB, ∆PAC. CMR :

a) Đường tròn ngoại tiếp ∆PI1 I2 đi qua một điểm cố định.

b) Đường tròn đường kính I1 I2 đi qua một điểm cố định.

c) Trung điểm của đoạn I1I2 nằm trên một đường tròn cố định.

Lời giải:Cho B1, C1 là điểm giữa của các cung AC, AB. Do I1, I2 là tâm đường trònnội tiếp của các tam giác ∆ABP, ∆ACP, ta có : C1A = C1B = C1I1, B1A =

B1C = B1 I2.Gọi I là tâm đường tròn ngoại tiếp ∆ABC và Q là giao điểm thứ 2 của đườngtròn ngoại tiếp ∆ABC và ∆PI1 I2. Do C1I1 và B1 I2 đi qua P nên ∆QI1C1 và∆QI2B1 đồng dạng, vậy : QC1

QB1= C1 I1

B1 I2= C1 A

B1 A không đổi.Do đó Q là giao của đường tròn ngoại tiếp ∆ABC với đường tròn Apolloniuscố định, nên Q cố định và phần a) được chứng minh.

Từ : I1QI2 = I1PI2 = C1PB1 =(B+C)

2

Các tam giác ∆QI1 I2 với Q 6= P đều đồng dạng với nhau. Vì thế nếu M làtrung điểm của I1I2 thì các tam giác ∆QI1M cũng đều đồng dạng.Nếu k = QM

QI1, α = MQI1, nghĩa là M là ảnh của I1 qua qua các phép dời hình

tâm Q với góc α và tỉ số k.Do C1I1 = C1A không đổi, I1 chuyển động trên một cung của đường tròncố định nên M nằm trên một đường tròn cố định và (c) được chứng minh.Cuối cùng ta tính được góc I1 I I2 = π

2 . Vì thế đường tròn đường kính I1 I2 điqua I cố định và phần (b) được chứng minh.

.3.13. Giả sử f : R+ → R+ là hàm liên tục, giảm sao cho ∀x, y ∈ R+,

f (x + y) + f ( f (x) + f (y)) = f ( f (x + f (y))) + f (y + f (x))

CMR : f ( f (x)) = x.

Lời giải:Đặt y = x, ta được:

f (2x) + f (2 f (x)) = f (2 f (x + f (x))) (1)

vnmath.com

Page 217: OLYMPIC TOÁN NĂM 2000 52 ĐỀ THI VÀ LỜI …»i nói đầu Để thử gói lệnh lamdethi.sty tôi biên soạn một số đề toán thi Olympic, mà các học trò của

Đề thi olympic Iran 17

Thay x bởi f (x) , ta được :

f (2 f (x)) + f (2 f ( f (x))) = f (2 f ( f (x) + f ( f (x)))) (2)

Lấy (2) trừ (1) , ta được :

f (2 f ( f (x)))− f (2x) = f (2 f ( f (x) + f ( f (x))))− f (2 f (x + f (x)))

Nếu f ( f (x)) > x thì vế trái của phương trình là âm, do đó :

f ( f (x) + f ( f (x))) > f (x + f (x))

Và f (x) + f ( f (x)) < x + f (x), mâu thuẫn.Điều mẫu thuẫn tương tự cũng xảy ra nếu f ( f (x)) < x.Vậy : f ( f (x)) = x.

.3.14. Cho A là một ma trận gồm các số 0 và 1 đối xứng(

Aij = Aji, ∀i, j)

sao cho

Aii = 1 với mọi i. Hãy chỉ ra rằng tồn tại một tập con của các dãy mà tổng tất cả

thành phần của véctơ là lẻ.

Lời giải:Giả sử ngược lại , tồn tại một véctơ (v1, ..., vn) sao cho ∑i viwi = 0 với mộtvài dãy (w1, ..., wn) nhưng ∑ vi 6= 0 (Tất cả các số ở đây là chia hết cho 2) .Cộng trên tất cả các dãy, ta có :

∑j

∑i

vi Aijvj = 0

Do ma trận là đối xứng, điều này được quy về ∑i

v2i Aii = 0 hoặc ∑

ivi = 0

(v1 ∈ {0, 1}), mâu thuẫn .

vnmath.com

Page 218: OLYMPIC TOÁN NĂM 2000 52 ĐỀ THI VÀ LỜI …»i nói đầu Để thử gói lệnh lamdethi.sty tôi biên soạn một số đề toán thi Olympic, mà các học trò của

Chương 4

Đề thi olympic Ireland

.4.15. Tìm tất cả các cặp số nguyên (x, y) sao cho 1 + 1996x + 1998y = xy

Lời giải: Ta có: (x − 1998) (y − 1996) = xy − 1998y − 1996x +

1996.1998 = 19972

Do 1997 là số nguyên tố, nên ta có: x − 1998 = ±1;±1997;±19972. Vậy có 6giá trị (x, y) thỏa mãn là

(x, y) =(

1999, 19972 + 1996)

,(

1997,−19972 + 1996)

,

(3995, 3993) , (1,−1)(

19972 + 1998, 1997)

,(−19972 + 1998, 195

)

.4.16. Cho ∆ABC, M là điểm trong tam giác. Goi D,E,F lần lượt là hình chiếu của

M xuống BC, CA, AD. Tìm tập hợp tất cả các điểm M thỏa mãn FDE = π2 .

Lời giải: Từ các tứ giác nội tiếp MDBF và MDCE ta có MDE = MCE vàMDF = MBE do đó FDE = π

2 ⇔ MCB + MBC = π6 hay BMC = 5π

6 ⇔ Mnằm trên cung tròn đi qua B và C.

.4.17. Tìm tất cả các đa thức P(x) sao cho đối với mọi x ta có :

(x − 16) P (2x) = 16 (x − 1) P (x) .

vnmath.com

Page 219: OLYMPIC TOÁN NĂM 2000 52 ĐỀ THI VÀ LỜI …»i nói đầu Để thử gói lệnh lamdethi.sty tôi biên soạn một số đề toán thi Olympic, mà các học trò của

Đề thi olympic Ireland 19

Lời giải: Goi d = degP và a là hệ số của x trong P(x) với số mũ lớn nhất. Khiđó hệ số của x mũ lớn nhất ở bên trái là 2da phải bằng 16a do đó d = 4

Do vế phải lúc này chia hết cho (x − 1), nhưng trong trường hợp đó vế phảilại chia hết cho (x − 2), tương tự là chia hết cho (x − 4) và (x − 8). Vậy đathức P(x) là bội của (x − 1)(x − 2)(x − 4)(x − 8) là tất cả các đa thức thỏamãn.

.4.18. Cho a, b, c là các số thức không âm sao cho a + b + c ≥ abc. Chứng minh

rằng a2 + b2 + c2 ≥ abc.

Lời giải: Giả sử phản chứng rằng với a, b, c > 0 mà a2 + b2 + c2< abc do

đó abc > a2 ⇒ a < bc. Làm tương tự ta cũng có b < ca, c < ab. Do đóabc ≥ a2 + b2 + c2 ≥ ab + bc + ca. Theo bất đẳng thức AM-GM và ab + bc +

ca > a + b + c suy ra abc > a + b + c. Trái với giả thiết. Vậy bài toán đượcchứng minh.

.4.19. Cho tập hợp S = {3, 5, 7, ...}. Với mỗi x ∈ S ta đặt δ(x) là xác định một số

nguyên duy nhât sao cho: 2δ(x)< x < 2δ(x)+1

Đối với a, b ∈ S ta định nghĩa phép toán

a ∗ b = 2δ(a)−1 (b − 3) + a

a, Chứng minh rằng nếu a, b ∈ S thì a ∗ b ∈ S

b, Chứng minh rằng nếu a, b, c ∈ S thì (a ∗ b) ∗ c = a ∗ (b ∗ c).

Lời giải: a, Hiển nhiênb, Nếu 2m

< a < 2m+1, 2n< b < 2n+1 thì

a ∗ b = 2m−1 (b − 3) + a ≥ 2m−1 (2n − 2) + 2m + 1 = 2n+m−1 + 1

và a ∗ b ≤ 2m−1(2n+1 − 4

)+ 2m+1 − 1 = 2m+n − 1. Vì vậy

δ(a ∗ b) = m + n − 1

Nếu 2p< c < 2p+1 thì

(a ∗ b) ∗ c =(

2m−1 (b − 3) + a)∗ c = 2m+n−2 (c − 3) + 2m−1(b − 3) + a

a ∗ (b ∗ c) = a ∗(

2m−1 (c − 3) + b)

= 2m−1(

2n−1(c − 3) + b − 3)

+ a = (a ∗ b) ∗ c.

vnmath.com

Page 220: OLYMPIC TOÁN NĂM 2000 52 ĐỀ THI VÀ LỜI …»i nói đầu Để thử gói lệnh lamdethi.sty tôi biên soạn một số đề toán thi Olympic, mà các học trò của

20 Nguyễn Hữu Điển, ĐHKHTN Hà Nội

.4.20. Cho tứ giác lồi ABCD có một đường tròn nội tiếp. Nếu

A = B =2π

3, D =

π

2, BC = 1

Tìm độ dài AD

Lời giải: Goi I là tâm đường tròn nôi tiếp . Do ∆ABC là tam giác đều, BIC =

1050, ICB = 150, AID = 750, IDA = 450 nên

AD =BI

BC

AD

AI=

sin 150

sin 1050

sin 750

sin 450=

√2 sin 150.

.4.21. Gọi A là tập con của {0, 1, 2, ..., 1997} gồm hơn 1000 phần tử. Chứng minh

rằng A chỉ gồm những lũy thừa của 2 hoặc hai phần tử phân biệt có tổng là lũy

thừa của 2.

Lời giải: Giả sử tập A không thỏa mãn bài toán. Khi đó A sẽ bao gồm hơnnửa số nguyên từ 51 tới 1997 mà chúng được chia thành từng cặp có tổng là2048 (VD : 51 + 1997 = 2048...). Tương tự như vậy, A bao gồm nhiều nhấtnửa số nguyên từ 14 tới 50, gồm nhiều nhất nửa số nguyên từ 3 tới 13, và cóthể cả số 0, do đó A có tổng cộng 937 + 18 + 5 + 1 = 997 số nguyên, trái vớigiả thiết A gồm hơn 1000 số nguyên từ tập {0, 1, 2, ..., 1997}.

.4.22. Xác định số tự nhiên n thỏa mãn những điều kiện sau:

a, Khai triển thập phân của n gồm 1000 số

b, Tất cả các số trong khai triển là số lẻ.

c, Hai phần tử bất kỳ liền nhau trong khai triển của n hơn kém nhau 2 đơn vị

Lời giải: Đặt an, bn, cn, dn, en là số trong khai triển của n, đó là những số lẻ vàhai số liên tiếp khác nhau 2 đơn vị do đó tận cùng theo thứ tự là 1, 3, 5, 7, 9

do đó

0 1 0 0 0

1 0 1 0 0

0 1 0 1 0

0 0 1 0 1

0 0 0 0 1

an

bn

cn

dn

en

=

an+1

bn+1

cn+1

dn+1

en+1

vnmath.com

Page 221: OLYMPIC TOÁN NĂM 2000 52 ĐỀ THI VÀ LỜI …»i nói đầu Để thử gói lệnh lamdethi.sty tôi biên soạn một số đề toán thi Olympic, mà các học trò của

Đề thi olympic Ireland 21

Gọi A là ma trận vuông trong biểu thức đó. Ta tìm giá trị riêng của của A,giả sử Av = λv với v = (v1, v2, v3, v4, v5). Do đó

v2 = λv1

v3 = λv2 − v1 =(λ2 − 1

)v1

v4 = λv3 − v2 =(λ3 − 2λ

)v1

v5 = λv4 − v3 =(λ4 − 3λ2 + 1

)v1

và v4 = λv5, do đó λ5 − 3λ3 + λ = λ3 − 2λ. Giải pt này ta được λ = 0, λ =

±1, λ = ±√

3 tương ứng ta có các vectơ riêng x1, x2, x3, x4, x5 là

(1, 0,−1, 0, 1) , (1, 1, 0,−1,−1) , (1,−1, 0, 1,−1) ,(

1,±√

3, 2,±√

3, 1)

(1, 1, 1, 1, 1) =1

3x1

2 +√

3

6x4 +

2 −√

3

6x5

Vì vậy

(a1000 ,b1000, c1000, d1000, e1000) =

= 39992

2 +√

3

6

(1,√

3, 2,√

3, 1)− 2 −

√3

6

(1,−

√3, 2,−

√3, 1)

=(

3499, 2.3499, 2.3499, 2.3499, 3499)

Vì thế kết quả của bài toán là 8.3499.

vnmath.com

Page 222: OLYMPIC TOÁN NĂM 2000 52 ĐỀ THI VÀ LỜI …»i nói đầu Để thử gói lệnh lamdethi.sty tôi biên soạn một số đề toán thi Olympic, mà các học trò của

Chương 5

Đề thi olympic Italy

.5.23. Cho một dải giấy hình chữ nhật có chiều rộng 3 cm, chiều dài vô tận. Gấp

dải giấy lại chỉ bằng một nếp gấp.Hỏi phần dải giấy bị phủ bởi việc gấp đó có thể có

diện tích nhỏ nhất là bao nhiêu ?

Lời giải: Phần dải giấy bị phủ là một tam giác.Kí hiệu ba đỉnh của tamgiác là A, B, C trong đó AB là nếp gấp và góc BAC nhọn. Hạ các đường caoAA′, BB′, CC′ của ∆ABC. Chú ý BAB′ = BAC. Đặt BAB′ = x. Ta xét haitrường hợp :Trường hợp 1 : Giả sử 0 < x ≤ π

4 , thế thì C nằm giữa A’ và B, ABA′ = x vàACA′ = 2x. Ta có:

S∆ABC = S∆ABA′ − S∆ACA′

=1

2.3.(3 cot x)− 1

2.3.(3 cot 2x)

=9

2(cot x − cot 2x)

=9

2csc 2x

Giá trị nhỏ nhất của csc2x chỉ có thể là 1 khi x = π4 . Do đó diện tích nhỏ

nhất là 92

.5.24. Cho f là một hàm giá trị thực sao cho với mỗi số thực x ta có

(a) f (10 + x) = f (10 − x)

vnmath.com

Page 223: OLYMPIC TOÁN NĂM 2000 52 ĐỀ THI VÀ LỜI …»i nói đầu Để thử gói lệnh lamdethi.sty tôi biên soạn một số đề toán thi Olympic, mà các học trò của

Đề thi olympic Italy 23

(b) f (20 + x) = − f (20 − x)

Chứng minh rằng f là hàm lẻ ( f (−x) = − f (x)) và tuần hoàn (tức là tồn tại T > 0

sao cho f (x + T) = f (x)).

Lời giải: Chọn x = n − 10, từ (a) ta có f (n) = f (20 − n)

Chọn x = n từ (b) ta có f (20 − n) = − f (20 + n)

Từ đó suy ra

f (n) = − f (n + 20)

f (n + 20) = − f (n + 40). Do đó f (n + 40) = f (n)). Vì vậy

f tuần hoàn và có chu kì là T = 40

Ta cũng có − f (n) = f (20 + n) = − f (20 − n) = f (n). Vậy f là hàm lẻ.

.5.25. Góc phần tư thứ nhất của mặt phẳng toạ độ được chia thành các hình vuông

dơn vị bởi các đường ô lưới.Có thể tô màu các hình vuông đơn vị thoả mãn các điều

kiện sau?

(a)Với mỗi hình vuông lớn có một đỉnh dặt tại gốc, và các cạnh song song với với

các trục toạ độ thì chứa nhiều hình vuông đơn vị được tô hơn các hình vuông dơn

vị không được tô.

(b)Mỗi đường song song với đường phân giác của góc phần tư thứ nhất chỉ đi qua

các hình vuông được tô màu.

Lời giải: Có thể thực hiện như sau: Trên mỗi đường y = x + D, ta tô màu|D| + 1 hình vuông gần trục nhất.Xét đường y = x + D với D ≥ 0. Dọc đường này hình vuông đầu tiên đượctô nằm trên cột thứ 1 và dòng thứ D + 1. Hình vuông cuối cùng được tômàu nằm trên cột thứ D + 1 và dòng thứ 2D + 1. Do các hình vuông nằmbên tay phải của hình vuông này ( phía trên đường y = x) là phần màcác đường chéo có các hình vuông được tô ít hơn, thế thì không một hìnhvuông nào trong số đó được tô cả. Nếu ta kí hiệu (i, j) là hình vuông ở dòngthứ i và cột thứ j thì hình vuông (i, j) được tô khi và chỉ khi:

j ≥ i, i ≤ D + 1 ⇒ i ≤ (j − i + 1) ⇒ i ≤ j + 1

2

hoặc j ≤ i+12 . Tổng các hình vuông được tô trong n.n hình vuông là

Cn = 2(n

∑k=1

[k + 1

2

])− 1

vnmath.com

Page 224: OLYMPIC TOÁN NĂM 2000 52 ĐỀ THI VÀ LỜI …»i nói đầu Để thử gói lệnh lamdethi.sty tôi biên soạn một số đề toán thi Olympic, mà các học trò của

24 Nguyễn Hữu Điển, ĐHKHTN Hà Nội

Nếu n chẵn, ta có Cn = 12 n2 + n − 1

Nếu n lẻ , thì Cn = 12n2 + n − 1

2

Do đó Cn ≥ 12 n2 với mọi n sẽ thoả mãn điều kiện đề bài.

.5.26. Cho tứ diện ABCD.Gọi a là độ dài của AB và S là diện tích hình chiếu của

tứ diện lên mặt phẳng vuông góc với AB.Hãy xác định thể tích của tứ diện ABCD

theo S và a.

Lời giải: Ta kí hiệu các toạ độ A = (0, 0, 0), B = (0, 0, n), C = (0, b, c), D =

(i, j, k). Khi đó mặt phẳng z = 0 vuông góc với AB, và hình chiếu của tứdiện lên mặt phẳng này là một tam giác có đỉnh A′ = B′ = (0, 0, 0), C′ =

(0, b, 0), D′ = (i, j, 0). Tam giác này có đáy là b và chiều cao tương ứng là i.Vậy S = b i

2 và a = AB = n.

Để tìm thể tích , ta xét tứ diện như là hình chóp có đáy là tam giác ABC.Khiđó mặt phẳng đáy có phương trình x = 0 và chiều cao hạ từ d có độ dài là i.Diện tích của tam giác ABC là b n

2 . Vậy thể tích của tứ diện là : b in6 = S a

3 .

.5.27. Cho X là tập hợp tất cả các sôs tự nhiên mà các chữ số của nó đôi một khác

nhau. Với mỗi n ∈ X, đặt An là tập hợp tất cả các số mà các chữ số của nó là một

hoán vị của các chữ số của n.

Ví dụ n = 47 thì An = {47, 74} , n = 125 thì An =

{125, 152, 251, 215, 521, 512}.

Gọi dn là ƯCLN của tất cả các số trong An. Tìm giá trị lớn nhất có thể của dn.

Lời giải: Giả sử n có nhiều hơn hoặc bằng 3 chữ số. Gọi AB là hai chữ sốcuối thế thì số có hai chữ số cuối theo thứ tự là BA cũng thuộc An. Vậy hiệucủa hai số trên là : |BA − AB| = |10B − A − 10A + b| = 9 |A − B| ≤ 81.Nếu dn là ước của hai số trên thì dn là ước của 81 vậy dn ≤ 81

Nếu n gồm hai chữ số , cả hai chữ số đều khác 0 thì lập luận như trên.Nếu n gồm hai chữ số mà một trong hai chữ số là 0 thì An chỉ chứa n.( Vdụ n = 90 thì An = {09, 90} = {90}) trong trường hợp này giá trị lớnnhất của dn = 90;nếu n = 10 thì An = {10}. Suy ra dn = 90 > 81

Vậy giá trị lớn nhất có thể của dn là 90.

vnmath.com

Page 225: OLYMPIC TOÁN NĂM 2000 52 ĐỀ THI VÀ LỜI …»i nói đầu Để thử gói lệnh lamdethi.sty tôi biên soạn một số đề toán thi Olympic, mà các học trò của

Chương 6

Đề thi olympic Japan

.6.28. Chứng minh rằng bất kỳ 9 điểm bất kỳ nằm trong một đường tròn đường

kính 5, tồn tại hai điểm có khoảng cách nhỏ hơn 2.

Lời giải:

O

Chia hình tròn thành 9 phần: một phần là hình tròn bán kính 1 đồng tâmvới đường tròn đã cho và 8 hình quạt bằng nhau là giao của phần còn lạivới đường tròn. Sau đó kiểm tra được rằnng hai điểm trong mỗi phần cókhoảng cách nhiều nhất là 2.

.6.29. Cho a, b, c là các số dương. Chứng minh bất đẳng thức sau:

(b + c − a)2

(b + c)2 + a2+

(c + a − b)2

(c + a)2 + b2+

(a + b − c)2

(a + b)2 + c2≤ 3

5

vnmath.com

Page 226: OLYMPIC TOÁN NĂM 2000 52 ĐỀ THI VÀ LỜI …»i nói đầu Để thử gói lệnh lamdethi.sty tôi biên soạn một số đề toán thi Olympic, mà các học trò của

26 Nguyễn Hữu Điển, ĐHKHTN Hà Nội

và xác định khi nào dấu bằng xảy ra.

Lời giải: Đầu tiên, rút gọn

2ab + 2ac

a2 + b2 + c2 + 2bc+

2ba + 2bc

a2 + b2 + c2 + 2ac+

2ca + 2cb

a2 + b2 + c2 + 2ab≤ 12

5

Đặt s = a2 + b2 + c2. Sau đó quy đồng khử mẫu số ta có

5s2(ab + bc + ca) + 10s(a2bc + ab2c + abc2) + 20(a3b2c + ab3c2 + a2bc3)

≤ 6s3 + 6s2(ab + bc + ca) + 12s(a2bc + ab2c + abc2) + 48a2b2c2

Đơn giản hai vế

6s3 + s2(ab + bc + ca) + 2s(a2bc + ab2c + abc2) + 48a2b2c2

≥ 10s(a2bc + ab2c + abc2) + 20(a3b2c + ab3c2 + a2bc3).

Thay s và khai triển biểu thức của s

(3a6 + 2a5b − 2a4b2 + 3a4bc + 2a3b3 − 12a3b2c + 12a2b2c2)+

+(3b6 + 2b5c − 2b4c2 + 3b4ca + 2b3c3 − 12b3c2a + 12b2c2a2)+

+(3c6 + 2c5a − 2c4a2 + 3c4ab + 2c3a3 − 12c3a2b + 12b2a2b2 ≥ 0. (∗)

Áp dụng bất đẳng thức Schur, ta có:

(4a4bc− 8a3b2c + 4a2b2c2)+ (4b4ca− 8b3c2a + 4b2c2a2)+ (4c4ab− 8c3a2b + 4c2a2b2) ≥ 0,

Để chứng minh bất đẳng thức (*), ta cần chứng minh:

(3a6 + 2a5b − 2a4b2 − a4bc + 2a3b3 − 4a3b2c)+

+(3b6 + 2b5c − 2b4c2 − b4ca + 2b3c3 − 4b3c2a)+

+(3c6 + 2c5a − 2c4a2 − c4ab + 2c3a3 − 4c3a2b) ≥ 0.

vnmath.com

Page 227: OLYMPIC TOÁN NĂM 2000 52 ĐỀ THI VÀ LỜI …»i nói đầu Để thử gói lệnh lamdethi.sty tôi biên soạn một số đề toán thi Olympic, mà các học trò của

Đề thi olympic Japan 27

Ta có thể chứng minh bất đẳng thức trên bằng bốn biểu thức không âm bởibất đẳng thức AM-GM:

0 ≤ 2 ∑a,b,c

2a6 + b6

3− a4b2

0 ≤ 2 ∑a,b,c

4a6 + b6 + c6

3− a4bc

0 ≤ 2 ∑a,b,c

2a3b3 + c3b3

3− a3b2c

0 ≤ 2 ∑a,b,c

2a5b + a5c + ab5 + ac5

6− a3b2c.

Dấu bằng xảy ra khai và chỉ khi a = b = c.

.6.30. Cho G là đồ thị 9 đỉnh. Giả sử rằng với bất kỳ 5 đỉnh của G đều tồn tại ít

nhất hai cạnh có điểm đầu và điểm cuối thuộc vào 5 điểm đó. Hỏi rằng số cạnh nhỏ

nhất có thể có của G là bao nhiêu?

Lời giải:

Số cạnh nhỏ nhất là 9, đạt được bởi 3 chu trình rời nhau.Thật vậy, gọi an là số cạnh nhỏ nhất của đồ thị n đỉnh thỏa mãn điều kiện

vnmath.com

Page 228: OLYMPIC TOÁN NĂM 2000 52 ĐỀ THI VÀ LỜI …»i nói đầu Để thử gói lệnh lamdethi.sty tôi biên soạn một số đề toán thi Olympic, mà các học trò của

28 Nguyễn Hữu Điển, ĐHKHTN Hà Nội

bài ra. Ta sẽ chứng minh rằng an+1 ≥ n + 1

n − 1an. Với mỗi đồ thị n đỉnh gọi li là

số cạnh của đồ thị nhận được bằng cách bỏ đi đỉnh thứ i và tất cả các cạnhgắn với đỉnh thứ i. (Ta có li ≥ an, mặt khác l1 + l2 + ... + ln+1 = (n − 1)an+1.Vì mỗi cạnh được đếm cho mọi cạnh khác trừ hai điểm đầu cuối của nó).Từ đó, a5 = 2, ta nhận được a6 ≥ 3, a7 ≥ 5, a8 ≥ 7, a9 ≥ 9.

.6.31. Cho A, B, C, D là bốn điểm không đồng phẳng. Giả sử rằng AX + BX +

CX + DX đạt giá trị nhỏ nhất tại X = X0 khác A, B, C, D. Chứng minh rằng

AX0B = CX0D.

Lời giải:

B

A C

D

X = X0

Giả sử A, B, C, D và P có các tọa độ (x1, y1, z1), ..., (x4, y4, z4) và (x, y, z). Tacó hàm số

f (P) = ∑i

√(x − xi)2 + (y − yi)2 + (z − zi)2

. Để đạt giá trị nhỏ nhất, ba đạo hàm riêng của nó phải bằng không, nhưng

có ba hàm tọa độ của ua + ub + uc + ud, ở đây ua là véctơ đơn vịP − A

||P − A||và tương tự. Do tổng này bằng không, và ua.ub = uc.ud tại điểm P = X0, từđó ta có điều phải chứng minh.

vnmath.com

Page 229: OLYMPIC TOÁN NĂM 2000 52 ĐỀ THI VÀ LỜI …»i nói đầu Để thử gói lệnh lamdethi.sty tôi biên soạn một số đề toán thi Olympic, mà các học trò của

Đề thi olympic Japan 29

.6.32. Cho n là một số nguyên dương. CMR: có thể gán cho mỗi đỉnh của một đa

giác 2n đỉnh, một trong các chữ cái A hoặc B sao cho các dãy n chữ cái nhận được

bằng cách đọc bắt đầu từ một đỉnh nào đó theo ngựơc chiều kim đồng hồ, là luôn

khác nhau.

Lời giải: Xét một đồ thị có hướng như sau: mỗi đỉnh của đồ thị là một dãycó độ dài n − 1, hai đỉnh là kề nhau nếu n − 2 chữ cái cuối của đỉnh nàytrùng với n − 2 chữ cái đầu của đỉnh kia. (Chú ý là: đây là một đồ thị có haivòng). Mỗi đỉnh của đồ thị có một cạnh đi vào và một cạnh đi ra, vì thế tồntại một đường đi có hướng đi qua mỗi cạnh đúng một lần. Ta có thể có mộtchu trình cần thiết bằng cách bắt đầu từ một đỉnh bất kì, viết ra dãy chữtương đương với nó, sau đó viết thêm vào chữ cuối cùng của mỗi dãy, ta sẽgặp hết các dãy trên đường đi đó.

vnmath.com

Page 230: OLYMPIC TOÁN NĂM 2000 52 ĐỀ THI VÀ LỜI …»i nói đầu Để thử gói lệnh lamdethi.sty tôi biên soạn một số đề toán thi Olympic, mà các học trò của

Chương 7

Đề thi olympic Korean

.7.33. Chứng minh rằng với bốn điểm bất kì trong một đường tròn đơn vị thì tồn

tại hai điểm mà khoảng cách giữa chúng không vượt quá√

2.

Lời giải: - Trường hợp 1:Nếu một trong 4 điểm là tâm đường tròn thì ta có ngay điều phải chứngminh.- Trường hợp 2:Nếu không có điểm nào trong bốn điểm trùng tâm đường tròn thì ta ký hiệucác điểm đó lần lượt là P1, P2, P3, P4

Ta có tứ giác lồi Q1Q2Q3Q4, với {Qi} là giao điểm của {OPi} với đườngtròn.Khi đó: ∠P1OP2 + ∠P2OP3 + ∠P3OP4 + ∠P4OP1 ≤ 2π

Ta có ∠PiOPi+1 ≤ π2

Đoạn PiPi+1 nằm trong tam giác OQiQi+1 vì vậy ta có:PiPi+1 ≤ max(OQi, QiQi+1, Qi+1O) = max(1, 2 sin ∠OQiQi+1) ≤

√2

.7.34. Cho hàm số: f : N → N thỏa mãn hai điều kiện:

a. n ∈ N, f(n + f(n)) = f(n)

b. n0 ∈ N, f(n0 ) = 1

Chứng minh rằng: f(n) = 1,∀ n ∈ N

Lời giải: Trước hết chú ý rằng nếu n ∈ N f(n) = 1, sau đóf(n + 1) = f(n + f(n)) = f(n) = 1

vnmath.com

Page 231: OLYMPIC TOÁN NĂM 2000 52 ĐỀ THI VÀ LỜI …»i nói đầu Để thử gói lệnh lamdethi.sty tôi biên soạn một số đề toán thi Olympic, mà các học trò của

Đề thi olympic Korean 31

Cho f(n0 ) = 1,f(n) = 1, ∀ n ≥ n0

Đặt S = {n ∈ N | f (n) 6= 1}Nếu S 6= ∅, gọi N = maxS. Ta có f(N + f(N)) = f(N) 6= 1⇒ N + f(N) ∈ S, N + f(N) > NĐiều này mâu thuấn với N = max S

Vậy S = ∅ và f(n) = 1,∀ n ∈ N

.7.35. Biểu thị tổngn

∑k=1

⌊√k⌋

theo các số hạng của n và a =⌊√

n⌋

Lời giải: Ta sẽ chứng minh kết quả là:

(n + 1)− a(a + 1)(2a + 1)

6

Thật vậy, ta quy ước việc sử dụng dấu [ ] như sau:Gọi P là một mệnh đề, [P] có giá trị bằng 1 nếu P đúng, ngược lại P có giá trị0 nếu P sai.Chú ý rằng bkc là số nguyên dương và bình phương của nó gần k nhất.

Vì vậy:⌊√

k⌋

=a

∑j=1

[j2 ≤ k

]

Do đó:n

∑k=1

⌊√k⌋

=n

∑k=1

a

∑j=1

[j2 ≤ k

]=

a

∑j=1

n

∑k=1

[j2 ≤ k

]

Bây giờ tổng:n

∑k=1

[j2 ≤ k

]đếm được

với k ∈ {1, ..., n} mà k ≥ j2, j ≤ a, j2 ≤ n thì số đó là n + 1 − j2

Vì vậyn

∑k=1

⌊√k⌋

=a

∑j=1

(n + 1 − j2) = (n + 1)− a(a+1)(2a+1)6

.7.36. Cho C là một đường tròn tiếp xúc với các cạnh của góc xOy và C1 cũng kà

một đường tròn tiếp xúc với các cạnh góc đó và đi qua tâm của C. Gọi A là giao

điểm thứ hai của đường kính của C1 qua tâm của C với C1 và gọi B là giao của

đường kính đó với C. Chứng minh rằng đường tròn tâm A đi qua B tiếp xúc với các

cạnh của góc xOy

Lời giải: Gọi T và T1 là tâm đường tròn C và C1 ; r, r1 lần lượt là các bán kínhcủa hai đường tròn đó. Vẽ đường vuông góc TT’ , T1 T′

1 và AA’ tới Ox.

vnmath.com

Page 232: OLYMPIC TOÁN NĂM 2000 52 ĐỀ THI VÀ LỜI …»i nói đầu Để thử gói lệnh lamdethi.sty tôi biên soạn một số đề toán thi Olympic, mà các học trò của

32 Nguyễn Hữu Điển, ĐHKHTN Hà Nội

Ta có: TT’ = r, T1T′1 = r1. Vì T1 là trung điểm của AT nên

T1T′1 =

AA′ + TT′

2⇒ AA′ = 2T1T′

1 − TT′ = 2r1 − r

Vì vậy AB = AT − BT = 2r1 − r. Suy ra đpcm.

.7.37. Tìm tất cả các số nguyên x, y, z thỏa mãn điều kiện x2+y2+z2 − 2xyz = 0

Lời giải: Ta chứng minh nghiệm duy nhất là x = y = z = 0.- Trước hết ta có x, y, z không thể là các số lẻ vì khi đó tổng x2+y2+z2 − 2xyz

là một số lẻ nên khác 0, do đó xyz...2

Mặt khác: x2+y2+z2 = 2xyz chia hết cho 4 khi tất cả các bình phương chiahết cho 4 hoặc chia cho 4 dư 1, x, y, z phải là các số chẵn, do đó ta lại viếtx = 2x1, y = 2y1, z = 2z1

Ta có: 4x21 + 4y2

1 + 4z21 = 16x1y1z1

Hay x21 + y2

1 + z21 = 4x1y1z1

Vì vế phải chia hết cho 4 nên nên ta lại viết được

x1 = 2x2, y1 = 2y2, z1 = 2z2

Thay vào ta có: 4x22 + 4y2

2 + 4z22 = 32x2y2z2

Hay x22 + y2

2 + z22 = 8x2y2z2

Tiếp tục quá trình đó ta được: ∀n ≥ 1, x2n + y2

n + z2n = 2n+1xnynzn

Tương tự các số xn ,yn ,zn là chẵn nên ta có thể viếtxn = 2xn+1, yn = 2yn+1, zn = 2zn+1 thỏa mãn: x2

n+1 + y2n+1 + z2

n+1 =

2n+2xn+1yn+1zn+1

Lặp lại quá trình này ta có dãy các số nguyên (x1, x2, ..., xn) thỏa mãnxi = 2xi+1

Lại có x = 2nxn ⇒ x...2n, ∀n ≥ 1 ⇒ x = 0.

Chứng minh tương tự ta có y = z = 0.

Ghi chú: Nếu thế x = yz - w ta được bài toán USAMO 76/3

.7.38. Tìm số nguyên k nhỏ nhất để tồn tại hai dãy {ai}, {bi}, thỏa mãn:

a.ai , bi ∈{

1, 1996, 19962, ...}

, i = 1...k

b.ai 6= bi, i = 1...k

vnmath.com

Page 233: OLYMPIC TOÁN NĂM 2000 52 ĐỀ THI VÀ LỜI …»i nói đầu Để thử gói lệnh lamdethi.sty tôi biên soạn một số đề toán thi Olympic, mà các học trò của

Đề thi olympic Korean 33

c.ai ≤ ai+1, bi ≤ bi+1

d.k

∑i=1

ai =k

∑i=1

bi

Lời giải: Ta chứng tỏ số k phải tìm là số 1997.Giả sử đã có hai dãy {ai}, {bi} thỏa mãn các điều kiện của bài toán, vớik ≤ 1996

Từ điều kiện b. cho ta a1 6= b1

Vì vậy không mất tính tổng quát, giải sử a1 < b1

Từ điều kiện a. ta có 0 ≤ m < n : a1 = 1996m, b1 = 1996n

Từ bi ≥ b1∀i = 1, 2, ..., (ĐK c.) và mỗi giá trị bi một lũy thừa của 1996,k

∑i=1

bi

chia hết cho 1996n

Do đó từ điều kiện d,k

∑i=1

ai...1996n

Ta kí hiệu t là một số jn mà aj = 1996m.

Ta có t.1996m =n

∑i=1

ai = 0(mod1996m+1)

vì thế t ≥ 1996 và t ≤ k ≤ 1996

Vì vậy ta có t = k = 1996.

Do đó 1996m+1 =k

∑i=1

ai =k

∑i=1

bi ≥k

∑i=1

b1 = 1996.1996n = 1996n+1

Điều này mâu thuẫn với m < n. Vì vậy k ≥ 1997

Với k = 1997 ta có ví dụ:a1 = a2 = ... = a1996 = 1, a1997 = 19962

b1 = b2 = ... = b1997 = 1996

.7.39. Đặt An là tập tất cả các số thực được hình thành từ tổng:

1 + α1√2+ α2

(√

2)2+ ... + αn

(√

2)n

Với αj = {−1, 1} , ∀j

Tìm số phần tử của Anvà tổng của tất cả các tích của hai phần tử phân biệt của An

Lời giải: Trước hết ta chứng minh bổ đề:∀n ≥ 1,

{β12 +

β24 + ... +

βn

2n |βi ∈ {−1, 1}}

={

j2n , jl, |j| < 2n

}

Thật vậyTa chứng minh bằng phương pháp quy nạp.Khi n = 1 thì cả hai tập hợp đều là

{− 1

2 , 12

}, n ≥ 1, β j ∈ {−1, 1}

vnmath.com

Page 234: OLYMPIC TOÁN NĂM 2000 52 ĐỀ THI VÀ LỜI …»i nói đầu Để thử gói lệnh lamdethi.sty tôi biên soạn một số đề toán thi Olympic, mà các học trò của

34 Nguyễn Hữu Điển, ĐHKHTN Hà Nội

Đặt j = 2n−1.β1 + ... + 20.βn, với j lẻ và β12 +

β24 + ... +

βn

2n =j

2n

Từ∣∣∣ β1

2 + β24 + ... + βn

2n

∣∣∣ ≤∣∣∣ β1

2

∣∣∣+∣∣∣ β2

4

∣∣∣+ ... +∣∣∣ βn

2n

∣∣∣ = 12 + 1

4 + ... + 12n < 1

ta có |j| < 2n

Khi đó tập hợp vế trái chứa trong tập hợp vế phải.Xét với j lẻ và |j| < 2n . Khi đó hoặc j−1

2 , hoặc j+12 là số lẻ vì chúng là hai số

nguyên liên tiếp.Đặtj0 là một trong hai số lẻ đó, khi đó |j0| ≤ |j|+1

2 ≤ 2n+1

Do |j| ≤ 2n , vì j0 là số lẻ, |j0| ≤ 2n−1

Vì vậy theo giả thiết quy nạp tồn tại β1 ,...,βn−1 sao choβ12 +

β24 + ... +

βn−1

2n−1 =j0

2n−1

Đặt βn = j − 2j0 ∈ {−1, 1}Khi đó β1

2 +β24 + ... +

βn−1

2n−1 +βn

2n =j0

2n−1 +j−2j0

2n =j

2n

Vậy bổ đề được chứng minh.Từ bổ đề ta có: An =

{1 +

j

2[n/2] + k√

22[n/2]

∣∣∣j, k, |j| ≤ 2bn/2c, |k| ≤ 2dn/2e}

j và k lẻ. Vì vậy An chứa phần tử 2b n2 cd n

2 e = 2n

Để tính tổng tất cả các phần tử khác nhau của An ta sử dụng công thức:

∑a,b∈An,a<b

ab = 12

(

∑a∈An

a

)2

− ∑a∈An

a2

Bây giờ ta có thể ghép đôi phần tử 1 + j

2[n/2] + k√

22[n/2] v1 − j

2[n/2] − k√

22[n/2]

Vì thế trung bình của các phần tử của An là 1; từ đó ∑a∈An

a = |An| = 2n

Bây giờ nếu X, Y là hai tập các số thực hữu hạn với ∑x∈X

x = ∑y∈Y

y = 0, ta có

∑x∈X

∑y∈Y

(1 + x + y)2 = |X| |Y| + |Y| ∑x∈X

x2 + |X| ∑y∈Y

y2

Từ ba số hạng khác không theo giả thiết ta có tổng:

∑jl?,|j|<2m

j2 = 132m((2m)2 − 1)

dễ dàng chứng minh được bằng phép quy nạp theo m. Vì thế

∑a∈An

a2 = ∑jle

|j|≤2bn/2c

∑kle|k|≤2dn/2e

(1 + j

2[n/2] + k√

22[n/2]

)2

∑a∈An

a2 = ∑jle

|j|≤2bn/2c

∑kle|k|≤2dn/2e

(1 +

j

2[n/2] + k√

22[n/2]

)

vnmath.com

Page 235: OLYMPIC TOÁN NĂM 2000 52 ĐỀ THI VÀ LỜI …»i nói đầu Để thử gói lệnh lamdethi.sty tôi biên soạn một số đề toán thi Olympic, mà các học trò của

Đề thi olympic Korean 35

∑a∈An

a2 = 2n + 13

(2dn/2e 2bn/2c(22dn/2e−1)

22bn/2c + 2bn/2c 2dn/2e(22bn/2c−1)

2dn/2e−1

)

∑a∈An

a2 = 2n+1 − 1

Chú ý n chẵn và n lẻ khác nhau ở bước cuối cùng, do đó:

∑a,b∈An,a<b

ab = 12

(

∑a∈An

a

)2

− ∑a∈An

a2

= 1

2(22n − 2n+1 + 1)

.7.40. Cho tam giác nhọn ABC với AB khác AC. Gọi V là giao điểm của đường phân

giác góc A với BC và gọi d là chân đường cao hạ từ A tới BC. Gọi E và F là các

giao điểm phân biệt của đường tròn ngoại tiếp tam giác AVD với CA và AB,chứng

minh rằng 3 đường thẳng AD, BE, CF đồng quy.

Lời giải: Từ ∠ADV = π2 và A, D,V, E, F nằm trên đường tròn, ∠BFV =

∠CEV = π2 ta có tam giác BFV và tam giác BDA đồng dạng; tam giác CEV

và CDA cũng đồng dạng.Vì vậy BD

BF = ABVB ; CD

CE = ACVC

Nhưng ABVB = AC

VC ( theo định lý đường phân giác)Vì thế BD

BF = CDCE

Lại có ∠FAV = ∠VAE, AE = AF, từ đó BDDC . CE

EA .AFFB =

BDBFCDCE

= 1 và AD,BE,CF

đồng quy theo định lý Ceva.

.7.41. Một từ là một dãy gồm chữ số 0 hoặc 1. Đặt x và y là 2 từ khác nhau ở đúng

3 vị trí. chứng minh rằng số từ khác nhau giữa x và y không quá 5 vị trí là 38.

Lời giải: Không mất tính tổng quát giả sử x=00000000, y=00000111.Gọi z là một từ khác giữa x và y mà có ít nhất 5 vị trí khác nhau nếu và chỉnếu a + b ≥ 5 với a là chữ số 1 đầu trên trong 5 chữ số khác nhau của z và blà chữ số 1 cuối cùng trong 3 chữ số của z. Hơn nữa ta có bất đẳng thức 2a≥ 7.Điều phải chứng minh là (4,1),(4,2 ) và (5,b) với ∈ {0, 1, 2, 3}Hai điều đầu tiên cho

5

4

3

1

+

3

2

= 30

vnmath.com

Page 236: OLYMPIC TOÁN NĂM 2000 52 ĐỀ THI VÀ LỜI …»i nói đầu Để thử gói lệnh lamdethi.sty tôi biên soạn một số đề toán thi Olympic, mà các học trò của

36 Nguyễn Hữu Điển, ĐHKHTN Hà Nội

từ cho z và các trường hợp khác cho 23 = 8.Vì thế có 38 từ khác nhau giữa x và y mà có 5 vị trí khác nhau.

.7.42. Tìm tất cả các cặp hàm số f,g:R → R thỏa mãn

a. Nếu x<y thì f(x) < f(y)

b. ∀x, y ∈ R, f (xy) = g(y). f (x) + f (y)

Lời giải: Xét cặp (f,g ) cho bởi: f (x = t(1 − g(t))) g(x) =

xm, x ≥ 0

− |x|m , x < 0

với t < 0, m > 0 là nghiệm duy nhất.Đặt x = 0, từ b ta có: f (0) = f (0)g(y) + f (y), suyra f (y) = f (0)(1 − g(y)).

Đặt t = f (0), ta có f (y) f (y) = t(1 − g(y)). Do f là hàm đồng biến nên takhông có t = 0.Thay công thức này cho f ở b cho ta t(1 − g(xy)) = g(y)t(1 − g(x)) + t(1 −g(y)). Từ đó ta có: 11 − g(xy) = g(y)(1 − g(x)) + 1 − g(y) = 1 − g(x)g(y)

hoặc g(xy) = g(x)g(y) với mọi x, y thuộc R.Từ g = 1 − f

t là hàm đơn điệu ngặt, vì vậy g(1) 6= 0

Nhưng lại có g(1) = g2(1), vì thế g(1) = 1.Ta có g là hàm tăng nên f cũng tăng, ta có t < 0..Vì g(x) > 0, ∀x > 0. Gọi h:R → R với h = log.g.exp.Ta cóh(x + y) = log(g(ex+y)) = log(g(ex)g(ey)) = log(g(ex)). log(g(ey))

= h(x) + h(y)

h(0) = log(g(e0)) = 0 và h đơn điệu ngặt.Ta cũng có h(x+y) = h(x) + h(y) suy ra h(nx) = n.h(x) với n ∈ N và h(-x) =-h(x), do đó h(alphax) = α.h(x) với α ∈ Q.Xét dãy hữu tỷ {yi} , {yi}xi < x

yi > x

và sử dụng tính đơn điệu chứng tỏ rằng h(x) = x.h(1), ∀ x ∈ RĐặt m = h(1), từ đó ta có m > 0 và h là hàm tăng vì g(x) = xm ,∀x > 0

Ta có g(−1) < 0, mà (g(−1))2 = g(1) = 1 nên g(-1) = 1.

Từ g(-x) = -g(x) nên g(x) =

xm, x ≥ 0

− |x|m , x < 0

Ta cũng có f(x) = t(1-g(x)). Dễ dàng kiểm tra được rằng cặp (f,g) là nghiệm

vnmath.com

Page 237: OLYMPIC TOÁN NĂM 2000 52 ĐỀ THI VÀ LỜI …»i nói đầu Để thử gói lệnh lamdethi.sty tôi biên soạn một số đề toán thi Olympic, mà các học trò của

Đề thi olympic Korean 37

∀m > 0 > t.

.7.43. Cho a1, a2, ...,an là các số dương và kí hiệu: A = a1+...+ann

G = (a1...an)1n

H = na−1

1 +...+a−1n

a. Nếu n chẵn, chứng minh rằng AH ≤ −1 + 2( A

G )n

b. Nếu n lể, chứng minh rằng: AH ≤ −n−2

n + 2(n−1)n ( A

G )n

Lời giải: Chú ý rằng theo bất đẳng thức Máclôranh ta có:

GH =

a1...an(a−11 +...+a−1

n )n = 1

n

n

∑j=1

a1...anaj

≤(

1n

n

∑j=1

aj

)n−1

= An−1

Vì thế AH ≤ ( A

G )n

Từ A ≥ G, ( AG )n ≥ 1 nên A

H ≤ ( AG )n ≤ −1 + 2( A

G )n

⇒ a.

Với b, AH ≤ ( A

G )n ≤ ( AG )n + n−2

n (( AG )n − 1)

⇒ AH ≤ −n−2

n + 2(n−1)n ( A

G )n

Ta có đpcm.

vnmath.com

Page 238: OLYMPIC TOÁN NĂM 2000 52 ĐỀ THI VÀ LỜI …»i nói đầu Để thử gói lệnh lamdethi.sty tôi biên soạn một số đề toán thi Olympic, mà các học trò của

Chương 8

Đề thi olympic Poland

.8.44. Cho các số nguyên x1, x2, ..., x7 thỏa mãn điều kiện x6 = 144, xn+3 =

xn+2(xn+1 + xn); n = 1, 2, 3, 4.

Tính x7.

Lời giải: Nhân phương trình đã cho với n = 1, 2, 3 và khử nhân tử chung tađược144 = x3(x1 + x2)(x2 + x3)(x3 + x4).[1]

Mặt khác từ phương trình đã cho ta được bất đẳng thức:x4 = x3(x2 + x1) ≥ 2x3

x5 = x4(x3 + x2) ≥ 2x23

144 = x6 ≥ x5(x4 + x3) ≥ 2x23(3x3) ⇒ 144 ≥ 6x3

3 ⇒ x3 = 1, 2.

+ Xét trường hợp 1: x3 = 1

Từ [1], 144 = (x1 + x2)(x2 + 1)(x1 + x2 + 1). Nhân cả 2 vế với 144 liên tiếpcác cặp số nguyên (1, 2), (2, 3), (3, 4), (8, 9). Từ x1 + x2 và x1 + x2 + 1 với hệsố 144 đó các số nguyên liên tiếp và khi đó x1 + x2 ≥ 2. Ta có 3 trường hợp:1a. x1 + x2 = 2 ⇒ 6(x2 + 1) = 144 từ đó suy ra x2 = 23, x1 = −21. Tuynhiên nó không thỏa mãn, vậy xi là số nguyên xác định.1b. x1 + x2 = 3 ⇒ 12(x2 + 1) = 144 từ đó suy ra x2 = 11, x1 = −8. Khôngthỏa mãn.1c. x1 + x2 = 8 ⇒ 72(x2 + 1) = 144 từ đó suy ra x2 = 1, x1 = 7. Thử nghiệmcó thể xảy ra: x4 = 8, x5 = 16, x6 = 144. Như vậy thử (x1, x2, x3) = (7, 1, 1)

vnmath.com

Page 239: OLYMPIC TOÁN NĂM 2000 52 ĐỀ THI VÀ LỜI …»i nói đầu Để thử gói lệnh lamdethi.sty tôi biên soạn một số đề toán thi Olympic, mà các học trò của

Đề thi olympic Poland 39

ta được x7 = 3456

+ Xét trường hợp 2: x3 = 2

144 = 2(x1 + x2)(x2 + 2)(2x1 + 2x2 + 2) ⇒ 36 = (x1 + x2)(x2 + 2)(x1 + x2 +

1). Nhân cả 2 vế với hệ số liên tiếp các cặp số nguyên (1, 2), (2, 3), (3, 4) ta cócác trường hợp sau:2a. x1 + x2 = 2 ⇒ x2 = 4, x1 = −2. (Không thỏa mãn)2b. x1 + x2 = 3 ⇒ x2 = 1, x1 = 2. Ta thử các nghiệm sau: x4 = 6, x5 =

18, x6 = 144. Như vậy với (x1, x2, x3) = (2, 1, 2) thì x7 = 3456.Vậy giá trị x7 cần tìm là 3456.

.8.45. Giải hệ phương trình sau với x, y, z là các số thực:

3(x2 + y2 + z2) = 1

x2y2 + y2z2 + z2x2 = xyz(x + y + z)3

Lời giải: Ta có x, y, z hoặc (x+y+z) không thể bằng 0 và xyz(x + y + z) =x2y2+y2z2+z2x2

(x+y+z)2 ≥ 0

Với 3 số thực a, b, c ta có: (a− b)2 +(a− c)2 +(b − c)2 ≥ 0 hay a2 + b2 + c2 ≥ab + ac + bc, dấu "=" xảy ra nếu và chỉ nếu a=b=c.Vậy 1 = 3(x2 + y2 + z2) ≥ (x + y + z)2 =

x2y2+y2z2+z2x2

xyz(x+y+z)≥ xy2z+x2yz+xyz2

xyz(x+y+z)= 1

Dấu "=" xảy ra khi và chỉ khi x = y = z, từ đó suy ra (x,y,z) là(13 ; 1

3 ; 13

);(−13 ; −1

3 ; −13

).

Đó chính là nghiệm của hệ phương trình.

.8.46. Bài 3: Trong tứ diện ABCD, ở giữa các mặt ABD, ACD, BCD, từ đỉnh D

nhìn các cạnh AB, AC, BC tạo thành các góc tương ứng bằng nhau. Chứng minh

rằng mỗi một tam giác này có diện tích nhỏ hơn hoặc bằng tổng của diện tích của 2

mặt khác.

Lời giải: Cho α ≤ 900 là 1 góc, đầu tiên ta chứng minh α 6= 900. Nếu a,b, c tương ứng là độ dài 3 cạnh BC, CA, AB và ma, mb, mc tương ứng là3 đường trung tuyến ứng với 3 cạnh BC, CA, AB thì diện tích của tamgiác DAB là 1

2 mcc sin α. Chú ý rằng giá trị tuyệt đối của những tích giữavectơ là

(D − A+B

2

)và A-B là mcccosα= 2cotα [DAB] =

∣∣DA2 − DB2∣∣.

Chúng ta có 3 diện tích [ABD] , [ACD] , [BCD] tương ứng tỉ lệ là∣∣DA2 − DB2∣∣ ,∣∣DA2 − DC2

∣∣ ,∣∣DB2 − DC2

∣∣Ta chứng minh α = 900. Đặt x =

vnmath.com

Page 240: OLYMPIC TOÁN NĂM 2000 52 ĐỀ THI VÀ LỜI …»i nói đầu Để thử gói lệnh lamdethi.sty tôi biên soạn một số đề toán thi Olympic, mà các học trò của

40 Nguyễn Hữu Điển, ĐHKHTN Hà Nội

ADB, y =BDC, z =ADC(0 < x, y, z < 180). Chú ý rằng 3 góc x, y, z là 3 góctam diện, chúng ta có: x + y > z, x + z > y, y + z > x và x + y + z ≤ 360.Khi đó diện tích của tam giác ADB là AD.BD.sinx

2 ( tương tự đối với diện tíchcác tam giác BDC và ADC) và AD=BD=CD( khi đó α = 900), chúng ta cầnchứng minh rằng: sin x + sin y > sin z

Ta có sin x + sin y = 2 sinx+y

2 cosx−y

2 , sinz = 2sin z2 cos z

2

Ta cần chứng minh: sinx+y

2 cosx−y

2 > sin z2 cos z

2

Từ x + y + z ≤ 360 ⇒ x+y2 ≤ 180 − z

2 . Để ý rằng 0 <z2 ≤ x+y

2 ≤ 180 − z2 ⇒

sinx+y

2 > sin z2 . Như vậy từ x−y

2 <z2 ⇒ cos

x−y2 > cos z

2 . Khi đó hàm cosingiảm trong [0, 180]

Từ đó suy ra sinx+y

2 cosx−y

2 > sin z2 cos z

2 .(đpcm)

.8.47. Cho dãy a1, a2, ..., xác định bởi :

a1 = 0, an = a[n/2] + (−1)n(n+1)/2, n > 1

Với mọi số nguyên k ≥ 0. Tìm n thỏa mãn:

2k ≤ n ≤ 2k+1van = 0

Lời giải: Đặt Bn có nghĩa là cơ số 2 đại diện của n số. Ta chứng minh bằngquy nạp. Giả sử an là số hạng của 00 hay 11 đoạn trong Bn trừ đi số của01 hay 11 trong Bn. Với k=1,2,3,...,n-1, ak là số hạng của 00 hay 11 trong Bk

trừ đi số 01 hay 10 trong Bk. Đầu tiên xét trường hợp khi n ≡ 0, 3(mod4)

thì Bn giới hạn trong 00 hay 11. Như vậy an bằng một số dương của số 00hay 11 trong tất cả các đoạn nhưng mà chữ số của Bn trừ đi các số 01 hay10 trong tất cả các đoạn nhưng mà số cuối cùng của Bn cho bởi a[ n

2 ]. Như vây:

an = a[ n2 ]

+ 1 = a[ n2 ]

+ (−1)n(n+1)/2

Tương tự với n = 1, 2(mod4) ta có:

a[ n2 ]− 1 = a[ n

2 ]+ (−1)n(n+1)/2

Vậy ta đã chứng minh xong.Như vậy với k số nguyên đã cho chúng ta cần tìm số của n số nguyên thỏamãn 2k ≤ n ≤ 2k+1 và số của 00 và 11 đoạn bằng số của 01 và 10 đoạn.

vnmath.com

Page 241: OLYMPIC TOÁN NĂM 2000 52 ĐỀ THI VÀ LỜI …»i nói đầu Để thử gói lệnh lamdethi.sty tôi biên soạn một số đề toán thi Olympic, mà các học trò của

Đề thi olympic Poland 41

Chú ý rằng Bn có k+1 số. Với mỗi Bn chúng ta xây dựng dãy mới Cn của 0′s

và 1′s giống như nhau.Bắt đầu từ các số của Bn và chữ số gần cuối của Bn, ta cộng thêm vào dãysố Cn giá trị tuyệt đối của số ở khác giữa và chữ số ở bên trái. Ví dụ vớiB11 = 1011 và C11 = 110. Khi đó 00 hay 11 là đoạn trong Bn sinh ra 0 trongdãy Cn và 01 hay 10 trong Bn sinh ra 1 trong Cn. Chúng ta cần tìm thấy sốcủa n số nguyên.Như vậy có điều phải chứng minh.

.8.48. Cho ngũ giác lồi ABCDE với DC=DE và BCD = DEA = π2 . Cho F là

trung điểm đoạn AB, khi đó AFBF = AE

BC . Chứng minh rằng:

FCE = FDE, FEC = BDC.

Lời giải: Gọi P = EA ∩ BC và xét điểm C, D, E, P. Gọi Q, R tương ứng làđiểm thuộc DA, DB với chu vi của tứ giác CDEF.Gọi G = QC ∩ RF ta có:

AG

GB=

sin DCQ

sin ERD

QC

RG

sin RBG

sin GAQ=

CD

DE

sin QRG

sin GQR

sin DBA

sin BAD

=sin ADE

sin CDB

AD

BD=

AE

BC

Suy ra ∆ADF = ∆BDC.

.8.49. Xét n điểm(n ≥ 2) trên đường tròn. Chứng minh rằng số lớn nhất n2

3 của

cung tròn với điểm cuối trong số n điểm có chiều dài lớn hơn hoặc bằng√

2.

Lời giải: Xây dựng đồ thị cho bởi các đỉnh bởi mọi cặp điểm có khoảng cáchlớn hơn hoặc bằng

√2

Chúng ta sẽ chứng minh rằng không tồn tại K4.Giả sử tồn tại K4, gọi các đỉnh theo thứ tự là ABCD.Cạnh có chiều dài lớn hơn

√2 đối diện cung có độ dài lớn hơn π

2 . Như vậymỗi cung AB, BC, CD, DA nhỏ hơn 2pi và chúng cùng nhỏ hơn 2pi, suy ramâu thuẫn. (đpcm)

vnmath.com

Page 242: OLYMPIC TOÁN NĂM 2000 52 ĐỀ THI VÀ LỜI …»i nói đầu Để thử gói lệnh lamdethi.sty tôi biên soạn một số đề toán thi Olympic, mà các học trò của

Nguyễn Hữu Điển

OLYMPIC TOÁN NĂM 1997-199848 ĐỀ THI VÀ LỜI GIẢI

(Tập 6)

NHÀ XUẤT BẢN GIÁO DỤC

vnmath.com

Page 243: OLYMPIC TOÁN NĂM 2000 52 ĐỀ THI VÀ LỜI …»i nói đầu Để thử gói lệnh lamdethi.sty tôi biên soạn một số đề toán thi Olympic, mà các học trò của

2

vnmath.com

Page 244: OLYMPIC TOÁN NĂM 2000 52 ĐỀ THI VÀ LỜI …»i nói đầu Để thử gói lệnh lamdethi.sty tôi biên soạn một số đề toán thi Olympic, mà các học trò của

Lời nói đầu

Để thử gói lệnh lamdethi.sty tôi biên soạn một số đề toán thi

Olympic, mà các học trò của tôi đã làm bài tập khi học tập LATEX.

Để phụ vụ các bạn ham học toán tôi thu thập và gom lại thành các

sách điện tử, các bạn có thể tham khảo. Mỗi tập tôi sẽ gom khoảng

51 bài với lời giải.

Rất nhiều bài toán dịch không được chuẩn, nhiều điểm không

hoàn toàn chính xác vậy mong bạn đọc tự ngẫm nghĩ và tìm hiểu

lấy. Nhưng đây là nguồn tài liệu tiếng Việt về chủ đề này, tôi đã có

xem qua và người dịch là chuyên về ngành Toán phổ thông. Bạn có

thể tham khảo lại trong [1].

Rất nhiều đoạn vì mới học TeX nên cấu trúc và bố trí còn xấu, tôi

không có thời gian sửa lại, mong các bạn thông cảm.

Hà Nội, ngày 2 tháng 1 năm 2010

Nguyễn Hữu Điển

51GD-05

89/176-05 Mã số: 8I092M5

vnmath.com

Page 245: OLYMPIC TOÁN NĂM 2000 52 ĐỀ THI VÀ LỜI …»i nói đầu Để thử gói lệnh lamdethi.sty tôi biên soạn một số đề toán thi Olympic, mà các học trò của

Mục lục

Lời nói đầu ................................................. 3

Mục lục ..................................................... 4

Chương 1. Đề thi olympic Russian........................... 5

Chương 2. Đề thi olympic Nam Phi .......................... 9

Chương 3. Đề thi olympic Tây Ban Nha.................... 12

Chương 4. Đề thi olympic Đài Loan........................ 16

Chương 5. Đề thi olympic Thổ Nhĩ Kỳ ..................... 23

Chương 6. Đề thi olympic Ukraina ......................... 27

Chương 7. Đề thi olympic Anh.............................. 34

vnmath.com

Page 246: OLYMPIC TOÁN NĂM 2000 52 ĐỀ THI VÀ LỜI …»i nói đầu Để thử gói lệnh lamdethi.sty tôi biên soạn một số đề toán thi Olympic, mà các học trò của

Chương 1

Đề thi olympic Russian

.1.1. Chứng minh rằng các số từ 1 đến 16 có thể viết được trên cùng 1

dòng nhưng không viết được trên 1 đường tròn, sao cho tổng của 2 số

bất kỳ đứng liền nhau là 1 số chính phương.

Lời giải: Nếu các số đó viết trên 1 đường tròn thì đứng cạnh số 16 là

số x, y khi đó 16+1 ≤ 16+x, 16+y ≤ 16+15, suy ra: 16+x = 16+y = 25

mâu thuẫn. Các số đó có thể được sắp xếp trên 1 dòng như sau:

16, 9, 7, 2, 14, 11, 5, 4, 12, 13, 3, 6, 10, 15, 1, 8.

.1.2. Trên cạnh AB và BC của tam giác đều ABC lấy điểm D và K

trên cạnh AC và lấy điểm E và M sao cho DA+AE = KC +CM = AB.

Chứng minh rằng góc giữa DM và KE bằngπ

3.

Lời giải: Ta có: CE = AC − AE = AD.

Và tương tự: CK = AM .

Xét phép quay tâm là tâm của tam giác ABC, góc quay2π

3biến K

thành M , biến E thành D, từ đó suy ra điều phải chứng minh.

.1.3. Một công ty có 50.000 công nhân, với mỗi công nhân tổng số

người cấp trên trực tiếp và cấp dưới trực tiếp của anh ta là 7. Vào thứ

2 mỗi công nhân đưa ra một số chỉ dẫn và gửi bản photo của nó cho

mỗi cấp dưới trực tiếp của anh ta (nếu anh ta có). Mỗi ngày sau đó mỗi

vnmath.com

Page 247: OLYMPIC TOÁN NĂM 2000 52 ĐỀ THI VÀ LỜI …»i nói đầu Để thử gói lệnh lamdethi.sty tôi biên soạn một số đề toán thi Olympic, mà các học trò của

6 Nguyễn Hữu Điển, ĐHKHTN Hà Nội

công nhân giữ tất cả các chỉ dẫn mà anh ta nhận được vào ngày hôm

trước và gửi bản photo của chúng cho tất cả cấp dưới trực tiếp của anh

ta nếu anh ta có hoặc anh ta phải tự thực hiện nếu không có cấp dưới

trực tiếp. Cứ như thế cho đến thứ 6 không còn chỉ dẫn nào đưa ra.

Hay chỉ ra rằng có ít nhất 97 công nhân ko có cấp trên trực tiếp.

Lời giải: Giả sử k là số công nhân ko có cấp trên trực tiếp, vào ngày

thứ 2 số chỉ dẫn được đưa ra nhiều nhất là 7k, vào ngày thứ 3 nhiều

nhất là 6.7k vào ngày thứ 4 nhiều nhất là 36.7k vào ngày thứ 5 mỗi

công nhân nhận được 1 chỉ dẫn ko có cấp dưới trực tiếp, vì vậy mỗi

công nhân có 7 cấp trên trực tiếp, mỗi người đưa ra nhiều nhất là 6

chỉ dẫn và có nhiều nhất là 216.7k/7 công nhân nhận được chỉ dẫn.

Chúng ta có:

50.000 ≤ k + 7k + 42k + 252k + 216k = 518k và k ≥ 97.

.1.4. Các cạnh của tam giác nhọn ABC là các đường chéo của hình

vuông K1, K2, K3. Chứng minh rằng miềm trong của tam giác ABC có

thể được phủ bởi 3 hình vuông.

Lời giải: Gọi I là giao điểm của 3 đường phân giác của tam giác

ABC, vì các góc là nhọn nên IAB. IBA < 45◦ vì vậy tam giác IAB có

thể được phủ bởi hình vuông mà đường chéo của nó là AB và tương

tự đối với tam giác IBC và tam giác ICA.

.1.5. Các số từ 1 tới 37 có thể được viết trên 1 dòng sao cho mỗi số là

ước của tổng tất cả các số đứng trước nó. Nếu số đầu tiên là 37 và số

thứ 2 là 1 thì số thứ 3 là bao nhiêu.

Lời giải: Gọi số cuối cùng là x, x phải là ước của tổng tất cả các số

là 37x19, vì x = 19 và số thứ 3 phải là ước của 38 khác 1 hoặc 19, vậy

số thứ 3 là 2.

.1.6. Tìm các cặp số nguyên tố p, q sao cho p3 − q5 = (p + q)2.

Lời giải: Chỉ có nghiệm duy nhất là (7, 3), đầu tiên giả sử cả p và q

đều không bằng 3. Nếu chúng đồng dư với Module 3, vế trái thì chia

hết cho 3 nhưng vế phải thì không, nếu chúng không cùng đồng dư

vnmath.com

Page 248: OLYMPIC TOÁN NĂM 2000 52 ĐỀ THI VÀ LỜI …»i nói đầu Để thử gói lệnh lamdethi.sty tôi biên soạn một số đề toán thi Olympic, mà các học trò của

Đề thi olympic Russian 7

module 3 thì vế phải chia hết cho 3 nhưng vế trái thì không vì thế

không xảy ra khả năng này. Nếu p = 3 ta có q5 < 27, không có số

nguyên tố nào thỏa mãn. Vì vậy q = 3 và p3 − 243 = (p + 3)2 chỉ có

nghiệm duy nhất là p = 7.

.1.7. (a) a. Ở thành phố Mehico để hạn chế giao thông mỗi xe oto

riêng đều phải đăng ký 2 ngày trong 1 tuần vào 2 ngày đó oto đó

không được lưu thông trong thành phố. Một gia đình cần sử dụng

ít nhất 10 chiếc oto mỗi ngày. Hỏi họ phải có ít nhất bao nhiêu

chiếc oto nếu họ có thể chọn ngày hạn chế cho mỗi chiếc oto.

(b) b. Luật được thay đổi để cấm mỗi oto chỉ 1 ngày trong 1 tuần

nhưng cảnh sát được quyền chọn ngày cấm đó. Một gia đình hối

lộ cảnh sát để gia đình đó được quyền chọn 2 ngày lien tiếp không

bị cấm cho mỗi xe và ngay lập tức cảnh sát cấm xe oto vào 1 trong

những ngày khác. Hỏi gia đình đó cần ít nhất bao nhiêu xe oto

nếu họ sử dụng 10 chiếc mỗi ngày.

Lời giải:(a) Nếu n xe oto được sử dụng, số ngày được sử dụng là 5n. Mà mỗi

gia đình sử dụng ít nhất là 10 xe nên 7x10 ≤ 5n vì thế n ≥ 14.

Trong thực tế 14 xe thỏa mãn yêu cầu của đầu bài toán: 4 xe bị

cấm vào ngày thứ 2 và thứ 3, 4 xe bị cấm vào ngày thứ 4 và thứ

5, 2 xe bị cấm vào ngày thứ 6 và thứ 7, 2 xe bị cấm vào ngày

thứ 7 và chủ nhật, 2 xe bị cấm vào ngày chủ nhật và thứ 6.

(b) 12 xe oto là số xe họ cần, đầu tiên chúng ta chỉ ra rằng n ≤ 11

xe không thỏa mãn. Khi đó có n ngày xe bị cấm, mỗi ngày nhiều

nhất làbn

7xe được lưu thông nhưng n ≤ 11,

bn

7< 10. Đối với

n = 12, gia đình đó cần đưa ra 2 ngày liên tiếp cho mỗi xe trong

những ngày đó xe ko bị cấm lưu thông.

.1.8. Một đa giác đều 1997 đỉnh được chia bởi các đường chéo ko cắt

nhau tạo thành các tam giác. Hãy chỉ ra rằng có ít nhất một tam giác

nhọn.

Lời giải: Đường tròn ngoại tiếp đa giác đều 1997 đỉnh cũng là đường

tròn ngoại tiếp mỗi tam giác. Vì tâm của các đường tròn không nằm

vnmath.com

Page 249: OLYMPIC TOÁN NĂM 2000 52 ĐỀ THI VÀ LỜI …»i nói đầu Để thử gói lệnh lamdethi.sty tôi biên soạn một số đề toán thi Olympic, mà các học trò của

8 Nguyễn Hữu Điển, ĐHKHTN Hà Nội

trên bất kỳ đường chéo nào nên nó phải nằm trong một tam giác, vì

thế tam giác đó phải là tam giác nhọn.

.1.9. Viết các số từ 1 đến 1000 trên bảng, Hai người chơi lần lượt xóa

đi 1 số trong các số đó, cuộc chơi kết thúc khi còn lại 2 số:

Người chơi thứ 1 thắng nếu tổng các số còn lại chia hết cho 3, các

trường hợp còn lại người chơi thứ 2 thắng. Người chơi nào có chiến

thuật chiến thắng.

Lời giải: Người chơi thứ 2 có chiến thuật chiến thắng, nếu người

chơi thứ 1 xóa đi số x, người chơi thứ 2 xóa đi số 1001− x vì thể tổng

của 2 số cuối cùng là 1001.

.1.10.Có 300 quả táo, không có quả nào nặng hơn 3 lần quả khác. Hãy

chỉ ra rằng có thể chia các quả táo này thành 4 nhóm mà không có

nhóm nào có cân nặng hơn11

2lần nhóm khác.

Lời giải: Sắp xếp các quả táo tăng dần theo trọng lượng, và ghép

từng đôi một quả nhẹ nhất với quả nặng nhất, sau đó lại đem 1 quả

nhẹ nhất tiếp theo với 1 quả nặng nhất tiếp theo tiếp tục cho đến

hết. Chú ý rằng không cặp nào nặng hơn 2 lần cặp khác. Nếu a, d và

b, c là 2 nhóm với

a ≤ b ≤ c ≤ d thì a + d ≤ 4a ≤ 2b + 2c

b + c ≤ 3a + d ≤ 2a + 2d.

Bây giờ các cặp nặng nhất và nhẹ nhất tạo thành 4 nhóm, không

cân nặng của nhóm nào gấp 2/3 lần nhóm khác vì

e ≤ f ≤ g ≤ h là các cặp thì e + h ≤ 3e ≤ 3

2(f + g)

f + g ≤ 2e + h ≤ 3

2(e + h).

vnmath.com

Page 250: OLYMPIC TOÁN NĂM 2000 52 ĐỀ THI VÀ LỜI …»i nói đầu Để thử gói lệnh lamdethi.sty tôi biên soạn một số đề toán thi Olympic, mà các học trò của

Chương 2

Đề thi olympic Nam Phi

.2.11.Cho ∆A0B0C0 và một dãy ∆A1B1C1,∆A2B2C2 ... được xây dựng

như sau:

Ak+1, Bk+1, Ck+1 là điểm tiếp xúc của đương tròn ngoại tiếp ∆AkBkCk với

các cạnh BkCk,CkAk,AkBk theo thứ tự.

(a) hãy xác định ∠Ak+1Bk+1Ck+1 từ ∠AkBkCk

(b) chứng minh: limk→∞

∠AkBkCk = 600

Lời giải:(a) Ta có AkBk+1 = AkCk+1(vì đây là 2 tiếp tuyến xuất phát từ

một điểm).Vì vậy ∆Ak+1Bk+1Ck+1 là tam giác cân với ∠AkBk+1Ck+1 =

900 − ∠Ak

2.Tương tự ta có: ∠CkBk+1Ck+1 = 900 − ∠Ck

2. Hơn nữa ∠Bk+1 =

(∠Ak+∠Ck)2

= 900 − ∠Bk

2

(b) Ta có ∠Bk+1 − 600 = 900 − ∠Bk

2− 600 = ∠Bk−600

−2

Vì ∠Bk − 600 = ∠B0−600

(−2)k Hiển nhiên limk→∞

∠Bk = 600

.2.12.Tìm tất cả các số tự nhiên thoả mãn:khi chuyển chữ số đầu tiên

xuống cuối,số mới bằng 3,5 lần số ban đầu.

Lời giải:Các chữ số như thế có dạng sau: 153846153846153846. . . 153846

Hiển nhiên những số thoả mãn giả thiết phải bắt đầu bởi 1 hoặc 2.

Trường hợp 1: Số đó có dạng: 10N + A vớiA < 10N .Vì 72(10N + A) =

vnmath.com

Page 251: OLYMPIC TOÁN NĂM 2000 52 ĐỀ THI VÀ LỜI …»i nói đầu Để thử gói lệnh lamdethi.sty tôi biên soạn một số đề toán thi Olympic, mà các học trò của

10 Nguyễn Hữu Điển, ĐHKHTN Hà Nội

10A + 1 ⇒ A = (7.10N−2)13

Ta có 10N ≡ 1, 3, 4, 9, 10, 12(mod13).Vì thế A sẽ là một số nguyên tố nếu:

N ≡ 5(mod6).Từ đó ta có kết quả như trên.

Trường hợp 2: Số đó có dạng 2.10N + A, A < 10N .Theo chứng minh

trên A = (14.10N−4)13

.Nhưng vì A < 10N ,tức 10N < 4.Vô lý.

.2.13.Tìm tất cả các hàm: f : Z → Z sao cho:

f(m + f(n)) = f(m) + n, ∀m, n ∈ Z

Lời giải:Rõ ràng: f(n) = kn với k = 1, k = −1.Ta sẽ chứng minh đây là kết quả

duy nhất.Cho n = 0 ta có f(m + f(0)) = f(m).Xét 2 trường hợp:

Trường hợp 1: f(0) = 0.cho m = 0 có f(f(n)) = n.Gán f(n) bởi n

ta có: f(m + f(f(n))) = f(m + n) = f(m) + f(n) ⇒ f(n) = nf(1) và

n = f(f(n)) = n(f(1))2 ⇒ f(1) = ±1.Đây là kết quả trên.

Trường hợp 2: f(0) 6= 0. f(n) là hàm tuần hoàn và bị chặn .Đặt f(M) ≥f(n), ∀n.nhưng f(M + f(1)) = f(M) + 1.Mâu thuẫn giải thiết.

.2.14.Cho một đường tròn và một điểm P phía trên đường tròn trong

mặt phẳng toạ độ.Một hạt nhỏ di chuyển dọc theo một đường thẳng từ

P đến điểm Q trên đường tròn dưới ảnh hưởng của trọng lực.Khoảng

cách đi từ P trong thời gian t là:12gt2 sin α với g không đổi và α là góc

giữa PQ với mặt phẳng nằm ngang.Hãy chỉ ra vị trí điểm Q sao cho

thời gian di chuyển từ P đến Q là ít nhất.

Lời giải:Câu hỏi là tìm giá trị nhỏ nhất của PQ

sin αhoặc giá trị lớn nhất của sin α

PQ.

Biểu diễn một phép nghịch đảo điểm P với ảnh là quĩ đạo của chính

nó.Điểm cực đại trên sơ đồ cũ(gọi là Q) sẽ vạch ra một điểm Q′

với giá

trị lớn nhất PQ′

sin α, với độ cao khác nhau giữa P và Q′

.Như vậy P là

điểm phía trên đường tròn„Q′

là điểm phía đáy đường tròn.Để tìm Q

hãy chú ý rằng P,Q,Q′

cùng thuộc một đường thẳng.Do đó cách tìm

như sau:

(a): Tìm điểm phía đáy đường tròn gọi là Q′

(b): Tìm giao của PQ′

với đường tròn ,đó là điểm cần tìm.

vnmath.com

Page 252: OLYMPIC TOÁN NĂM 2000 52 ĐỀ THI VÀ LỜI …»i nói đầu Để thử gói lệnh lamdethi.sty tôi biên soạn một số đề toán thi Olympic, mà các học trò của

Đề thi olympic Nam Phi 11

.2.15.Có 6 điểm được nối với nhau từng đôi một bởi những đoạn màu

đỏ hoặc màu xanh..chứng minh rằng:Có một chu trình 4 cạnh cùng

màu.

Lời giải:Gọi các điểm là A,B,C,D,E,F.Dễ thấy luôn có một chu trình tam giác

cùng màu(Thật vậy:Xét một số đỉnh nào đó,sẽ có 5 cạnh từ các đỉnh

ấy mà ít nhất 3 trong số chúng cùng màu.giả sử đó là màu đỏ và các

cạnh này đi đến A,B,C.Nếu một số cạnh giữa A,B,C là màu đỏ ta có

điều phải chứng minh,nếu không ta cũng có điều phải chứng minh)

Không mất tính tổng quát,gọi các cạnh AB,BC,CA là màu đỏ.Nếu

một trong các đỉnh của cạnh màu đỏ khác chạy đến A,B,C,ta có điều

cần tìm.Nếu 2 trong 3 điểm D,E,F của cạnh màu xanh chạy đến 2

trong số các đỉnh cùng màu A,B,C,ta cũng có điều cần tìm.Trường

hợp duy nhất không xảy ra là nếu một trong các điểm D,E,F của

cạnh màu đỏ tạo bởi những điểm khác A,B,C;không mất tính tổng

quát giả sử AD,BE,CF là màu đỏ.Các cạnh không theo lý thuyết là

DE,EF,FD.Nếu một cạnh trong số chúng màu đỏ ta có dạng hình

tròn(ví dụ nếu DE màu đỏ thì DABE là đỏ)Nếu một trong số chúng

màu xanh thì DCEF màu xanh.Ta đã chứng minh được bài toán.

vnmath.com

Page 253: OLYMPIC TOÁN NĂM 2000 52 ĐỀ THI VÀ LỜI …»i nói đầu Để thử gói lệnh lamdethi.sty tôi biên soạn một số đề toán thi Olympic, mà các học trò của

Chương 3

Đề thi olympic Tây Ban Nha

.3.16.Tính tổng bình phương của 100 số hạng đầu tiên của một cấp số

cộng, với giả thiết tổng 100 số hạng bằng −1 và tổng các số hạng thứ

hai, thứ tư, ..., thứ một trăm bằng 1

Lời giải:Gọi 100 số hạng đầu tiên của cấp số cộng là x1, x2, x3, · · · , x100 và d là

công sai của cấp số cộng theo giả thiết thứ nhất ta có:

x1 + x2 + x3 + · · · + x100 =1

2(x1 + x100) .100 = −1 ⇒ x1 + x100 = − 1

50.

Theo giả thiết thứ 2 ta có

x2 + x4 + · · ·+ x100 =1

2((x1 + d) + x100) .50 = 1 ⇒ x1 + x100 + d =

1

25.

Suy ra d = 350

và x1 + x100 = x1 + (x1 + 99d) = − 150

hay x1 = −14950

. Từ đó

ta tìm được:

x21 + x2

2 + · · ·+ x2100 = 100x2

1 + 2dx1 (1 + · · ·+ 99) +(12 + · · ·+ 992

).

Vậy

x21 + x2

2 + · · ·+ x2100 =

14999

50

.3.17.A là môt tập gồm 16 điểm tạo thành một hình vuông trên mỗi cạnh

có 4 điểm. Tìm số điểm lớn nhất của tập A mà không có 3 điểm nào

trong số các điểm đó tạo thành một tam giác cân

vnmath.com

Page 254: OLYMPIC TOÁN NĂM 2000 52 ĐỀ THI VÀ LỜI …»i nói đầu Để thử gói lệnh lamdethi.sty tôi biên soạn một số đề toán thi Olympic, mà các học trò của

Đề thi olympic Tây Ban Nha 13

Lời giải: Số điểm lớn nhất cần tìm là 6 có được bằng cách lấy các

điểm ở hai cạnh kề nhau nhưng bỏ đi điểm chung của hai cạnh đó.

Đầu tiên giả 4 điểm bên trong không được chọn, những điểm còn lại

tạo thành 3 hình vuông, nên nhiều nhất 2 đường thẳng đứng từ mỗi

hình vuông được chọn. Như vậy chúng ta có thể cho rằng một trong

số các điểm trong được chọn trong sơ đồ sau là điểm O

D A1 A2 A3

C Z1 O Z2

E B1 B2 B2

C D C E

Không có điểm nào cùng tên gọi A, B, C, D, E được chọn, vậy nếu ta

không chọn Z1, Z2 một lần nữa nhiều nhất 6 điểm có thể được chọn.

Nếu chọn Z1 nhưng không chọn Z2 thì A1, A2, B1, B2 cũng không được

chọn, và cả A3 và B3 cũng không được chọn, vì vậy một trong hai

điểm A và B phải bỏ đi, môt lần nữa số điểm lớn nhất là 6. Trường

hợp chọn Z2 nhưng không chọn Z1 tương tự. Cuối cùng nếu Z1 và Z2

được chọn thì cả Ai và Bi đều không được chọn, vì vậy số điểm lớn

nhất là 6.

.3.18.Với mỗi Parabol y = x2 + px + q cắt hai trục tọa độ tai 3 điểm phân

biết, vẽ một đường tròn đi qua 3 điểm đó. Chứng minh rằng tất cả các

đường tròn đó đều đi qua một điểm cố định

Lời giải: Tất cả các đường tròn đều đi qua điểm (0, 1). Giả sử

(0, q) , (r1, 0) , (r2, 0) là 3 điểm mà Parabol đí qua, do đó r1 + r2 = −p.

Giả sử (x − a)2 + (y − b)2 = r2 là đường tròn luôn đi qua 3 điểm trên

do đó a = −p2

1

4p2 + (q − b)2 =

(r − p

2

)2

+ b2 =1

4(r1 − r2)

2 + b2

hay q2 − 2.qb = −q do đó b = q+12

, khi đó điểm đối xứng với điểm (0, q)

qua đường kính nằm ngang là điểm (0, 1)

.3.19.Cho p là số nguyên tố. Tìm tất cả k ∈ Z sao cho√

k2 − pk là số

nguyên dương.

vnmath.com

Page 255: OLYMPIC TOÁN NĂM 2000 52 ĐỀ THI VÀ LỜI …»i nói đầu Để thử gói lệnh lamdethi.sty tôi biên soạn một số đề toán thi Olympic, mà các học trò của

14 Nguyễn Hữu Điển, ĐHKHTN Hà Nội

Lời giải: Giá trị k cần tìm là k = (p±1)2

4với p là số lẻ ( trừ trường hợp

p =2 ). Trước hết xét p = 2, trong trường hợp này ta cần k2 − 2k =

(k − 1)2 − 1 là một số chính phương dương trường hợp này không thể

xảy ra vì chỉ có duy nhất hai số chính phương liên tiếp là 0 và 1

Giả sử p là số lẻ. Đầu tiên ta xét trường hợp k chia hết cho p, hay

k = np, khi đó k2 − pk = p2n (n − 1), n và n − 1 là hai số nguyên tố liên

tiếp. Do đó cả hai không thể là số chính phương.

Giả sử k và p là hai số nguyên tố cùng nhau, khi đó k và k − p cũng

là hai số nguyên tố cùng nhau. Để k2 − pk là số chính phương khi và

chỉ khi k và k − p là các số chính phương, k = m2, k − p = n2. Do đó

p = m2 − n2 = (m + n) (m − n). Suy ra m + n = p, m− n = 1 và k = (p+1)2

4,

hoặc m + n = 1, m − n = p và k = (p−1)2

4

.3.20.Chứng minh rằng trong tất cả các tứ giác lồi có diện tích bằng

1, thì tổng độ dài các cạnh và các đường chéo lớn hơn hoặc bằng

2(2 +

√2)

Lời giải: Thực tế ta cần chỉ ra rằng tổng độ dài các cạnh của tứ giác

lồi lớn hơn hoặc bằng 4 và tổng độ dài các đường chéo của tứ giác lồi

lớn hơn hoặc bằng 2√

2. Đối với trường hợp đường chéo ta sử dụng

công thức tính diện tích A = 12d1d2 sin θ, với θ là góc giữu hai đường

chéo. Từ giả thiết cho diện tích tứ giác bằng 1 suy ra d1d2 ≥ 2, áp

dụng bất đẳng thức AG-GM suy ra d1 + d2 ≥ 2√

2, đẳng thức xảy ra

khi và chỉ khi độ dài hai đường chéo bằng nhau và vuông góc với

nhau.

Đối với trường hợp cạnh ta sử dụng công thức tính diện tích

A = (s − a) (s − b) (s − c) (s − d) − abcd cos2 B + D

2,

với s = a+b+c+d2

, B và D là hai góc đối diện nhau. Từ giả thiết cho

diện tích tứ giác bằng 1 ta suy ra (s − a) (s − b) (s − c) (s − d) ≥ 1, lại

sử dụng bất đẳng thức AG-MG ta lại suy ra

4 ≤ (s − a) + (s − b) + (s − c) + (s − d) = a + b + c + d

, đẳng thức xảy ra khi và chỉ khi a = b = c = d.

vnmath.com

Page 256: OLYMPIC TOÁN NĂM 2000 52 ĐỀ THI VÀ LỜI …»i nói đầu Để thử gói lệnh lamdethi.sty tôi biên soạn một số đề toán thi Olympic, mà các học trò của

Đề thi olympic Tây Ban Nha 15

Từ đó ta suy ra kết luận, để cả hai đẳng thức xảy ra khi và chỉ khi

tứ giác lồi là hình vuông.

.3.21.Lượng gas chính xác để một chiếc ôtô hoàn thành một vòng đường

đua được đạt trong n bình gas đặt dọc đường đua. Chứng minh rằng

có một vị trí mà xe có thể bắt đầu ở đó với một bình gas rỗng, có thể

hoàn thành một vòng đường đua mà không sợ hết gas ( giả sử xe có

thể chứa một lượng gas không giới hạn)

Lời giải: Ta sử dụng phương pháp qui nạp theo n, trường hợp n = 1

dễ dàng thấy được. Cho n+1 bình chứa phải có một bình chứa A mà

từ đó ôtô có thể tới được bình chứa B mà bình đó không có đủ gas

cho một vòng đua. Nếu chúng ta dồn bình B vào bình A và bỏ bình

B đi, theo giả thiết quy nạp có 1 điểm xuất phát mà xe có thể hoàn

thành vòng đua, cùng điểm xuất phát như thế cho hoàn thành vòng

đua với lượng phân phát ban đầu của bình chứa.

vnmath.com

Page 257: OLYMPIC TOÁN NĂM 2000 52 ĐỀ THI VÀ LỜI …»i nói đầu Để thử gói lệnh lamdethi.sty tôi biên soạn một số đề toán thi Olympic, mà các học trò của

Chương 4

Đề thi olympic Đài Loan

.4.22.Cho a là một số hữu tỷ, b, c, d là các số thực và f : R → [−1; 1] là 1

hàm thỏa mãn:

f(x + a + b) − f(x + b) = c[x + 2a + [x] − 2[x + a] − [b]] + d

với mọi x ∈ R. Chứng minh rằng hàm f tuần hoàn, tức là tồn tại số

p > 0 sao cho f(x + p) = f(x) với mọi x ∈ R.

Lời giải: Với mọi số nguyên n ta có:

f(x + n + a) − f(x + n)

= c [x − b + n] + 2a + [x − b + n] − 2 [x − b + n + a] − [b] + d

= c [x − b] + n + 2a + [x − b] + n − 2 [x − b + a] + n − [b] + d

= c [x − b] + 2a + [x − b] − 2 [x − b + a] − [b] + d

= f (x + a) − f (x)

Lấy số nguyên dương m sao cho am là một số nguyên. Khi đó với mọi

số tự nhiên k ta có:

vnmath.com

Page 258: OLYMPIC TOÁN NĂM 2000 52 ĐỀ THI VÀ LỜI …»i nói đầu Để thử gói lệnh lamdethi.sty tôi biên soạn một số đề toán thi Olympic, mà các học trò của

Đề thi olympic Đài Loan 17

f(x + kam) − f(x)

=x∑

j=1

m∑

i=1

(f (x + jam + ai) − f (x + jam + a (i − 1)))

= k

m∑

i=1

(f (x + ai) − f (x + a (i − 1)))

= k (f (x + am) − f (x))

Do f(x) ∈ [−1; 1], f(x + kam) − f(x) bị chặn nên f(x + kam) − f(x) phải

bằng 0. Suy ra f(x + kam) = f(x), vì vậy f(x) là hàm tuần hoàn.

.4.23.Cho đoạn thẳng AB. Tìm tất cả các điểm C trong mặt phẳng sao

cho tam giác ABC sao cho đường cao kẻ từ A và trung tuyến kẻ từ B

có độ dài bằng nhau.

Lời giải: Gọi D là chân đường cao kẻ từ A và E là chân đường trung

tuyến kẻ từ B. Gọi F là chân đường vuông góc kẻ từ E xuống BC.

Khi đó EF//AD và E là trung điểm của AC, vì vậy EF = 1/2(AD) =

1/2(BE) và EBC = ±π/6 (tất cả các góc đều có hướng trừ khi được ).

Bây giờ, cho P là một điểm sao cho B là trung điểm của AP . Khi đó

BE//PC, vì thế PCB = EBC và không đổi. Quỹ tích tất cả các điểm

C sao cho PCB không đổi là một đường tròn. Do đó, quỹ tích các

điểm C bao gồm hai đường tròn bằng nhau, cắt nhau tại B, P (Một

tương ứng với góc π/6 và một tương ứng với góc −π/6 ). Trong trường

hợp đặc biệt, khi tam giác ABC cân ta thấy rằng mỗi đường tròn đều

có bán kính AB và tâm sao cho ABQ = 2π/3 (không có hướng).

.4.24.Cho số nguyên n ≥ 3, giả thiết rằng dãy số thực dương a1, a2, ..., an

thoả mãn ai−1 + ai+1 = kiai với dãy k1, k2, ..., kn là dãy số nguyên dương

bất kỳ. (trong đó a0 = an và an+1 = a1 ). Chứng minh rằng

2n ≤ k1 + k2 + ... + kn ≤ 3n

Lời giải: Bất đẳng

2n ≤ k1 + k2 + ... + kn

vnmath.com

Page 259: OLYMPIC TOÁN NĂM 2000 52 ĐỀ THI VÀ LỜI …»i nói đầu Để thử gói lệnh lamdethi.sty tôi biên soạn một số đề toán thi Olympic, mà các học trò của

18 Nguyễn Hữu Điển, ĐHKHTN Hà Nội

được chứng minh dựa vào AM-GM với chú ý rằng:

k1 + k2 + ... + kn =n∑

i=1

ai

ai+1

+ai+1

ai

Để chứng minh được bất đẳng k1 + k2 + ... + kn ≤ 3n, ta cần chứng

minh k1 + k2 + ... + kn ≤ 3n − 2 với n ≥ 2 , bằng phương pháp quy nạp

theo n. Với n = 2, nếu a1 ≥ a2 thì 2a2 = k1a1 , vì thế hoặc a1 = a2 và

k1 + k2 = 4 = 3.2 − 2, hoặc a − 1 = 2a2 và k1 + k2 = 4 = 3.2− 2. Với n > 2,

ta có thể giả thiết tất cả các ai không bằng nhau, khi tồn tại i sao

cho ai ≥ aa−1, ai+1 mà dấu bằng không xảy ra ở ít nhất một trong hai

trường hợp. Khi đó aiaa−1 +ai+1 < 2ai và do đó ki = 1. Ta kết luận rằng

dãy mà bỏ đi số hạng ai cũng thỏa mãn điều kiện đã cho với ki−1 và

ki+1 giảm đi 1 đơn vị và bỏ đi số hạng ki. Theo giả thiết quy nạp, tổng

của ki số hạng nhỏ hơn hoặc bằng 3(n−1)−2, tổng của các số ki ban

đầu nhỏ hơn hoặc bằng 3n − 2, do đó ta có điều phải chứng minh.

.4.25.Cho k = 22n

+ 1 với n là số nguyên dương bất kỳ. Chứng minh

rằng k là một số nguyên tố khi và chỉ khi k là một ước của 3(k−1)/2 +1.

Lời giải: Giả sử k là một ước của 3(k−1)/2 + 1. Điều này tương đương

với 3(k−1)/2 ≡ −1( mod k). Vì vậy 3k−1 ≡ 1( mod k). Với d sao cho 3d ≡ 3

mod k. Vì vậy, d không là ước của (k − 1)/2 nhưng là ước của k − 1,

mặt khác (k − 1) là ước của d nên d = k − 1 (bởi vi d phải nhỏ hơn k).

Do đó, k là số nguyên tố.

Ngược lại, nếu k là số nguyên tố(

3k

)=

(k3

)=

(23

)= −1

3(k−1)/2 ≡(

3k

)≡ −1

.4.26.Cho tứ diện ABCD. Chứng minh rằng:

(a) Nếu AB = CD, AD = BC, AC = BDthì các tam giác

ABC, ACD, ABD, BCD là các tam giác nhọn.

(b) Nếu các tam giác ABC, ACD, ABD, BCD có cùng diện tích thì

AB = CD, AD = BC, AC = BD

Lời giải: (a) Theo giả thiết 4 mặt của tứ diện bằng nhau, ta có góc

tam diện ở mỗi đỉnh được tạo bởi ba góc khác nhau của một mặt.

Gọi M là trung điểm của BC. Theo bất đẳng thức trong tam giác,

vnmath.com

Page 260: OLYMPIC TOÁN NĂM 2000 52 ĐỀ THI VÀ LỜI …»i nói đầu Để thử gói lệnh lamdethi.sty tôi biên soạn một số đề toán thi Olympic, mà các học trò của

Đề thi olympic Đài Loan 19

AM + MD > AD = BC = 2MC. Các tam giác ABC và DBC là bằng

nhau, vì thế AM = DM . Do đó 2MD > 2MC; nghĩa là, MD lớn hơn

bán kính của đường tròn nằm trên mặt phẳng BCD với đường kinh

BC. Do đó, D nằm ngoài đường tròn và góc BDC là góc nhọn. Tương

tự như vậy, ta chứng minh được các góc còn lại (bài toán này là bài

toán USAMO tháng 2/1972, tham khảo các cách giải khác ở quyển

sách USAMO của Klamkin).

(b) Vì AB và CD không song song (vì ABCD)là hình tứ diện), ta có

thể chọn hai mặt phẳng song song là mặt phẳng (P ) chứa AB và (Q)

chứa CD. Gọi khoảng cách giữa mặt phẳng (P ) và (Q) là d. Gọi A′, B′

lần lượt là hình chiếu của A, B trên (Q), gọi C ′, D′ lần lượt là hình

chiếu của C, D trên (P ). Vì các tam giác ACD và BCD có cùng diện

tích và chung đáy CD, nên chúng có cùng chiều cao h. Ta vẽ hình

trụ với trục CD và bán kính h; rõ ràng A, B thuộc hình trụ này. Hai

điểm này cũng thuộc mặt phẳng (P ) và mặt phẳng (P ) giao với hình

trụ tại một hoặc hai đường thẳng song song với CD.

A và B không thể cùng nằm trên một trong hai đường thẳng này,

vì thế hình trụ và mặt phẳng (P ) sẽ giao nhau tại hai đường thẳng,

một đường thẳng đi qua A và một đường thẳng đi qua B. Hai đường

thẳng song song với nhau và cách đều đường thẳng C ′D′ một khoảng

là (h2 − d2)1/2. Vì vậy, đường thẳng C ′D′ chia đôi đoạn thẳng AB.

Tương tự, ta chứng minh được đường thẳng A′B′ chia đôi đoạn thẳng

CD, qua phép chiếu, mặt phẳng (Q) biến thành mặt phẳng (P ),

đường thẳng AB chia đôi đoạn thẳng C ′D′. AB và C ′D′ cắt nhau

tại trung điểm mỗi đường nên AC ′BD′ là hình bình hành, do đó ta

có AC ′ = BD′ (gọi khoảng cách giữa chúng là x) và BC ′ = AD′(gọi

khoảng cách giữa chúng là y). Vậy ta có:

AC =(AC ′2 + C ′C2

)1/2=

(x2 + d2

)1/2

=(BD′2 + D′D2

)1/2= BD

AC =(BC ′2 + C ′C2

)1/2=

(y2 + d2

)1/2

=(AD′2 + D′D2

)1/2= AD

vnmath.com

Page 261: OLYMPIC TOÁN NĂM 2000 52 ĐỀ THI VÀ LỜI …»i nói đầu Để thử gói lệnh lamdethi.sty tôi biên soạn một số đề toán thi Olympic, mà các học trò của

20 Nguyễn Hữu Điển, ĐHKHTN Hà Nội

Sử dụng hai cặp cạnh khác lúc đầu, ta cũng có AB = CD

.4.27.Cho X là một tập hợp các số nguyên cho bởi công thức:

a2k102k + a2k−2102k−2 + ... + a2102 + a0

Với k là một số nguyên không âm và a2i ∈ {1, 2, ..., 9} với i = 0, 1, ..., k.

Chứng minh rằng, mọi số nguyên có dạng 2p3q , với p, q là những số

nguyên không âm, chia hết một phần tử nào đó của X.

Lời giải: Mọi số nguyên không chia hết cho 10 thì đều chia hết cho

một số hạng nào đó của X. Ta nhận thấy rằng luôn tồn tại một phần

tử trong Xcó2p − 1 chữ số là bội của 4p, với mọi số nguyên p không

âm. Điều này dẫn tới phép quy nạp theo p: với p = 0, 1 và nếu x là

một bội số với p = k, thì ta có thể chọn a2k sao cho x + a2k102k ≡ 0(

mod 4k+1) vì 102k ≡ 1( mod 4k)

Bây giờ, ta chứng minh rằng bất kỳ số nguyên n nào không chia

hết cho 10 thì chia hết cho một số hạng nào đó của x. Gọi n = 2pk

với k lẻ. Khi đó với bổ đề bên trên ta có thể tìm ra bội số của 2p

trong tập hợp X. Gọi m là bội số và d là số chữ số của m và f =

10d+1 − 1. Bằng hệ quả Euler của định lý Fermat, 10φfk( mod fk) . Vì

vậy m(10(d+1)φ(fk) − 1

)/(10d+1 − 1

)chia hết cho 2pk và thuộc X

.4.28.Xác định tất cả các số nguyên dương k để tồn tại một hàm f :

N → Z thỏa mãn:

(a)f(1997) = 1998

(b) Với mọi a, b ∈ N, f(ab) = f(a) + f(b) + kf(gcd(a, b))

Lời giải: Hàm f như trên tồn tại với k = 0 và k = −1. Ta lấy a = b,

thay vào (b) ta có f(a2) = (k + 2)f(a). Áp dụng hai lần ta có:

f(a4) = (k + 2)f(a2) = (k + 2)2f(a)

Mặt khác ta có:

f(a4) = f(a) + f(a3) + kf(a) = (k + 1)f(a) + f(a3)

= (k + 1)f(a) + f(a) + f(a2) + kf(a)

= (2k + 2)f(a) + f(a2) = (3k + 4)f(a)

vnmath.com

Page 262: OLYMPIC TOÁN NĂM 2000 52 ĐỀ THI VÀ LỜI …»i nói đầu Để thử gói lệnh lamdethi.sty tôi biên soạn một số đề toán thi Olympic, mà các học trò của

Đề thi olympic Đài Loan 21

Chọn a = 1997 để f (a) 6= 0 suy ra (k + 2)2 = 3k + 4 có nghiệm k = 0; k =

−1.

Với k = 0 ta có:

f (pe11 ...pen

n ) = e1g (p1) + ... + eng (pn)

với m là ước nguyên tố của 1997, g(m) = 1998 và g(p) = 0 với mọi số

nguyên tố p 6= m.

Với k = 1 ta có

f (pe11 ....pen

n ) = g (p1) + ... + g (pn)

.4.29.Cho tam giác ABC nhọn với O là tâm đường tròn ngoại tiếp và

bán kính R. AO cắt đường tròn ngoại tiếp tam giác OBC ở D, BO cắt

đường tròn ngoại tiếp tam giác OCA ở E, và CO cắt đường tròn ngoại

tiếp tam giác OAB ở F . Chứng minh rằng OD.OE.OF ≤ 8R3.

Lời giải: Gọi D′, E ′, F ′ lần lượt là giao điểm của AO và BC , BO và CA,

CO và AB. Do đó chúng là ảnh của D, E, F qua phép của đường tròn

ngoại tiếp tam giác ABC, Do đó OD′.OD = OE ′.OE = OF ′.OF = R2. Vì

vậy, bất đẳng thức của bài toán tương đương với:

AO

OD′BO

OE ′CO

OF ′ > 8

Gọi h1, h2, h3 lần lượt là khoảng cách từ O đến AB, BC, CA. Khi

đó AO/OD = [AOB]/[BOD′] = (ABh1)/(BD′h2), tương tự BO/OE ′ =

(BCh2)/(CE′h3) và CO/OF ′ = (CAh3)/(AF ′h1). Do đó ta có:

AB.BC.CA

AF ′.BD′.CE′ =(AF ′ + F ′B) (BD′ + D′C) (CE′ + E ′A)

AF ′.BD′.CE′

> 8

√AF ′.F ′B.BD′.D′C.CE′.E ′A

AF ′.BD′.CE′

= 8

√F ′B.D′C.′A

AF ′.BD′.CE′ = 8

Dấu bằng xảy ra khi và chỉ khi AF ′ = F ′B, BD′ = D′C, CE′ = E ′A, hay

ABC là tam giác đều.

.4.30.Cho X = 1, 2, ..., n với n ≥ k ≥ 3 và Fk là một tập con gồm k phần

tử của X sao cho hai tập Fk bất kỳ có nhiều nhất k − 2 phần tử chung.

vnmath.com

Page 263: OLYMPIC TOÁN NĂM 2000 52 ĐỀ THI VÀ LỜI …»i nói đầu Để thử gói lệnh lamdethi.sty tôi biên soạn một số đề toán thi Olympic, mà các học trò của

22 Nguyễn Hữu Điển, ĐHKHTN Hà Nội

Chỉ ra rằng tồn tại một tập Mk của X có ít nhất [log2 n] + 1 phần tử

không được chứa trong một tập con nào của Fk.

Lời giải: Nếu k ≥ log2 n thì điều phải chứng minh hiển nhiên đúng,

vì thế ta giả sử k < log2 n. Đặt m = [log2 n] + 1. Với mỗi tập con k− 1 số

hạng của tập Xthuộc tối đa một tập con của Fk và mỗi số hạng của

Fk bao gồm các tập con k(k − 1) phần tử. Ta có:

(Fk) ≤1

k

n

k − 1

=

1

n − k + 1

n

k

Mặt khác, chọn một tập con m số hạng bất kỳ của X, số số hạng của

Fk là: m

k

(Fk)

n

k

≤ 1

n − k + 1

m

k

Điều đó có thể chứng minh rằng con số sau nhỏ hơn 1, vì thế một

tập con m phần tử nào đó không được chứa số hạng nào của Fk.

Hiển nhiên

m

k

∑i

m

i

= 2m và ta cũng có thể chứng minh

m

k

≤ 3.2m−3 với m ≥ k ≥ 3 bằng phương pháp quy nạp theo m. Vì

vậy ta có:

1

n − k + 1

m

k

≤ 3n

4 (n − k + 1)< 1

với n ≥ 3. Ta có điều phải chứng minh.

vnmath.com

Page 264: OLYMPIC TOÁN NĂM 2000 52 ĐỀ THI VÀ LỜI …»i nói đầu Để thử gói lệnh lamdethi.sty tôi biên soạn một số đề toán thi Olympic, mà các học trò của

Chương 5

Đề thi olympic Thổ Nhĩ Kỳ

.5.31.Cho tam giác ABC vuông tại A, gọi H là chân đường cao kẻ A.

Chứng minh rằng tổng bán kính các đường tròn nội tiếp các tam giác

ABC, ABH, ACH bằng AH.

Lời giải: Đặt a = BC, b = CA, c = AB và s = a+b+c2

. Các tam giác ABH

và ACH đồng dạng với tam giác ABC với tỉ số tương ứng a/c và b/c

Áp dụng công thức diện tích tam giác bằng bán kính đường

tròn nội tiếp nhân với nửa chu vi, suy ra bán kính cần tìm làab

a+b+c; a

cab

a+b+c; b

cab

a+b+cvà tổng của chúng là ab

c= AH

.5.32.Dãy số {an}∞n=1 , {bn}∞n=1 được cho bởi:

a1 = α, b1 = β, an+1 = αan − βbn, bn+1 = αan + αbn với mọi n ≥ 1

Có bao nhiêu bộ số thực (α, β) thỏa mãn a1997 = b1 và b1997 = a1?

Lời giải: Lưu ý rằng a2n+1 + b2

n+1 = (α2 + β2)(a

2

n + b2

n

), trừ α = β = 0.

Chúng ta cần α2 + β2 = 1. Vì vậy có thể đặt α = cos θ, β = sin θ, từ đó

bằng phương pháp quy nạp ta chỉ ra α = cos nθ, β = sin nθ. Từ đó có

1998 bộ số: (0; 0) và (cos θ; sin θ) với θ = π k3998

, k = 1, 3, ..., 3997

.5.33.Trong một hiệp hội bóng đá, khi một cầu thủ chuyển từ đội X có

x cầu thủ sang đội Y có y cầu thủ, liên đoàn nhận được y−x triệu đôla

từ đội Y nếu y ≥ x nhưng phải trả lại x − y triệu đôla cho đội X nếu

vnmath.com

Page 265: OLYMPIC TOÁN NĂM 2000 52 ĐỀ THI VÀ LỜI …»i nói đầu Để thử gói lệnh lamdethi.sty tôi biên soạn một số đề toán thi Olympic, mà các học trò của

24 Nguyễn Hữu Điển, ĐHKHTN Hà Nội

x > y. Một cầu thủ có thể di chuyển tùy thích trong suốt mùa chuyển

nhượng. Hiệp hội bao gồm 18 đội, tất cả các đội đều bắt đầu mùa

chuyển nhượng với 20 cầu thủ. Kết thúc mùa chuyển nhượng, 12 đội

kết thúc với 20 cầu thủ, 6 đội còn lại kết thúc với 16,16,21,22,22,23

cầu thủ. Tổng số tiền lớn nhất mà liên đoàn có thể kiếm được trong

suốt mùa chuyển nhượng là bao nhiêu?

Lời giải: Chúng ta thừa nhận rằng số tiền lớn nhất kiếm được bởi

không bao giờ cho phép một cầu thủ chuyển đến đội nhỏ hơn. Chúng

ta cũng có thể giữ kỷ lục đó bằng một cách khác. Một đội bóng có x

cầu thủ thì được ghi là −x trước khi giao dịch một cầu thủ hoặc x

trước khi nhận một cầu thủ và số tiền mà liên đoàn kiếm được bằng

tổng của các số đó. Bây giờ ta xem xét các số được ghi bởi một đội

mà kết thúc có nhiều hơn 20 cầu thủ. Nếu số lượng cầu thủ tối đa

của đội đó trong suốt quá trình chuyển nhượng là k > n thì các số

k−1 và −k xuất hiện liên tiếp và bỏ đi 2 số đó thì tổng sẽ tăng lên. Vì

thế tổng của các số trong đội bóng đó ít nhất là 20 + 21 + ... + (n − 1).

Tương tự như vậy, tổng của các số trong đội bóng kết thúc có n < 20

cầu thủ ít nhất là −20 − 19 − ... − (n + 1). Vì những con số này chính

xác là những con số có được bởi việc luân chuyển cầu thủ từ đội kết

thúc ít hơn 20 cầu thủ sang đội kết thúc có nhiều hơn 20 cầu thủ.

Sự sắp xếp đó dẫn đến số tiền kiếm được là lớn nhất. Trong trường

hợp đó, tổng là:

(20 + 20 + 21 + 20 + 21 + 20 + 21 + 22)− 2(20 + 19 + 18 + 17) = 17

.5.34.Ngũ giác ABCDE lồi có các đỉnh nằm trên đường tròn đơn vị,

cạnh AE đi qua tâm đường tròn đó. Giả sử AB = a, BC = b, CD =

c, DE = d và ab = cd = 1/4. Tính AC + CE theo a, b, c.

Lời giải: Nếu gọi 2α, 2β, 2γ, 2δ là các cung chắn bởi các cạnh a, b, c, d

tương ứng thì:

AC = 2 sin (α + β) =a

2

√1 − b2

4+

b

2

√1 − a2

4

Tương tự với CD.

Tổng quát với R là bán kính đường tròn ngoại tiếp ngũ giác, thế thì

vnmath.com

Page 266: OLYMPIC TOÁN NĂM 2000 52 ĐỀ THI VÀ LỜI …»i nói đầu Để thử gói lệnh lamdethi.sty tôi biên soạn một số đề toán thi Olympic, mà các học trò của

Đề thi olympic Thổ Nhĩ Kỳ 25

AC2 + BD2 = 1 mà

AC = a√

R2 − b2 + b√

R2 − a2

Khi đó, dẫn đến biểu thức chứa R2 dưới dấu căn và ta giải phương

trình đối với R theo các số a, b, c, d.

.5.35.Chứng minh rằng với mỗi số nguyên tố p ≥ 7, tồn tại một số

nguyên dương n và các số nguyên x1, x2, ..., xn, y1, y2, ..., ym không chia

hết cho p sao cho:

x21 + y2

1 ≡ x22 (modp)

x22 + y2

2 ≡ x23 (modp)

.....

x2n + y2

n ≡ x21 (modp)

Lời giải: Gọi n là cấp của 5/3 mod p, và đặt xi = 3n−1−i5i−1, yi =

43n−15i−1 thì mọi đồng dư trên là tương đương trừ hệ thức cuối cùng.

Hệ thức đó có dạng 52n ≡ 32n (modp) (đúng).Vậy ta có điều phải chứng

minh.

.5.36.Cho các số nguyên n ≥ 2. Tìm giá trị nhỏ nhất của :

x51

x2 + x3 + ... + xn+

x52

x3 + x4 + ... + xn + x1+ ... +

x5n

x1 + x2 + ... + xn−1

Với x1, x2, ..., xn là các số thực thỏa mãn x21 + x2

2 + ... + x2n = 1.

Lời giải: Đặt S = x1 + x2 + ... + xn

Sử dụng BĐT Chebyshev’s cho hai dãy xi

S−xivà x4

i (cả hai dãy đều là

dãy tăng). Ta có:

∑ x5i

S − xi≥

(x1

S − x1+

x2

S − x2+ ... +

xn

S − xn

)(x4

1 + x42 + ... + x4

n

n

)

Áp dụng bất đẳng thức hàm lồi ta có:

x1

S − x1

+x2

S − x2

+ ... +xn

S − xn

≥ 1

n − 1

vnmath.com

Page 267: OLYMPIC TOÁN NĂM 2000 52 ĐỀ THI VÀ LỜI …»i nói đầu Để thử gói lệnh lamdethi.sty tôi biên soạn một số đề toán thi Olympic, mà các học trò của

26 Nguyễn Hữu Điển, ĐHKHTN Hà Nội

Áp dụng bất đẳng thức giá trị trung bình ta có:

(∑ x4i

n

)1/2

≥∑ x2

i

n=

1

n

Ta có kết luận:

∑ x5i

S − xi

≥ n1

n − 1.1

n2=

1

n (n − 1)

Đẳng thức xảy ra khi x1 = x2 = ... = xn = 1√n.

vnmath.com

Page 268: OLYMPIC TOÁN NĂM 2000 52 ĐỀ THI VÀ LỜI …»i nói đầu Để thử gói lệnh lamdethi.sty tôi biên soạn một số đề toán thi Olympic, mà các học trò của

Chương 6

Đề thi olympic Ukraina

.6.37.Một lưới hình chữ nhật được tô màu theo kiểu bàn cờ, và trong

mỗi ô có một số nguyên. Giả sử rằng tổng các số trong mỗi hàng và

tổng các số trong mỗi cột là số chẵn. Chứng minh rằng tổng tất cả các

số trong ô đen là chẵn.

Lời giải: Giả sử các màu tô là đỏ và đen, trong đó ô vuông góc trái

trên là màu đỏ. (Vì tổng tất cả các số trong lưới hình chữ nhật là

chẵn nên điều cần chứng minh cũng tương đương với tổng các ô

màu đỏ là số chẵn.)

Tổng các hàng thứ nhất, thứ ba, . . . (từ trên xuống), và các cột thứ

nhất, thứ ba, . . . (từ trái sang) bằng tổng các số trong các ô màu đen

trừ đi hai lần tổng tất cả các số trong các ô màu đỏ. Vì tổng này là

số chẵn nên tổng các số trong các ô đen là chẵn.

vnmath.com

Page 269: OLYMPIC TOÁN NĂM 2000 52 ĐỀ THI VÀ LỜI …»i nói đầu Để thử gói lệnh lamdethi.sty tôi biên soạn một số đề toán thi Olympic, mà các học trò của

28 Nguyễn Hữu Điển, ĐHKHTN Hà Nội

.6.38.Tìm tất cả các nghiệm thực của hệ phương trình sau:

x1 + x2 + · · ·+ x1997 = 1997

x41 + x4

2 + · · ·+ x41997 = x3

1 + x32 + · · ·+ x3

1997.

Lời giải: Ta sẽ chứng minh hệ trên chỉ có nghiệm x1 = x2 = · · · =

x1997 = 1.

Đặt Sn = xn1 + · · ·+ xn

1997. Theo bất đẳng thức luỹ thừa trung bình 1,

(S4

1997

)1/4

≥ S1

1997= 1

và (S4

1997

)1/4

≥(

S3

1997

)1/3

=

(S4

1997

)1/3

do đóS4

1997≤ 1.

Vì vậy bất đẳng thức luỹ thừa trung bình xảy ra dấu bằng, nghĩa là

x1 = · · · = x1997 = 1

.6.39.Kí hiệu d(n) là số lẻ lớn nhất trong các ước số của số tự nhiên n.

Ta xác định hàm f : N −→ N sao cho

f(2n − 1) = 2n và f(2n) = n +2n

d(n)với mọi n ∈ N

Tìm tất cả các số k sao cho f(f(. . . f(1) . . .)) = 1997, ở đó f được lặp k

lần.

Lời giải: Chỉ có một giá trị của k là 499499.

Với mỗi số chẵn (2a)b, ở đó b là số lẻ và a ≥ 1,

f((2a)b) = (2a−1)b +(2a)b

b= (2a−1)b + 2a = (2a−1)(b + 2).

Do đó lũy thừa cao nhất của hai trong f((2a)b) bằng một nửa luỹ

thừa cao nhất của hai trong (2a)b. Vì vậy, lấy lặp a lần của f tại (2a)b

để được một số lẻ.

1"power mean inequality" (trong bản tiếng Anh), tạm dịch là "bất đẳng thức luỹ

thừa trung bình" (ND)

vnmath.com

Page 270: OLYMPIC TOÁN NĂM 2000 52 ĐỀ THI VÀ LỜI …»i nói đầu Để thử gói lệnh lamdethi.sty tôi biên soạn một số đề toán thi Olympic, mà các học trò của

Đề thi olympic Ukraina 29

Ngoài ra, ước lẻ lớn nhất của f((2a)b) là b + 2, lớn hơn ước lẻ lớn nhất

của (2a)b hai đơn vị. Vì vậy sau a lần lặp f tại (2a)b, ước lẻ lớn nhất là

b + 2a. Do sau a lần lặp f ta nhận được số lẻ nên fa((2a)b) phải bằng

b + 2a.

Sau một lần lặp f tại 2n − 1, ở đó n ≥ 1, ta nhận được 2n. Đặt a = n

và b = 1. Ta thấy phải lặp thêm n lần nữa mới nhận được số lẻ, số đó

là, b + 2a = 2n + 1. Vì vậy phải lặp n + 1 lần cho f tại 2n − 1 để nhận

được số lẻ tiếp theo, 2n + 1.

Ta dễ thấy là không có số lẻ nào xuất hiện hai lần khi lặp f tại 1, bởi

vì dãy các số lẻ nhận được từ việc lặp f tại bất kì số nguyên dương

nào cũng là một dãy tăng ngặt. Đặc biệt, tồn tại một giá trị k sao cho

f (k)(1) = 1997, và số k đó là duy nhất.

Bây giờ ta chứng minh bằng quy nạp rằng

fn(n+1)

2−1(1) = 2n − 1.

Dễ thấy điều đó đúng tại n = 1. Sau đó giả sử nó đúng với n = k,

f(k+1)(k+2)

2−1(1) = fk+1

(f

k(k+1)2

−1(1))

= fk+1(2k − 1) = 2k + 1

Vì 1997 = 2(999) − 1, ta có

f999.1000

2−1(1) = f 499499(1) = 1997,

nên k = 499499 thoả mãn yêu cầu đề bài, và do lập luận trên, k là

duy nhất.

.6.40.Hai ngũ giác đều ABCDE và AEKPL trong không gian sao cho

DAK = 60o. Chứng minh rằng hai mặt phẳng ACK và BAL vuông góc.

A

B

C

D E

KP

L

T

O

vnmath.com

Page 271: OLYMPIC TOÁN NĂM 2000 52 ĐỀ THI VÀ LỜI …»i nói đầu Để thử gói lệnh lamdethi.sty tôi biên soạn một số đề toán thi Olympic, mà các học trò của

30 Nguyễn Hữu Điển, ĐHKHTN Hà Nội

Lời giải: Nếu ta quay AEKPL quanh trục AE, bắt đầu ở vị trí trùng

nhau với ABCDE, thì góc DAK tăng cho đến khi AEKPL lại nằm

trên mặt phẳng chứa ABCDE. (Điều này có thể nhận được bằng

cách tính toán thấy tích vô hướng của vectơ AD và AK tăng trong

một khoảng xác định). Vì vậy có một góc duy nhất giữa các mặt

phẳng sao cho góc DAK = 60◦. Thật vậy, điều này xảy ra khi các

hình ngũ giác là hai mặt của khối mười hai mặt đều 2 (do tính chất

đối xứng, tam giác DAK là đều trong trường hợp này). Đặc biệt, mặt

phẳng BAL là mặt phẳng chứa mặt thứ ba qua đỉnh A của khối mười

hai mặt đều, ta gọi mặt đó là BALTO.

Sử dụng thêm dấu phẩy sau tên điểm để ký hiệu hình chiếu tương

ứng của các điểm xuống mặt phẳng ABCDE. Khi đó K ′ nằm trên

đường phân giác của góc DEA. Lập hệ trục tọa độ sao cho ABCDE

nằm trên mặt phẳng Oxy theo chiều kim đồng hồ, và−−→K ′E cùng chiều

với chiều dương của trục Ox. Xét hai vectơ−−→EK và

−→EA. Vectơ thứ nhất

không có thành phần y, và vectơ thứ hai không có thành phần z. Do

đó tích vô hướng của hai vectơ đó bằng tích các thành phần x của

chúng. Vì AE và K ′E tạo với nhau góc 54◦,−→EA có thành phần x được

tính bởi AE(cos 54◦) = cos 54o.−−→EK có thành phần x là −K ′E. Do đó

tích vô hướng của chúng bằng −K ′E(cos 54◦). Mặt khác−−→EK.

−→EA =

EK.EA. cos KEA = (1).(1). cos(108◦) = − cos 72◦. Lập phương trình từ

hai kết quả trên ta nhận được K ′E = cos 72◦

cos 54◦

Các tính toán trên vectơ sau đây sẽ chứng tỏ K ′A và AC vuông góc:

−−→K ′A.

−→AC = (

−−→K ′E +

−→EA).

−→AC

= K ′E · AC cos 54◦ − CA · AC cos 72◦

=cos 72◦

cos 54◦cos 54◦ − cos 72◦ = 0

Vì BO′ vuông góc với AC và BO song song với AT , nên AT ′ vuông góc

với AC. Vì vậy các điểm K ′, A, T ′ thẳng hàng. Vì K, A, T không thẳng

hàng nên mặt phẳng AKT vuông góc với mặt phẳng chứa ABCDE.

Quay xung quanh A, ta suy ra mặt phẳng ACK vuông góc với mặt

2khối thập nhị diện đều (ND)

vnmath.com

Page 272: OLYMPIC TOÁN NĂM 2000 52 ĐỀ THI VÀ LỜI …»i nói đầu Để thử gói lệnh lamdethi.sty tôi biên soạn một số đề toán thi Olympic, mà các học trò của

Đề thi olympic Ukraina 31

phẳng chứa BALTO, từ đó có điều phải chứng minh.

.6.41.Cho phương trình ax3 + bx2 + cx + d = 0 có ba nghiệm thực phân

biệt. Hỏi phương trình sau có bao nhiêu nghiệm thực:

4(ax3 + bx2 + cx + d)(3ax + b) = (3ax2 + 2bx + c)2

Lời giải: Đặt P (x) = ax3 + bx2 + cx + d và Q(x) = 2P (x)P ′′(x) − [P ′(x)]2,

ta sẽ đếm số nghiệm thực của Q(x) = 0. Ta có thể thực hiện một số

phép rút gọn mà không thay đổi số nghiệm thực của Q(x). Trước tiên

ta chia P cho một hệ số tỉ lệ sao cho a = 1. Sau đó ta tịnh tiến x sao

cho nghiệm đứng giữa của P là 0, tức là d = 0 và c < 0. Khi đó:

Q(x) = 3x4 + 4bx3 + 6cx2 − c2.

Bây giờ ta áp dụng quy tắc dấu Descartes để đếm số nghiệm của

Q(x). Dấu của Q(x) là +, s,−,− và của Q(−x) là +,−s,−,−, ở đó s là

dấu của b. Trong trường hợp nào thì mỗi dãy trên cũng chỉ có đúng

một lần đổi dấu. Vì vậy Q có một nghiệm âm và một nghiệm dương,

tức là Q có tất cả hai nghiệm thực.

.6.42.Kí hiệu Q+ là tập tất cả các số hữu tỉ dương. Tìm tất cả các hàm

số f : Q+ → Q+ sao cho với mọi x ∈ Q+:

(a) f(x + 1) = f(x) + 1

(b) f(x2) = f(x)2.

Lời giải: Chỉ có một hàm số thoả mãn là f(x) = x. Từ (a), f(x + n) =

f(x) + n với mọi n nguyên dương. Đặt x = pq

với p, q nguyên dương. Ta

có:

f

(p + q2

q

)2

=

(q + f

(p

q

))2

= q2 + 2qf

(p

q

)+ f

(p

q

)2

.

Mặt khác,

f

(p + q2

q

)2

= f

((p + q2)2

q2

)

= f

(q2 + 2p +

p2

q2

)= q2 + 2p + f

(p

q

)2

.

vnmath.com

Page 273: OLYMPIC TOÁN NĂM 2000 52 ĐỀ THI VÀ LỜI …»i nói đầu Để thử gói lệnh lamdethi.sty tôi biên soạn một số đề toán thi Olympic, mà các học trò của

32 Nguyễn Hữu Điển, ĐHKHTN Hà Nội

Từ hai phương trình trên suy ra

2qf

(p

q

)= 2p

vì vậy f(p

q) =

p

q.

.6.43.Tìm số nguyên n nhỏ nhất sao cho với mỗi n số nguyên tuỳ ý, tồn

tại 18 số nguyên trong đó có tổng chia hết cho 18.

Lời giải: Giá trị nhỏ nhất của n là n = 35; tập hợp 34 phần tử gồm 17

số không và 17 số một cho ta thấy rằng n ≥ 35. Vì vậy ta chỉ còn phải

chứng minh rằng với 35 số nguyên bất kì luôn tìm được 18 số trong

đó có tổng chia hết cho 18. Thực ra ta sẽ chứng minh rằng với mọi n,

trong 2n − 1 số nguyên luôn tìm được n số có tổng chia hết cho n.

Ta sẽ chứng minh khẳng định trên bằng quy nạp theo n. Dễ thấy

điều đó đúng với n = 1. Nếu n là hợp số, viết n = pq, ta có thể lấy ra

tập p số nguyên mà tổng chia hết cho p cho đến khi còn lại ít nhất

2p − 1 số; ta có 2q − 1 tập như vậy, và lại theo giả thiết quy nạp, có q

tập trong số đó có tổng (của pq số) chia hết cho q.

Bây giờ giả sử n = p là số nguyên tố. Số x chia hết cho p nếu và chỉ

nếu xp−1 6≡ 1 (mod p). Vì vậy nếu khẳng định trên sai thì tổng các

số dạng (a1 + · · · + ap)p−1 trên mọi tập con {a1, . . . , ap} của các số đã

cho đồng dư với Cp−12p−1 ≡ 1 (mod p). Mặt khác, tổng của các số dạng

ae11 · · ·aep

p với e1 + · · ·+ ep ≤ p − 1 luôn chia hết cho p: nếu k ≤ p − 1 các

số ei khác không thì mỗi tích được lặp lại Cp−k2p−1−k lần, và cuối cùng

là một bội của p. Sự mâu thuẫn đó chứng tỏ khẳng định ở trên là

đúng trong trường hợp này. (Chú ý: để chứng minh bài toán đã cho,

cần phải chứng minh trực tiếp cho trường hợp p = 2, 3.)

.6.44.Các điểm K, L, M, N nằm trên các cạnh AB, BC, CD, DA

của hình hộp (không cần là hình hộp đứng) ABCDA1B1C1D1.

Chứng minh rằng các tâm mặt cầu ngoại tiếp của các tứ diện

A1AKN, B1BKL, C1CLM, D1DMN là các đỉnh của một hình bình hành.

Lời giải: Đưa vào hệ tọa độ với ABCD song song với z =

0. Gọi E, F, G, H là tâm đường tròn ngoại tiếp các tam giác

vnmath.com

Page 274: OLYMPIC TOÁN NĂM 2000 52 ĐỀ THI VÀ LỜI …»i nói đầu Để thử gói lệnh lamdethi.sty tôi biên soạn một số đề toán thi Olympic, mà các học trò của

Đề thi olympic Ukraina 33

AKN, BKL, CLM, DMN và gọi W, X, Y, Z là tâm mặt cầu ngoại tiếp

các tứ diện A1AKN, B1BKL, C1CLM, D1DMN . Với mỗi điểm Q ta kí

hiệu Q1, Q2, Q3 là các tọa độ x, y, z của Q.

1

A

B C

D

A

B C

D1

1

KL

MN

Trước tiên ta sẽ chứng minh EFGH là hình bình hành, bằng cách

chứng minh trung điểm của EG và FH trùng nhau. Chỉ cần chứng

minh điều đó với hình chiếu của các đoạn thẳng trên hai phương

khác nhau (chẳng hạn, xây dựng hệ trục theo các phương đó).

Nhưng rõ ràng với phép chiếu trên AB, vì E và F chiếu tương ứng

xuống trung điểm của AK và BK, nên đoạn thẳng giữa chúng có

chiều dài AB2

, tương tự với CD. Ta cũng có lập luận như vậy với phép

chiếu trên CD.

Bây giờ ta có E1 + G1 = F1 + H1 và E2 + G2 = F2 + H2. Hơn nữa, vì

W và E cách đều AKN , WE vuông góc với AKN nên vuông góc với

mặt phẳng z = 0. Vậy W1 = E1 và W2 = E2, tương tự với X, Y, Z. Vật

W1 + Y1 = X1 + Z1 và W2 + Y2 = X2 + Z2. Ta chỉ còn phải chứng minh

W3 + Y3 = X3 + Z3. Chú ý rằng cả W và X đều nằm trên mặt phẳng

vuông góc với ABB1A1 và đi qua trung điểm của AA1 và BB1. Do đó

W3 = aW1+bW2+c và X3 = aX1+bX2+c với a, b, c là các hằng số. Tương

tự, Y và Z đều nằm trên mặt phẳng vuông góc CDD1C1 và qua trung

điểm của CC1, DD1. Vì DCC1D1 song song và bằng ABB1A1, ta có

Y3 = aY1 + bY2 + d và Z3 = aZ1 + bZ2 + d với d là một hằng số khác, còn

a, b là các hằng số ở công thức trên. Bởi vậy W3 + Y3 = X3 + Z3, từ đó

hoàn thành chứng minh rằng WXY Z là hình bình hành.

vnmath.com

Page 275: OLYMPIC TOÁN NĂM 2000 52 ĐỀ THI VÀ LỜI …»i nói đầu Để thử gói lệnh lamdethi.sty tôi biên soạn một số đề toán thi Olympic, mà các học trò của

Chương 7

Đề thi olympic Anh

.7.45.Giả sử M và N là hai số nguyên dương có 9 chữ số có tính chất là

nếu bất kì chữ số của M được thay bởi chữ số của N tương ứng thì ta

được một bội của 7. Chứng minh rằng với bất kì một số đạt được bằng

cách thay một chữ số của N tương ứng bởi một chữ số của M cũng là

một bội của 7.

Tìm một số nguyên d > 9 sao cho kết quả trên vẫn còn đúng khi M và

N là hai số nguyên dương có d chữ số.

Lời giải: Kết quả đúng với bất kì d ≡ 2 ( mod 7). Viết M =∑

k 10k, N =

nk10k, ở đây mk, nk là các chữ số. Thì với bất kì k, 10k(nk−mk) ≡ 0−M (

mod 7). Lấy tổng theo k, chúng ta được M − N ≡ dM ≡ 2M ( mod 7),

vì vậy N ≡ −M ( mod 7), được 10k(mk − nk) ≡ −N ( mod 7). Vậy khi

thay bất kì chữ số trong N bởi chữ số tương ứng trong M chúng ta

đạt được một số chia hết cho 7.

.7.46.Trong tam giác nhọn ABC, CF là một đường cao, với F trên AB,

và BM là một trung tuyến, với M trên CA. Cho BM = CF và ∠MBC =

∠FCA, chứng minh rằng tam giác ABC là đều.

Lời giải: Giả sử ∠ACF = ∠CBM = A, và giả sử CM = AM = m. Thì

MB = CF = 2m cos A. Theo định lí hàm số Sin,

CM

sin ∠CBM=

MB

sin ∠MCB,

vnmath.com

Page 276: OLYMPIC TOÁN NĂM 2000 52 ĐỀ THI VÀ LỜI …»i nói đầu Để thử gói lệnh lamdethi.sty tôi biên soạn một số đề toán thi Olympic, mà các học trò của

Đề thi olympic Anh 35

và vì vậy sin ∠MCB = 2 cos A sin A = sin 2A.

Điều này đưa đến hai khả năng. Nếu ∠MCB+2A = 180◦, thì ∠CMB =

A = ∠MBC. Khi đó CB = MC và MB = 2MC sin A. Cũng có MB =

CF = AC cos A = 2MC cos A. Do đó sin A = cos A vì vậy A ≥ 45◦ ≥∠MCB ≥ 90◦, mâu thuẫn.

Vì vậy chúng ta có ∠MCB = 2A, vì vậy ∠ACF = ∠BCF . Do đó tam

giác ACF đồng dạng với tam giác CBM , vì vậy ∠CAF = ∠BCM . Do

đó BC = AB, vậy tam giác ABC là đều.

.7.47.Tìm số các đa thức bậc 5 với các hệ số khác nhau từ tập

{1, 2, . . . , 9} mà chia hết cho x2 − x + 1.

Lời giải: Cho phương trình bậc 5 là ax5 + bx4 + cx3 + dx2 + ex + f = 0.

Các nghiệm của x2 − x + 1 không phải là các nghiệm thực của x3 + 1,

đó là eπi/3 và e5πi/3. Do đó đa thức bậc 5 là chia hết cho x2 − x + 1 nếu

và chỉ nếu

ae5πi/3 + be4πi/3 + ceπi + de2πi/3 + eeπi/3 + f = 0.

Nói các khác, vì vậy i sin 60(−a − b + d + e = 0), hoặc a − d = e − b và

a/2 − b/2 − c − d/2 + e/2 + f = 0, hay e + 2f + a = b + 2c + d hoặc (vì

a − d = e − b) a − d = c− f = e − b. Điều này kéo theo 1/12 của đa thức

sẽ có các hệ số p + k, q, r + k, p, q + k, r với k > 0 và p ≤ q ≤ r.

Với k đã cho, có C39−k các giá trị của p, q, r sao cho r+k ≤ 9. Tuy nhiên,

các hệ số phải khác nhau, nên chúng ta phải trừ đi. Có 9 − 2k cách

lựa chọn 2 số khác nhau theo k, và 7 − k cách lựa chọn các số còn

lại. Tuy nhiên, chúng ta đếm được các số dạng x, x + d, x + 2d sinh

đôi, và có 9 − 2k số.

Do đó, với k cho trước, chúng ta có

C39−k − (9 − 2k)(7 − k) + 9 − 3k

đa thức. Cộng lại, có (1+4+10+20+35+56)−(42+25+12+3)+(3+6) = 53

đa thức dạng trên, và 53.12 = 636 các đa thức tất cả.

.7.48.Tập S = {1/r : r = 1, 2, 3, . . .} các số nguyên dương nghịc đảo lập

thành cấp số cộng với độ dài tùy ý. Chẳng hạn, 1/20, 1/8, 1/5 là một

vnmath.com

Page 277: OLYMPIC TOÁN NĂM 2000 52 ĐỀ THI VÀ LỜI …»i nói đầu Để thử gói lệnh lamdethi.sty tôi biên soạn một số đề toán thi Olympic, mà các học trò của

36 Nguyễn Hữu Điển, ĐHKHTN Hà Nội

cấp số như vậy, với độ dài là 3 với phương sai là 3/40. Hơn nữa, có

một cấp số cực đại trong S với độ dài 3 vì nó không thể mở rộng thêm

vè bên phải và bên trái của S (−1/40 và 11/40 không là phần tử của S).

(a) Tìm một cấp số cực đại trong S có độ dài 1996.

(b) Có hay không một cấp số trong S có độ dài 1997?

Lời giải: Có một cấp số cực đại có độ dài n, với mọi n > 1. Theo định

lí Dirichlet kéo theo rằng có một số nguyên tố p có dạng 1 + dn với

mỗi số nguyên dương d. Bây giờ xét cấp số

1

(p − 1)!,

1 + d

(p − 1)!, . . . ,

1 + (n − 1)d

(p − 1)!.

Vì các mẫu số chia hết từ số, nên mỗi phân thức là một số nguyên

nghịc đảo, nhưng với (1 + nd)/(p − 1)! = p/(p − 1)! không phải vì p là

số nguyên tố. Do đó có dãy cấp số cực đại. (Để giải (a), đơn giản lấy

p = 1997.)

vnmath.com